You are on page 1of 115

!"#$%&"'( *+,- .

ABOITIZ SHIPPING CORPORATION V NEW


INDIA ASSURANCE COMPANY, LTD.
[CITATION]
QUISUMBING; May 2, 2006

NATURE
Petition for review on certiorari

FACTS
- Societe Francaise Des Colloides loaded a cargo of
textiles and auxiliary chemicals from France on board
a vessel owned by Franco-Belgian Services, Inc.
- The cargo was consigned to General Textile, Inc.,
in Manila and insured by respondent New India
Assurance Company, Ltd.
- While in Hongkong, the cargo was transferred to
M/V P. Aboitiz for transshipment to Manila.
- Before departing, the vessel was advised that it
was safe to travel to its destination, but while at sea,
the vessel received a report of a typhoon moving
within its path.
- To avoid the typhoon, the vessel changed its
course. However, it was still at the fringe of the
typhoon when its hull leaked.
- On October 31, 1980, the vessel sank, but the
captain and his crew were saved.
- On November 3, 1980, the captain of M/V P.
Aboitiz filed his Marine Protest, stating that the
wind force was at 10 to 15 knots at the time the ship
foundered and described the weather as moderate
breeze, small waves, becoming longer, fairly
frequent white horse
- Petitioner notified the consignee of the total loss of
the vessel and all of its cargoes.
- General Textile, lodged a claim with respondent for
the amount of its loss.
- Respondent paid General Textile and was
subrogated to the rights of the latter.
- Respondent hired a surveyor to investigate, and
the same concluded that the cause was the flooding
of the holds brought about by the vessels
questionable seaworthiness.
- Respondent filed a complaint for damages against
petitioner Aboitiz, Franco-Belgian Services and the
latters local agent, F.E. Zuellig, Inc. (Zuellig)
- On November 20, 1989, the trial court ruled in
favor of respondent and held petitioner Aboitiz liable
for the total value of the lost cargo plus legal interest
- The complaint with respect to Franco and Zuellig
was dismissed
- Petitioner elevated the case to the Court of
Appeals, which in turn, affirmed in toto the trial
courts decision.
- Petitioner moved for reconsideration but the same
was denied.
- Hence, this petition for review
Petitioners Claim
> Petitioner contends that respondents claim for
damages should only be against the insurance
proceeds and limited to its pro-rata share in view of
the doctrine of limited liability
Respondents Comments
> Respondent counters that the doctrine of real and
hypothecary nature of maritime law is not applicable
in the present case because petitioner was found to
have been negligent. Hence, according to
respondent, petitioner should be held liable for the
total value of the lost cargo

ISSUE
WON the limited liability doctrine applies in this case

HELD
NO
Ratio Where the shipowner fails to overcome the
presumption of negligence, the doctrine of limited
liability cannot be applied.

Reasoning
- From the nature of their business and for reasons
of public policy, common carriers are bound to
observe extraordinary diligence over the goods they
transport according to all the circumstances of each
case. In the event of loss, destruction or
deterioration of the insured goods, common carriers
are responsible, unless they can prove that the loss,
destruction or deterioration was brought about by
the causes specified in Article 1734
1
of the Civil
Code. In all other cases, common carriers are
presumed to have been at fault or to have acted
negligently, unless they prove that they observed
extraordinary diligence. Moreover, where the vessel
is found unseaworthy, the shipowner is also
presumed to be negligent since it is tasked with the
maintenance of its vessel. Though this duty can be

!
#$%& !'()& *+,,+- ./$$01$2 /$1 $123+-20451 6+$ %71 5+228 912%$:.%0+-8 +$ 91%1$0+$/%0+- +6 %71
;++928 :-5122 %71 2/,1 02 9:1 %+ /-< +6 %71 6+55+=0-; ./:212 +-5<>
?!@ A5++98 2%+$,8 1/$%7B:/C18 50;7%-0-;8 +$ +%71$ -/%:$/5 902/2%1$ +$ ./5/,0%<D
?E@ #.% +6 %71 3:450. 1-1,< 0- =/$8 =71%71$ 0-%1$-/%0+-/5 +$ .0F05D
?(@ #.% +6 +,0220+- +6 %71 270331$ +$ +=-1$ +6 %71 ;++92D
?)@ G71 .7/$/.%1$ +6 %71 ;++92 +$ 9161.%2 0- %71 3/.C0-; +$ 0- %71 .+-%/0-1$2D
?H@ I$91$ +$ /.% +6 .+,31%1-% 3:450. /:%7+$0%<&

delegated, still, the shipowner must exercise close
supervision over its men.
- In the present case, petitioner has the burden of
showing that it exercised extraordinary diligence in
the transport of the goods it had on board in order to
invoke the limited liability doctrine. Differently put,
to limit its liability to the amount of the insurance
proceeds, petitioner has the burden of proving that
the unseaworthiness of its vessel was not due to its
fault or negligence.
- Considering the evidence presented and the
circumstances obtaining in this case, we find that
petitioner failed to discharge this burden. Both the
trial and the appellate courts, in this case, found that
the sinking was not due to the typhoon but to its
unseaworthiness. Evidence on record showed that
the weather was moderate when the vessel sank.
These factual findings of the Court of Appeals,
affirming those of the trial court are not to be
disturbed on appeal, but must be accorded great
weight. These findings are conclusive not only on
the parties but on this Court as well.
Disposition Petition is denied for lack of merit.

PHILAMCARE HEALTH SYSTEMS, INC. V CA
(TRINOS)
379 SCRA 357
YNARES-SANTIAGO; March 18, 2002

NATURE
Petition for review of CA decision

FACTS
- Ernani TRINOS, deceased husband of respondent
Julita, applied for a health care coverage with
Philamcare Health Systems, Inc. In the standard
application form, he answered no to the question:
Have you or any of your family members ever
consulted or been treated for high blood pressure,
heart trouble, diabetes, cancer, liver disease, asthma
or peptic ulcer? (If Yes, give details).
- The application was approved for period of one
year; upon termination, it was extended for another
2 years. Amount of coverage was increased to a
maximum sum of P75T per disability.
- During this period, Ernani suffered a HEART
ATTACK and was confined at the Manila Medical
Center (MMC) for one month. While her husband was
in the hospital, Julita tried to claim the
hospitalization benefits.
- Petitioner treated the Health Care Agreement
(HCA) as void since there was a concealment
!"#$%&"'( *+,- /

regarding Ernanis medical history. Doctors at
the MMC allegedly discovered at the time of his
confinement, he was hypertensive, diabetic and
asthmatic. Julita then paid the hospitalization
expenses herself, amounting to about P76T.
- After her husband died, Julita instituted action for
damages against Philamcare and its Pres. After
trial, the lower court ruled in her favor and ordered
Philamcare to reimburse medical and hospital
coverage amounting to P76T plus interest, until fully
paid; pay moral damages of P10T; pay exemplary
damages of P10T; attys fees of P20T.
- CA affirmed the decision of the trial court but
deleted all awards for damages and absolved
petitioner Reverente.
Petitioners Claims
(1) Agreement grants living benefits such as
medical check-ups and hospitalization which a
member may immediately enjoy so long as he is
alive upon effectivity of the agreement until its
expiration.
(2) Only medical and hospitalization benefits are
given under the agreement without any
indemnification, unlike in an insurance contract
where the insured is indemnified for his loss.
(3) HCAs are only for a period of one year;
therefore, incontestability clause does not apply, as
it requires effectivity period of at least 2 yrs.
(4) It is not an insurance company, governed by
Insurance Commission, but a Health Maintenance
Organization under the authority of DOH.
(5) Trinos concealed a material fact in his
application.
(6) Julita was not the legal wife since at the time
of their marriage, the deceased was previously
married to another woman who was still alive.*

ISSUES
1. WON a health care agreement is an insurance
contract (If so, incontestability clause under the
Insurance Code is applicable)
2. WON the HCA can be invalidated on the basis of
alleged concealment

HELD
YES
Ratio Every person has an insurable interest in the
life and health of himself
2
. The health care

E
!"#$%&$ ()"*+ ,"*-./ 01- 1/ 2/-3*145" 2/6"*"-6 2/ 60" 527" 1/8 0"15609
?!@ +6 70,21568 +6 702 23+:21 /-9 +6 702 .7059$1-D
?E@ +6 /-< 31$2+- +- =7+, 71 9131-92 =7+55< +$ 0- 3/$% 6+$ 19:./%0+- +$ 2:33+$%8 +$ 0- =7+, 71
7/2 / 31.:-0/$< 0-%1$12%D
agreement was in the nature of non-life insurance,
which is primarily a contract of indemnity. Once the
member incurs hospital, medical or any other
expense arising from sickness, injury or other
stipulated contingent, the health care provider must
pay for the same to the extent agreed upon under
the contract.
Reasoning
- A contract of insurance
3
is an agreement whereby
one undertakes for a consideration to indemnify
another against loss, damage or liability arising from
an unknown or contingent event.
- An insurance contract exists where the following
elements concur:
(a) The insured has an insurable interest;
(b) The insured is subject to a risk of loss by the
happening of the peril;
(c) The insurer assumes the risk;
(d) Such assumption of risk is part of a general
scheme to distribute actual losses among a large
group of persons bearing a similar risk; and
(e) In consideration of the insurers promise, the
insured pays a premium.
2. NO
Ratio Where matters of opinion or judgment are
called for, answers made in good faith and without
intent to deceive will not avoid a policy even though
they are untrue; since in such case the insurer is not
justified in relying upon such statement, but is
obligated to make further inquiry.
Reasoning
- The fraudulent intent on the part of the insured
must be established to warrant rescission of the
insurance contract. The right to rescind should be
exercised previous to the commencement of an
action on the contract. No rescission was made.
Besides, the cancellation of health care agreements
as in insurance policies requires:
(a) Prior notice of cancellation to insured;
(b) Notice must be based on the occurrence after
effective date of the policy of one or more of the
grounds mentioned;
(c) Must be in writing, mailed or delivered to the
insured at the address shown in the policy;
(d) Must state the grounds relied upon provided in
Section 64 of the Insurance Code and upon request
of insured, to furnish facts on which cancellation is
based.

?(@ +6 /-< 31$2+- :-91$ / 51;/5 +450;/%0+- %+ 70, 6+$ %71 3/<,1-% +6 ,+-1<8 $1231.%0-; 3$+31$%< +$
21$F0.18 +6 =70.7 91/%7 +$ 055-122 ,0;7% 915/< +$ 3$1F1-% %71 31$6+$,/-.1D /-9
?)@ +6 /-< 31$2+- :3+- =7+21 5061 /-< 12%/%1 +$ 0-%1$12% F12%19 0- 70, 9131-92&
(
!"#62./ : ;%< .7 60" =/-3*1/#" >.8"
- These conditions have not been met. When the
terms of insurance contract contain limitations on
liability, courts should construe them in such a way
as to preclude insurer from non-compliance of
obligation. Being a contract of adhesion, terms of an
insurance contract are to be construed strictly
against the party which prepared it the insurer.
- Also, Philamcare had 12 months from the date of
issuance of the Agreement within which to contest
the membership of the patient if he had previous
ailment of asthma, and six months from the issuance
of the agreement if the patient was sick of diabetes
or hypertension.
* The health care agreement is in the nature of a
contract of indemnity. Hence, payment should be
made to the party who incurred the expenses. It is
clear that respondent paid all the hospital and
medical bills; thus, she is entitled to reimbursement.
Disposition Petition DENIED.

PINEDA V CA (INSULAR LIFE INSURANCE
COMPANY)
226 SCRA 755
DAVIDE; September 27, 1993

NATURE
Appeal by certiorari for review and set aside the
Decision of the public respondent Court of Appeals
and its Resolution denying the petitioners' motion for
reconsideration

FACTS
- In 1983, Prime Marine Services, Inc. (PMSI)
procured a group policy from Insular Life to provide
life insurance coverage to its sea-based employees
enrolled under the plan. During the effectivity of the
policy, 6 covered employees perished at sea. They
were survived by complainants-appellees, the
beneficiaries under the policy.
- complainants-appellees sought to claim death
benefits due them and approached Capt. Roberto
Nuval, President and GM of PMSI, then executed
special powers of attorney authorizing Capt. Nuval to
follow up, ask, demand, collect and receive for
their benefit indemnities of sums of money due
them
- Unknown to the complainants, PMSI filed with
Insular Life claims for and in behalf of them through
Capt. Nuval, even using the 5 special powers of
attorney that they executed as documents. Insular
Life then released 6 checks, payable to the order of
the complainant-appellees, to the treasurer of PMSI
!"#$%&"'( *+,- 0

(who happened to be Capt. Nuvals son-in-law).
Capt. Nuval then endorsed and deposited these
checks (which were for the complainants) in his bank
account.
- 3 years after, the complainants-appellees found out
that they were entitled, as beneficiaries, to life
insular benefits under a group policy with
respondent-appellant so they sought to recover
these benefits from Insular Life. Insular Life denied
the claim, saying that their liability to complainants
was already extinguished upon delivery to and
receipt by PMSI of the 6 checks issued in the
complainants names. Complainants filed case with
the Insurance Commission which decided in their
favor.
- Insurance Commission held that the special powers
of attorney executed by complainants in favor of the
complainants do not contain in unequivocal and clear
terms authority to Capt. Nuval to obtain, receive,
receipt from respondent company insurance
proceeds arising from the death of the seaman-
insured; also, that Insular Life did not convincingly
refuted the claim of Mrs. Alarcon that neither she nor
her husband executed a special power of authority in
favor of Capt. Nuval (and therefore, the company
should have not released the check to Capt. Nuval-
PMSI); and that it did not observe Sec 180(3), as
repealed by Art. 225 of the Family Code, when it
released the benefits due to the minor children of
Ayo and Lontok, when the said complainants did not
post a bond as required
- Insular Life appealed to the CA; CA modified the
decision of the Insurance Commission, eliminating
the award to the Lontoks and Ayo

ISSUES
1. WON Insular Life should be liable to the
complainants when they relied on the special powers
of attorney, which Capt. Nuval presented as
documents, when they released the checks to the
latter
2. WON Insular Life should be liable to the
complainants when they released the check in favor
of Ayo and LOntok, even if no bond was posted as
required

HELD
1. YES
Ratio Third persons deal with agents at their peril
and are bound to inquire as to the extent of the
power of the agent with whom they contract. The
person dealing with an agent must also act
with ordinary prudence and reasonable
diligence. Obviously, if he knows or has good
reason to believe that the agent is exceeding his
authority, he cannot claim protection. So if the
suggestions of probable limitations be of such a clear
and reasonable quality, or if the character assumed
by the agent is of such a suspicious or unreasonable
nature, or if the authority which he seeks to exercise
is of such an unusual or improbable character, as
would suffice to put an ordinarily prudent man upon
his guard, the party dealing with him may not shut
his eyes to the real state of the case, but should
either refuse to deal with the agent at all, or should
ascertain from the principal the true condition of
affairs.
Reasoning
- The execution by the principals of special powers of
attorney, which clearly appeared to be in prepared
forms and only had to be filled up with their names,
residences, dates of execution, dates of
acknowledgement and others, excludes any intent to
grant a general power of attorney or to constitute a
universal agency. Being special powers of attorney,
they must be strictly construed. Insular Life knew
that a power of attorney in favor of Capt. Nuval for
the collection and receipt of such proceeds was a
deviation from its practice with respect to group
policies (that the employer-policyholder is the agent
of the insurer).
- The employer acts as a functionary in the collection
and payment of premiums and in performing related
duties. Likewise falling within the ambit of
administration of a group policy is the disbursement
of insurance payments by the employer to the
employees. Most policies, such as the one in this
case, require an employee to pay a portion of the
premium, which the employer deducts from wages
while the remainder is paid by the employer. This is
known as a contributory plan as compared to a
non-contributory plan where the premiums are solely
paid by the employer.
- the labor of the employees is the true source
of the benefits, which are a form of additional
compensation to them.
- the employer is the agent of the insurer in
performing the duties of administering group
insurance policies. It cannot be said that the
employer acts entirely for its own benefit or for the
benefit of its employees in undertaking
administrative functions. While a reduced premium
may result if the employer relieves the insurer of
these tasks, and this, of course, is advantageous to
both the employer and the employees, the insurer
also enjoys significant advantages from the
arrangement. The reduction in the premium which
results from employer-administration permits the
insurer to realize a larger volume of sales, insurer to
realize a larger volume of sales, and at the same
time the insurer's own administrative costs are
markedly reduced.
- the employee has no knowledge of or control over
the employer's actions in handling the policy or its
administration. An agency relationship is based upon
consent by one person that another shall act in his
behalf and be subject to his control. It is clear from
the evidence regarding procedural techniques here
that the insurer-employer relationship meets this
agency test with regard to the administration of the
policy, whereas that between the employer and its
employees fails to reflect true agency. The insurer
directs the performance of the employer's
administrative acts, and if these duties are not
undertaken properly the insurer is in a position to
exercise more constricted control over the
employer's conduct.
- ON GROUP INSURANCE: Group insurance is
essentially a single insurance contract that
provides coverage for many individuals. In its
original and most common form, group insurance
provides life or health insurance coverage for
the employees of one employer. The coverage
terms for group insurance are usually stated in a
master agreement or policy that is issued by the
insurer to a representative of the group or to an
administrator of the insurance program, such as an
employer. Although the employer may be the titular
or named insured, the insurance is actually
related to the life and health of the employee.
Indeed, the employee is in the position of a real
party to the master policy, and even in a non-
contributory plan, the payment by the employer of
the entire premium is a part of the total
compensation paid for the services of the employee.
2. YES
Ratio Regardless of the value of the unemancipated
common child's property, the father and mother ipso
jure become the legal guardian of the child's
property. However, if the market value of the
property or the annual income of the child exceeds
P50,000,00, a bond has to be posted by the parents
concerned to guarantee the performance of the
obligations of a general guardian.
Reasoning
!"#$%&"'( *+,- 1

- Sec 180, Insurance Code: 'In the absence of a
judicial guardian, the father, or in the latter's
absence or incapacity, the mother of any minor, who
is an insured or a beneficiary under a contract of life,
health or accident insurance, may exercise, in behalf
of said minor, any right under the policy, without
necessity of court authority or the giving of a bond
where the interest of the minor in the particular act
involved does not exceed twenty thousand pesos "
- repealed by Art. 225, Family Code: "ART. 225.
The father and the mother shall jointly exercise legal
guardianship over the property of their
unemancipated common child without the necessity
of a court appointment. In case of disagreement, the
father's decision shall prevail, unless there is judicial
order to the contrary.
Where the market value of the property or the
annual income of the child exceeds P50,000, the
parent concerned shall be required to furnish a bond
in such amount as the court may determine, but not
less than ten per centum (10%) of the value of the
property or annual income, to guarantee the
performance of the obligations prescribed for general
guardians."
-"market value of the property or the annual
income of the child": the aggregate of the child's
property or annual income; if this exceeds
P50,000.00, a bond is required - There is no
evidence that the share of each of the minors in the
proceeds of the group policy in question is the
minor's only property. Without such evidence, it
would not be safe to conclude that, indeed, that is
his only property.
Disposition the instant petition is GRANTED. The
Decision of 10 October 1991 and the Resolution of
19 May 1992 of the public respondent in CA-G.R. SP
No. 22950 are SET ASIDE and the Decision of the
Insurance Commission in IC Case No. RD-058 is
REINSTATED. Costs against the private respondent.
SO ORDERED.

CEBU SHIPYARD ENGINEERING WORKS, INC. V
WILLIAM LINES, INC. and PRUDENTIAL
GUARANTEE and ASSURANCE COMPANY, INC.
[CITATION]
PURISIMA; May 5, 1999

NATURE
Petition for review on certiorari

FACTS
- Cebu Shipyard and Engineering Works, Inc.
(CSEW) is engaged in the business of dry-docking
and repairing of marine vessels while the Prudential
Guarantee and Assurance, Inc. (Prudential) is in the
non-life insurance business.
- William Lines, Inc. is in the shipping business. It
was the owner of M/V Manila City, a luxury
passenger-cargo vessel, which caught fire and sank
on Feb. 16, 1991. At the time of the unfortunate
occurrence sued upon, subject vessel was insured
with Prudential for P45M for hull and machinery. The
Hull Policy included an Additional Perils
(INCHMAREE) Clause covering loss of or damage to
the vessel through the negligence of, among others,
ship repairmen
- Petitioner CSEW was also insured by Prudential for
third party liability under a Shiprepairers Legal
Liability Insurance Policy. The policy was for P10
million only, under the limited liability clause, to wit:
- On Feb. 5, 1991, William Lines, Inc. brought its
vessel, M/V Manila City, to the Cebu Shipyard in
Lapulapu City for annual dry-docking and repair.
- On Feb. 6, 1991, an arrival conference was held
between representatives of William Lines, Inc. and
CSEW to discuss the work to be undertaken on the
M/V Manila City. The contracts, denominated as
Work Orders, were signed thereafter., with the
following stipulations:
10. The Contractor shall replace at its own work and
at its own cost any work or material which can be
shown to be defective and which is communicated in
writing within one (1) month of redelivery of the
vessel or if the vessel was not in the Contractors
Possession, the withdrawal of the Contractors
workmen, or at its option to pay a sum equal to the
cost of such replacement at its own works. These
conditions shall apply to any such replacements.
11. Save as provided in Clause 10, the Contractor
shall not be under any liability to the Customer
either in contract or for delict or quasi-delict or
otherwise except for negligence and such liability
shall itself be subject to the following overriding
limitations and exceptions, namely:
(a) The total liability of the Contractor to the
Customer (over and above the liability to replace
under Clause 10) or of any sub-contractor shall be
limited in respect of any defect or event (and a
series of accidents arising out of the same defect or
event shall constitute one defect or event) to the
sum of Pesos Philippine Currency One Million only.
x x x
20. The insurance on the vessel should be
maintained by the customer and/or owner of the
vessel during the period the contract is in effect.
- While the M/V Manila City was undergoing dry-
docking and repairs within the premises of CSEW,
the master, officers and crew of M/V Manila City
stayed in the vessel, using their cabins as living
quarters. Other employees hired by William Lines to
do repairs and maintenance work on the vessel were
also present during the dry-docking.
- On February 16, 1991, after subject vessel was
transferred to the docking quay, it caught fire and
sank, resulting to its eventual total loss.
- On February 21, 1991, William Lines, Inc. filed a
complaint for damages against CSEW, alleging that
the fire which broke out in M/V Manila City was
caused by CSEWs negligence and lack of care.
- On July 15, 1991 was filed an Amended Complaint
impleading Prudential as co-plaintiff, after the latter
had paid William Lines, Inc. the value of the hull and
machinery insurance on the M/V Manila City. As a
result of such payment Prudential was subrogated to
the claim of P45 million, representing the value of
the said insurance it paid.
On June 10, 1994, the trial court a quo came out
with a judgment against CSEW:
1. To pay unto plaintiff Prudential Guarantee and
Assurance, Inc., the subrogee, the amount of P45M,
with interest at the legal rate until full payment is
made; the amount of P56,715,000 representing loss
of income of M/V MANILA CITY, with interest at the
legal rate until full payment is made;
2. To pay unto plaintiff, William Lines, Inc. the
amount of P11M as payment, in addition to what it
received from the insurance company to fully cover
the injury or loss, in order to replace the M/V
MANILA CITY, with interest at the legal rate until full
payment is made; the sum of P927,039 for the loss
of fuel and lub oil on board the vessel when she was
completely gutted by fire at defendant, Cebu
Shipyards quay, with interest at the legal rate until
full payment is made; the sum of P3,054,677.95 as
payment for the spare parts and materials used in
the M/V MANILA CITY during dry-docking with
interest at the legal rate until full payment is made;
P500,000 in moral damages;the amount of P10Min
attorneys fees; and to pay the costs of this suit.
- On September 3, 1997, the Court of Appeals
affirmed the appealed decision of the trial court,
ordering CSEW to pay Prudential, the subrogee, the
sum of P45 Million, with interest at the legal rate
until full payment is made.
!"#$%&"'( *+,- 2

CSEWs version:
On Feb. 13, 1991, the CSEW completed the
drydocking of M/V Manila City at its grave dock. It
was then transferred to the docking quay of CSEW
where the remaining repair to be done was the
replating of the top of Water Ballast Tank No. 12
which was subcontracted by CSEW to JNB General
Services. Tank Top No. 12 was at the rear section of
the vessel, on level with the flooring of the crew
cabins located on the vessels second deck.
At around 7AM of Feb. 16, 1991, the JNB workers
trimmed and cleaned the tank top framing which
involved minor hotworks (welding/cutting works).
The said work was completed at about 10AM. The
JNB workers then proceeded to rig the steel plates,
after which they had their lunch break. The rigging
was resumed at 1PM
While in the process of rigging the second steel
plate, the JNB workers noticed smoke coming from
the passageway along the crew cabins. When one of
the workers, Mr. Casas, proceeded to the
passageway to ascertain the origin of the smoke, he
noticed that smoke was gathering on the ceiling of
the passageway but did not see any fire as the crew
cabins on either side of the passageway were locked.
He immediately sought out the proprietor of JNB,
Mr. Buenavista, and the Safety Officer of CSEW, Mr.
Aves, who sounded the fire alarm. CSEWs fire
brigade immediately responded as well as the other
fire fighting units in Metro Cebu. However, there
were no WLI representative, officer or crew to guide
the firemen inside the vessel.
- Despite the combined efforts of the firemen of the
Lapulapu City Fire Dept., Mandaue Fire Dept.,
Cordova Fire Dept. Emergency Rescue Unit
Foundation, and fire brigade of CSEW, the fire was
not controlled until 2AM of the following day.
- On the early morning of Feb. 17, 1991, gusty winds
rekindled the flames on the vessel and fire again
broke out. Then the huge amounts of water pumped
into the vessel, coupled with the strong current,
caused the vessel to tilt until it capsized and sank
- When M/V Manila City capsized, steel and angle
bars were noticed to have been newly welded along
the port side of the hull of the vessel, at the level of
the crew cabins. William Lines did not previously
apply for a permit to do hotworks on the said portion
of the ship as it should have done pursuant to its
work order with CSEW.
Prudentials version
> At around 7AM of Feb. 16, 1991, the Chief Mate of
M/V Manila City was inspecting the various works
being done by CSEW on the vessel, when he saw
that some workers of CSEW were cropping out steel
plates on Tank Top No. 12 using acetylene, oxygen
and welding torch. He also observed that the rubber
insulation wire coming out of the air-conditioning
unit was already burning, prompting him to scold the
workers.
> At 2:45 PM of the same day, witnesses saw smoke
coming from Tank No. 12. The vessels reeferman
reported such occurence to the Chief Mate who
immediately assembled the crew members to put
out the fire. When it was too hot for them to stay on
board and seeing that the fire cannot be controlled,
the vessels crew were forced to withdraw from
CSEWs docking quay.
- In the morning of Feb. 17, 1991, M/V Manila City
sank. As the vessel was insured with Prudential
Guarantee, William Lines filed a claim for
constructive total loss, and after a thorough
investigation of the surrounding circumstances of the
tragedy, Prudential found the said insurance claim to
be meritorious and issued a check in favor of
William Lines in the amount of P45 million pesos
representing the total value of M/V Manila Citys hull
and machinery insurance.

ISSUES
1. WON CSEW had management and supervisory
control of the m/v manila city at the time the fire
broke out
2. WON the doctrine of res ipsa loquitur applies
against the crew
3. WON CSEWS expert evidence is admissible or of
probative value
4. WON Prudential has the right of subrogation
against its own insured THE CONTRACTUAL 5. 5. 5.
5. WON the provisions limiting csews liability for
negligence to a maximum of p1 million are valid

HELD
1. YES
- The that factual findings by the CA are conclusive
on the parties and are not reviewable by this Court.
They are entitled to great weight and respect, even
finality, especially when, as in this case, the CA
affirmed the factual findings arrived at by the trial
court. When supported by sufficient evidence,
findings of fact by the CA affirming those of the trial
court, are not to be disturbed on appeal. The
rationale behind this doctrine is that review of the
findings of fact of the CA is not a function that the
Supreme Court normally undertakes.
- The CA and the Cebu RTC are agreed that the fire
which caused the total loss of subject M/V Manila
City was due to the negligence of the employees and
workers of CSEW. Both courts found that the M/V
Manila City was under the custody and control of
petitioner CSEW, when the ill-fated vessel caught
fire. The decisions of both the lower court and the CA
set forth clearly the evidence sustaining their finding
of actionable negligence on the part of CSEW. This
factual finding is accorded great weight and is
conclusive on the parties. The court discerns no basis
for disturbing such finding firmly anchored on
enough evidence.
- Furthermore, in petitions for review on certiorari,
only questions of law may be put into issue.
Questions of fact cannot be entertained. The finding
of negligence by the CA is a question which this
Court cannot look into as it would entail going into
factual matters on which the finding of negligence
was based. Such an approach cannot be allowed by
this Court in the absence of clear showing that the
case falls under any of the exceptions to the well-
established principle.
The finding by the trial court and the Court of
Appeals that M/V Manila City caught fire and sank by
reason of the negligence of the workers of CSEW,
when the said vessel was under the exclusive
custody and control of CSEW is accordingly upheld.
2. YES
- For the doctrine of res ipsa loquitur to apply to a
given situation, the following conditions must
concur: (1) the accident was of a kind which does
not ordinarily occur unless someone is negligent; and
(2) that the instrumentality or agency which caused
the injury was under the exclusive control of the
person charged with negligence.
The facts and evidence on record reveal the
concurrence of said conditions in the case under
scrutiny. First, the fire that occurred and consumed
M/V Manila City would not have happened in the
ordinary course of things if reasonable care and
diligence had been exercised. In other words, some
negligence must have occurred. Second, the agency
charged with negligence, as found by the trial court
and the CA and as shown by the records, is the
herein petitioner, CSEW, which had control over
subject vessel when it was docked for annual repairs.
So also, as found by the RTC, other responsible
causes, including the conduct of the plaintiff, and
third persons, are sufficiently eliminated by the
evidence.
!"#$%&"'( *+,- 3

What is more, in the present case the trial court
found direct evidence to prove that the workers
and/or employees of CSEW were remiss in their duty
of exercising due diligence in the care of subject
vessel. The direct evidence substantiates the
conclusion that CSEW was really negligent. Thus,
even without applying the doctrine of res ipsa
loquitur, in light of the direct evidence on record, the
ineluctable conclusion is that CSEW was negligent
and consequently liable for damages to the
respondent, William Lines, Inc.
3. NO
- Petitioner maintains that the CA erred in
disregarding the testimonies of the fire experts,
Messrs. David Grey and Gregory Michael Southeard,
who testified on the probable origin of the fire in M/V
Manila City. Petitioner avers that since the said fire
experts were one in their opinion that the fire did not
originate in the area of Tank Top No. 12 where the
JNB workers were doing hotworks but on the crew
accommodation cabins on the portside No. 2 deck,
the RTC and the CA should have given weight to
such finding based on the testimonies of fire experts;
petitioner argues.
But courts are not bound by the testimonies of
expert witnesses. Although they may have probative
value, reception in evidence of expert testimonies is
within the discretion of the court, under Section 49,
Rule 130 of the Revised Rules of Court. It is never
mandatory for judges to give substantial weight to
expert testimonies. If from the facts and evidence on
record, a conclusion is readily ascertainable, there is
no need for the judge to resort to expert opinion
evidence. In the case under consideration, the
testimonies of the fire experts were not the only
available evidence on the probable cause and origin
of the fire. There were witnesses who were actually
on board the vessel when the fire occurred. Between
the testimonies of the fire experts who merely based
their findings and opinions on interviews and the
testimonies of those present during the fire, the
latter are of more probative value.



4. YES
- Petitioner contends that Prudential is not entitled to
be subrogated to the rights of William Lines, Inc.,
theorizing that (1) the fire which gutted M/V Manila
City was an excluded risk and (2) it is a co-assured
under the Marine Hull Insurance Policy.
- It is petitioners submission that the loss of M/V
Manila City or damage thereto is expressly excluded
from the coverage of the insurance because the
same resulted from want of due diligence by the
Assured, Owners or Managers which is not included
in the risks insured against. Again, this theory of
petitioner is bereft of any factual or legal basis. It
proceeds from a wrong premise that the fire which
gutted subject vessel was caused by the negligence
of the employees of William Lines, Inc. To repeat,
the issue of who between the parties was negligent
has already been resolved against CSEW. Upon proof
of payment by Prudential to William Lines, Inc., the
former was subrogated to the right of the latter to
indemnification from CSEW. As aptly ruled by the
Court of Appeals, the law on the matter is succinct
and clear, to wit:
- Art. 2207. If the plaintiffs property has been
insured, and he has received indemnity from the
insurance company for the injury or loss arising out
of the wrong or breach of contract complained of,
the insurance company shall be subrogated to the
rights of the insured against the wrongdoer or the
person who has violated the contract. If the amount
paid by the insurance company does not fully cover
the injury or loss, the aggrieved party shall be
entitled to recover the deficiency from the person
causing the loss or injury.
- Thus, when Prudential, after due verification of the
merit and validity of the insurance claim of William
Lines, Inc., paid the latter the total amount covered
by its insurance policy, it was subrogated to the right
of the latter to recover the insured loss from the
liable party, CSEW.
- Petitioner theorizes further that there can be no
right of subrogation as it is deemed a co-assured
under the subject insurance policy. To buttress its
stance that it is a co-assured, petitioner placed
reliance on Clause 20 of the Work Order which
states:
20. The insurance on the vessel should be
maintained by the customer and/or owner of the
vessel during the period the contract is in effect.
- According to petitioner, under the aforecited
clause, William Lines, Inc., agreed to assume the risk
of loss of the vessel while under drydock or repair
and to such extent, it is benefited and effectively
constituted as a co-assured under the policy.
- This theory of petitioner is devoid of sustainable
merit. Clause 20 of the Work Order in question is
clear in the sense that it requires William Lines to
maintain insurance on the vessel during the period of
dry-docking or repair. Concededly, such a
stipulation works to the benefit of CSEW as the
shiprepairer. However, the fact that CSEW benefits
from the said stipulation does not automatically
make it as a co-assured of William Lines. The
intention of the parties to make each other a co-
assured under an insurance policy is to be gleaned
principally from the insurance contract or policy itself
and not from any other contract or agreement
because the insurance policy denominates the
assured and the beneficiaries of the insurance. The
hull and machinery insurance procured by William
Lines, Inc. from Prudential named only William
Lines, Inc. as the assured. There was no
manifestation of any intention of William Lines, Inc.
to constitute CSEW as a co-assured under subject
policy. It is axiomatic that when the terms of a
contract are clear its stipulations control.
i]
Thus,
when the insurance policy involved named only
William Lines, Inc. as the assured thereunder, the
claim of CSEW that it is a co-assured is unfounded.
- Then too, in the Additional Perils Clause of the
same Marine Insurance Policy, it is provided that:
Subject to the conditions of this Policy, this insurance
also covers loss of or damage to vessel directly
caused by the following:
xxx
Negligence of Charterers and/or Repairers, provided
such Charterers and/or Repairers are not an
Assured hereunder.
- As correctly pointed out by respondent Prudential,
if CSEW were deemed a co-assured under the policy,
it would nullify any claim of William Lines, Inc. from
Prudential for any loss or damage caused by the
negligence of CSEW. Certainly, no shipowner would
agree to make a shiprepairer a co-assured under
such insurance policy; otherwise, any claim for loss
or damage under the policy would be invalidated.
Such result could not have been intended by William
Lines, Inc.
5. NO
- Although in this jurisdiction, contracts of adhesion
have been consistently upheld as valid per se; as
binding as an ordinary contract, the Court recognizes
instances when reliance on such contracts cannot be
favored especially where the facts and circumstances
warrant that subject stipulations be disregarded.
Thus, in ruling on the validity and applicability of the
stipulation limiting the liability of CSEW for
negligence to P1M only, the facts and circumstances
vis-a-vis the nature of the provision sought to be
!"#$%&"'( *+,- 4

enforced should be considered, bearing in mind the
principles of equity and fair play.
- It is worthy to note that M/V Manila City was
insured with Prudential for P45M. To determine the
validity and sustainability of the claim of William
Lines, Inc., for a total loss, Prudential conducted its
own inquiry. Upon thorough investigation by its hull
surveyor, M/V Manila City was found to be beyond
economical salvage and repair. The evaluation of the
average adjuster also reported a constructive total
loss. The said claim of William Lines, Inc., was then
found to be valid and compensable such that
Prudential paid the latter the total value of its
insurance claim. Furthermore, it was ascertained
that the replacement cost of the vessel (the price of
a vessel similar to M/V Manila City), amounts to
P55M.
- Considering the aforestated circumstances, let
alone the fact that negligence on the part of
petitioner has been sufficiently proven, it would
indeed be unfair and inequitable to limit the liability
of petitioner to One Million Pesos only. As aptly held
by the trial court, it is rather unconscionable if not
overstrained. To allow CSEW to limit its liability to
P1M notwithstanding the fact that the total loss
suffered by the assured and paid for by Prudential
amounted to P45M would sanction the exercise of a
degree of diligence short of what is ordinarily
required because, then, it would not be difficult for
petitioner to escape liability by the simple expedient
of paying an amount very much lower than the
actual damage or loss suffered by William Lines, Inc.
Disposition Petition is DENIED. Resolution of the
CA is AFFIRMED.

NEW LIFE ENTERPRISES V CA
207 SCRA 669
REGALADO; March 31, 1992

NATURE
Appeal by certiorari

FACTS
- The antecedents of this case show that Julian Sy
and Jose Sy Bang have formed a business
partnership in the City of Lucena. Under the business
name of New Life Enterprises, the partnership
engaged in the sale of construction materials at its
place of business, a two storey building situated at
Iyam, Lucena City. The facts show that Julian Sy
insured the stocks in trade of New Life Enterprises
with Western Guaranty Corporation, Reliance Surety
and Insurance Co. Inc., and Equitable Insurance
Corporation.
- On May 15, 1981, Western Guaranty Corporation
issued Fire Insurance Policy No. 37201 in the amount
of P350,000.00. This policy was renewed on May 13,
1982.
- On July 30, 1981, Reliance Surety and Insurance
Co., Inc. issued Fire Insurance Policy No. 69135 in
the amount of P300,000.00 (Renewed under
Renewal Certificate No. 41997). An additional
insurance was issued by the same company on
November 12, 1981 under Fire Insurance Policy No.
71547 in the amount of P700,000.00.
- On February 8, 1982, Equitable Insurance
Corporation issued Fire Insurance Policy No. 39328
in the amount of P200,000.00.
- Thus when the building occupied by the New Life
Enterprises was gutted by fire at about 2:00 o'clock
in the morning of October 19, 1982, the stocks in
trade inside said building were insured against fire in
the total amount of P1,550,000.00. According to the
certification issued by the Headquarters, Philippine
Constabulary/Integrated National Police, Camp
Crame, the cause of fire was electrical in nature.
According to the plaintiffs, the building and the
stocks inside were burned. After the fire, Julian Sy
went to the agent of Reliance Insurance whom he
asked to accompany him to the office of the
company so that he can file his claim. He averred
that in support of his claim, he submitted the fire
clearance, the insurance policies and inventory of
stocks.
He further testified that the three insurance
companies are sister companies, and as a matter of
fact when he was following-up his claim with
Equitable Insurance, the Claims Manager told him to
go first to Reliance Insurance and if said company
agrees to pay, they would also pay. The same
treatment was given him by the other insurance
companies. Ultimately, the three insurance
companies denied plaintiffs' claim for payment.
Respondents comments
> Western Guaranty Corporation through Claims
Manager Bernard S. Razon told the plaintiff that his
claim 'is denied for breach of policy conditions.'
Reliance Insurance purveyed the same message as
well as Equitable Insurance Corporation.
- The said policy in question follows:
"The insured shall give notice to the Company of any
insurance or insurances already effected, or which
may subsequently be effected, covering any of the
property or properties consisting of stocks in trade,
goods in process and/or inventories only hereby
insured, and unless such notice be given and the
particulars of such insurance or insurances be stated
therein or endorsed on this policy pursuant to
Section 50 of the Insurance Code, by or on behalf of
the Company before the occurrence of any loss or
damage, all benefits under this policy shall be
deemed forfeited, provided however, that this
condition shall not apply when the total insurance or
insurances in force at the time of loss or damage is
not more than P200,000.00."
Petitioners comments
> Petitioners contend that they are not to be blamed
for the omissions, alleging that insurance agent Leon
Alvarez (for Western) and Yap Kam Chuan (for
Reliance and Equitable) knew about the existence of
the additional insurance coverage and that they were
not informed about the requirement that such other
or additional insurance should be stated in the
policy, as they have not even read said policies.

ISSUE
WON New Life Enterprises claim for payment be
denied

HELD
YES
Ratio Furthermore, when the words and language of
documents are clear and plain or readily
understandable by an ordinary reader thereof, there
is absolutely no room for interpretation or
construction anymore. Courts are not allowed to
make contracts for the parties; rather, they will
intervene only when the terms of the policy are
ambiguous, equivocal, or uncertain. The parties
must abide by the terms of the contract because
such terms constitute the measure of the insurer's
liability and compliance therewith is a condition
precedent to the insured's right of recovery from the
insurer.
- While it is a cardinal principle of insurance law that
a policy or contract of insurance is to be construed
liberally in favor of the insured and strictly against
the insurer company, yet contracts of insurance, like
other contracts, are to be construed according to the
sense and meaning of the terms which the parties
themselves have used. If such terms are clear and
unambiguous, they must be taken and understood in
their plain, ordinary and popular sense. Moreover,
obligations arising from contracts have the force of
law between the contracting parties and should be
complied with in good faith.
!"#$%&"'( *+,- 5

Reasoning
a. The terms of the contract are clear and
unambiguous. The insured is specifically required to
disclose to the insurer any other insurance and its
particulars which he may have effected on the same
subject matter. The knowledge of such insurance by
the insurer's agents, even assuming the acquisition
thereof by the former, is not the "notice" that would
stop the insurers from denying the claim. Besides,
the so-called theory of imputed knowledge, that is,
knowledge of the agent is knowledge of the principal,
aside from being of dubious applicability here has
likewise been roundly refuted by respondent court
whose factual findings we find acceptable.
b. Petitioners should be aware of the fact that a
party is not relieved of the duty to exercise the
ordinary care and prudence that would be exacted in
relation to other contracts. The conformity of the
insured to the terms of the policy is implied from his
failure to express any disagreement with what is
provided for.

FIRST QUEZON CITY INSURANCE CO. v. CA (DE
DIOS MARIKINA TRANSPORT CO)
218 SCRA 526
GRINO-AQUINO; February 28, 1993

NATURE
PETITION for review of the decision of the Court of
Appeals. FQCIC seeks to limit to P12000, the amount
specified in the insurance contract, its liability to
indemnify the respomdemt DMTC, for the damages
suffered by a passenger, who accidentally fell off the
bug.

FACTS
- After sending off certain seamen at the departure
area of MIA, Jose V. del Rosario proceeded to the
public utility bus stop. While at the bus stop, the
plaintiff saw a DMTC bus. While moving at a crawling
pace, it was taking several passengers, all of whom
managed to board the bus while it was already at the
bus stop; plaintiff was the last one to board the bus.
While the plaintiff was still on the bus with his hand
on the bus door, the slowly moving bus sped forward
at a high speed, as a result of which, the plaintiff lost
balance and fell from the bus. As plaintiff clung
instinctively to the handle bar, he was dragged by
the bus along the asphalted road. The bus driver, Gil
Agpalo, abruptly stopped the bus. Then fled from the
scene, leaving the bus and the injured plaintiff
behind.
- The plaintiff was brought to the Manila Sanitarium
and Hospital where the doctors performed 2 major
surgical operations on plaintiffs right leg.
- Plaintiff was confined at the hospital for (40) days,
from June 10, 1984 to August 26, 1984. Medical
expenses totaled the amount of P69,444.41.
Plaintiffs medical expenses were advanced by his
employer Maglines but he was required to reimburse
Maglines on a staggered basis by way of salary
deductions. After his release from the hospital, he
returned to the hospital for further treatment and
checkup. The injuries had left plaintiff with a huge
scar on his right leg. Also, the plaintiff incurred lost
earning by way of unearned salaries amounting to
P7,500.00 due to said physical injuries and the
consequent hospital confinement.
- Plaintiff filed on June 26, 1985 the complaint
against DMTC and its driver. Agpalo was later
dropped as a party defendant because he could not
be served with summons. Upon filing its answer,
defendant DMTC filed a thirdparty complaint against
First Quezon City Insurance Co., Inc. September 17,
1985, third-party defendant filed its answer to the
third-party complaint.
- TC held DMTC complaint dismissed for lack of merit
and as regards the third-party complaint First
Quezon City Insurance Co., Inc. was to indemnify
third-party plaintiff DMTC in the sum of P12,000.00
with interest. There being no satisfactory warrant the
court dismissed the rest of the claims in the
complaint and third-party complaint.
- The bus company appealed to the CA, which
modified the dispositive as regards the third-party
complaint, that the third-party defendant First
Quezon City Insurance Co., Inc. be ordered to
indemnify third-party plaintiff DMTC the SUM of
P50,000.00 with legal interest. Insurance company
filed a MFR which was denied.
Hence, this petition for review, assailing the
appellate courts' interpretation of the provision of
the insurance contract on the limit of the insurer's
liability.

ISSUE
WON the CA erred in the interpretation of the
insurance contract on the limit of the insurers
liability

HELD
YES
- The insurance policy clearly placed the maximum
limit of the petitioner's liability for damages arising
from death or bodily injury at P12,000.00 per
passenger and its maximum liability per accident at
(P50,000.00. Since only one passenger was injured
in the accident, the insurer's liability for the damages
suffered by said passenger is pegged to the amount
of P12,000.00 only.
- The limit of P50,000.00 per accident means that
the insurer's maximum liability for any single
accident will not exceed P50,000.00 regardless of the
number of passengers killed or injured therein.
The bus company may not recover from the
insurance company more than P12,000.00 per
passenger killed or injured, or (P50,000.00) per
accident even if under the judgment of the court, the
erring bus operator will have to pay more than
P12,000.00 to each injured passenger. The trial
court's interpretation of the insurance contract was
the correct interpretation.
Disposition petition for review is GRANTED. The
decision promulgated by the CA, ordering the third
party defendent, First Quezon City Insurance Co.,
Inc. to indemnify theI private respondent, (DMTC),
the sum of P50,000.00 for the damages of the
passenger, Jose V. Del Rosario, is hereby modified
by reducing the award to 12,000.00 only. Costs
against the private respondent De Dios Marikina
Transportation Co., Inc.

TY V FIRST NATIONAL SURETY
1 SCRA 1324
LABRADOR; April 29, 1961

FACTS
- At different times within a period of two months
prior to 24 December 1953, Diosdado C. Ty,
employed as operator mechanic foreman in the
Broadway Cotton Factory insured himself in 18 local
insurance companies, among which being the 8
above-named defendants, which issued to him
personal accident policies. Plaintiffs beneficiary was
his employer, Broadway Cotton Factory, which paid
the insurance premiums. On 24 December 1953, a
fire broke out which totally destroyed the Broadway
Cotton Factory. Fighting his way out of the factory,
plaintiff was injured on the left hand by a heavy
object. He was brought to the Manila Central
University hospital, and after receiving first-aid, he
went to the National Orthopedic Hospital for
treatment of his injuries (fractures in index, middle,
fourth, and fifth fingers of left hand). From 26
December 1953 to 8 February 1954, he underwent
medical treatment in the hospital. The above-
!"#$%&"'( *+,- 6

described physical injuries have caused temporary
total disability of plaintiffs left hand. Plaintiff filed
the corresponding notice of accident and notice of
claim with all of the above-named defendants to
recover indemnity. Defendants rejected plaintiffs
claim for indemnity for the reason that there being
no severance of amputation of the left hand, the
disability suffered by him was not covered by his
policy.
- Plaintiff sued the defendants in the Municipality
Court of this City, which dismissed his complaints.
Thereafter, the plaintiff appealed to the Court of First
Instance Manila, presided by Judge Gregorio S.
Narvasa, which absolved the defendants from the
complaints. Hence, the appeal.

ISSUE
WON Diosdado Ty is entitled to indemnity under the
insurance policy for the disability of his left hand

HELD
- The agreement contained in the insurance policies
is the law between the parties. As the terms of the
policies are clear, express and specific that only
amputation of the left hand should be considered as
a loss thereof, an interpretation that would include
the mere fracture or other temporary disability not
covered by the policies would certainly be
unwarranted. In the case at bar, due to the clarity of
the stipulation, distinction between temporary
disability and total disability need not be made in
relation to ones occupation means that the condition
of the insurance is such that common prudence
requires him to desist from transacting his business
or renders him incapable of working. While the Court
sympathizes with the plaintiff or his employer, for
whose benefit the policies were issued, it can not go
beyond the clear and express conditions of the
insurance policies, all of which define partial
disability as loss of either hand by a amputation
through the bones of the wrist. There was no such
amputation in the case at bar.
- The Supreme Court affirmed the appealed decision,
with costs against the plaintiff-appellant.

MISAMIS LUMBER V CAPITAL INSURANCE
17 SCRA 288
REYES; May 20, 1966

NATURE
Direct appeal on a point of law from the judgment of
the Court of First Instance of Manila

FACTS
- Misamis Lumber Corporation, under its former
name, Lanao Timber Mills, Inc., insured its Ford
Falcon motor car for the amount of P14,000 with
Capital Insurance & Surety Company, Inc. The
pertinent provisions of the policy provided, as
follows:
1. The Company will subject to the Limits of
Liability indemnify the Insured against loss or
damage to the Motor Vehicle and its accessories
and spare parts whilst thereon.
2. (a) by accidental collision or overturning or
collision or overturning consequent when
mechanical breakdown or consequent upon wear
and tear.
3. At its option, the Company may pay in cash the
amount of the loss or damage or may repair,
reinstate or replace the Motor Vehicle or any part
thereof or its accessories or spare parts. The
liability of the Company shall not exceed the value
of the parts lost or damaged and the reasonable
cost of fitting such parts or the value of the Motor
Vehicle at the time of the loss or damage
whichever is the loss. The Insured's estimate of
value stated in the schedule shall be the maximum
amount payable by the Company in respect of any
claim for loss or damage.
4. The Insured may authorize the repair of the
Motor Vehicle necessitated by damage for which
the Company may be liable under this policy
provided that:
(a) the estimated cost of such repair does not
exceed the authorized Repair Limit.
(b) a detailed estimate of the cost is forwarded
to the Company without delay and providing
also that the authorized repair limit is P150.00.
- One night, the insured car, while traveling along in
Aurora Boulevard, passed over a water hole which
the driver did not see because an oncoming car did
not dim its light. The crankcase and flywheel housing
of the car broke when it hit a hollow block lying
alongside the water hole. The car was towed and
repaired by Morosi Motors at a total cost of P302.27.
- When the repairs on the car had already been
made, Misamis made a report of the accident to
Capital Insurance.
- Since Capital refused to pay for the total cost of to
wage and repairs, suit was filed in the municipal
court originally.
- The defendant-appellant admits liability in the
amount of P150, but not for any excess thereof. The
lower court did not exonerate the said appellant for
the excess because the company's absolution would
render the insurance contract one-sided and that the
said insurer had not shown that the cost of repairs in
the sum of P302.27 is unreasonable, excessive or
padded, nor had it shown that it could have
undertaken the repairs itself at less expense.

ISSUE
WON Capital Insurance can be made to pay more
than P150

HELD
NO
- The insurance policy stipulated in paragraph 4 that
if the insured authorizes the repair the liability of the
insurer, per its sub-paragraph (a), is limited to
P150.00. The literal meaning of this stipulation must
control, it being the actual contract, expressly and
plainly provided for in the policy.
- Recourse to legal hermeneutics is not called for
because paragraph 4 of the policy is clear and
specific and leaves no room for interpretation.
- The option to undertake the repairs is accorded to
the insurance company per paragraph 2. The said
company was deprived of the option because the
insured took it upon itself to have the repairs made,
and only notified the insurer when the repairs were
done. As a consequence, paragraph 4, which limits
the company's liability to P150.00, applies.
- The insurance contract may be rather onerous
("one-sided", as the lower court put it), but that in
itself does not justify the abrogation of its express
terms, terms which the insured accepted or adhered
to and which is the law between the contracting
parties.
- To require the insurer to prove that the cost of the
repairs ordered by the insured is unreasonable, when
the insurer was not given an opportunity to inspect
and assess the damage before the repairs were
made, is contrary to elementary justice and equity.

SUN INSURANCE OFFICE LTD. V CA (TAN)
195 SCRA 193
PARAS; March 13, 1991

NATURE
Petition for certiorari to review the decision of the CA

FACTS
- Private respondent Emilio Tan took from the
petitioner a Peso 300,000 property insurance policy
!"#$%&"'( *+,- .7

to cover his interest in the electrical insurance store
of his brother housed in a building in Iloilo City on
August 15, 1983. Four days after the issuance of the
policy, the building including the insured store
burned.
- On August 20, 1983, Tan filed his claim for fire
loss. Sun Insurance, on February 29, 1984, wrote
the private respondent denying the claim. On April 3,
1984, private respondent wrote another letter to the
insurance company requesting reconsideration of the
denial. Tans lawyer wrote another letter to the
insurance company inquiring about the April 3 letter
which sought for a reconsideration of the denial. In
its reply to the lawyers letter, Sun Insurance
reiterated its denial of the claim and enclosed therein
copies of the two previous denials dated February
29, 1984 and May 17, 1985.
- On November 20, 1985, Tan filed a civil case with
the RTC. Petition filed a motion to dismiss on the
alleged ground that the action has already prescribed
based on Condition 27 of the Insurance Policy which
stated that the window to file the appropriate action
with either the Insurance Commission or in any court
of competent jurisdiction is twelve months from the
rejection of the claim. RTC denied the motion and
the subsequent motion for reconsideration. The CA
likewise denied the petition of Sun Insurance.

ISSUE
1. WON the court the filing of a motion for
reconsideration interrupts the 12 months prescription
period to contest the denial of the insurance claim
2. WON the rejection of the claim shall be deemed
final only if it contains words to the effect that the
denial is final


HELD
1. NO
- The SC held that Condition 27 of the Insurance
policy is very clear and free from any doubt or
ambiguity. It has to be taken in its plain, ordinary,
and popular sense. The rejection letter of February
29, 1984 was clear and plain. The Court noted that
the one year period is likewise in accord with Section
23 of the Insurance Code which states that any
condition which limits the time for commencing an
action to a period of less than one year when the
cause of action accrues is void. The right of action,
according to the SC, accrues at the time that the
claim is rejected at the first instance. A request for
reconsideration of the denial cannot suspend the
running of the prescriptive period. The Court noted
that the rationale for the one year period is to ensure
that the evidence as to the origin and cause of the
destruction have not yet disappeared.
2. NO
- The Court clarified its ruling in Eagle Star Insurance
Co. vs Chia Yu where it ruled that the cause of
action in an insurance contract does not accrue until
the Insureds claim is finally rejected by the Insurer
by stating the use of the word finally cannot be
construed to mean the rejection of a petition for
reconsideration. What the court referred to in effect
is the rejection in the first instance as claimed by
Sun Insurance
Disposition The decision of the CA is reversed and
set aside. The case is dismissed

FORTUNE INSURANCE AND SURETY CO. INC.V
CA (PRODUCERS BANK OF THE PHILIPPINES)
244 SCRA 308
DAVIDE; May 23, 1995

NATURE
Petition for Review on certiorari of CA decision

FACTS
- Producers Bank of the Philippines filed a complaint
against Fortune Insurance and Surety Co., Inc. for
recovery of P725,000.00 under the policy issued by
Fortune. The sum was allegedly lost on June 29,
1987 during a robbery of Producer's armored vehicle
while it was in transit to transfer the money from its
Pasay City Branch to its head office in Makati under
the custody of its teller, Maribeth Alampay. The
armored car was driven by Benjamin Magalong Y de
Vera, escorted by Security Guard Saturnino Atiga Y
Rosete. Driver Magalong was assigned by PRC
Management Systems.
- After an investigation by the Pasay police, driver
Magalong and guard Atiga were charged, together
with Batigue , Aquino and John Doe, with violation of
P.D. 532 (Anti-Highway Robbery Law)
- Demands were made by the Producers upon the
Fortune to pay the amount of the loss of
P725,000.00 but the latter refused to pay as the loss
is excluded from the coverage of the insurance policy
specifically under "General Exceptions"
> The company shall not be liable under this
policy in respect of x x x (b) any loss caused by
any dishonest, fraudulent or criminal act of the
insured or any officer, employee, partner,
director, trustee or authorized representative of
the Insured whether acting alone or in
conjunction with others.
- Fortune opposes the contention of Producers that
Atiga and Magalong are not its "officer, employee, x
x x trustee or authorized representative x x x at the
time of the robbery
- Trial Court
> On being EMPLOYEES
Magalong and Atiga were not employees or
representatives of Producers as their services as
armored car driver and as security guard having
been merely offered by PRC Management and by
Unicorn Security and which latter firms assigned
them to plaintiff. The wages and salaries of both
Magalong and Atiga are presumably paid by their
respective firms, which alone wields the power to
dismiss them
> On being AUTHORIZED REPRESENTATIVE
They were merely an assigned armored car driver
and security guard for the money transfer. It was
teller Maribeth Alampay who had "custody" of the
P725,000.00 cash being transferred along a specified
money route
- Court of Appeals
> affirmed in toto
> A policy or contract of insurance is to be construed
liberally in favor of the insured and strictly against
the insurance company (New Life Enterprises vs.
Court of Appeals; Sun Insurance Office, Ltd. vs.
Court of Appeals). Contracts of insurance, like other
contracts, are to be construed according to the sense
and meaning of the terms which the parties
themselves have used. If such terms are clear and
unambiguous, they must be taken and understood in
their plain, ordinary and popular sense (New Life
Enterprises Case; Sun Insurance Office).
> The language used by Fortune in the policy is
plain, ordinary and simple. No other interpretation is
necessary. The word "employee" should be taken to
mean in the ordinary sense. The Labor Code is a
special law specifically dealing with/and specifically
designed to protect labor and therefore its definition
as to employer-employee relationships insofar as the
application/enforcement of said Code is concerned
must necessarily be inapplicable to an insurance
contract. Had it intended to apply the Labor Code in
defining what the word "employee" refers to, it
must/ should have so stated expressly in the
insurance policy. Said driver and security guard
cannot be considered as employees of Producers
bank because it has no power to hire or to dismiss
said driver and security guard under the contracts
!"#$%&"'( *+,- ..

except only to ask for their replacements from the
contractors.
- Fortunes Contention
> when Producers commissioned a guard and a
driver to transfer its funds from one branch to
another, they effectively and necessarily became its
authorized representatives in the care and custody of
the money. Assuming that they could not be
considered authorized representatives, they were,
nevertheless, employees of Producers. It asserts that
the existence of an employer-employee relationship
"is determined by law and being such, it cannot be
the subject of agreement." Thus, if there was in
reality an employer-employee relationship between
Producers, on the one hand, and Magalong and
Atiga, on the other, the provisions in the contracts of
Producers with PRC Management System for
Magalong and with Unicorn Security Services for
Atiga which state that Producers is not their
employer and that it is absolved from any liability as
an employer, would not obliterate the relationship.
> an employer-employee relationship depends upon
four standards:
(1) the manner of selection and engagement of the
putative employee
(2) the mode of payment of wages
(3) the presence or absence of a power to dismiss
and
(4) the presence and absence of a power to control
the putative employee's conduct.
> Of the four, the right-of-control test has been held
to be the decisive factor. It asserts that the power of
control over Magalong and Atiga was vested in and
exercised by Producers. Fortune further insists that
PRC Management System and Unicorn Security
Services are but "labor-only" contractors under
Article 106 of the Labor Code which provides:
Art. 106. Contractor or subcontractor. - There is
"labor-only" contracting where the person
supplying workers to an employer does not have
substantial capital or investment in the form of
tools, equipment, machineries, work premises,
among others, and the workers recruited and
placed by such persons are performing activities
which are directly related to the principal business
of such employer. In such cases, the person or
intermediary shall be considered merely as an
agent of the employer who shall be responsible to
the workers in the same manner and extent as if
the latter were directly employed by him.
> International Timber Corp. vs. NLRC - a "labor-
only" contractor is equivalent to a finding that there
is an employer-employee relationship between the
owner of the project and the employee of the "labor-
only" contractor
- Producers Contention
> Magalong and Atiga were not its employees since
it had nothing to do with their selection and
engagement, the payment of their wages, their
dismissal, and the control of their conduct.
> International Timber Corp. is not applicable to all
cases but only when it becomes necessary to prevent
any violation or circumvention of the Labor Code, a
social legislation whose provisions may set aside
contracts entered into by parties in order to give
protection to the working man.
> American President Lines vs. Clave should be
applied which stated
In determining the existence of employer-
employee relationship, the following elements are
generally considered, namely: (1) the selection
and engagement of the employee; (2) the
payment of wages; (3) the power of dismissal;
and (4) the power to control the employee's
conduct.
- Since under Producers' contract with PRC
Management Systems it is the latter which assigned
Magalong as the driver of Producers' armored car
and was responsible for his faithful discharge of his
duties and responsibilities, and since Producers paid
the monthly compensation of P1,400.00 per driver to
PRC Management Systems and not to Magalong, it is
clear that Magalong was not Producers' employee. As
to Atiga, Producers relies on the provision of its
contract with Unicorn Security Services which
provides that the guards of the latter "are in no
sense employees of the CLIENT."

ISSUE
WON Fortune Insurance and Surety Co. Inc. is liable
under the Money, Security, and Payroll Robbery
policy it issued to Producers Bank of the Philippines
or WON recovery is precluded under the general
exceptions clause of the policy

HELD
NO
Ratio A contract of insurance is a contract of
adhesion, thus any ambiguity therein should be
resolved against the insurer, or it should be
construed liberally in favor of the insured and strictly
against the insurer. Limitations of liability should be
regarded with extreme jealousy and must be
construed in such a way as to preclude the insurer
from non-compliance with its obligation. It goes
without saying then that if the terms of the contract
are clear and unambiguous, there is no room for
construction and such terms cannot be enlarged or
diminished by judicial construction.
- An insurance contract is a contract of indemnity
upon the terms and conditions specified therein. It is
settled that the terms of the policy constitute the
measure of the insurer's liability. In the absence of
statutory prohibition to the contrary, insurance
companies have the same rights as individuals to
limit their liability and to impose whatever conditions
they deem best upon their obligations not
inconsistent with public policy.
Reasoning
- It should be noted that the insurance policy
entered into by the parties is a theft or robbery
insurance policy which is a form of casualty
insurance. Section 174 of the Insurance Code
provides:
Sec. 174. Casualty insurance is insurance covering
loss or liability arising from accident or mishap,
excluding certain types of loss which by law or
custom are considered as failing exclusively within
the scope of insurance such as fire or marine. It
includes, but is not limited to, employer's liability
insurance, public liability insurance, motor vehicle
liability insurance, plate glass insurance, burglary
and theft insurance, personal accident and health
insurance as written by non-life insurance
companies, and other substantially similar kinds of
insurance. (italics supplied)
- Except with respect to compulsory motor vehicle
liability insurance, the Insurance Code contains no
other provisions applicable to casualty insurance or
to robbery insurance in particular. These contracts
are, therefore, governed by the general provisions
applicable to all types of insurance. Outside of these,
the rights and obligations of the parties must be
determined by the terms of their contract, taking
into consideration its purpose and always in
accordance with the general principles of insurance
law.
- With the foregoing principles in mind, it may now
be asked whether Magalong and Atiga qualify as
employees or authorized representatives has been
aptly observed that in burglary, robbery, and theft
insurance, "the opportunity to defraud the insurer -
the moral hazard - is so great that insurers have
found it necessary to fill up their policies with
countless restrictions, many designed to reduce this
hazard. Seldom does the insurer assume the risk of
!"#$%&"'( *+,- ./

all losses due to the hazards insured against."
Persons frequently excluded under such provisions
are those in the insured's service and employment.
The purpose of the exception is to guard against
liability should the theft be committed by one having
unrestricted access to the property. In such cases,
the terms specifying the excluded classes are to be
given their meaning as understood in common
speech. The terms "service" and "employment" are
generally associated with the idea of selection,
control, and compensation.
- There is marked disagreement between the parties
on the correct meaning of the terms "employee"
and "authorized representatives."
It is clear to us that insofar as Fortune is concerned,
it was its intention to exclude and exempt from
protection and coverage losses arising from
dishonest, fraudulent, or criminal acts of persons
granted or having unrestricted access to Producers'
money or payroll. When it used then the term
"employee," it must have had in mind any person
who qualifies as such as generally and universally
understood, or jurisprudentially established in the
light of the four standards in the determination of
the employer-employee relationship or as statutorily
declared even in a limited sense as in the case of
Article 106 of the Labor Code which considers the
employees under a "labor-only" contract as
employees of the party employing them and not of
the party who supplied them to the employer.
- But even granting for the sake of argument that
these contracts were not "labor-only" contracts, and
PRC Management Systems and Unicorn Security
Services were truly independent contractors, we are
satisfied that Magalong and Atiga were, in respect of
the transfer of Producer's money from its Pasay City
branch to its head office in Makati, its "authorized
representatives" who served as such with its teller
Maribeth Alampay. Howsoever viewed, Producers
entrusted the three with the specific duty to safely
transfer the money to its head office, with Alampay
to be responsible for its custody in transit; Magalong
to drive the armored vehicle which would carry the
money; and Atiga to provide the needed security for
the money, the vehicle, and his two other
companions. In short, for these particular tasks, the
three acted as agents of Producers. A
"representative" is defined as one who represents or
stands in the place of another; one who represents
others or another in a special capacity, as an agent,
and is interchangeable with "agent."
Disposition instant petition is hereby GRANTED. CA
decision and RTC Makati decision are REVERSED and
SET ASIDE. Civil Case is DISMISSED.

VERENDIA V CA (FIDELITY & SURETY CO. OF
THE PHILS)
217 SCRA 417
MELO; January 22, 1993

NATURE
Petition to review decision of the CA

FACTS
- Fidelity Co. issued a Fire Insurance Policy covering
Verendias residential building in the amount of
P385k. Verendia also insured the same building with
two other companies (Country Bankers Insurance for
P56k, and Development Insurance for P400k).
- While all 3 policies were in force, the insured
property was completely destroyed by fire. Verendia
filed a claim against Fidelity, but the latter refused
payment, thus a complaint was filed in the RTC.
Fidelitys reason for refusal: the policy was avoided
by reason of over-insurance, and that Verendia
maliciously represented that the building was under
lease to a Roberto Garcia, when it was actually a
Marcelo Garcia who was the lessee.
- RTC: policy was violated by Verendia when it failed
to inform Fidelity of his other insurance coverages,
thus no need to pay.
- CA: reversed decision

ISSUE
(There is a procedural issue involved here, but is
irrelevant to our discussion. It concerns the filing of
a motion for extension of time to file a motion for
reconsideration, where the court said that although it
now prohibits filing of such motion, the instant
motion was filed before the effectivity of this rule,
thus allowing the adjudication of the case)
WON Fidelity was liable to pay Verendia considering
the circumstances

HELD
1. NO
Ratio As the insurance contract is the law between
the parties, Verendia is deemed to have forfeited his
right to claim by the misrepresentation he made.
Reasoning
- the court reviewed the factual findings of the courts
below, since it appears that there was a
misapprehension of the facts by the CA.
- Verendia is found to have concocted the lease
contract to deflect responsibility for the fire towards
an alleged lessee, even making it appear that the
alleged lessee had disappeared, inflated the value of
the property, and insured same property with two
other companies.
- An insurance contract is the law between the
parties, its terms and conditions constitute the
measure of the insurers liability and compliance
therewith is a condition precedent to the insureds
right to recovery from the insurer.
- As it is also a contract of adhesion, an insurance
contract should be liberally construed in favor of the
insured and strictly against the insurer company
which usually prepares it.
- Considering, however, the fact that Verendia used
a false lease contract to support his claim, the terms
of the policy should be strictly construed against the
insured. Verendia failed to live by the terms of the
policy, specifically Section 13 thereof which is
expressed in terms that are clear and unambiguous,
that all benefits under the policy shall be forfeited If
the claim be in any respect fraudulent, or if any false
declaration be made or used in support thereof, or if
any fraudulent means or devises are used by the
Insured or anyone acting in his behalf to obtain any
benefit under the policy. Verendia, having presented
a false declaration to support his claim for benefits in
the form of a fraudulent lease contract, he forfeited
all benefits therein by virtue of Section 13 of the
policy in the absence of proof that Fidelity waived
such provision. Worse yet, by presenting a false
lease contract, Verendia reprehensibly disregarded
the principle that insurance contracts are uberrimae
fidae and demand the most abundant good faith.
Disposition Decision of CA reversed, and that of
RTC is reinstated.
FIELDMEN'S INSURANCE CO. INC V VDA. DE
SONGCO
25 SCRA 20
FERNANDO; 1968

FACTS
- An insurance firm, petitioner Fieldmen's Insurance
Co., Inc., was not allowed to escape liability under a
common carrier insurance policy on the pretext that
what was insured, not once but twice, was a private
vehicle and not a common carrier, the policy being
issued upon the insistence of its agent who
discounted fears of the insured that his privately
owned vehicle might not fall within its terms, the
insured moreover being "a man of scant education,"
!"#$%&"'( *+,- .0

finishing only the first grade. So it was held in a
decision of the lower court thereafter affirmed by
respondent Court of Appeals. Petitioner in seeking
the review of the above decision of respondent Court
of Appeals cannot be so sanguine as to entertain the
belief that a different outcome could be expected. To
be more explicit, we sustain the Court of Appeals.
- The facts as found by respondent Court of Appeals,
binding upon us, follow: "This is a peculiar case.
Federico Songco of Floridablanca, Pampanga, a man
of scant education being only a first grader ...,
owned a private jeepney with Plate No. 41-289 for
the year 1960. On September 15, 1960, as such
private vehicle owner, he was induced by Fieldmen's
Insurance Company Pampanga agent Benjamin
Sambat to apply for a Common Carrier's Liability
Insurance Policy covering his motor vehicle ... Upon
paying an annual premium of P16.50, defendant
Fieldmen's Insurance Company, Inc. issued on
September 19, 1960, Common Carriers Accident
Insurance Policy No. 45-HO- 4254 ... the duration of
which will be for one (1) year, effective September
15, 1960 to September 15, 1961. On September 22,
1961, the defendant company, upon payment of the
corresponding premium, renewed the policy by
extending the coverage from October 15, 1961 to
October 15, 1962. This time Federico Songco's
private jeepney carried Plate No. J-68136-
Pampanga-1961. ... On October 29, 1961, during the
effectivity of the renewed policy, the insured vehicle
while being driven by Rodolfo Songco, a duly
licensed driver and son of Federico (the vehicle
owner) collided with a car in the municipality of
Calumpit, province of Bulacan, as a result of which
mishap Federico Songco (father) and Rodolfo Songco
(son) died, Carlos Songco (another son), the latter's
wife, Angelita Songco, and a family friend by the
name of Jose Manuel sustained physical injuries of
varying degree."
1

- It was further shown according to the decision of
respondent Court of Appeals: "Amor Songco, 42-
year-old son of deceased Federico Songco, testifying
as witness, declared that when insurance agent
Benjamin Sambat was inducing his father to insure
his vehicle, he butted in saying: 'That cannot be, Mr.
Sambat, because our vehicle is an "owner" private
vehicle and not for passengers,' to which agent
Sambat replied: 'whether our vehicle was an "owner"
type or for passengers it could be insured because
their company is not owned by the Government and
the Government has nothing to do with their
company. So they could do what they please
whenever they believe a vehicle is insurable' ... In
spite of the fact that the present case was filed and
tried in the CFI of Pampanga, the defendant
company did not even care to rebut Amor Songco's
testimony by calling on the witness-stand agent
Benjamin Sambat, its Pampanga Field
Representative."
2

- The plaintiffs in the lower court, likewise
respondents here, were the surviving widow and
children of the deceased Federico Songco as well as
the injured passenger Jose Manuel. On the above
facts they prevailed, as had been mentioned, in the
lower court and in the respondent Court of
Appeals.1awphl.nt
- The basis for the favorable judgment is the doctrine
announced in Qua Chee Gan v. Law Union and Rock
Insurance Co., Ltd.,
3
with Justice J. B. L. Reyes
speaking for the Court. It is now beyond question
that where inequitable conduct is shown by an
insurance firm, it is "estopped from enforcing
forfeitures in its favor, in order to forestall fraud or
imposition on the insured."
4

- As much, if not much more so than the Qua Chee
Gan decision, this is a case where the doctrine of
estoppel undeniably calls for application. After
petitioner Fieldmen's Insurance Co., Inc. had led the
insured Federico Songco to believe that he could
qualify under the common carrier liability insurance
policy, and to enter into contract of insurance paying
the premiums due, it could not, thereafter, in any
litigation arising out of such representation, be
permitted to change its stand to the detriment of the
heirs of the insured. As estoppel is primarily based
on the doctrine of good faith and the avoidance of
harm that will befall the innocent party due to its
injurious reliance, the failure to apply it in this case
would result in a gross travesty of justice.
- That is all that needs be said insofar as the first
alleged error of respondent Court of Appeals is
concerned, petitioner being adamant in its far-from-
reasonable plea that estoppel could not be invoked
by the heirs of the insured as a bar to the alleged
breach of warranty and condition in the policy. lt
would now rely on the fact that the insured owned a
private vehicle, not a common carrier, something
which it knew all along when not once but twice its
agent, no doubt without any objection in its part,
exerted the utmost pressure on the insured, a man
of scant education, to enter into such a contract.
- Nor is there any merit to the second alleged error
of respondent Court that no legal liability was
incurred under the policy by petitioner. Why liability
under the terms of the policy
5
was inescapable was
set forth in the decision of respondent Court of
Appeals. Thus: "Since some of the conditions
contained in the policy issued by the defendant-
appellant were impossible to comply with under the
existing conditions at the time and 'inconsistent with
the known facts,' the insurer 'is estopped from
asserting breach of such conditions.' From this
jurisprudence, we find no valid reason to deviate and
consequently hold that the decision appealed from
should be affirmed. The injured parties, to wit,
Carlos Songco, Angelito Songco and Jose Manuel, for
whose hospital and medical expenses the defendant
company was being made liable, were passengers of
the jeepney at the time of the occurrence, and
Rodolfo Songco, for whose burial expenses the
defendant company was also being made liable was
the driver of the vehicle in question. Except for the
fact, that they were not fare paying passengers,
their status as beneficiaries under the policy is
recognized therein."
6

- Even if it be assumed that there was an ambiguity,
an excerpt from the Qua Chee Gan decision would
reveal anew the weakness of petitioner's contention.
Thus: "Moreover, taking into account the well known
rule that ambiguities or obscurities must be strictly
interpreted against the party that caused them, the
'memo of warranty' invoked by appellant bars the
latter from questioning the existence of the
appliances called for in the insured premises, since
its initial expression, 'the undernoted appliances for
the extinction of fire being kept on the premises
insured hereby, ... it is hereby warranted ...,' admits
of interpretation as an admission of the existence of
such appliances which appellant cannot now
contradict, should the parol evidence rule apply."
7

- To the same effect is the following citation from the
same leading case: "This rigid application of the rule
on ambiguities has become necessary in view of
current business practices. The courts cannot ignore
that nowadays monopolies, cartels and concentration
of capital, endowed with overwhelming economic
power, manage to impose upon parties dealing with
them cunningly prepared 'agreements' that the
weaker party may not change one whit, his
participation in the 'agreement' being reduced to the
alternative to 'take it or leave it' labelled since
Raymond Saleilles 'contracts by adherence' (contrats
d'adhesion), in contrast to those entered into by
parties bargaining on an equal footing, such
contracts (of which policies of insurance and
international bills of lading are prime examples)
!"#$%&"'( *+,- .1

obviously call for greater strictness and vigilance on
the part of courts of justice with a view to protecting
the weaker party from abuses and imposition, and
prevent their becoming traps for the unwary (New
Civil Code. Article 24; Sent. of Supreme Court of
Spain, 13 Dec. 1934, 27 February 1942)."
8

- The last error assigned which would find fault with
the decision of respondent Court of Appeals insofar
as it affirmed the lower court award for exemplary
damages as well as attorney's fees is, on its face, of
no persuasive force at all.
- The conclusion that inescapably emerges from the
above is the correctness of the decision of
respondent Court of Appeals sought to be reviewed.
For, to borrow once again from the language of the
Qua Chee Gan opinion: "The contract of insurance is
one of perfect good faith (uberima fides) not for the
insured alone,but equally so for the insurer; in fact,
it is more so for the latter, since its dominant
bargaining position carries with it stricter
responsibility."
9

- This is merely to stress that while the morality of
the business world is not the morality of institutions
of rectitude like the pulpit and the academe, it
cannot descend so low as to be another name for
guile or deception. Moreover, should it happen thus,
no court of justice should allow itself to lend its
approval and support.1awphl.nt
- We have no choice but to recognize the monetary
responsibility of petitioner Fieldmen's Insurance Co.,
Inc. It did not succeed in its persistent effort to avoid
complying with its obligation in the lower court and
the Court of Appeals. Much less should it find any
receptivity from us for its unwarranted and
unjustified plea to escape from its liability.

MALAYAN INSURANCE CORP. V CA (TKC
MARKETING CORP.)
270 SCRA 242
ROMERO; March 20, 1997

NATURE
Petition for review on certiorari

FACTS
- TKC Marketing Corp. was the owner/consignee of
some 3,189.171 metric tons of soya bean meal
which was loaded on board the ship MV Al Kaziemah
for carriage from the port of Rio del Grande, Brazil,
to the port of Manila. Said cargo was insured against
the risk of loss by petitioner Malayan Insurance
Corporation for which it issued two (2) Marine Cargo
Policies.
- While the vessel was docked in Durban, South
Africa the civil authorities arrested and detained it
because of a lawsuit on a question of ownership and
possession. TKC Marketing notified Malayan of the
arrest of the vessel and made a formal claim for the
dollar equivalent on the policies (US$916,886.66) for
non-delivery of the cargo. It likewise sought the
assistance of Malayan on what to do with the cargo.
- Malayan replied that the arrest of the vessel by civil
authority was not a peril covered by the policies. TKC
advised Malayan that it might tranship the cargo and
requested an extension of the insurance coverage
until actual transhipment, which extension was
approved upon payment of additional premium. The
insurance coverage was extended under the same
terms and conditions embodied in the original
policies while in the process of making arrangements
for the transhipment of the cargo from Durban to
Manila. However the cargo was sold in Durban,
South Africa, for US$154.40 per metric ton or a total
of P10,304,231.75 due to its perishable nature which
could no longer stand a voyage of twenty days to
Manila and another twenty days for the discharge
thereof. It reduced its claim to US$448,806.09 (or its
peso equivalent of P9,879,928.89 at the exchange
rate of P22.0138 per $1.00) representing its loss
after the proceeds of the sale were deducted from
the original claim.Malayan maintained its position
that the arrest of the vessel by civil authorities on a
question of ownership was an excepted risk under
the marine insurance policies.
Petitioners Claim
- an arrest by civil authority is not compensable
since the term "arrest" refers to "political or
executive acts" and does not include a loss caused
by riot or by ordinary judicial process as in this case
- the deletion of the Free from Capture or Seizure
Clause would leave the assured covered solely for
the perils specified by the wording of the policy itself
- the rationale for the exclusion of an arrest pursuant
to judicial authorities is to eliminate collusion
between unscrupulous assured and civil authorities.
- any loss which private respondent may have
incurred was in the nature and form of unrecovered
acquisition value brought about by a voluntary
sacrifice sale and not by arrest, detention or seizure
of the ship.
- its act of rejecting the claim was a result of its
honest belief that the arrest of the vessel was not a
compensable risk under the policies issued
Respondents Comments
- petitioner, being the sole author of the policies,
"arrests" should be strictly interpreted against it
because the rule is that any ambiguity is to be taken
contra proferentum. Risk policies should be
construed reasonably and in a manner as to make
effective the intentions and expectations of the
parties.
- the policies clearly stipulate that they cover the
risks of non-delivery of an entire package and that it
was petitioner itself that invited and granted the
extensions and collected premiums thereon.

ISSUES
1. WON the arrest of the vessel was a risk covered
under the subject insurance policies
2. WON insurance policies should be strictly
construed against the insurer

HELD
1.YES
- With the incorporation of subsection 1.1 of Section
1 of the Institute War Clauses, "arrest" caused by
ordinary judicial process is deemed included among
the covered risks. This interpretation becomes
inevitable when subsection 1.1 of Section 1 of the
Institute War Clauses provided that "this insurance
covers the risks excluded from the Standard Form of
English Marine Policy by the clause 'Warranted free
of capture, seizure, arrest, etc. x x x'" or the F.C. &
S. Clause. Jurisprudentially, "arrests" caused by
ordinary judicial process is also a risk excluded from
the Standard Form of English Marine Policy by the
F.C. & S. Clause.
- Petitioner cannot adopt the argument that the
"arrest" caused by ordinary judicial process is not
included in the covered risk simply because the F.C.
& S. Clause under the Institute War Clauses can only
be operative in case of hostilities or warlike
operations on account of its heading "Institute War
Clauses."
2. YES
Ratio Insurance Policies should be construed
liberally in favor of the insured and strictly against
the insurer.
Reasoning
- An insurance contract should be so interpreted as
to carry out the purpose for which the parties
entered into the contract which is, to insure against
risks of loss or damage to the goods. Such
interpretation should result from the natural and
reasonable meaning of language in the policy. Where
!"#$%&"'( *+,- .2

restrictive provisions are open to two interpretations,
that which is most favorable to the insured is
adopted.
Indemnity and liability insurance policies are
construed in accordance with the general rule of
resolving any ambiguity therein in favor of the
insured, where the contract or policy is prepared by
the insurer. A contract of insurance, being a contract
of adhesion, par excellence, any ambiguity therein
should be resolved against the insurer.Limitations of
liability should be regarded with extreme jealousy
and must be construed in such a way as to preclude
the insurer from noncompliance with its obligations
- It must be borne in mind that such contracts are
invariably prepared by the companies and must be
accepted by the insured in the form in which they
are written. Any construction of a marine policy
rendering it void should be avoided. Such policies
will, therefore, be construed strictly against the
company in order to avoid a forfeiture, unless no
other result is possible from the language used.
- If a marine insurance company desires to limit or
restrict the operation of the general provisions of its
contract by special proviso, exception, or exemption,
it should express such limitation in clear and
unmistakable language.
Be that as it may, exceptions to the general
coverage are construed most strongly against the
company. Even an express exception in a policy is to
be construed against the underwriters by whom the
policy is framed, and for whose benefit the exception
is introduced.

WESTERN GUARANTY CORPORATION V CA
(RODRIGUEZ, and DE DIOS TRANSPORTATION
CO)
187 SCRA 652
FELICIANO; July 20, 1990

FACTS
- At around 4:30 in the afternoon of 27 March 1982,
while crossing Airport Road on a pedestrian lane on
her way to work, respondent Priscilla E. Rodriguez
was struck by a De Dios passenger bus owned by
respondent De Dios Transportation Co., Inc., then
driven by one Walter Saga y Aspero. The bus driver
disregarded the stop signal given by a traffic
policeman to allow pedestrians to cross the road.
Priscilla was thrown to the ground, hitting her
forehead. She was treated at the Protacio Emergency
Hospital and later on hospitalized at the San Juan De
Dios Hospital. Her face was permanently disfigured,
causing her serious anxiety and moral distress.
- Respondent bus company was insured with
petitioner Western Guaranty Corporation ("Western")
under its Master Policy which enumerated specific
liabilities of the insurance company and ended with a
clause to clarify the limitations of the amount which
could be granted as indemnity.
- Respondent Priscilla Rodriguez filed a complaint for
damages before the Regional Trial Court of Makati
against De Dios Transportation Co. and Walter A.
Saga. Respondent De Dios Transportation Co., in
turn, filed a third-party complaint against its
insurance carrier, petitioner Western.
- On 6 August 1985, the trial court rendered a
decision in favor of respondent Priscilla E. Rodriguez,
- On appeal, the Court of Appeals affirmed in toto
the decision of the trial court. Petitioner moved for
the reconsideration of the appellate court's decision.
In a Resolution dated 10 January 1990, the Court of
Appeals denied the motion for reconsideration for
lack of merit. Petitioner Western is now before us on
a Petition for Review alleging that the Court of
Appeals erred in holding petitioner liable to pay
beyond the limits set forth in the Schedule
Indemnities and in finding Western liable for loss of
earnings, moral damages and attorney's fees.
Succinctly stated, it is petitioner Western's position
that it cannot be held liable for loss of earnings,
moral damages and attorney's fees because these
items are not among those included in the Schedule
Indemnities set forth in the insurance policy.
- Petitioner Western in effect contends before this
Court, as it did before the Court of Appeals, that
because the Schedule of Indemnities limits the
amount payable for certain kinds of expenses
"hospital room", "surgical expenses", "an
aesthesiologists' fee", "operating room" and "medical
expenses" that Schedule should be read as
excluding liability for any other type of expense or
damage or loss even though actually sustained or
incurred by the third party victim. We are not
persuaded by Western's contention.

ISSUE
WON the Schedule of indemnities as stated in the
insurance policy should be construed strictly to
exclude all others not explicitly stated therein



HELD
NO
Ratio An insurance policy being in the nature of an
adhesion contract is to be strictly construed against
the insurer and liberally in favor of the insured.
Reasoning
- Firstly, the Schedule of Indemnities does not
purport to restrict the kinds of damages that may be
awarded against Western once liability has arisen.
Section 1, quoted above, does refer to certain
"Limits of Liability" which in the case of the third
party liability section of the Master Policy, is
apparently P50,000.00 per person per accident.
Within this over-all quantitative limit, all kinds of
damages allowable by law "actual or compensatory
damages"; "moral damages"; "nominal damages";
"temperate or moderate damages"; "liquidated
damages"; and "exemplary damages" may be
awarded by a competent court against the insurer
once liability is shown to have arisen, and the
essential requisites or conditions for grant of each
species of damages are present. It appears to us
self-evident that the Schedule of Indemnities was
not intended to be an enumeration, much less a
closed enumeration, of the specific kinds of damages
which may be awarded under the Master Policy
Western has issued.
- Secondly, the reading urged by Western of the
Schedule of Indemnities comes too close to working
fraud upon both the insured and the third party
beneficiary of Section 1, quoted above. For
Western's reading would drastically and without
warning limit the otherwise unlimited (save for the
over-all quantitative limit of liability of P50,000.00
per person per accident) and comprehensive scope
of liability assumed by the insurer Western under
Section 1: "all sums necessary to discharge liability
of the insured in respect of [bodily injury to a third
party]". This result which is not essentially
different from taking away with the left hand what
had been given with the right hand we must avoid
as obviously repugnant to public policy. If what
Western now urges is what Western intended to
achieve by its Schedule of Indemnities, it was
incumbent upon Western to use language far more
specific and precise than that used in fact by
Western, so that the insured, and potential
purchasers of its Master Policy, and the Office of the
Insurance Commissioner, may be properly informed
and act accordingly.
- Petitioner Western would have us construe the
Schedule of Indemnities as comprising contractual
limitations of liability which, as already noted, is
!"#$%&"'( *+,- .3

comprehensively defined in Section 1 "Liability to
the Public" of the Master Policy. It is well-settled,
however, that contractual limitations of liability found
in insurance contracts should be regarded by courts
with a jaundiced eye and extreme care and should
be so construed as to preclude the insurer from
evading compliance with its just obligations.
- Finally, an insurance contract is a contract of
adhesion. The rule is well entrenched in our
jurisprudence that the terms of such contract are to
be construed strictly against the party which
prepared the contract, which in this case happens to
be petitioner Western.

QUA CHEE GAN V LAW UNION AND ROCK
INSURANCE CO., LTD.
96 PHIL 85
REYES; December 17, 1955

NATURE
An appeal by defendant insurance company from the
decision of CFI in favor of the plaintiff

FACTS
- before the last war, plaintiff-appellee owned 4
warehouses or bodegas in Tabaco, Albay, used for
the storage of stocks of copra and of hemp, baled
and loose, in which the appellee dealt extensively.
They had been, with their contents, insured with the
defendant Company since 1937, and the loose made
payable to the Philippine National Bank as mortgage
of the hemp and crops, to the extent of its interest.
- Fire of undetermined origin that broke out in the
early morning of July 21, 1940, and lasted almost
one week, gutted and completely destroyed Bodegas
Nos. 1, 2 and 4, with the merchandise stored
therein. Plaintiff-appellee informed the insurer by
telegram on the same date; and on the next day, the
fire adjusters engaged by appellant insurance
company arrived and proceeded to examine and
photograph the premises, pored over the books of
the insured and conducted an extensive
investigation. The plaintiff having submitted the
corresponding fire claims, totalling P398,562.81 (but
reduced to the full amount of the insurance,
P370,000), the Insurance Company resisted
payment, claiming violation of warranties and
conditions, filing of fraudulent claims, and that the
fire had been deliberately caused by the insured or
by other persons in connivance with him.
- Que Chee Gan, with his brother, Qua Chee Pao,
and some employees of his, were indicted and tried
in 1940 for the crime of arson, it being claimed that
they had set fire to the destroyed warehouses to
collect the insurance. They were, however, acquitted
by the trial court.
- the civil suit to collect the insurance money
proceeded to its trial with the CFI holding that:
judgment is rendered for the plaintiff and against the
defendant condemning the latter to pay the former
(a) Under the first cause of action, the sum of
P146,394.48; (b) Under the second cause of action,
the sum of P150,000; (c) Under the third cause of
action, the sum of P5,000; (d) Under the fourth
cause of action, the sum of P15,000; and (e) Under
the fifth cause of action, the sum of P40,000; all of
which shall bear interest at the rate of 8% per
annum in accordance with Section 91 (b) of the
Insurance Act from September 26, 1940, until each
is paid, with costs against the defendant.
- In its first assignment of error, the insurance
company alleges that the trial Court should have
held that the policies were avoided for breach of
warranty, specifically the one appearing on a rider
pasted (with other similar riders) on the face of the
policies.
4

- It is argued that since the bodegas insured had an
external wall perimeter of 500 meters or 1,640 feet,
the appellee should have 11 fire hydrants in the
compound, and that he actually had only 2, with a
further pair nearby, belonging to the municipality of
Tabaco.

ISSUES
1. WON the defendant-appellant can claim the
policies it had issued as void ab initio
2. WON the insured violated the "Hemp Warranty"
provisions of Policy No. 2637165 against the storage
of gasoline
3. WON the insured connived at the loss and
fraudulently inflated the quantity of the insured stock
in the burnt bodegas


4
!"#$% $' ()**)+,-& G71 :-91$-+%19 #3350/-.12 6+$ %71 1J%0-.%0+- +6 60$1 410-; C13% +- %71
3$1,0212 0-2:$19 71$14<8 /-9 0% 410-; 91.5/$19 /-9 :-91$2%++9 %7/% %71$1 02 /- /,351 /-9
.+-2%/-% =/%1$ 2:335< =0%7 2:660.01-% 3$122:$1 /F/05/451 /% /55 21/2+-2 6+$ %71 2/,18 0% 02 71$14<
=/$$)+,"9 %7/% %71 2/09 /3350/-.12 27/55 41 ,/0-%/0-19 0- 1660.01-% =+$C0-; +$91$ 9:$0-; %71
.:$$1-.< +6 %702 3+50.<8 4< $1/2+- =71$1+6 / 902.+:-% +6 E !KE 31$ .1-% 02 /55+=19 +- %71 3$1,0:,
.7/$;1/451 :-91$ %702 3+50.<&
.-/*)+,0 0- %71 .+,3+:-98 -+% 5122 0- -:,41$ %7/- +-1 6+$ 1/.7 !HL 611% +6 1J%1$-/5 =/55
,1/2:$1,1-% +6 4:0590-;8 3$+%1.%198 =0%7 -+% 5122 %7/- !LL 611% +6 7+21 3030-; /-9 -+MM512 6+$
1F1$< %=+ 7<9$/-%2 C13% :-91$ .+F1$ 0- .+-F1-01-% 35/.128 %71 7<9$/-%2 410-; 2:335019 =0%7 =/%1$
3$122:$1 4< / 3:,30-; 1-;0-18 +$ 6$+, 2+,1 +%71$ 2+:$.18 ./3/451 +6 902.7/$;0-; /% %71 $/%1 +6 -+%
5122 %7/- ELL ;/55+-2 +6 =/%1$ 31$ ,0-:%1 0-%+ %71 :331$ 2%+$< +6 %71 70;712% 4:0590-; 3$+%1.%198 /-9
/ %$/0-19 4$0;/91 +6 -+% 5122 %7/- EL ,1- %+ =+$C %71 2/,1&N

HELD
1. NO
Ratio It is usually held that where the insurer, at the
time of the issuance of a policy of insurance, has
knowledge of existing facts which, if insisted on,
would invalidate the contract from its very inception,
such knowledge constitutes a waiver of conditions in
the contract inconsistent with the facts, and the
insurer is stopped thereafter from asserting the
breach of such conditions. The law is charitable
enough to assume, in the absence of any showing to
the contrary, that an insurance company intends to
executed a valid contract in return for the premium
received; and when the policy contains a condition
which renders it voidable at its inception, and this
result is known to the insurer, it will be presumed to
have intended to waive the conditions and to execute
a binding contract, rather than to have deceived the
insured into thinking he is insured when in fact he is
not, and to have taken his money without
consideration.
Reasoning
- The appellant is barred estoppel to claim violation
of the so-called fire hydrants warranty, for the
reason that knowing fully all that the number of
hydrants demanded therein never existed from the
very beginning, the appellant neverthless issued the
policies in question subject to such warranty, and
received the corresponding premiums. The insurance
company was aware, even before the policies were
issued, that in the premises insured there were only
two fire hydrants installed by Qua Chee Gan and two
others nearby, owned by the municipality of Tabaco,
contrary to the requirements of the warranty in
question
- The plain, human justice of this doctrine is
perfectly apparent. To allow a company to
accept one's money for a policy of insurance
which it then knows to be void and of no effect,
though it knows as it must, that the assured
believes it to be valid and binding, is so
contrary to the dictates of honesty and fair
dealing, and so closely related to positive
fraud, as to the abhorrent to fair-minded men.
It would be to allow the company to treat the
policy as valid long enough to get the premium
on it, and leave it at liberty to repudiate it the
next moment. This cannot be deemed to be the
real intention of the parties. To hold that a
literal construction of the policy expressed the
true intention of the company would be to
!"#$%&"'( *+,- .4

indict it, for fraudulent purposes and designs
which we cannot believe it to be guilty of.
- The appellant company so worded the policies that
while exacting the greater number of fire hydrants
and appliances, it kept the premium discount at the
minimum of 2 1/2%, thereby giving the insurance
company a double benefit. Such abnormal treatment
of the insured strongly points at an abuse of the
insurance company's selection of the words and
terms of the contract, over which it had absolute
control.
- Receipt of Premiums or Assessments after Cause
for Forfeiture Other than Nonpayment. It is a well
settled rule of law that an insurer which with
knowledge of facts entitling it to treat a policy as no
longer in force, receives and accepts a premium on
the policy, estopped to take advantage of the
forfeiture. It cannot treat the policy as void for the
purpose of defense to an action to recover for a loss
thereafter occurring and at the same time treat it as
valid for the purpose of earning and collecting further
premiums.
- Moreover, taking into account the well known rule
that ambiguities or obscurities must be strictly
interpreted against the party that caused them,

the
"memo of warranty" invoked by appellant bars the
latter from questioning the existence of the
appliances called for in the insured premises
On the alleged violations of the plaintiff The
alleged violation of the warranty of 100 feet of fire
hose for every two hydrants, must be equally
rejected, since the appellant's argument thereon is
based on the assumption that the insured was bound
to maintain no less than eleven hydrants, which
requirement appellant is estopped from enforcing.
- As to maintenance of a trained fire brigade of 20
men, the record is preponderant that the same was
organized, and drilled, from time to give, although
not maintained as a permanently separate unit,
which the warranty did not require.
2. NO
Ratio Here, again, by reason of the exclusive control
of the insurance company over the terms and
phraseology of the contract, the ambiguity must be
held strictly against the insurer and liberally in favor
of the insured, specially to avoid a forfeiture.
Insurance is, in its nature, complex and
difficult for the layman to understand. Policies
are prepared by experts who know and can
anticipate the hearing and possible
complications of every contingency. So long as
insurance companies insist upon the use of
ambiguous, intricate and technical provisions,
which conceal rather than frankly disclose,
their own intentions, the courts must, in
fairness to those who purchase insurance,
construe every ambiguity in favor of the
insured. An insurer should not be allowed, by
the use of obscure phrases and exceptions, to
defeat the very purpose for which the policy
was procured.
Reasoning
- Appellee admitted that there were 36 cans of
gasoline in the building designed. It However,
gasoline is not specifically mentioned among the
prohibited articles listed in the so-called "hemp
warranty." The cause relied upon by the insurer
speaks of "oils (animal and/or vegetable and/or
mineral and/or their liquid products having a flash
point below 300 Fahrenheit)", and is decidedly
ambiguous and uncertain; for in ordinary parlance,
"Oils" mean "lubricants" and not gasoline or
kerosene. And how many insured, it may well be
wondered, are in a position to understand or
determine "flash point below 300 Fahrenheit.
- If the company intended to rely upon a condition of
that character, it ought to have been plainly
expressed in the policy.
- The contract of insurance is one of perfect good
faith not for the insured alone, but equally so for the
insurer; in fact, it is mere so for the latter, since its
dominant bargaining position carries with it stricter
responsibility.
- Another point that is in favor of the insured is that
the gasoline kept in Bodega No. 2 was only incidental
to his business, being no more than a customary 2
day's supply for the five or six motor vehicles used
for transporting of the stored merchandise. "It is well
settled that the keeping of inflammable oils on the
premises though prohibited by the policy does not
void it if such keeping is incidental to the business."
On the submission of books, voucbers, etc. The
charge that the insured failed or refused to submit to
the examiners of the insurer the books, vouchers,
etc. demanded by them was found unsubstantiated
by the trial Court, and no reason has been shown to
alter this finding. The insured gave the insurance
examiner all the date he asked for, and the examiner
even kept and photographed some of the examined
books in his possession. What does appear to have
been rejected by the insured was the demand that
he should submit "a list of all books, vouchers,
receipts and other records", but the refusal of the
insured in this instance was well justified, since the
demand for a list of all the vouchers (which were not
in use by the insured) and receipts was positively
unreasonable, considering that such listing was
superfluous because the insurer was not denied
access to the records, that the volume of Qua Chee
Gan's business ran into millions, and that the
demand was made just after the fire when
everything was in turmoil. That the representatives
of the insurance company were able to secure all the
date they needed is proved by the fact that the
adjuster Alexander Stewart was able to prepare his
own balance sheet that did not differ from that
submitted by the insured except for the valuation of
the merchandise, as expressly found by the Court in
the criminal case for arson.
3. NO
Ratio Both defenses are predicted on the
assumption that the insured was in financial
difficulties and set the fire to defraud the insurance
company, presumably in order to pay off the
Philippine National Bank, to which most of the
insured hemp and copra was pledged. Both defenses
are fatally undermined by the established fact that,
notwithstanding the insurer's refusal to pay the value
of the policies the extensive resources of the insured
enabled him to pay off the National Bank in a short
time; and if he was able to do so, no motive appears
for attempt to defraud the insurer. While the
acquittal of the insured in the arson case is not res
judicata on the present civil action, the insurer's
evidence, to judge from the decision in the criminal
case, is practically identical in both cases and must
lead to the same result, since the proof to establish
the defense of connivance at the fire in order to
defraud the insurer "cannot be materially less
convincing than that required in order to convict the
insured of the crime of arson."
- As to the defense that the burned bodegas could
not possibly have contained the quantities of copra
and hemp stated in the fire claims, the insurer's case
rests almost exclusively on the estimates, inferences
and conclusions of its adjuster investigator who
examined the premises during and after the fire. His
testimony, however, was based on inferences from
the photographs and traces found after the fire, and
must yield to the contradictory testimony of those
who actually saw the contents of the bodegas shortly
before the fire, while inspecting them for the
mortgagee Bank.
Disposition We find no reversible error in the
judgment appealed from, wherefore the same is
hereby affirmed.
!"#$%&"'( *+,- .5


DEL ROSARIO V EQUITABLE INSURANCE &
CASUALTY CO., INC
8 SCRA 343
PAREDES; June 29, 1963

NATURE
Appeal from judgment of CFI Rizal

FACTS
- Francisco del Rosario was insured by Equitable
Insurance and Casualty Co. Inc under Personal
Accident Policy no. 7136. The Company bound itself
to pay P1000 to P3000 as indemnity for the death
of the insured.
- Under the policy:
Part I. Indemnity for Death
If the insured sustains any bodily injury
which is effected solely through violent,
external, visible and accidental means,
and which shall result, independently of
all other causes and within sixty days
from the occurrence thereof, in the Death
of the Insured, the Company shall pay the
amount set opposite such injury:

Section 1. Injury sustained other than
those specified below unless excepted
hereinafter P1000
Section 2. Injury sustained by the
wrecking or disablement of a railroad
passenger car or street railway car in or
on which the Insured is traveling as a
farepaying passenger P1500
Section 3. Injury sustained by the burning
of a church, theatre, public library or
municipal administration building while
the Insured is therein at the
commencement of the fire P2000
Section 4. Injury sustained by the
wrecking or disablement of a regular
passenger elevator car in which the
Insured is being conveyed as a passenger
(Elevator in mines exluded) P2500
Section 5. Injury sustained by a stroke of
lightning or by a cyclone P3000
x x x x x x x x
x x x x
Part VI. Exceptions
This policy shall not cover disappearance
of the Insured nor shall it cover Death,
Disability, Hospital fees, or Loss of time,
caused to the insured:
x x x (h) By drowning except as a
consequence of the wrecking or
disablement in the Philippine waters of a
passenger steam or motor vessel in
which the Insured is traveling as a
farepaying passenger; x x x
- A rider to the Policy contained the following;
IV. DROWNING
It is hereby declared and agreed that
exemption clause Letter (h) in PART VI of
the policy is hereby waived by the
company, and to form a part of the
provision covered by the policy.
- Feb 24, 1957, Francisco del Rosario while on board
the motor launch ISLAMA, with his beneficiary to the
policy, Remedios Jayme, were forced to jump off said
launch on account of fire which broke out on said
vessel, resulting in the death by drowning of the
insured and his beneficiary.
- Simeon del Rosario, the insureds father, filed a
claim for payment with the company. The company
paid him P1000 pursuant to section 1 Part I of the
policy.
- On the same date, Atty. Francisco wrote to the
company acknowledging receipt by his client of the
P1000 but informing said company that said amount
was not the correct one. He claimed that the amount
payable should be P1500 under the provision of
Section 2 Part I, based on the rule of pari materia.
- The company referred the matter to the Insurance
Coomissioner, who was of the opinion that the
liability of the company was only P1000. thus the
company refused to pay more that P1000. Atty.
Francisco wrote a subsequent letter to company
asking for p3000, which the company refused to pay.
- A complaint for recovery of the balance of P2000
was instituted with the CFI Rizal, praying for a
further sum of P10000 as attorneys fees, expenses
of litigation and costs.
- CFI ruled in favor of petitioner, ordering the
company to pay P2000 to del Rosario.

ISSUE
How much should the indemnity be

HELD
- All the parties agree that indemnity has to be paid,
but the conflict centers on how much it should be.
- Where there is ambiguity with respect to the terms
and conditions of the policy, the same will be
resolved against the one responsible thereof.
Generally, the insured has little, if any, participation
in the preparation of the policy, together with the
drafting of its terms and conditions. The
interpretation of obscure stipulations in a contract
should not favor the party who caused the obscurity.
- SC agreed with the ruling of the lower court:
x x x death by drowning is a ground for recovery
apart from the bodily injury because death by
bodily injury is covered by Part I of the policy
while death by drowning is covered by Part VI
thereof. But while the policy mentions specific
amounts that may be recovered for death for
bodily injury, yet, there is not specific amount
mentioned in the policy for death thru drowning
although the latter is, under Part VI of the policy,
a ground for recovery thereunder. Since the
defendant has bound itself to pay P1000 to
P3000 as indemnity for the death of the insured
but the policy does not positively state any
definite amount that may be recovered in case of
death by drowning, there is an ambiguity in this
respect in the policy, which ambiguity must be
interpreted in favor of the insured and strictly
against the insurer so as to allow a greater
indemnity. x x x plaintiff is therefore entitled to
recover P3000.
Disposition Judgment appealed from is affirmed.

GEAGONIA v. CA (COUNTRY BANKERS
INSURANCE)
8 SCRA 343
DAVIDE; February 6 1995

FACTS
-Geagonia is the owner of Norman's Mart located in
the public market of San Francisco, Agusan del Sur.
On 22 Dec 1989, he obtained from the private
respondent fire insurance policy for P100,000.00.
The period of the policy was from 22 Dec 1989 to 22
Dec 1990 and covered the ff: "Stock-in-trade
consisting principally of dry goods such as RTW's for
men and women wear and other usual to assured's
business.
-The policy contained the following condition:
"3. The insured shall give notice to the Company of
any insurance or insurances already effected, or
which may subsequently be effected, covering any of
the property or properties consisting of stocks in
trade, goods in process and/or inventories only
hereby insured, and unless notice be given and the
particulars of such insurance or insurances be stated
!"#$%&"'( *+,- .6

therein or endorsed in this policy pursuant to Section
50 of the Insurance Code, by or on behalf of the
Company before the occurrence of any loss or
damage, all benefits under this policy shall be
deemed forfeited, provided however, that this
condition shall not apply when the total insurance or
insurances in force at the time of the loss or damage
is not more than P200,000.00."
-On 27 May 1990, fire of accidental origin broke out
at around 7:30 p.m. at the public market of San
Francisco, Agusan del Sur. The petitioner's insured
stocks-in-trade were completely destroyed
prompting him to file w/ the private respondent a
claim under the policy. On 28 Dec 1990, the private
respondent denied the claim because it found that at
the time of the loss the petitioner's stocks-in-trade
were likewise covered by two fire insurance policies
for P100,000.00 each, issued by the Cebu Branch of
the Philippines First Insurance Co., Inc. (PFIC).
-The basis of the private respondent's denial was the
petitioner's alleged violation of Condition 3 of the
policy.
- Geagonia then filed a complaint against the private
respondent w/ the Insurance Commission for the
recovery of P100,000.00 under fire insurance policy,
for attorney's fees, and costs of litigation. He claims
that the time he obtained the private respondent's
fire insurance policy he knew that the two policies
issued by the PFIC were already in existence;
however, he had no knowledge of the provision in
the private respondent's policy requiring him to
inform it of the prior policies; this requirement was
not mentioned to him by the private respondent's
agent; and had it been so mentioned, he would not
have withheld such information. He further asserted
that the total of the amounts claimed under the
three policies was below the actual value of his
stocks at the time of loss, w/c was P1M.
- The Insurance Commission found that the
petitioner did not violate Condition 3 as he had no
knowledge of the existence of the two fire insurance
policies obtained from the PFIC; that it was Cebu
Tesing Textiles w/c procured the PFIC policies w/o
informing him or securing his consent; and that Cebu
Tesing Textile, as his creditor, had insurable interest
on the stocks. These findings were based on the
petitioner's testimony that he came to know of the
PFIC policies only when he filed his claim with the
private respondent and that Cebu Tesing Textile
obtained them and paid for their premiums w/o
informing him. The Insurance Commission then
ordered the respondent company to pay complainant
the sum of P100,000.00 with legal interest from the
time the complaint was filed until fully satisfied plus
the amount of P10,000.00 as attorney's fees.
-CA reversed the decision of the Insurance
Commission because it found that the petitioner
knew of the existence of the two other policies
issued by the PFIC

ISSUES
1. WON the petitioner had prior knowledge of the
two insurance policies issued by the PFIC when he
obtained the fire insurance policy from the private
respondent, thereby, for not disclosing such fact,
violating Condition 3 of the policy
2. if he had, WON he is precluded from recovering
therefrom

HELD
1. YES
- We agree w/ the CA that the petitioner knew of the
prior policies issued by the PFIC. His letter of 18
January 1991 to the private respondent conclusively
proves this knowledge. His testimony to the contrary
before the Insurance Commissioner and which the
latter relied upon cannot prevail over a written
admission made ante litem motam. It was, indeed,
incredible that he did not know about the prior
policies since these policies were not new or original.
2. NO
- It must, however, be underscored that unlike the
"other insurance" clauses involved in General
Insurance and Surety Corp. vs. Ng Hua or in
Pioneer Insurance & Surety Corp. vs. Yap, which
read:
"The insured shall give notice to the company of any
insurance or insurances already effected, or which
may subsequently be effected covering any of the
property hereby insured, and unless such notice be
given and the particulars of such insurance or
insurances be stated in or endorsed on this Policy by
or on behalf of the Company before the occurrence
of any loss or damage, all benefits under this Policy
shall be forfeited." or in the 1930 case of Santa Ana
vs. Commercial Union Assurance Co. which
provided "that any outstanding insurance upon the
whole or a portion of the objects thereby assured
must be declared by the insured in writing and he
must cause the company to add or insert it in the
policy, without which such policy shall be null and
void, and the insured will not be entitled to
indemnity in case of loss," Condition 3 in the
private respondent's policy No. F-14622 does
not absolutely declare void any violation
thereof. It expressly provides that the
condition "shall not apply when the total
insurance or insurances in force at the time of
the loss or damage is not more than
P200,000.00."
- Interpretation: It is a cardinal rule on insurance
that a policy or insurance contract is to be
interpreted liberally in favor of the insured and
strictly against the company, the reason being,
undoubtedly, to afford the greatest protection which
the insured was endeavoring to secure when he
applied for insurance. It is also a cardinal principle of
law that forfeitures are not favored and that any
construction which would result in the forfeiture of
the policy benefits for the person claiming
thereunder, will be avoided, if it is possible to
construe the policy in a manner which would permit
recovery, as, for example, by finding a waiver for
such forfeiture. Stated differently, provisions,
conditions or exceptions in policies which tend
to work a forfeiture of insurance policies
should be construed most strictly against those
for whose benefits they are inserted, and most
favorably toward those against whom they are
intended to operate. The reason for this is that,
except for riders which may later be inserted, the
insured sees the contract already in its final form and
has had no voice in the selection or arrangement of
the words employed therein. On the other hand, the
language of the contract was carefully chosen and
deliberated upon by experts and legal advisers who
had acted exclusively in the interest of the insurers
and the technical language employed therein is
rarely understood by ordinary laymen.
- With these principles in mind, we are of the
opinion that Condition 3 of the subject policy is
not totally free from ambiguity and must be
meticulously analyzed. Such analysis leads us
to conclude that (a) the prohibition applies only
to double insurance, and (b) the nullity of the
policy shall only be to the extent exceeding
P200,000.00 of the total policies obtained.
- Furthermore, by stating within Condition 3 itself
that such condition shall not apply if the total
insurance in force at the time of loss does not
exceed P200,000.00, the private respondent was
amenable to assume a co-insurer's liability up to a
loss not exceeding P200,000.00. What it had in mind
was to discourage over-insurance. Indeed, the
rationale behind the incorporation of "other
insurance" clause in fire policies is to prevent over-
!"#$%&"'( *+,- /7

insurance and thus avert the perpetration of fraud.
When a property owner obtains insurance policies
from two or more insurers in a total amount that
exceeds the property's value, the insured may have
an inducement to destroy the property for the
purpose of collecting the insurance. The public as
well as the insurer is interested in preventing a
situation in which a fire would be profitable to the
insured.
Disposition Petition granted. The decision of the
Court of Appeals in CA-G.R. SP No. 31916 is SET
ASIDE and the decision of the Insurance Commission
in Case No. 3340 is REINSTATED.

SUN INSURANCE OFFICE, LTD. V CA (LIM)
211 SCRA 554
CRUZ; July 17, 1992

NATURE
Petition for review from the decision of the Court of
Appeals

FACTS
- Felix Lim was issued a Personal Accident Policy
insurance with petitioner company with a face value
of P200,000. His beneficiary was his wife Nerissa.
- October 6, 1982 Felix accidentally shot himself in
the head with his own gun.
- He was playing with the handgun after he had
removed the guns magazine (kasi naman).
- He pointed the gun at his secretary and only
witness Pilar Nalagon as a joke and assured her
that the gun was not loaded (are you sure).
- He then put the gun to his temple and fired it
(haaay, sabi ko na nga ba).
- Both parties are in agreement that there was no
suicide.
- Nerissa claimed as Felixs beneficiary but Sun
Insurance would not grant her claim, saying that her
husbands death was not an accident.
- Nerissa sued Sun Insurance and won the case.
Sun Insurance was ordered to pay her P200,000
representing the face value of the claim along with
moral, exemplary and compensatory damages and
attorneys fees. The decision was affirmed by the
CA.
Petitioners Claim
- Sun Insurance cites one of the four exceptions in
the contract of insurance which includes bodily injury
consequent upon the insured person attempting to
commit suicide or willfully exposing himself to
needless peril in an attempt to save a human life.
- There mere act of pointing the gun to his temple
showed that Felix willfully exposed himself to danger
because a gun should always be handled with
caution.
Respondents Comments
- Felix believed the gun to be safe because he had
removed the magazine.
- He repeatedly assured his secretary that the gun
was not loaded.

ISSUES
1. WON Felix Lims death was an accident, thus
making his widow Nerissa liable to claim the accident
insurance
2. WON the award of damages to Nerissa Lim was
justified

HELD
1. YES, Felix Lims death was an accident.
Ratio There is no accident when a deliberate act is
performed unless some additional, unexpected,
independent and unforeseen happening occurs which
produces or brings bout their injury or death.
Reasoning
- An accident has been defined to be that which
happens by chance or fortuitously without intention
or design and which is unexpected, unusual and
unforeseen. It an event that takes pace without
ones foresight or expectastion an event that
proceeds from an unknown cause or is an unusual
effect of a known case and therefore not expected.
It happens without any human agency, an event
which, under the circumstances, is unusual to and
not expected by the person to whom it happens.
- The firing of the gun was deemed to be the
unexpected and independent and unforeseen
occurrence that led to the insured persons death.
- There was no willful exposure to needless peril for
the part of Felix. Suicide and exposure to needless
peril are similar in the sense that both signify
disregard for ones life. Suicide imparts a positive
act of ending ones life whereas the latter indicates
recklessness that is almost suicidal in intent.
- Accident insurance policies were never meant to
reward the insured for his tendency to show off or
for his miscalculations. They were intended to
provide for contingencies.
- Lim was unquestionably negligent but it should not
prevent his widow from recovering from the
insurance policy he obtained precisely against
accident.
- Insurance contracts are, as a rule, supposed to be
interpreted liberally in favor of the assured.
2. NO, the claim for damages should not be granted
for being unjust.
Ratio A person may be made liable to the payment
of moral damages if his act is wrongful. The adverse
result of an action does not per se make the act
wrongful and subject the act or to the payment of
moral damages.
Reasoning
- Petitioner was acting in good faith when it resisted
the private respondents claim on the ground that
the death of the insured was covered by the
exception.
- The issue was debatable and was clearly not raised
only for the purpose of evading a legitimate
obligation.

RIZAL SURETY & INSURANCE COMPANY V CA
(TRANSWORLD KNITTING MILLS, INC.)
336 SCRA 12
PURISIMA; July 18, 2000

NATURE
Petition for Review on Certiorari under Rule 45 of the
Rules of Court

FACTS
- Rizal Surety & Insurance Company (Rizal
Insurance) issued Fire Insurance Policy No. 45727 in
favor of Transworld Knitting Mills, Inc. (Transworld).
- Pertinent portions of subject policy on the buildings
insured, and location thereof, read:
"On stocks of finished and/or unfinished
products, raw materials and supplies of every
kind and description, the properties of the
Insureds and/or held by them in trust, on
commission or on joint account with others
and/or for which they (sic) responsible in case
of loss whilst contained and/or stored during
the currency of this Policy in the premises
occupied by them forming part of the
buildings situate (sic) within own Compound
at MAGDALO STREET, BARRIO UGONG,
PASIG, METRO MANILA, PHILIPPINES, BLOCK
NO. 601.
x
xx............
...xxx.......
........xxx
Said building of four-span lofty one storey in
height with mezzanine portions is constructed
!"#$%&"'( *+,- /.

of reinforced concrete and hollow blocks
and/or concrete under galvanized iron roof
and occupied as hosiery mills, garment and
lingerie factory, transistor-stereo assembly
plant, offices, warehouse and caretaker's
quarters.
'Bounds in front partly by one-storey concrete
building under galvanized iron roof occupied
as canteen and guardhouse, partly by building
of two and partly one storey constructed of
concrete below, timber above undergalvanized
iron roof occupied as garage and quarters and
partly by open space and/or tracking/
packing, beyond which is the aforementioned
Magdalo Street; on its right and left by
driveway, thence open spaces, and at the rear
by open spaces.'"
- The same pieces of property insured with the
petitioner were also insured with New India
Assurance Company, Ltd., (New India).
- Fire broke out in the compound of Transworld,
razing the middle portion of its four-span building
and partly gutting the left and right sections
thereof. A two-storey building (behind said four-
span building) where fun and amusement
machines and spare parts were stored, was also
destroyed by the fire.
- Transworld filed its insurance claims with Rizal
Surety & Insurance Company and New India
Assurance Company but to no avail.
- Private respondent brought against the said
insurance companies an action for collection of
sum of money and damages.
- Petitioner Rizal Insurance countered that its fire
insurance policy sued upon covered only the
contents of the four-span building, which was
partly burned, and not the damage caused by the
fire on the two-storey annex building.
- The trial court dismissed the case as against The
New India Assurance Co., Ltd. but ordered
defendant Rizal Surety And Insurance Company to
pay Transwrold (sic) Knitting Mills, Inc.
- Both the petitioner, Rizal Insurance Company,
and private respondent, Transworld Knitting Mills,
Inc., went to the Court of Appeals, which required
New India Assurance Company to pay plaintiff-
appellant the amount of P1,818,604.19 while the
Rizal Surety has to pay the plaintiff-appellant
P470,328.67.
- New India appealed to the Court theorizing inter
alia that the private respondent could not be
compensated for the loss of the fun and
amusement machines and spare parts stored at
the two-storey building because it (Transworld)
had no insurable interest in said goods or items.
- The Court denied the appeal with finality.
- Petitioner Rizal Insurance and private respondent
Transworld, interposed a Motion for
Reconsideration before the Court of Appeals, which
reconsidered its decision of July 15, 1993, as
regards the imposition of interest.
- Undaunted, petitioner Rizal Surety & Insurance
Company found its way to the Court.

ISSUE
WON the fire insurance policy litigated upon
protected only the contents of the main building
(four-span), and did not include those stored in the
two-storey annex building

HELD
NO
- Resolution of the issue posited hinges on the
proper interpretation of the stipulation in subject fire
insurance policy regarding its coverage, which reads:
"xxx contained and/or stored during the currency of
this Policy in the premises occupied by them forming
part of the buildings situate (sic) within own
Compound xxx"
- It can be gleaned unerringly that the fire insurance
policy in question did not limit its coverage to what
were stored in the four-span building. As opined by
the trial court of origin, two requirements must
concur in order that the said fun and amusement
machines and spare parts would be deemed
protected by the fire insurance policy under scrutiny,
to wit:
"First, said properties must be contained and/or
stored in the areas occupied by Transworld and
second, said areas must form part of the building
described in the policy xxx"
- Said building of four-span lofty one storey in height
with mezzanine portions is constructed of reinforced
concrete and hollow blocks and/or concrete under
galvanized iron roof and occupied as hosiery mills,
garment and lingerie factory, transistor-stereo
assembly plant, offices, ware house and caretaker's
quarter.
- The Court is mindful of the well-entrenched
doctrine that factual findings by the Court of Appeals
are conclusive on the parties and not reviewable by
this Court, and the same carry even more weight
when the Court of Appeals has affirmed the findings
of fact arrived at by the lower court.
- In the case under consideration, both the trial court
and the Court of Appeals found that the so called
"annex " was not an annex building but an integral
and inseparable part of the four-span building
described in the policy and consequently, the
machines and spare parts stored therein were
covered by the fire insurance in dispute.
- Verily, the two-storey building involved, a
permanent structure which adjoins and
intercommunicates with the "first right span of the
lofty storey building", formed part thereof, and
meets the requisites for compensability under the
fire insurance policy sued upon.
- So also, considering that the two-storey building
aforementioned was already existing when subject
fire insurance policy contract was entered into,
petitioner should have specifically excluded the said
two-storey building from the coverage of the fire
insurance if minded to exclude the same but if did
not, and instead, went on to provide that such fire
insurance policy covers the products, raw materials
and supplies stored within the premises of
respondent Transworld which was an integral part of
the four-span building occupied by Transworld,
knowing fully well the existence of such building
adjoining and intercommunicating with the right
section of the four-span building.
- Indeed, the stipulation as to the coverage of the
fire insurance policy under controversy has created a
doubt regarding the portions of the building insured
thereby. Article 1377 of the New Civil Code provides:
"Art.1377. The interpretation of obscure words
or stipulations in a contract shall not favor the
party who caused the obscurity"
- Conformably, it stands to reason that the doubt
should be resolved against the petitioner, Rizal
Surety Insurance Company, whose lawyer or
managers drafted the fire insurance policy contract
under scrutiny. Citing the aforecited provision of law
in point, the Court in Landicho vs. Government
Service Insurance System, ruled:
"This is particularly true as regards insurance
policies, in respect of which it is settled that the
'terms in an insurance policy, which are
ambiguous, equivocal, or uncertain x x x are to
be construed strictly and most strongly against
the insurer, and liberally in favor of the insured
so as to effect the dominant purpose of
indemnity or payment to the insured, especially
where forfeiture is involved' and the reason for
this is that the 'insured usually has no voice in
the selection or arrangement of the words
!"#$%&"'( *+,- //

employed and that the language of the contract
is selected with great care and deliberation by
experts and legal advisers employed by, and
acting exclusively in the interest of, the
insurance company.' "
- Equally relevant is the following disquisition of the
Court in Fieldmen's Insurance Company, Inc. vs.
Vda. De Songco, to wit:
"'This rigid application of the rule on ambiguities
has become necessary in view of current
business practices. The courts cannot ignore
that nowadays monopolies, cartels and
concentration of capital, endowed with
overwhelming economic power, manage to
impose upon parties dealing with them
cunningly prepared 'agreements' that the
weaker party may not change one whit, his
participation in the 'agreement' being reduced to
the alternative to 'take it or leave it' labelled
since Raymond Saleilles 'contracts by adherence'
(contrats [sic] d'adhesion), in contrast to these
entered into by parties bargaining on an equal
footing, such contracts (of which policies of
insurance and international bills of lading are
prime example) obviously call for greater
strictness and vigilance on the part of courts of
justice with a view to protecting the weaker
party from abuses and imposition, and prevent
their becoming traps for the unwary.'"
- The issue of whether or not Transworld has an
insurable interest in the fun and amusement
machines and spare parts, which entitles it to be
indemnified for the loss thereof, had been settled in
G.R. No. L-111118, entitled New India Assurance
Company, Ltd., vs. Court of Appeals, where the
appeal of New India from the decision of the Court of
Appeals under review, was denied with finality by
this Court on February 2, 1994.
- The rule on conclusiveness of judgment, which
obtains under the premises, precludes the relitigation
of a particular fact or issue in another action
between the same parties based on a different claim
or cause of action. "xxx the judgment in the prior
action operates as estoppel only as to those matters
in issue or points controverted, upon the
determination of which the finding or judgment was
rendered. In fine, the previous judgment is
conclusive in the second case, only as those matters
actually and directly controverted and determined
and not as to matters merely involved therein."
Disposition Decision, and the Resolution of the CA
WERE AFFIRMED in toto. No pronouncement as to
costs.

PAN MALAYAN INSURANCE CORPORATION vs.
COURT OF APPEALS (ERLINDA FABIE & HER
UNKNOWN DRIVER)
184 SCRA 55; G.R. No. 81026
CORTES; April 3, 1990

FACTS
- December 10, 1985: PANMALAY filed a complaint
for damages with the RTC of Makati against private
respondents Erlinda Fabie and her driver. PANMALAY
averred the following: that it insured a Mitsubishi
Colt Lancer car registered in the name of Canlubang
Automotive Resources Corporation [CANLUBANG];
that on May 26, 1985, due to the "carelessness,
recklessness, and imprudence" of the unknown
driver of a pick-up, the insured car was hit and
suffered damages in the amount of P42,052.00; that
PANMALAY defrayed the cost of repair of the insured
car and, therefore, was subrogated to the rights of
CANLUBANG against the driver of the pick-up and his
employer, Erlinda Fabie; and that, despite repeated
demands, defendants, failed and refused to pay the
claim of PANMALAY. PANMALAY clarified that the
damage caused to the insured car was settled under
the "own damage", coverage of the insurance policy.
- Private respondents filed a Motion to Dismiss
alleging that PANMALAY had no cause of action
against them. RTC dismissed PANMALAY's complaint
for no cause of action and denied PANMALAY's
motion for reconsideration. CA affirmed. Hence, this
petition for review.

ISSUE
WON the insurer PANMALAY may institute an action
to recover the amount it had paid its assured in
settlement of an insurance claim against private
respondents as the parties allegedly responsible for
the damage caused to the insured vehicle

HELD
YES
- Article 2207 of the Civil Code is founded on the
well-settled principle of subrogation. If the insured
property is destroyed or damaged through the fault
or negligence of a party other than the assured, then
the insurer, upon payment to the assured, will be
subrogated to the rights of the assured to recover
from the wrongdoer to the extent that the insurer
has been obligated to pay.
- General Rule: Payment by the insurer to the
assured operates as an equitable assignment to the
former of all remedies which the latter may have
against the third party whose negligence or wrongful
act caused the loss. The right of subrogation is not
dependent upon, nor does it grow out of, any privity
of contract or upon written assignment of claim. It
accrues simply upon payment of the insurance claim
by the insurer.
- Exceptions:
a.) if the assured by his own act releases the
wrongdoer or third party liable for the loss or
damage, from liability
b.) where the insurer pays the assured the value of
the lost goods without notifying the carrier who has
in good faith settled the assured's claim for loss
c.) where the insurer pays the assured for a loss
which is not a risk covered by the policy, thereby
effecting "voluntary payment"
- None of the exceptions are availing in the present
case.
- When PANMALAY utilized the phrase "own damage"
a phrase which, incidentally, is not found in the
insurance policy to define the basis for its
settlement of CANLUBANG's claim under the policy, it
simply meant that it had assumed to reimburse the
costs for repairing the damage to the insured
vehicle.
- It is a basic rule in the interpretation of contracts
that the terms of a contract are to be construed
according to the sense and meaning of the terms
which the parties thereto have used. In the case of
property insurance policies, the evident intention of
the contracting parties, i.e., the insurer and the
assured, determine the import of the various terms
and provisions embodied in the policy. It is only
when the terms of the policy are ambiguous,
equivocal or uncertain, such that the parties
themselves disagree about the meaning of particular
provisions, that the courts will intervene. In such an
event, the policy will be construed by the courts
liberally in favor of the assured and strictly against
the insurer.
- Considering that the very parties to the policy were
not shown to be in disagreement regarding the
meaning and coverage of Section III-1, specifically
sub-paragraph (a) thereof, it was improper for the
appellate court to indulge in contract construction, to
apply the ejusdem generis rule, and to ascribe
!"#$%&"'( *+,- /0

meaning contrary to the clear intention and
understanding of these parties.
- Although the terms "accident" or "accidental" as
used in insurance contracts have not acquired a
technical meaning, the Court has on several
occasions defined these terms to mean that which
takes place "without one's foresight or expectation,
an event that proceeds from an unknown cause, or is
an unusual effect of a known cause and, therefore,
not expected." The concept "accident" is not
necessarily synonymous with the concept of "no
fault". It may be utilized simply to distinguish
intentional or malicious acts from negligent or
careless acts of man.
- Obiter Dicta: Even if under the above
circumstances PANMALAY could not be deemed
subrogated to the rights of its assured under Article
2207 of the Civil Code, PANMALAY would still have a
cause of action against private respondents. The
insurer who may have no rights of subrogation due
to "voluntary" payment may nevertheless recover
from the third party responsible for the damage to
the insured property under Article 1236 of the Civil
Code.
Disposition Petition is GRANTED. Petitioner's
complaint for damages against private respondents
is REINSTATED. Case remanded to the lower court
for trial on the merits.

AMERICAN HOME ASSURANCE COMPANY V
TANTUCO ENTERPRISES
366 SCRA 740
PUNO; October 8, 2001

NATURE
Petition for Review on Certiorari assailing the
Decision of the Court of Appeals.

FACTS
- Respondent Tantuco Enterprises, Inc. is engaged in
the coconut oil milling and refining industry. It owns
two oil mills. Both are located at its factory
compound at Iyam, Lucena City. Respondent
commenced its business operations with only one oil
mill. In 1988, it started operating its second oil mill
( the new oil mill).
- The two oil mills were separately covered by fire
insurance policies issued by petitioner American
Home Assurance Co. The first oil mill was insured
Policy No. 306-7432324-3 for the period March 1,
1991 to 1992.The new oil mill was insured under
Policy No. 306-7432321-9 for the same term. Official
receipts indicating payment for the full amount of the
premium were issued by the petitioner's agent.
- Policy description:
Front: by a driveway thence at 18 meters distance
by Bldg. No. 2.
Right: by an open space thence by Bldg. No. 4.
Left: Adjoining thence an imperfect wall by Bldg.
No. 4.
Rear: by an open space thence at 8 meters
distance.
- A fire that broke out in the early morning of
September 30,1991 gutted and consumed the new
oil mill. Petitioner rejected respondents claim for
the insurance proceeds on the ground that no policy
was issued by it covering the burned oil mill. It
stated that the description of the insured
establishment referred to another building.
Petitioners Claim
The policies referred to the old mill, as stated in the
description contained in the policy.

ISSUE
WON new oil mill is insured by fire insurance policy

HELD
YES, new oil mill is insured.
Ratio In construing the words used descriptive of a
building insured, the greatest liberality is shown by
the courts in giving effect to the insurance. In view
of the custom of insurance agents to examine
buildings before writing policies upon them, and
since a mistake as to the identity and character of
the building is extremely unlikely, the courts are
inclined to consider that the policy of insurance
covers any building which the parties manifestly
intended to insure, however inaccurate the
description may be.
Reasoning
- The parties manifestly intended to insure the new
oil mill. On machineries and equipment with
complete accessories usual to a coconut oil mill
including stocks of copra, copra cake and copra mills
whilst contained in the new oil mill building, situate
(sic) at UNNO. ALONG NATIONAL HIGH WAY, BO.
IYAM, LUCENA CITY UNBLOCKED.
- If the parties really intended to protect the first oil
mill, then there is no need to specify it as new. It
would be absurd to assume that respondent would
protect its first oil mill for different amounts and
leave uncovered its second one.
- As may be gleaned from the testimony of the
petitioners employee, the source of the discrepancy
happened during the preparation of the written
contract.
- Respondent is not estopped from claiming that the
policy description is wrong. Evidence on record
reveals that respondents operating manager, Mr.
Edison Tantuco, notified the petitioners agent with
whom respondent negotiated for the contract about
the inaccurate description in the policy. However,
Mr. Borja assured Mr. Tantuco that the use of the
adjective new will distinguish the insured property.
- Regarding policy requirements that fire
extinguishment appliances should be available and in
good working condition, warranty did not require
respondent to provide for all the fire extinguishing
appliances enumerated therein. Neither did it
require that the appliances are restricted to those
mentioned in the warranty. (Within the vicinity of the
new oil mill can be found the following devices:
numerous portable fire extinguishers, two fire hoses,
fire hydrant, and an emergency fire engine.)
- The object of the court in construing a contract is
to ascertain the intent of the parties to the contract
and to enforce the agreement which the parties have
entered into. In determining what the parties
intended, the courts will read and construe the policy
as a whole and if possible, give effect to all the parts
of the contract.
Disposition Petition is dismissed.

PERLA COMPANIA DE SEGUROS, INC. v, CA
(MILAGROS CAYAS)
185 SCRA 741
FERNAN; May 28, 1990

NATURE
Petition for review on certiorari of a decision of the
Court of Appeals

FACTS
- Private respondent Milagros Cayas was the
registered owner of a Mazda bus, insured with Perla
Compania de Seguros, Inc. (PCSI) under a policy
issued on February 3, 1978.
- On December 17, 1978, the bus figured in an
accident in Naic, Cavite injuring several of its
passengers.
- One of them, 19-year old Edgardo Perea, sued
Milagros Cayas for damages in the CFI of Cavite,
while three others agreed to a settlement of
P4,000.00 each.
- After trial, the court rendered a decision in favor of
Perea, ordering Cayas to compensate him, with an
!"#$%&"'( *+,- /1

award of exemplary and moral damages, as well as
attorneys fees. ( P32,000 total)
- On November 11, 1981, Milagros Cayas filed a
complaint for a sum of money and damages against
PCSI in the Court of First Instance of Cavite.
- In view of Milagros Cayas' failure to prosecute the
case, the court motu propio ordered its dismissal
without prejudice.
- Alleging that she had not received a copy of the
answer to the complaint, and that "out of
sportsmanship", she did not file a motion to hold
PCSI in default, Milagros Cayas moved for the
reconsideration of the dismissal order. Said motion
for reconsideration was acted upon favorably by the
court.
- About two months later, Milagros Cayas filed a
motion to declare PCSI in default for its failure to file
an answer.
- The motion was granted and plaintiff was allowed
to adduce evidence ex-parte.
- On July 13, 1982, the court rendered judgment by
default ordering PCSI to pay Milagros Cayas P50,000
as compensation for the injured passengers, P5,000
as moral damages and P5,000 as attorney's fees.
- Said decision was set aside after the PCSI filed a
motion therefor. Trial of the case ensued.
- In due course, the court promulgated a decision
ordering defendant Perla Compania de Seguros, Inc.
to pay plaintiff Milagros Cayas the sum of
P50,000.00 under its maximum liability as provided
for in the insurance policy; and the sum of P5,000.00
as reasonable attorney's fee
- PCSI appealed to the Court of Appeals, which
affirmed in toto the lower court's decision.
- Its motion for reconsideration having been denied
by said appellate court, PCSI filed this petition

ISSUE
WON, as maintained by petitioner, its liability is
limited only to the payment made by private
respondent to Perea and only up to the amount of
P12,000.00

HELD
YES
- The insurance policy involved explicitly limits
petitioner's liability to P12,000.00 per person and to
P50,000.00 per accident.
- In Stokes vs. Malayan Insurance Co., Inc., the
Court held that the terms of the contract constitute
the measure of the insurer's liability and compliance
therewith is a condition precedent to the insured's
right of recovery from the insurer.
- In the case at bar, the insurance policy clearly and
categorically placed petitioner's liability for all
damages arising out of death or bodily injury
sustained by one person as a result of any one
accident at P12,000.00.
- Said amount complied with the minimum fixed by
the law then prevailing, Section 377 of Presidential
Decree No. 612, which provided that the liability of
land transportation vehicle operators for bodily
injuries sustained by a passenger arising out of the
use of their vehicles shall not be less than P12,000.
- In other words, under the law, the minimum
liability is P12,000 per passenger. Petitioner's liability
under the insurance contract not being less than
P12,000.00, and therefore not contrary to law,
morals, good customs, public order or public policy,
said stipulation must be upheld as effective, valid
and binding as between the parties.
- In like manner, we rule as valid and binding upon
private respondent the condition requiring her to
secure the written permission of petitioner before
effecting any payment in settlement of any claim
against her.
- There is nothing unreasonable, arbitrary or
objectionable in this stipulation as would warrant its
nullification. The same was obviously designed to
safeguard the insurer's interest against collusion
between the insured and the claimants.
- It being specifically required that petitioner's
written consent be first secured before any payment
in settlement of any claim could be made, private
respondent is precluded from seeking reimbursement
of the payments made to the three other passangers
in view of her failure to comply with the condition
contained in the insurance policy.
- Clearly, the fundamental principle that contracts
are respected as the law between the contracting
parties finds application in the present case.
- It was error on the part of the trial and appellate
courts to have disregarded the stipulations of the
parties and to have substituted their own
interpretation of the insurance policy.
- In Phil. American General Insurance Co., Inc vs.
Mutuc, we ruled that contracts which are the private
laws of the contracting parties should be fulfilled
according to the literal sense of their stipulations, if
their terms are clear and leave no room for doubt as
to the intention of the contracting parties, for
contracts are obligatory, no matter what form they
may be, whenever the essential requisites for their
validity are present.
- In Pacific Oxygen & Acetylene Co. vs. Central
Bank," it was stated that the first and fundamental
duty of the courts is the application of the law
according to its express terms, interpretation being
called for only when such literal application is
impossible.
- We observe that although Milagros Cayas was able
to prove a total loss of only P44,000.00, petitioner
was made liable for the amount of P50,000.00, the
maximum liability per accident stipulated in the
policy. This is patent error. An insurance indemnity,
being merely an assistance or restitution insofar as
can be fairly ascertained, cannot be availed of by any
accident victim or claimant as an instrument of
enrichment by reason of an accident.
Disposition Petition granted. The decision of the
Court of Appeals is modified in that petitioner shall
pay Milagros Cayas the amount of Twelve Thousand
Pesos (P12,000. 00) plus legal interest from the
promulgation of the decision of the lower court until
it is fully paid and attorney's fees in the amount of
P5,000.00.

POLTAN v. BPI & JOHN DOE
G.R. No. 164307
CHICO-NAZARIO; March 5, 2007

NATURE
Petition for review of CA decision

FACTS
- Petitioners POLTAN obtained a loan evidenced by a
promissory note from the MANTRADE Devt Corp.
This was secured by a chattel mortgage over a 1-unit
Nissan Sentra vehicle.
- With notice to petitioners, MANTRADE assigned to
BPI, by way of a Deed of Assignment, all its rights,
title and interest to the promissory note and chattel
mortgage.
- Petitioners defaulted and so BPI demanded the
whole balance of P286.5T including accrued interest,
or to return to BPI the possession of the motor
vehicle for foreclosure. It is specifically provided in
the promissory note and chattel mortgage that
failure to pay any installment when due shall make
subsequent installments and the entire balance of
the obligation due and payable.
- After they refused to do so, BPI then filed
complaint.
!"#$%&"'( *+,- /2

- Petitioners claimed that BPI required them to
obtain a motor vehicle insurance policy from FGU
Insurance Corporation (FGU Insurance). This is a
sister company of BPI. They had been paying the
monthly installments on the vehicle until it figured in
an accident where it became a total wreck. Under
the terms of the insurance policy from FGU
Insurance, the vehicle had to be replaced or its value
paid to them. Due to the failure and refusal of FGU
Insurance to replace the vehicle or pay its value,
they stopped payment of the monthly installments.
- RTC ordered POLTANS to pay BPI the said amount.
- CA reversed and remanded case to RTC for trial on
the merits.
- RTC again ruled in favor of BPI. CA affirmed.

ISSUES
1. WON contracts presented in evidence by BPI were
unjust and unacceptable contracts of adhesion
2. WON the terms and conditions of the
comprehensive car insurance policy issued by FGU
should be deemed as having automatically operated
in favor of BPI as the assured mortgagee, and if so,
it should be deemed as resulting in the
extinguishment of petitioners obligation

HELD
1. NO
Ratio A contract of adhesion is one in which one of
the parties imposes a ready-made form of contract,
which the other party may accept or reject, but
which the latter cannot modify. It is just as binding
as ordinary contracts.
Reasoning
- Petitioners failed to show that they were under
duress or forced to sign the loan documents. The
natural presumption is that one does not sign a
document without first informing himself of its
contents and consequences.
- Contracts of adhesion are not entirely prohibited
even as the courts remain careful in scrutinizing the
factual circumstances underlying each case to
determine the respective claims of contending
parties on their efficacy.
2. NO
Reasoning
- Petitioners failed to show any provision in the
insurance policy or mortgage contract providing that
the loss of the mortgaged vehicle extinguishes their
principal obligation to BPI.
- While it is true that the proceeds from the
insurance policy over the mortgaged chattel is for
the benefit of BPI, this will result in partial or full
satisfaction of the obligation only if the insurer pays
the mortgagee, BPI, or if the insurance proceeds
were paid to BPI. In this case, upon the loss of the
vehicle due to total wreck, the petitioners filed a
claim under the insurance policy, collected and
received the proceeds thereof, but did not settle
their obligation with BPI which remained outstanding
despite the loss of the vehicle.
Disposition CA decision AFFIRMED with the
modification that the interest rate be reduced to
12% per annum from 24 May 1994 until fully paid,
and the award of attorneys fees be reduced to P50T.

FILIPINO MERCHANTS INS. v. CA (CHOA TIEK
SENG)
179 SCRA 638
REGALADO; November 28, 1989

NATURE
Review of the decision of the CA

FACTS
- Plaintiff insured said shipment with defendant
insurance company under said cargo for the goods
described as 600 metric tons of fishmeal in new
gunny bags of 90 kilos each from Bangkok, Thailand
to Manila against all risks under warehouse to
warehouse terms.
- Some of the goods arrived in bad condition.
Plaintiff made a claim against Filipino Merchants
Insurance Company. The latter refused to pay.
Plaintiff brought an action against them. The
defendant insurance company presented a third
party complaint against the vessel and the arrastre
contractor.
- Judgment was rendered against the insurance
company. On the third party complaint, the third
party defendants were ordered to pay the third party
plaintiffs. The CA affirmed, but modified the same
with regard to the adjudication of the third-party
complaint

ISSUES
1. WON some fortuity, casualty or accidental cause is
needed to be proved despite the all risks policy (as
asserted by the insurance company)
2. WON the respondent has an insurable interest

HELD
1. NO
- The very nature of the term "all risks" must be
given a broad and comprehensive meaning as
covering any loss other than a willful and fraudulent
act of the insured.
7
This is pursuant to the very
purpose of an "all risks" insurance to give protection
to the insured in those cases where difficulties of
logical explanation or some mystery surround the
loss or damage to property.
- Generally, the burden of proof is upon the insured
to show that a loss arose from a covered peril, but
under an "all risks" policy the burden is not on the
insured to prove the precise cause of loss or damage
for which it seeks compensation. The insured under
an "all risks insurance policy" has the initial burden
of proving that the cargo was in good condition when
the policy attached and that the cargo was damaged
when unloaded from the vessel; thereafter, the
burden then shifts to the insurer to show the
exception to the coverage. As we held in Paris-Manila
Perfumery Co. vs. Phoenix Assurance Co., Ltd. the
basic rule is that the insurance company has the
burden of proving that the loss is caused by the risk
excepted and for want of such proof, the company is
liable. In the present case, there being no showing
that the loss was caused by any of the excepted
perils, the insurer is liable under the policy.
2. YES
- Section 13 of the Insurance Code defines insurable
interest in property as every interest in property,
whether real or personal, or any relation thereto, or
liability in respect thereof, of such nature that a
contemplated peril might directly damnify the
insured. In principle, anyone has an insurable
interest in property who derives a benefit from its
existence or would suffer loss from its destruction
whether he has or has not any title in, or lien upon
or possession of the property y.
16
Insurable interest
in property may consist in (a) an existing interest;
(b) an inchoate interest founded on an existing
interest; or (c) an expectancy, coupled with an
existing interest in that out of which the expectancy
arises.
- Respondents interest over the goods is based on
the perfected contract of sale. The perfected contract
of sale between him and the shipper of the goods
operates to vest in him an equitable title even before
delivery or before be performed the conditions of the
sale.
- Further, Article 1523 of the Civil Code provides that
where, in pursuance of a contract of sale, the seller
is authorized or required to send the goods to the
buyer, delivery of the goods to a carrier, whether
!"#$%&"'( *+,- /3

named by the buyer or not, for, the purpose of
transmission to the buyer is deemed to be a delivery
of the goods to the buyer, the exceptions to said rule
not obtaining in the present case. The Court has
heretofore ruled that the delivery of the goods on
board the carrying vessels partake of the nature of
actual delivery since, from that time, the foreign
buyers assumed the risks of loss of the goods and
paid the insurance premium covering them
- Moreover, the issue of lack of insurable interest
was not raised in petitioners answer.
Disposition Petition denied

GAISANO CAGAYAN v. INSURANCE Co. OF
NORTH AMERICA
490 SCRA 296
Austria-Martinez; June 8, 2006

NATURE
Petition for review on certiorari of the Decision of the
Court of Appeals

FACTS
- Intercapitol Marketing Corporation (IMC) is the
maker of Wrangler Blue Jeans. Levi Strauss (Phils.)
Inc. (LSPI) is the local distributor of products bearing
trademarks owned by Levi Strauss & Co.. IMC and
LSPI separately obtained from respondent fire
insurance policies with book debt endorsements.
The insurance policies provide for coverage on book
debts in connection with ready-made clothing
materials which have been sold or delivered to
various customers and dealers of the Insured
anywhere in the Philippines. The policies defined
book debts as the unpaid account still appearing in
the Book of Account of the Insured 45 days after the
time of the loss covered under this Policy. The
policies also provide for the following conditions:
1. Warranted that the Company shall not
be liable for any unpaid account in
respect of the merchandise sold and
delivered by the Insured which are
outstanding at the date of loss for a
period in excess of six (6) months from
the date of the covering invoice or
actual delivery of the merchandise
whichever shall first occur.
2. Warranted that the Insured shall submit to the
Company within twelve (12) days after the close of
every calendar month all amount shown in their
books of accounts as unpaid and thus become
receivable item from their customers and dealers. x
x x
- Petitioner is a customer and dealer of the products
of IMC and LSPI. On February 25, 1991, the Gaisano
Superstore Complex in Cagayan de Oro City, owned
by petitioner, was consumed by fire. Included in the
items lost or destroyed in the fire were stocks of
ready-made clothing materials sold and delivered by
IMC and LSPI. On February 4, 1992, respondent
filed a complaint for damages against petitioner. It
alleges that IMC and LSPI filed with respondent their
claims under their respective fire insurance policies
with book debt endorsements; that as of February
25, 1991, the unpaid accounts of petitioner on the
sale and delivery of ready-made clothing materials
with IMC was P2,119,205.00 while with LSPI it was
P535,613.00; that respondent paid the claims of IMC
and LSPI and, by virtue thereof, respondent was
subrogated to their rights against petitioner; that
respondent made several demands for payment
upon petitioner but these went unheeded. In its
Answer with Counter Claim dated July 4, 1995,
petitioner contends that it could not be held liable
because the property covered by the insurance
policies were destroyed due to fortuities event or
force majeure; that respondents right of subrogation
has no basis inasmuch as there was no breach of
contract committed by it since the loss was due to
fire which it could not prevent or foresee; that IMC
and LSPI never communicated to it that they insured
their properties; that it never consented to paying
the claim of the insured.
- At the pre-trial conference the parties failed to
arrive at an amicable settlement. Thus, trial on the
merits ensued. On August 31, 1998, the RTC
rendered its decision dismissing respondents
complaint. It held that the fire was purely accidental;
that the cause of the fire was not attributable to the
negligence of the petitioner; that it has not been
established that petitioner is the debtor of IMC and
LSPI; that since the sales invoices state that it is
further agreed that merely for purpose of securing
the payment of purchase price, the above-described
merchandise remains the property of the vendor
until the purchase price is fully paid, IMC and LSPI
retained ownership of the delivered goods and must
bear the loss. Dissatisfied, petitioner appealed to the
CA. On October 11, 2000, the CA rendered its
decision setting aside the decision of the RTC. The
CA held that the sales invoices are proofs of sale,
being detailed statements of the nature, quantity
and cost of the thing sold; that loss of the goods in
the fire must be borne by petitioner since the proviso
contained in the sales invoices is an exception under
Article 1504 (1) of the Civil Code, to the general rule
that if the thing is lost by a fortuitous event, the risk
is borne by the owner of the thing at the time the
loss under the principle of res perit domino; that
petitioners obligation to IMC and LSPI is not the
delivery of the lost goods but the payment of its
unpaid account and as such the obligation to pay is
not extinguished, even if the fire is considered a
fortuitous event; that by subrogation, the insurer has
the right to go against petitioner; that, being a fire
insurance with book debt endorsements, what was
insured was the vendors interest as a creditor.
Petitioner filed a motion for reconsideration but it
was denied by the CA in its Resolution dated April
11, 2001.

ISSUES
1. WON the CA erred in construing a fire insurance
policy on book debts as one covering the unpaid
accounts of IMC and LSPI since such insurance
applies to loss of the ready-made clothing materials
sold and delivered to petitioner.
2. WON IMC bears the risk of loss because it
expressly reserved ownership of the goods by
stipulating in the sales invoices that [i]t is further
agreed that merely for purpose of securing the
payment of the purchase price the above described
merchandise remains the property of the vendor
until the purchase price thereof is fully paid.
3. WON the petitioner liable for the unpaid accounts

HELD
1. NO
- It is well-settled that when the words of a contract
are plain and readily understood, there is no room
for construction. In this case, the questioned
insurance policies provide coverage for book debts
in connection with ready-made clothing materials
which have been sold or delivered to various
customers and dealers of the Insured anywhere in
the Philippines.; and defined book debts as the
unpaid account still appearing in the Book of
Account of the Insured 45 days after the time of the
loss covered under this Policy. Nowhere is it
provided in the questioned insurance policies that
the subject of the insurance is the goods sold and
delivered to the customers and dealers of the
insured.
- Indeed, when the terms of the agreement are clear
and explicit that they do not justify an attempt to
!"#$%&"'( *+,- /4

read into it any alleged intention of the parties, the
terms are to be understood literally just as they
appear on the face of the contract. Thus, what were
insured against were the accounts of IMC and LSPI
with petitioner which remained unpaid 45 days after
the loss through fire, and not the loss or destruction
of the goods delivered.
2. NO
- The present case clearly falls under paragraph (1),
Article 1504 of the Civil Code:
ART. 1504. Unless otherwise agreed, the goods
remain at the sellers risk until the ownership
therein is transferred to the buyer, but when the
ownership therein is transferred to the buyer the
goods are at the buyers risk whether actual
delivery has been made or not, except that:
(1) Where delivery of the goods has been made to
the buyer or to a bailee for the buyer, in
pursuance of the contract and the ownership in
the goods has been retained by the seller
merely to secure performance by the buyer of
his obligations under the contract, the goods
are at the buyers risk from the time of such
delivery; (Emphasis supplied)
- Thus, when the seller retains ownership only to
insure that the buyer will pay its debt, the risk of
loss is borne by the buyer. Accordingly, petitioner
bears the risk of loss of the goods delivered.
- IMC and LSPI did not lose complete interest over
the goods. They have an insurable interest until full
payment of the value of the delivered goods. Unlike
the civil law concept of res perit domino, where
ownership is the basis for consideration of who bears
the risk of loss, in property insurance, ones
interest is not determined by concept of title,
but whether insured has substantial economic
interest in the property.
- Section 13 of our Insurance Code defines insurable
interest as every interest in property, whether real
or personal, or any relation thereto, or liability in
respect thereof, of such nature that a contemplated
peril might directly damnify the insured.
Parenthetically, under Section 14 of the same Code,
an insurable interest in property may consist in: (a)
an existing interest; (b) an inchoate interest founded
on existing interest; or (c) an expectancy, coupled
with an existing interest in that out of which the
expectancy arises.
- Therefore, an insurable interest in property does
not necessarily imply a property interest in, or a lien
upon, or possession of, the subject matter of the
insurance, and neither the title nor a beneficial
interest is requisite to the existence of such an
interest, it is sufficient that the insured is so situated
with reference to the property that he would be
liable to loss should it be injured or destroyed by the
peril against which it is insured. Anyone has an
insurable interest in property who derives a benefit
from its existence or would suffer loss from its
destruction. Indeed, a vendor or seller retains an
insurable interest in the property sold so long as he
has any interest therein, in other words, so long as
he would suffer by its destruction, as where he has a
vendors lien. In this case, the insurable interest of
IMC and LSPI pertain to the unpaid accounts
appearing in their Books of Account 45 days after the
time of the loss covered by the policies.
3. YES
- Petitioners argument that it is not liable because
the fire is a fortuitous event under Article 1174 of
the Civil Code is misplaced. As held earlier,
petitioner bears the loss under Article 1504 (1) of
the Civil Code.
- Moreover, it must be stressed that the insurance in
this case is not for loss of goods by fire but for
petitioners accounts with IMC and LSPI that
remained unpaid 45 days after the fire. Accordingly,
petitioners obligation is for the payment of money.
Where the obligation consists in the payment of
money, the failure of the debtor to make the
payment even by reason of a fortuitous event shall
not relieve him of his liability. The rationale for this is
that the rule that an obligor should be held exempt
from liability when the loss occurs thru a fortuitous
event only holds true when the obligation consists in
the delivery of a determinate thing and there is no
stipulation holding him liable even in case of
fortuitous event. It does not apply when the
obligation is pecuniary in nature.
- Under Article 1263 of the Civil Code, [i]n an
obligation to deliver a generic thing, the loss or
destruction of anything of the same kind does not
extinguish the obligation. If the obligation is generic
in the sense that the object thereof is designated
merely by its class or genus without any particular
designation or physical segregation from all others of
the same class, the loss or destruction of anything of
the same kind even without the debtors fault and
before he has incurred in delay will not have the
effect of extinguishing the obligation. This rule is
based on the principle that the genus of a thing can
never perish. Genus nunquan perit. An obligation
to pay money is generic; therefore, it is not
excused by fortuitous loss of any specific
property of the debtor.
- Thus, whether fire is a fortuitous event or
petitioner was negligent are matters immaterial to
this case. What is relevant here is whether it has
been established that petitioner has outstanding
accounts with IMC and LSPI.
- With respect to IMC, the respondent has
adequately established its claim. Petitioner has an
outstanding account with IMC in the amount of
P2,119,205.00, check voucher evidencing payment
to IMC, subrogation receipt executed by IMC in favor
of respondent upon receipt of the insurance
proceeds. All these documents have been properly
identified, presented and marked as exhibits in
court. The subrogation receipt, by itself, is sufficient
to establish not only the relationship of respondent
as insurer and IMC as the insured, but also the
amount paid to settle the insurance claim. The right
of subrogation accrues simply upon payment by the
insurance company of the insurance claim.
Respondents action against petitioner is squarely
sanctioned by Article 2207 of the Civil Code which
provides:
Art. 2207. If the plaintiffs property has been
insured, and he has received indemnity from the
insurance company for the injury or loss arising
out of the wrong or breach of contract complained
of, the insurance company shall be subrogated to
the rights of the insured against the wrongdoer or
the person who has violated the contract. x x x
- Petitioner failed to refute respondents evidence.
- As to LSPI, respondent failed to present sufficient
evidence to prove its cause of action. No evidentiary
weight can be given to Exhibit F Levi Strauss, a
letter dated April 23, 1991 from petitioners General
Manager, Stephen S. Gaisano, Jr., since it is not an
admission of petitioners unpaid account with LSPI.
It only confirms the loss of Levis products in the
amount of P535,613.00 in the fire that razed
petitioners building on February 25, 1991.
- Moreover, there is no proof of full settlement of the
insurance claim of LSPI; no subrogation receipt was
offered in evidence. Thus, there is no evidence that
respondent has been subrogated to any right which
LSPI may have against petitioner. Failure to
substantiate the claim of subrogation is fatal to
petitioners case for recovery of the amount of
P535,613.00.
Disposition Petition is partly GRANTED. The
assailed Decision dated October 11, 2000 and
Resolution dated April 11, 2001 of the Court of
!"#$%&"'( *+,- /5

Appeals in CA-G.R. CV No. 61848 are AFFIRMED
with the MODIFICATION that the order to pay the
amount of P535,613.00 to respondent is DELETED
for lack of factual basis.

TAI TONG CHUACHE & CO v. INSURANCE
COMMISSION and TRAVELLERS MULTI-
INDEMNITY CORPORATION
158 SCRA 366
GANCAYCO; February 29, 1988

NATURE
Petition for review on certiorari of the decision of the
Insurance Commission

FACTS
- Complainants Palomo acquired a parcel of land and
a building located in Davao City. They assumed the
mortgage of the building in favor of SSS, which
building was insured with respondent SSS Accredited
Group of Insurers for P25K.
- On April 19, 1975, Azucena Palomo obtained a
P100K loan from Tai Tong Chuache Inc. (TTCC) and
executed a mortgage over the land and the building
in favor of Tai Tong Chuache & Co. as security of
payment .On April 25, 1975, Arsenio Chua,
representative of TTCC insured the latter's interest
with Travellers Multi-Indemnity Corporation
(Travellers) for P100K (P70K for bldg and P30K for
the contents thereof)
- On June 11, 1975, Pedro Palomo secured a Fire
Insurance Policy, covering the building for P50K with
respondent Zenith Insurance Corporation (ZIC).
Another Fire Insurance Policy was later procured
from respondent Philippine British Assurance
Company (PBAC), covering the same building for
P50K and contents thereof for P70K. On July 31,
1975, the building and the contents were totally
razed by fire.
- Based on the computation of the loss, including the
Travellers, respondents, ZIC, PBAC, and SSS paid
their corresponding shares of the loss. Complainants
were paid the following: P41,546.79 by PBAC,
P11,877.14 by ZIC, and P5,936.57 by SSS. Demand
was made from respondent Travellers for its share in
the loss but was refused. Hence, complainants
demanded from the other 3 respondents the balance
of each share in the loss based on the computation
excluding Travellers Multi-Indemnity in the amount
of P30,894.31 (P5,732.79-ZIC: P22,294.62, PBAC:
and P2,866.90, SSS) but was refused, hence, this
action.

ISSUE
WON petitioner Tai Tong has insurable interest in the
said policy

HELD
YES
- First, respondent insurance commission based its
findings on mere inference. Respondent Insurance
Commission absolved respondent insurance company
from liability on the basis of the certification issued
by the then CFI, that in a certain civil action against
the Palomos, Arsenio Lopez Chua stands as the
complainant and not Tai Tong Chuache. From said
evidence respondent commission inferred that the
credit extended by herein petitioner to the Palomos
secured by the insured property must have been
paid. Such is a glaring error which this Court cannot
sanction.
- Second, it has been held in a long line of cases that
when the creditor is in possession of the document of
credit, he need not prove non-payment for it is
presumed. The validity of the insurance policy taken
b petitioner was not assailed by private respondent.
Moreover, petitioner's claim that the loan extended
to the Palomos has not yet been paid was
corroborated by Azucena Palomo who testified that
they are still indebted to herein petitioner. So at the
time of the fire, petitioner as mortgagee still had
insurable interest therein.
- And third, petitioner's declaration that Arsenio
Lopez Chua acts as the managing partner of the
partnership was corroborated by respondent
insurance company. Thus Chua as the managing
partner of the partnership may execute all acts of
administration including the right to sue debtors of
the partnership in case of their failure to pay their
obligations when it became due and demandable. Or
at the least, Chua being a partner of petitioner Tai
Tong Chuache & Company is an agent of the
partnership. Being an agent, it is understood that he
acted for and in behalf of the firm.
Disposition Appealed decision SET ASIDE and
ANOTHER judgment is rendered order private
respondent Travellers to pay petitioner the face
value of Fire Insurance Policy in the amount of
P100K. Costs against said private respondent.


PEREZ v. CA (BF LIFEMAN INSURANCE CORP.)
323 SCRA 613
YNARES-SANTIAGO; January 28, 2000

NATURE
Petition for review on certiorari

FACTS
- Primitivo Perez has been insured with the BF
Lifeman Insurance Corporation (BF hereafter) since
1980 for Php20,000.
Sometime in 1987, Rodolfo Lalog (agent of BF)
convinced him to apply for additional insurance
coverage of Php50,000.
Perez accomplished the application form and passed
the required medical examination. He also paid
Php2,075 premium) to Lalog.
- On November 25, 1987, Perez died while riding a
banca which capsized during a storm. During this
time his application papers for the additional
insurance coverage was still with the Gumaca,
Quezon
office of BF.
- Without knowing that Perez died on November 25,
1987, BF approved Perez's application and issued the
corresponding policy for the Php50,000 on December
2, 1987.
- Virginia Perez (wife of the deceased) claimed the
benefits under the insurance policies of the
deceased, but she was only able to receive
Php40,000 under the first insurance policy.
BF refused to pay the proceeds amounting to
Php150,000 under the additional policy coverage of
Php50,000 because they maintain that such policy
had not been perfected.
- On September 21, 1990, BF filed a complaint
against Mrs. Perez seeking recission and declaration
of nullity of the insurance contract in question. Mrs.
Perez filed a conterclaim
for the collection of Php150,000 plus damages.

ISSUE
WON there was a consummated contract of
insurance between Perez and BF

HELD
NO
- An essential requisite of a valid contract is consent.
Consent must be manifested by the meeting of the
offer and the acceptance upon the thing and the
cause which are to constitute the contract.
- The offer must be certain and the acceptance
absolute. When Perez filed the application, it was
subject to the acceptance of BF. The perfection was
!"#$%&"'( *+,- /6

also further conditioned upon (1) the issuance of the
policy,
(2) the payment of the premium, and (3) the
delivery to and acceptance by the applicant in good
health.
- The delivery and acceptance by the applicant was a
suspensive condition which was not fulfilled
inasmuch as the applicant was already dead at the
time the policy was issued. The non-fulfillment of the
condition resulted
in the non-perfection of the contract.
- An application for insurance is merely an offer
which requires the overt act of the insurer for it to
ripen to a contract. Delay in acting on the application
does not constitute acceptance even though the
insured has forwarded
his first premium with his application. Delay, in this
case, does not constitute gross negligence because
the application was granted within the normal
processing time.
Disposition Decision of CA affirmed in so far as it
declared the insurance policy for Php50,000 issued
by BF null and void (no recission because it
presupposes a valid contract)

VDA. DE SINDAYEN v. INSULAR
62 Phil 51
BUTTE; September 4, 1935

FACTS
- Arturo Sindayen, up to the time of his death on
January 19, 1933, was employed as a linotype
operator in the Bureau of Printing at Manila and had
been such for eleven years prior thereto. While there
he made a written application on December 26,
1932, to the defendant Insular Life Assurance Co.,
Ltd., through its agent, Cristobal Mendoza, for a
policy of insurance on his life in the sum of P1,000
and he paid to the agent P15 cash as part of the first
premium. It was agreed with the agent that the
policy, when and if issued, should be delivered to his
aunt. Felicidad Estrada, with whom Sindayen left the
sum of P26.06 to complete the payment of the first
annual premium of P40.06.
- On January 1, 1933, Sindayen, who was then
twenty-nine years of age, was examined by the
company's doctor who made a favorable report, to
the company. On January 11, 1933, The company
accepted the risk and issued policy No. 47710 dated
back to December 1, 1932, and mailed the same to
its agent, Cristobal Mendoza, in Camiling, Tarlac, for
delivery to the insured. -On January 11, 1933,
Sindayen was at work in the Bureau of Printing. On
January 12, he complained of a severe headache and
remained at home. On January 15, he called a
physician who found that he was suffering from
acute nephritis and uremia and on January 19, 1933,
he died.
- On January 18, 1933, the agent, in accordance
with his agreement with the insured, delivered the
policy to Felicidad Estrada upon her payment of the
balance of the first year's annual premium. The
agent asked Felicidad Estrada if her nephew was in
good health and she replied that she believed so
because she had no information that he was sick and
he thereupon delivered to her the policy.
- On January 20, 1933, the agent learned of the
death of Arturo Sindayen and called on Felicidad
Estrada and asked her to return the policy. But he
did not return or offer to return the premium paid.
Felicidad Estrada on his aforesaid statement gave
him the policy.
- On February 4, 1933 Insular Life obtained from the
beneficiary, Sindayens wife, her signature to a legal
document entitled "ACCORD, SATISFACTION AND
RELEASE" whereby in consideration of the sum of
P40.06 paid to her by a check of the company, she
"assigns, releases and forever discharges said Isular
Life Assurance Co., Ltd., its successors and assigns,
of all claims, obligation in or indebtedness. The said
check for P40.06 was never cashed but returned to
the company and appears in the record of this case
as Exhibit D. Thereupon this action was brought to
enforce payment of the policy.
By the terms of the policy, an annual premium of
P40.06 is due on the first day of December of each
year, the first premium already paid by the insured
covering the period from December 1, 1932. It is to
December 1, 1933. It is to be noted that the policy
was not issued and the company assumed no actual
risk prior to January 11, 1933.The application which
the insured signed in Camiling, Tarlac, on December
26, 1932, contained among others the following
provisions:
3 That the said policy shall not take effect until the
first premium has been paid and the policy has been
delivered to and accepted by me, while I am in good
health.
-Main defense of the company in this case, namely,
that the said policy never took effect because of
paragraph 3 of the application above quoted, for at
the time of its delivery by the agent as aforesaid the
insured was not in good health

ISSUE
WON the insurance policy is valid

HELD
YES
- There is one line of cases which holds that the
stipulation contained in paragraph 3 is in the nature
of a condition precedent, that is to say, that there
can be no valid delivery to the insured unless he is in
good health at the time; that this condition
precedent goes to the very essence of the contract
and cannot be waived by the agent making delivery
of the policy
- On the other hand, a number of American decisions
hold that an agent to whom a life insurance policy
similar to the one here involved was sent with
instructions to deliver it to the insured has authority
to bind the company by making such delivery,
although the insured was not in good health at the
time of delivery, on the theory that the delivery of
the policy being the final act to the consummation of
the contract, the condition as to the insurer's good
health was waived by the company.
- we are inclined to the view that it is more
consonant with the well known practice of life
insurance companies and the evidence in the present
case to rest our decision on the proposition that
Mendoza was authorized by the company to make
the delivery of the policy when he received the
payment of the first premium and he was satisfied
that the insured was in good health. As was well said
in the case of MeLaurin vs. Mutual Life Insurance Co.
It is plain, therefore, that upon the facts it is not
necessarily a case of waiver or of estoppel, but a
case where the local agents, in the exercise of the
powers lodged in them, accepted the premium and
delivered the policy. That act binds their principal,
the defendant.
- Mendoza was duly licensed by the Insurance
Commissioner to act as the agent of the defendant
insurance company. The well known custom of the
insurance business and the evidence in this case
prove that Mendoza was not regarded by the
company as a mere conduit or automaton for the
performance of the physical act of placing the policy
in the hands of the insured
- Granted that Mendoza's decision that the condition
had been met by the insured and that it was proper
to make a delivery of the policy to him is just as
binding on the company as if the decision had been
made by its board of directors. Granted that
Mendoza made a mistake of judgement because he
!"#$%&"'( *+,- 07

acted on insufficient evidence as to the state of
health of the insured. But it is not charged that the
mistake was induced by any misconduct or omission
of duty of the insured.
- It is the interest not only the applicant but of all
insurance companies as well that there should be
some act which gives the applicant the definite
assurance that the contract has been consummated.
This sense of security and of peace of mind that
one's defendants are provided for without risk either
of loss or of litigation is the bedrock of life insurance.
When the policy is issued and delivered, in the
absence of fraud or other grounds for rescission, it is
plainly not within the intention of the parties that
there should be any questions held in abeyance or
reserved for future determination that leave the very
existence of the contract in suspense and doubt.
- It is therefore in the public interest, for the public is
profoundly and generally interested in life insurance,
as well as in the interest of the insurance companies
themselves by giving certainly and security to their
policies, that we are constrained to hold, as we, do,
that the delivery of the policy to the insured by an
agent of the company who is authorized to make
delivery or without delivery is the final act which
binds the company (and the insured as well) in the
absence of fraud or other legal ground for rescission
- The company therefore having decided that all the
conditions precedent to the taking effect of the policy
had been complied with and having accepted the
premium and delivered the policy thereafter to the
insured, the company is now estopped to assert that
it never intended that the policy should take effect.

SEPARATE OPINION

IMPERIAL [dissent]
- "A local agent of an insurance company, whose
only power is to solicit applications for insurance,
and forward them to the company for approval,
when, if approved to the insured, has no power to
waive any of the provision of the policy so
delivered."
- It is clear, therefore, that the delivery of the policy
by Mendoza does not bind the defendant, nor is the
defendant estopped from alleging its defense, for the
simple reason that Mendoza was not an agent with
authority to issue policies or to accept risks in the
name of his principle.
-There is another ground upon which the majority
opinion is based, namely, that the defendant waived
the defense it now invokes, by reason of the delivery
of the policy by its invokes, by reason of the delivery
of the policy by its agent. It is admitted that if the
delivery of the policy was due to fraud, legally there
could have been no waiver. In view of the facts
established and admitted, there is no doubt, as to
the existence of the fraud. -Estrada, as a
representative of the insured was not only bound to
give a truthful information on the state of health of
the insured, but it was her duty to find out it his true
state of health in order to give true and correct
information. When she gave Mendoza an incorrect
information tending to create the impression that the
insured was well when in fact he was seriously ill,
there is no doubt that she committed fraud and
imparted a deceitful information to the defendant
agent

ENRIQUEZ v. SUN LIFE OF CANADA
41 PHIL 269
MALCOLM; November 29, 1920

NATURE
Appeal from judgment of trial court denying
plaintiffs (administrator of the estate of the late
Joaquin Ma. Herrer) action to recover from the
defendant life insurance company the sum of pesos
6,000 paid by the deceased for a life annuity.

FACTS
- On September 24, 1917, Joaquin Herrer made
application to the Sun Life Assurance Company of
Canada through its office in Manila for a life annuity.
Two days later he paid the sum of P6,000 to the
manager of the company's Manila office and was
given a receipt.
- The application was immediately forwarded to the
head office of the company at Montreal, Canada. On
November 26, 1917, the head office gave notice of
acceptance by cable to Manila. (Whether on the
same day the cable was received, notice was sent by
the Manila office of Herrera that the application had
been accepted, is a disputed point, which will be
discussed later.) On December 4, 1917, the policy
was issued at Montreal. On December 18, 1917,
attorney Aurelio A. Torres wrote to the Manila office
of the company stating that Herrer desired to
withdraw his application. The following day the local
office replied to Mr. Torres, stating that the policy
had been issued, and called attention to the
notification of November 26, 1917. This letter was
received by Mr. Torres on the morning of December
21, 1917. Mr. Herrer died on December 20, 1917.
- The chief clerk of the Manila office of Sun Life
testified that he prepared the letter and handed it to
the local manager, Mr. E. E. White, for signature.
The local manager, Mr. White, testified to having
received the cablegram accepting the application of
Mr. Herrer from the home office on November 26,
1917. He said that on the same day he signed a
letter notifying Mr. Herrer of this acceptance. The
witness further said that letters, after being signed,
were sent to the chief clerk and placed on the
mailing desk for transmission. Mr. Tuason, who was
the chief clerk on November 26, 1917, was not
called as a witness.
- For the defense, attorney Manuel Torres testified to
having prepared the will of Joaquin Ma. Herrer. That
on this occasion, Mr. Herrer mentioned his
application for a life annuity, and that he said that
the only document relating to the transaction in his
possession was the provisional receipt. Rafael
Enriquez, the administrator of the estate, testified
that he had gone through the effects of the deceased
and had found no letter of notification from the
insurance company to Mr. Herrer.

ISSUE
WON there exists a contract for life annuity between
Herrer and defendant

HELD
NO
Ratio The law applicable to the case is found to be
the second paragraph of article 1262 of the Civil
Code providing that an acceptance made by letter
shall not bind the person making the offer except
from the time it came to his knowledge.
Reasoning
- Until quite recently, all of the provisions concerning
life insurance in the Philippines were found in the
Code of Commerce and the Civil Code. After July 1,
1915, there was, however, in force the Insurance
Act. No. 2427. Chapter IV of this Act concerns life
and health insurance. The Act expressly repealed
Title VIII of Book II and Section III of Title III of
Book III of the code of Commerce. The law of
insurance is consequently now found in the
Insurance Act and the Civil Code.
- While, as just noticed, the Insurance Act deals with
life insurance, it is silent as to the methods to be
followed in order that there may be a contract of
insurance. On the other hand, the Civil Code, in
article 1802, not only describes a contact of life
annuity markedly similar to the one we are
!"#$%&"'( *+,- 0.

considering, but in two other articles, gives strong
clues as to the proper disposition of the case. For
instance, article 16 of the Civil Code provides that
"In matters which are governed by special laws, any
deficiency of the latter shall be supplied by the
provisions of this Code." On the supposition,
therefore, which is incontestable, that the special law
on the subject of insurance is deficient in enunciating
the principles governing acceptance, the subject-
matter of the Civil code, if there be any, would be
controlling. In the Civil Code is found article 1262
providing that "Consent is shown by the concurrence
of offer and acceptance with respect to the thing and
the consideration which are to constitute the
contract. An acceptance made by letter shall not bind
the person making the offer except from the time it
came to his knowledge.
- According to the provisional receipt, three things
had to be accomplished by the insurance company
before there was a contract: (1) There had to be a
medical examination of the applicant; (2) there had
to be approval of the application by the head office
of the company; and (3) this approval had in some
way to be communicated by the company to the
applicant. The further admitted facts are that the
head office in Montreal did accept the application, did
cable the Manila office to that effect, did actually
issue the policy and did, through its agent in Manila,
actually write the letter of notification and place it in
the usual channels for transmission to the
addressee.
- The contract for a life annuity in the case at bar
was not perfected because it has not been proved
satisfactorily that the acceptance of the application
ever came to the knowledge of the applicant.
Disposition Judgment is reversed, and the plaintiff
shall have and recover from the defendant the sum
of P6,000 with legal interest from November 20,
1918, until paid, without special finding as to costs in
either instance.




VELASCO and ACOSTA v. APOSTOL and
MAHARLIKA INSURANCE CO., INC.
173 SCRA 228
REGALADO.; May 9, 1989

NATURE
Petition for review on certiorari

FACTS
- Petitioners were plaintiffs in a civil case of which
public respondent Hon. Apostol was the judge.
- The case was an offshoot of an incident: plaintiffs
were riding in their car, when a taxicab crossed a
center island in the road and collided with their car.
Private respondent Maharlika was eventually
impleaded as a defendant in this case, with an
allegation that the taxicab involved was insured
against third party liability for P20,000.00 with
private respondent at the time of the accident
- Maharlika claimed there was no cause of action
against it because at the time of the accident, the
alleged insurance policy was not in force due to the
non-payment of the premium thereon. Also, even if
the cab had been insured, the complaint would be
premature since the policy provides that the insurer
would be liable only when the insured becomes
legally liable.
- Trial court ruled in favor of the plaintiff, holding the
defendants liable for repair of the car, medical
expenses, etc. BUT Maharlike was exonerated on the
gnd that the policy was not in force.
- Petitioners elevated this case to this court, faulting
the respondent judge for considering the defense of
late payment of premium when the same was
waived at the pre-trial, hence the evidence of late
payment should be disregarded supposedly because
the private respondent had admitted that such fact
was not in issue.
- (More pertinent to this class: ) petitioners assert
that the private respondent had agreed to grant the
then prospective insured a credit extension of the
premium due.
- This controversy arose under the old insurance law,
Act No. 2427.
- The accident occurred in 1973. The complaint was
filed on July 20, 1974. both before the
effectivity of Presidential Decree no. 612, the
subsequent insurance law which repealed its
predecessor
- The former insurance law, which applies to the case
here, provided: An insurer is entitled to the payment
of premium as soon as the thing insured is exposed
to the peril insured against, unless there is clear
agreement to grant the insured credit extension of
the premium due. No policy issued by an insurance
company is valid and binding unless and until the
premium thereof has been paid.

ISSUE
WON the insurance policy would be valid and binding
notwithstanding the non-payment of the premium

HELD
NO
Ratio Act No. 2427: an insurance policy would be
valid and binding notwithstanding the non-payment
of the premium if there was a clear agreement to
grant to the insured credit extension. Such
agreement may be express or implied.
Reasoning
- Petitioners maintain that in spite of their late
payment, the policy is binding because there was an
implied agreement to grant a credit extension so as
to make the policy effective. To them, the
subsequent acceptance of the premium and delivery
of the policy estops the respondent company from
asserting that the policy is ineffective.
The court however sees no proof of any such implied
agreement. The purported nexus between the
delivery of the policy and the grant of credit
extension is too tenuous to support the conclusion
for which petitioners contend.
Parenthetically mention: in the present law,
Section 77 of the Insurance Code of 1978 has
deleted the clause "unless there is clear agreement
to grant the insured credit extension of the premium
due" which was then involved in this controversy.
Disposition Fnding no reversible error, the
judgment appealed from is hereby AFFIRMED.





TIBAY v. CA (FORTUNE LIFE & GENERAL
INSURANCE)
257 SCRA 126
BELLOSILLO; May 24, 1996

FACTS
- On 22 January 1987 Fortune Life and General
Insurance Co., Inc. (FORTUNE) issued Fire Insurance
Policy No. 136171 in favor of Violeta R. Tibay and/or
Nicolas Roraldo on their two-storey residential
building located at 5855 Zobel Street, Makati City,
together with all their personal effects therein. The
insurance was for P600,000 covering the period from
23 January 1987 to 23 January 1988. On 23 January
1987, of the total premium of P2,983.50, Violeta
Tibay only paid P600 thus leaving a considerable
balance unpaid.
!"#$%&"'( *+,- 0/

- On 8 March 1987 the insured building was
completely destroyed by fire. Two days later, Violeta
Tibay paid the balance of the premium. On the same
day, she filed with FORTUNE a claim on the fire
insurance policy. Her claim was accordingly referred
to its adjuster, Goodwill Adjustment Services, Inc.
(GASI), which immediately wrote Violeta requesting
her to furnish it with the necessary documents for
the investigation and processing of her claim.
Petitioner forthwith complied. On 28 March 1987 she
signed a nonwaiver agreement with GASI to the
effect that any action taken by the companies shall
not be, or be claimed to be, an admission of liability.
- FORTUNE denied the claim of Violeta for violation
of Policy Condition No. 2

and of Sec. 77 of the


Insurance Code. Efforts to settle the case before the
Insurance Commission proved futile. On 3 March
1988 Violeta and the other petitioners sued
FORTUNE for damages in the amount of P600,000
representing the total coverage of the fire insurance
policy plus 12% interest per annum, P100,000
moral damages, and attorney's fees equivalent to
20% of the total claim. The trial court ruled for
petitioners. CA reversed.

ISSUE
WON a fire insurance policy is valid, binding and
enforceable upon mere partial payment of premium

HELD
NO
Ratio Where the insurer and the insured expressly
stipulated that the policy is not in force until the
premium has been fully paid the payment of partial
premium by the assured in this particular instance
should not be considered the payment required by
the law and the stipulation of the parties. Rather, it
must be taken in the concept of a deposit to be held
in trust by the insurer until such time that the full
amount has been tendered and duly receipted for.
Reasoning
- As expressly agreed upon in the contract, full
payment must be made before the risk occurs for the
policy to be considered effective and in force. Thus,
no vinculum juris whereby the insurer bound itself to
indemnify the assured according to law ever resulted
from the fractional payment of premium. The
insurance contract itself expressly provided that the
policy would be effective only when the premium was
paid in full. It would have been altogether different

G702 3+50.< 0-.5:90-; /-< $1-1=/5 %71$1+6 /-9K+$ /-< 1-9+$21,1-% %71$1+- 02 -+% 0- 6+$.1 :-%05 %71
3$1,0:, 7/2 411- 6:55< 3/09 %+ /-9 9:5< $1.103%19 4< %71 *+,3/-< 0- %71 ,/--1$ 3$+F0919 71$10-&
were it not so stipulated. Ergo, petitioners had
absolute freedom of choice whether or not to be
insured by FORTUNE under the terms of its policy
and they freely opted to adhere thereto.
- Indeed, and far more importantly, the cardinal
polestar in the construction of an insurance contract
is the intention of the parties as expressed in the
policy. Courts have no other function but to enforce
the same. The rule that contracts of insurance will be
construed in favor of the insured and most strongly
against the insurer should not be permitted to have
the effect of making a plain agreement ambiguous
and then construe it in favor of the insured. Verily, it
is elemental law that the payment of premium is
requisite to keep the policy of insurance in force. If
the premium is not paid in the manner prescribed in
the policy as intended by the parties the policy is
ineffective. Partial payment even when accepted as a
partial payment will not keep the policy alive even
for such fractional part of the year as the part
payment bears to the whole payment.
Disposition Petition is DENIED. Decision of the CA
is AFFIRMED.



SEPARATE OPINION

VITUG [dissent]
- The law neither requires, nor measures the
strength of the vinculum juris by, any specific
amount of premium payment. It should thus be
enough that payment on the premium, partly or in
full, is made by the insured which the insurer
accepts. In fine, it is either that a juridical tie exists
(by such payment) or that it is not extant at all (by
an absence thereof). Once the juridical relation
comes into being, the full efficacy, not merely pro
tanto, of the insurance contract naturally follows.
Verily, not only is there an insurance perfected but
also a partially performed contract. In case of loss,
recovery on the basis of the full contract value, less
the unpaid premium can accordingly be had;
conversely, if no loss occurs, the insurer can demand
the payment of the unpaid balance of the premium.
The insured, on the one hand, cannot avoid the
obligation of paying the balance of the premium
while the insurer, upon the other hand, cannot treat
the contract as valid only for the purpose of
collecting premiums and as invalid for the purpose of
indemnity.

MAKATI TUSCANY v. CA ( AMERICAN HOME
ASSURANCE CO.)
215 SCRA 462
BELLOSILLO; November 6, 1992

NATURE
Appeal from decision of the CA

FACTS
- American Home Assurance Co. (AHAC),
represented by American International Underwriters
(Phils.), Inc., issued in favor of petitioner Makati
Tuscany Condominium Corporation an insurance
policy on the latter's building and premises, for the
period 1 March 1982 to1 March 1983. The premium
was paid on installments all of which were accepted
by AHAC.
- A second policy was issued to renew the first one,
this time covering the period 1 March 1983 to 1
March 1984. This was also pain in installment basis.
- A third policy was again issued for the period 1
March 1984 to 1 March 1985. For this, petitioner
made two installment payments, both accepted by
AHAC. Thereafter, petitioner refused to pay the
balance of the premium. AHAC filed an action to
recover the unpaid balance of P314,103.05.
- Petitioner explained that it discontinued the
payment of premiums because the policy did not
contain a credit clause in its favor and the receipts
for the installment payments covering the policy for
1984-85, as well as the two (2) previous policies,
stated the following reservations:
2. Acceptance of this payment shall not waive
any of the company rights to deny liability on any
claim under the policy arising before such
payments or after the expiration of the credit
clause of the policy; and
3. Subject to no loss prior to premium payment.
If there be any loss such is not covered.
- Petitioner further claimed that the policy was never
binding and valid, and no risk attached to the policy.
It then pleaded a counterclaim for P152k for the
premiums already paid for 1984-85, and in its
answer with amended counterclaim, sought the
refund of P924,206.10 representing the premium
payments for 1982-85.
- Trial court dismissed the complaint and the
counterclaim upon the following findings: (1)
payment of the premiums of the three policies were
made during the term of said policies, hence, it could
not be said, inspite of the reservations, that no risk
attached under the policies; (2) as regards the
!"#$%&"'( *+,- 00

unpaid premiums, in view of the reservation in the
receipts ordinarily issued by AHAC on premium
payments the only plausible conclusion is that AHAC
has no right to demand their payment after the lapse
of the term of said policy on March 1, 1985.
Therefore, Tuscany was justified in refusing to pay
the same.
- CA modified the decision by ordering Tuscany to
pay the balance of the premiums due on the third
policy plus legal interest until fully paid, and
affirming the denial of the counterclaim.
Petitioners Claims
Petitioner argues that where the premiums is not
actually paid in full, the policy would only be
effective if there is an acknowledgment in the policy
of the receipt of premium pursuant to Sec. 78 of the
Insurance Code. The absence of an express
acknowledgment in the policies of such receipt of the
corresponding premium payments, and petitioner's
failure to pay said premiums on or before the
effective dates of said policies rendered them invalid.
Petitioner thus concludes that there cannot be a
perfected contract of insurance upon mere partial
payment of the premiums because under Sec. 77 of
the Insurance Code, no contract of insurance is valid
and binding unless the premium thereof has been
paid, notwithstanding any agreement to the
contrary.

ISSUE
WON payment by installment of the premiums due
on an insurance policy invalidates the contract of
insurance

HELD
Ratio Where the risk is entire and the contract is
indivisible, the insured is not entitled to a refund of
the premiums paid if the insurer was exposed to the
risk insured for any period, however brief or
momentary.
Reasoning
- The obligation to pay premiums when due is
ordinarily as indivisible obligation to pay the entire
premium. Here, the parties herein agreed to make
the premiums payable in installments, and there is
no pretense that the parties never envisioned to
make the insurance contract binding between them.
And the insured never informed the insurer that it
was terminating the policy because the terms were
unacceptable.
- There is nothing in Section 77 which suggests that
the parties may not agree to allow payment of the
premiums in installment, or to consider the contract
as valid and binding upon payment of the first
premium.
- The records clearly show that petitioner and private
respondent intended subject insurance policies to be
binding and effective notwithstanding the staggered
payment of the premiums. Acceptance of payments
speaks loudly of the insurer's intention to honor the
policies it issued to petitioner.
- Section 78 of the Insurance Code in effect allows
waiver by the insurer of the condition of prepayment
by making an acknowledgment in the insurance
policy of receipt of premium as conclusive evidence
of payment so far as to make the policy binding
despite the fact that premium is actually unpaid.
Section 77 merely precludes the parties from
stipulating that the policy is valid even if premiums
are not paid, but does not expressly prohibit an
agreement granting credit extension, and such an
agreement is not contrary to morals, good customs,
public order or public policy.
- At the very least, both parties should be deemed in
estoppel to question the arrangement they have
voluntarily accepted.
Disposition Judgment affirmed. Costs against
petitioner.

SOUTH SEA SURETY AND INSURANCE v. CA
(VALENZUELA HARDWOOD)
244 SCRA 744
VITUG; June 2, 1995

NATURE
Petition for review on certiorari

FACTS
- Hardwood entered into agreement with Seven Bros
Shipping, where latter undertook to load the formers
logs on vessel. Hardwood insured the logs with
South Sea Surety which issued Marine Cargo
Insurance Policy. The vessel sank Jan 25, 1984.
- Hardwood filed claim with South Sea and Seven
Bros. Trial Court favored Hardwood. CA decided
against South Sea, but absolved Seven Bros. South
Sea filed this instant petition.

ISSUES
WON the insurance contract was already in effect
when the vessel sank

HELD
YES
- It is already in effect because Hardwood has
already paid the insurance premium.
It delivered the check to Victorio Chua before the
vessel sank, but Victorio Chua was only to deliver
the check to South Sea five days after the vessel
sank.
Appellant argues that Chua was not its broker, but it
was found that Chua was authorized by South Sea to
receive the premium on its behalf.


AREOLA v. CA (PRUDENTIAL GUARANTEE AND
ASSURANCE, INC.)
236 SCRA 643
ROMERO; September 22, 1994

NATURE CERTIORARI

FACTS
- June 29, 1985- 7 months after the issuance of
Santos Areola's Personal Accident Insurance Policy
No. PA-20015 (covering a period of one year),
Prudential unilaterally cancelled the same since
company records revealed that Areola failed to pay
his premiums.
! Under the terms of the statement of account
issued by Prudential, Areola was supposed to
pay the total amount of P1,609.65 which
included the premium of P1,470.00,
documentary stamp of P110.25 and 2%
premium tax of P29.40.
! The statement of account stated that it must not
be considered a receipt as an official receipt will
be issued upon payment of the account. And if
payment was made to a representative, the
client must demand for a Provisional Receipt and
if Official Receipts arent received within 7 days,
Prudential should be notified. If payment is
made to their office, clients should demand for
an OR.
- August 3, 1985- Prudential offered to reinstate
same policy it had previously cancelled and even
proposed to extend its lifetime to December 17,
1985, upon a finding that the cancellation was
erroneous and that the premiums were paid in full by
Areola but were not remitted by Teofilo M. Malapit,
Prudential's branch manager.
Petitioners Claims
- The fraudulent act of in misappropriating Areolas
premium payments is the proximate cause of the
cancellation of the insurance policy.
!"#$%&"'( *+,- 01

- Areola theorized that Malapit's act of signing and
even sending the notice of cancellation himself,
notwithstanding his personal knowledge of
petitioner-insured's full payment of premiums,
further reinforces the allegation of bad faith.
- Such fraudulent act committed by Malapit is
attributable to Prudential.
- Malapit's actuations are therefore not separate and
distinct from that of Prudentials. It must, therefore,
bear the consequences of the erroneous cancellation
of subject insurance policy caused by the non-
remittance by its own employee of the premiums
paid.
- Subsequent reinstatement could not possibly
absolve respondent insurance company from liability,
there being an obvious breach of contract. After all
damage had already been inflicted on him and no
amount of rectification could remedy the same.
Respondents Argument
- Prudential argues that where reinstatement, the
equitable relief sought by Areola was granted at an
opportune moment, i.e. prior to the filing of the
complaint, Areola is left without a cause of action on
which to predicate his claim for damages.
- Reinstatement effectively restored Areola to all his
rights under the policy.

ISSUES
1. WON the erroneous act of canceling subject
insurance policy entitle petitioner-insured to
payment of damages
2. WON the subsequent act of reinstating the
wrongfully cancelled insurance policy obliterate
whatever liability for damages Prudential has

HELD
1. YES
2. NO
Reasoning
- Malapit's fraudulent act of misappropriating the
premiums paid by petitioner-insured is beyond doubt
directly imputable to Prudential.
- A corporation, such as respondent insurance
company, acts solely thru its employees. The latters
acts are considered as its own for which it can be
held to account.
- The facts are clear as to the relationship between
private respondent insurance company and Malapit.
His act of receiving the premiums collected is well
within the province of his authority as manager.
Thus, his receipt of said premiums is receipt by
private respondent insurance company who, by
provision of law, particularly under Article 1910 of
the Civil Code, is bound by the acts of its agent.
- Article 1910 thus reads:
Art. 1910. The principal must comply with all the
obligations which the agent may have contracted
within the scope of his authority.
As for any obligation wherein the agent has
exceeded his power, the principal is not bound
except when he ratifies it expressly or tacitly.
- Malapit's failure to remit the premiums he received
cannot constitute a defense for private respondent
insurance company; no exoneration from liability
could result therefrom.
- Prudentials earlier act of reinstating the insurance
policy can not obliterate the injury inflicted on
petitioner-insured.
- Respondent company should be reminded that a
contract of insurance creates reciprocal obligations
for both insurer and insured.
- Reciprocal obligations are those which arise from
the same cause and in which each party is both a
debtor and a creditor of the other, such that the
obligation of one is dependent upon the obligation of
the other.
- Under the circumstances of instant case, the
relationship as creditor and debtor between the
parties arose from a common cause: i.e., by reason
of their agreement to enter into a contract of
insurance under whose terms, Prudential promised
to extend protection to Areola against the risk
insured for a consideration in the form of premiums
to be paid by the latter.
- Under the law governing reciprocal obligations,
particularly the second paragraph of Article 1191,
the injured party, Areola in this case, is given a
choice between fulfillment or rescission of the
obligation in case one of the obligors, such as
respondent insurance company, fails to comply with
what is incumbent upon him.
- However, said article entitles the injured party to
payment of damages, regardless of whether he
demands fulfillment or rescission of the obligation.
- Untenable then is reinstatement insurance
company's argument, namely, that reinstatement
being equivalent to fulfillment of its obligation,
divests petitioner-insured of a rightful claim for
payment of damages. Such a claim finds no support
in our laws on obligations and contracts.
DAMAGES:
- The nature of damages to be awarded, however,
would be in the form of nominal damages
- Although the erroneous cancellation of the
insurance policy constituted a breach of contract,
Prudential within a reasonable time took steps to
rectify the wrong committed by reinstating the
insurance policy of petitioner.
- Moreover, no actual or substantial damage or
injury was inflicted on petitioner Areola at the time
the insurance policy was cancelled.
- Nominal damages are "recoverable where a legal
right is technically violated and must be vindicated
against an invasion that has produced no actual
present loss of any kind, or where there has been a
breach of contract and no substantial injury or actual
damages whatsoever have been or can be shown.
Disposition Petition for review on certiorari is
hereby GRANTED. RTC s DECISION is REINSTATED.

UCPB GENERAL INSURANCE CO., INC. v.
MASAGANA TELAMART, INC.
308 SCRA 259
PARDO; June 15, 1999

NATURE
Petition for review on certiorari of a decision of the
Court of Appeals.

FACTS
- On April 15, 1991, petitioner issued five (5)
insurance policies covering respondent's various
property described therein against fire, for the period
from May 22, 1991 to May 22, 1992.
- In March 1992, petitioner evaluated the policies
and decided not to renew them upon expiration of
their terms on May 22, 1992. Petitioner advised
respondent's broker, Zuellig Insurance Brokers, Inc.
of its intention not to renew the policies.
- On April 6, 1992, petitioner gave written notice to
respondent of the non-renewal of the policies at the
address stated in the policies.
- On June 13, 1992, fire razed respondent's property
covered by three of the insurance policies petitioner
issued.
- On July 13, 1992, respondent presented to
petitioner's cashier at its head office five (5)
manager's checks in the total amount of
P225,753.95, representing premium for the renewal
of the policies from May 22, 1992 to May 22, 1993.
No notice of loss was filed by respondent under the
policies prior to July 14, 1992.
- On July 14, 1992, respondent filed with petitioner
its formal claim for indemnification of the insured
property razed by fire. On the same day, petitioner
!"#$%&"'( *+,- 02

returned to respondent the five manager's checks
that it tendered, and at the same time rejected
respondent's claim for the reasons (a) that the
policies had expired and were not renewed, and (b)
that the fire occurred on June 13, 1992, before
respondent's tender of premium payment.
- On July, 21, 1992, respondent filed with the
Regional Trial Court, Branch 58, Makati City, a civil
complaint against petitioner for recovery, of
P18.645,000.00, representing the face value of the
policies covering respondent's insured property razed
by fire, and for attorney's fees.
- On October 23, 1992, after its motion to dismiss
had been denied, petitioner filed an answer to the
complaint. It alleged that the complaint "fails to
state a cause of action"; that petitioner was not
liable to -respondent for insurance proceeds under
the policies because at the time of the loss of
respondent's property due to fire, the policies had
long expired and were not renewed.
After due trial, on March 10, 1993, the Regional Trial
Court, Branch 58, Makati, rendered decision, the
dispositive portion of which reads:
"WHEREFORE, premises considered, judgment is
hereby rendered in favor of the plaintiff and against
the defendant, as follows.
"(1) Authorizing and allowing the plaintiff to
consign/deposit with this Court the sum of
P225,753.95 (refused by the defendant) as full
payment of the corresponding premiums for the
replacement-renewal policies for Exhibits A, B, C, D
and E; "(2) Declaring plaintiff to have fully complied
with its obligation to pay the premium thereby
rendering the replacement-renewal policy of Exhibits
A, B, C, D and E effective and binding for the
duration May 22, 1992 until May 22, 1993; and,
ordering defendant to deliver forthwith to plaintiff
the said replacement-renewal policies; "(3) Declaring
Exhibits A & B, in force from August 22, 1991 up to
August 23, 1992 and August 9, 1991 to August 9,
1992, respectively; and "(4) Ordering the defendant
to pay plaintiff the sums of. (a) P18,645,000.00
representing the latter's claim for indemnity under
Exhibits A, B & C and/or its replacement-renewal
policies; (b) 25% of the total amount due as and for
attorney's fees; (c) P25,000.00 as necessary
litigation expenses; and, (d) the costs of suit.
xxx
- In due time, petitioner appealed to the Court of
Appeals (CA). The CA promulgated its decision
affirming that of the Regional Trial Court with the
modification that item No. 3 of the dispositive
portion was deleted, and the award of attorney's fees
was reduced to 10% of the total amount due.
It held that following previous practice, respondent
was allowed a 60- to 90-day credit term for the
renewal of its policies, and that the acceptance of the
late premium payment suggested an understanding
that payment could be made later. Hence, this
appeal.

ISSUE
WON the fire insurance policies issued by petitioner
to the respondent covering the period May 22, 1991
to May 22, 1992, had expired on the latter date or
had been extended or renewed by an implied credit
arrangement though actual payment of premium was
tendered on a later date after the occurrence of the
risk (fire) insured against

HELD
NO
- An insurance policy, other than life, issued
originally or on renewal, is not valid and binding until
actual payment of the premium. Any agreement to
the contrary is void. The parties may not agree
expressly or impliedly on the extension of credit or
time to pay the premium and consider the policy
binding before actual payment.
Disposition Judgment reversed and set aside










UCPB GENERAL INSURANCE CO., INC. v.
MASAGANA TELAMART, INC. (EN BANC)
356 SCRA 307
DAVIDE; April 4, 2001

NATURE
Motion for reconsideration of the decision of the
Supreme Court.

FACTS
- In its decision of 15 June 1999, the SC defined the
main issue to be whether the fire insurance policies
issued by petitioner to the respondent covering the
period from May 22, 1991 to May 22, 1992 had been
extended or renewed by an implied credit
arrangement though actual payment of premium was
tendered on a later date and after the occurrence of
the (fire) risk insured against. The Court resolved
this issue in the negative in view of Section 77 of the
Insurance Code and its decisions in Valenzuela v.
Court of Appeals; South Sea Surety and Insurance
Co., Inc. v. Court of Appeals; and Tibay v. Court of
Appeals. Accordingly, it reversed and set aside the
decision of the Court of Appeals.
- Respondent seasonably filed a motion for the
reconsideration of the adverse verdict. It alleges in
the motion that the SC had made in the decision its
own findings of facts, which are not in accord with
those of the trial court and the Court of Appeals.
The courts below correctly found that no notice of
non-renewal was made within 45 days before 22 May
1992, or before the expiration date of the fire
insurance policies. Thus, the policies in question
were renewed by operation of law and were effective
and valid on 30 June 1992 when the fire occurred,
since the premiums were paid within the 60- to 90-
day credit term.
- Respondent likewise disagrees with its ruling that
parties may neither agree expressly or impliedly on
the extension of credit or time to pay the premium
nor consider a policy binding before actual payment.
It urges the Court to take judicial notice of the fact
that despite the express provision of Section 77 of
the Insurance Code, extension of credit terms in
premium payment has been the prevalent practice in
the insurance industry. Most insurance companies,
including Petitioner, extend credit terms because
Section 77 of the Insurance Code is not a prohibitive
injunction but is merely designed for the protection
of the parties to an insurance contract. The Code
itself, in Section 78, authorizes the validity of a
policy notwithstanding non-payment of premiums.
- Respondent also asserts that the principle of
estoppel applies to Petitioner. Despite its awareness
of Section 77 Petitioner persuaded and induced
Respondent to believe that payment of premium on
the 60- to 90-day credit term was perfectly alright;
in fact it accepted payments within 60 to 90 days
after the due dates. By extending credit and
habitually accepting payments 60 to 90 days from
the effective dates of the policies, it has implicitly
agreed to modify the tenor of the insurance policy
and in effect waived the provision therein that it
would pay only for the loss or damage in case the
same occurred after payment of the premium.
- Petitioner filed an opposition to the Respondents
!"#$%&"'( *+,- 03

motion for reconsideration. It argues that both the
trial court and the Court of Appeals overlooked the
fact that on 6 April 1992 Petitioner sent by ordinary
mail to Respondent a notice of non-renewal and sent
by personal delivery a copy thereof to Respondents
broker, Zuellig. Both courts likewise ignored the fact
that Respondent was fully aware of the notice of
non-renewal. A reading of Section 66 of the
Insurance Code readily shows that in order for an
insured to be entitled to a renewal of a non-life
policy, payment of the premium due on the effective
date of renewal should first be made. Respondents
argument that Section 77 is not a prohibitive
provision finds no authoritative support.
- The following facts, as found by the trial court and
the Court of Appeals, are indeed duly established:
1. For years, Petitioner had been issuing fire
policies to the Respondent, and these policies
were annually renewed.
2. Petitioner had been granting Respondent a
60- to 90-day credit term within which to pay
the premiums on the renewed policies.
3. There was no valid notice of non-renewal of
the policies in question, as there is no proof at
all that the notice sent by ordinary mail was
received by Respondent, and the copy thereof
allegedly sent to Zuellig was ever transmitted
to Respondent.
4. The premiums for the policies in question in
the aggregate amount of P225,753.95 were
paid by Respondent within the 60- to 90-day
credit term and were duly accepted and
received by Petitioners cashier.

ISSUE
WON Sec. 77 of the Insurance Code of 1978 must be
strictly applied to Petitioners advantage despite its
practice of granting a 60- to 90-day credit term for
the payment of premiums
HELD
NO
- Section 77 of the Insurance Code of 1978 provides:
SEC. 77. An insurer is entitled to payment of the
premium as soon as the thing insured is exposed
to the peril insured against. Notwithstanding any
agreement to the contrary, no policy or contract of
insurance issued by an insurance company is valid
and binding unless and until the premium thereof
has been paid, except in the case of a life or an
industrial life policy whenever the grace period
provision applies.
- This Section is a reproduction of Section 77 of P.D.
No. 612 (The Insurance Code) promulgated on 18
December 1974. In turn, this Section has its source
in Section 72 of Act No. 2427 otherwise known as
the Insurance Act as amended by R.A. No. 3540,
approved on 21 June 1963, which read:
SEC. 72. An insurer is entitled to payment of
premium as soon as the thing insured is exposed
to the peril insured against, unless there is clear
agreement to grant the insured credit extension of
the premium due. No policy issued by an
insurance company is valid and binding unless and
until the premium thereof has been paid.
(Underscoring supplied)
- It can be seen at once that Section 77 does not
restate the portion of Section 72 expressly
permitting an agreement to extend the period to pay
the premium. But there are exceptions to Section
77.
The first exception is provided by Section 77 itself,
and that is, in case of a life or industrial life policy
whenever the grace period provision applies.
The second is that covered by Section 78 of the
Insurance Code, which provides:
SEC. 78. Any acknowledgment in a policy or
contract of insurance of the receipt of premium is
conclusive evidence of its payment, so far as to
make the policy binding, notwithstanding any
stipulation therein that it shall not be binding until
premium is actually paid.
- A third exception was laid down in Makati Tuscany
Condominium Corporation vs. Court of Appeals,
wherein we ruled that Section 77 may not apply if
the parties have agreed to the payment in
installments of the premium and partial payment has
been made at the time of loss. Tuscany has
provided a fourth exception to Section 77, namely,
that the insurer may grant credit extension for the
payment of the premium. This simply means that if
the insurer has granted the insured a credit term for
the payment of the premium and loss occurs before
the expiration of the term, recovery on the policy
should be allowed even though the premium is paid
after the loss but within the credit term.
Moreover, there is nothing in Section 77 which
prohibits the parties in an insurance contract to
provide a credit term within which to pay the
premiums. That agreement is not against the law,
morals, good customs, public order or public policy.
The agreement binds the parties. Article 1306 of the
Civil Code provides:
ART. 1306. The contracting parties may establish
such stipulations clauses, terms and conditions as
they may deem convenient, provided they are not
contrary to law, morals, good customs, public
order, or public policy.
- Finally, it would be unjust and inequitable if
recovery on the policy would not be permitted
against Petitioner, which had consistently granted a
60- to 90-day credit term for the payment of
premiums despite its full awareness of Section 77.
Estoppel bars it from taking refuge under said
Section since Respondent relied in good faith on such
practice. Estoppel then is the fifth exception to
Section 77.
Disposition Judgment reconsidered and set aside,
that of the Court of Appeals affirmed in toto.

SEPARATE OPINION

VITUG
- An essential characteristic of an insurance is its
being synallagmatic, a highly reciprocal contract
where the rights and obligations of the parties
correlate and mutually correspond.
- By weight of authority, estoppel cannot create a
contract of insurance, neither can it be successfully
invoked to create a primary liability, nor can it give
validity to what the law so procribes as a matter of
public policy.

PARDO [dissent]
- An assureds failure to give notice of the fire
immediately upon its occurrence blatantly showed
the fraudulent character of its claims. Respondent is
required by law and by express terms of the policy to
give immediate written notice of loss. This must be
complied with in the utmost good faith.
- Assuming arguendo that the 60- to 90-day credit
has been agreed between the parties, respondent
could not still invoke estoppel to back up its claim.
Estoppel cannot give validity to an act that is
prohibited by law or against public policy. The actual
payment of premiums is a condition precedent to the
validity of an insurance contract other than life
insurance policy. Any agreement to the contrary is
void as against law and public policy.

ACME SHOE RUBBER & PLASTIC CORP. v. CA
(DOMESTIC INSURANCE COMPANY OF THE
PHILS.)
134 SCRA 155
MELENCIO-HERRERA; January 17, 1985.

NATURE
!"#$%&"'( *+,- 04

Petition for Review on Certiorari of the Decision of
the then Court of Appeals (CA-G. R. No. 58917-R),
denying recovery on an insurance policy, thereby
reversing the judgment of the Court of First Instance
of Rizal, Branch XII, at Caloocan City, which had
allowed such recovery.

FACTS
- ACME Shoe Rubber and Plastic Corporation (ACME)
had been insuring yearly against fire its building,
machines and general merchandise with Domestic
Insurance Company (INSURER) since 1946. On May
14, 1962, ACME continued to insure its properties
with INSURER in the amount of P200,000 for the
period May 15, 1962 up to May 15, 1963.
- On May 14, 1963, INSURER issued Renewal Receipt
to cover the period May 15, 1963 to May 15, 1964.
- On January 8, 1964, ACME paid P3,331.26 as
premium. The INSURER applied the payment as
renewal premium for the period of May 15, 1963 to
May 15, 1964.
- On May 15, 1964, INSURER issued a Renewal
Receipt for the period of May 15, 1964 to May 15,
1965 (for renewal premium of P3,331.26 yet to be
paid) with a stamped note that says that the
insurance will be deemed valid and binding only
when the premium and documentary stamps have
actually been paid in full and duly acknowledged in
an official receipt. ACME was given 90 days to pay
otherwise the policy would automatically become
void and ineffective. (ACME should pay short period
premium for 90 days before the period expires. If
they are able to pay the whole amount before the
90-day period, the automatic termination wont
apply anymore).
- On May 26, 1964, ACME, through its President,
signed a promissory note saying that they promise to
pay the premium and documentary stamps and
agreed to the automatic cancellation penalty for not
complying.
- On October 13, 1964, ACMEs properties were
completely destroyed by fire. ACME filed insurance
claim but the INSURER disclaimed liability on the
ground that as of the date of loss, the properties
burned were not covered by insurance.
- ACME claims that the January 8, 1964 payment
was for the period 1964-1965 and that INSURER had
no right to apply it to the period 1963-1964 because
under RA 3540, the policy was void and INSURER
could have validly disclaimed liability for loss had one
occurred then.
- TC found INSURER liable for P200k and opined that
there was a clear intention on the INSURER's part to
grant ACME a credit extension for the payment of the
premium due; and that to allow the INSURER to
apply the premium ACME paid on January 8, 1964.
CA reversed TC and dismissed the suit on the ground
that, as of the moment of loss, ACME's properties
were not insured and the INSURER could not be held
liable for any indemnity as a result of the loss.

ISSUE
WON the premium payment for 1964-1965 was paid

HELD
NO
- Not having paid the 1964-1965 premium within the
extension granted, and pursuant to R.A. No. 3540,
the policy was automatically cancelled and there was
no insurance coverage to speak of as of the date of
the fire on October 13, 1964.
- The pertinent provision of Republic Act No. 3540
reads:
"Sec. 72. An insurer is entitled to payment of the
premium as soon as the thing insured is exposed
to the peril insured against, unless there is clear
agreement to grant the insured credit extension of
the premium due. No policy issued by an insurance
company is valid and binding unless and until the
premium thereof has been paid."
- RA 3540 was approved on June 20, 2963 and was
put into effect on Oct 1, 1963. It could not be
applied retroactively to the renewal of the policy for
the 1963-1964 period because said policy was
renewed on May 14, 1963. (Laws have no retroactive
effect unless the contrary is provided.) Therefore,
the Jan 8, 1964 payment was properly applied to the
1963-1964 premium. The Trial Court's opinion that
there was a clear agreement to grant ACME credit
extension for 1964-1965 is negated by ACME's
Promissory Note binding itself to pay within ninety
days from the effective date of this policy, 15th May,
1964. The credit extension was granted for 90 days
only. (So wala na by August 16, 1964.)
- If ACME was granted credit extensions in the past,
the promissory note it signed did away with such
credit arrangement. Also, before RA 3540, the
Renewal Receipts issued by INSURER did not contain
the auto-cancellation after 90 days note. By 1964,
however, the situation had changed by the passage
of the RA: no policy could be valid and binding
unless and until the premium thereof had been
paid.
- What became automatically cancelled by R.A. No.
3540 was the 1964-1965 policy for ACME's failure to
pay the premium within the 90-day extension
granted, and in accordance with the express terms of
the Promissory Note that it had signed.
Disposition The judgment under review is hereby
affirmed. Without pronouncement as to costs.

PEDRO ARCE v. THE CAPITAL INSURANCE &
SURETY CO., INC.
11 SCRA 63
ABAD SANTOS; September 30, 1982.

NATURE
Appeal from CFI decision on question of law.

FACTS
- Arce (INSURED) owned a residential house which
was insured with the appellant COMPANY since 1961.
In November 1965, the COMPANY sent to the
INSURED a Renewal Certificate to cover the period
from December 5, 1965 to December 5,1966, and
requested payment of the corresponding premium.
Anticipating that the premium could not be paid on
time, the INSURED asked for an extension which was
granted by the COMPANY. After the lapse of the
requested extension, INSURED still failed to pay the
premium. Thereafter, the house of the INSURED was
totally destroyed by fire. Upon INSURED's
presentation of claim for indemnity, he was told that
no indemnity was due because the premium was not
paid. The INSURED sued the COMPANY for
indemnity.
- The trial court held the COMPANY liable to
indemnify the INSURED on the ground that since the
COMPANY could have demanded payment of the
premium, mutuality of obligation required that it
should be liable on the policy.

ISSUE
WON the COMPANY can be held liable on its policy

HELD
NO.
- The Court commiserates with the INSURED. They
are well aware that many insurance companies have
fallen into the condemnable practice of collecting
premiums promptly but resort to all kinds of excuses
to deny or delay payment of just claims. Unhappily
the instant case is one where the insurer has the law
on its side.
!"#$%&"'( *+,- 05

- Sec. 72 of the Insurance Act, as amended by R.A.
No. 3540 reads:
"SEC. 72. An insurer is entitled to payment of
premium as soon as the thing insured is exposed
to the perils insured against, unless there is clear
agreement to grant credit extension for the
premium due. No policy issued by an insurance
company is valid and binding unless and until the
premium thereof has been paid."
- It is obvious from both the Insurance Act, as
amended, and the stipulation of the parties that time
is of the essence in respect of the payment of the
insurance premium so that if it is not paid the
contract does not take effect unless there is still
another stipulation to the contrary. In the instant
case, the INSURED was given a grace period to pay
the premium but the period having expired with no
payment made, he cannot insist that the COMPANY
is nonetheless obligated to him.
- Prior to the amendment (italicized portion above),
an insurance contract was effective even if the
premium had not been paid so that an insurer was
obligated to pay indemnity in case of loss and
correlatively he had also the right to sue for payment
of the premium. But the amendment to Sec. 72 has
radically changed the legal regime in that unless the
premium is paid there is no insurance.
Disposition The decision of the court a quo is
reversed; the appellee's complaint is dismissed. No
special pronouncement as to costs.
- Irrelevant facts: The premium costs P38.10. After
the fire, the COMPANY issued a check for P300 to
Arce as donation. Arce accepted the check, but still
sued the company.
CAPITAL INC. v. PLASTIC ERA CO.
65 SCRA 134
MARTIN; July 18, 1975

NATURE
Petition for review of a decision of the CA affirming
the decision of the CFI of Manila

FACTS
- On December 17, 1960, petitioner Capital
Insurance & Surety Co., Inc. delivered to the
respondent Plastic Era Manufacturing Co., Inc., its
open Fire Policy No. 22760 wherein the former
undertook to insure the latter's building, equipments,
raw materials, products and accessories located at
Sheridan Street, Mandaluyong, Rizal. The policy
expressly provides that if the property insured would
be destroyed or damaged by fire after the payment
of the premiums, at anytime between the 15th day
of December 1960 and one o'clock in the afternoon
of the 15th day of December 1961, the insurance
company shall make good all such loss or damage in
an amount not exceeding P100,000.00. When the
policy was delivered, Plastic Era failed to pay the
corresponding insurance premium. On January 8,
1961, in partial payment of the insurance premium,
Plastic Era delivered to Capital Insurance, a check for
the amount of P1,000.00 postdated January 16,
1961. However, Capital Insurance tried to deposit
the check only on February 20, 1961 and the same
was dishonored by the bank for lack of funds.
- Two days after the insurance premium became
due, at about 4:00 to 5:00 o'clock in the morning,
the property insured by Plastic Era was destroyed by
fire. In less than a month Plastic Era demanded from
Capital Insurance the payment of the sum of
P100,000.00 as indemnity for the loss of the insured
property under Policy No. 22760 but the latter
refused for the reason that, among others, Plastic
Era failed to pay the insurance premium.

ISSUES
1. WON a contract of insurance has been duly
perfected between petitioner and respondent
2. WON the dishonored check constituted payment

HELD
1. YES
- Tender of draft or check in order to effect payment
that would extinguish the debtor's liability should be
actually cashed. If the delivery of the check of Plastic
Era to Capital Insurance were to be viewed in the
light of the foregoing, no payment of the premium
had been effected. Significantly, Capital Insurance
accepted the promise of Plastic Era to pay the
insurance premium within 30 days from the effective
date of policy. By so doing, it has implicitly agreed to
modify the tenor of the insurance policy and in
effect, waived the provision therein that it would only
pay for the loss or damage in case the same occurs
after the payment of the premium. Considering that
the insurance policy is silent as to the mode of
payment, Capital Insurance is deemed to have
accepted the promissory note in payment of the
premium. This rendered the policy immediately
operative on the date it was delivered.
2. YES
- Although the check was due for payment on
January 16, 1961 and Plastic Era had sufficient funds
to cover it as of January 19, 1961, Capital Insurance
decided to hold the same for thirty-five (35) days
before presenting it for payment. Having held the
check for such an unreasonable period of time,
Capital Insurance was estopped from claiming a
forfeiture of its policy for non-payment even if the
check had been dishonored later. Where the check is
held for an unreasonable time before presenting it
for payment, the insurer may be held estopped from
claiming a forfeiture if the check is dishonored.
Disposition The decision of the CA is AFFIRMED in
toto.

MALAYAN INSURANCE CO., INC. v. ARNALDO
and PINCA
154 SCRA 672
CRUZ; October 12, 1987

FACTS
- On June 7, 1981, Malayan Insurance Co. (MICO),
issued fire insurance for the amount of P14,000 on
the property of private respondent, Pinca, effective
July 1981-1982. MICO later allegedly cancelled the
policy for non-payment of the premium and sent a
notice to Pinca. On Dec. 24 Adora, an agent of MICO,
received Pincas payment, which was remitted to
MICO. On Jan. 18, 1982, Pincas property was
completely burned. On Feb. 5, MICO returned Pincas
payment to Adora on the ground that her policy had
been cancelled; the latter refused to accept it. Her
demand for payment having been rejected by MICO,
Pinca went to the Insurance Commission. Public
respondent Arnaldo, the Insurance Commissioner,
sustained Pinca, hence this petition from MICO.
Records show MICO received Arnaldos decision on
April 10; MICO filed a MFR on April 25 which was
denied on June 4; MICO received notice of this denial
on June 14; instant petition was filed on July 2.

ISSUES
Procedural
1. WON the petition should be dismissed for late
filing
Substantive
2. WON there was a valid insurance contract at the
time of the loss
3. WON Adora was authorized to receive such
payment
4. WON an adjuster is indispensable in the
valuation of the loss

HELD
Procedural
!"#$%&"'( *+,- 06

1. YES
- Petitioner invokes Sec 416 of the Insurance Code
which grants it 30 days from notice of the Insurance
Commission within which to appeal by certiorari with
the Court. MICO filed its MFR on April 25, 15 days
after the notice; the reglementary period began to
run again after June 13. Since the petition was filed
only on July 2, it was tardy by 4 days. Alternatively it
invokes Rule 45 of the Rules of Court for certiorari
but the petition still exceeds the 15 day limit from
the June 13 notice.
-Respondents, on the other hand, invoke Sec. 39 of
B.P. 129 which pegs the period for appeal from
decisions of any court in all cases at 15 days from
the notice of the decision appealed from. Since the
MFR was filed only 15 days after receiving notice of
the decision, it was already 18 days late by July 2.
So whichever is applied, the petition is still late.
Substantive
2. YES
- A valid cancellation requires the following
conditions based on Sections 64-65 of the Code:
prior notice which must be based on the occurrence
of one or more of the grounds mentioned in Sec 64
(in this case, non-payment of premium), after the
effective date of the policy; the notice must be
written and mailed to the address on the policy; it
must state the ground(s) for cancellation and the
insurer must furnish details upon the request of the
insured.
- It is undisputed that payment of premium was
made. Petitioner relies heavily on Sec 77 of the
Insurance Code to contest this, the said provision
requiring payment of premium as soon as the thing
is exposed to the peril insured against and that the
policy is invalid without it. However, this is not
applicable in the instant case as payment was
eventually made. It is to be noted that the premium
invoice was stamped Payment Received, indicating
an understanding between the parties that payment
could be made later. This is furthered by the fact
that Adora had earlier told her to call him anytime
she was ready with her payment. The Court also
finds it strange that MICO only sought to return
Pincas Jan. 15 payment only on Feb. 5, long after
her house had burned downthis makes petitioners
motives highly suspect.
- MICO claims to have sent a notice to Pinca, who
flatly denied receiving one. Pinca did not have to
prove this since the strict language of Sec 64
requires that MICO ensure the cancellation was
actually sent to and received by the insured.
- MICO also suggests that Pinca knew the policy had
been cancelled and was paying the premium in order
to renew the policy. A close study of the transcripts
show, however, that Pinca only meant to renew the
policy had it been cancelled but not if it was still in
effectit was conditional. Payment was thus legally
made on the original transaction and validly received
by Adora, who was not informed of the alleged
cancellation and thus saw no reason to reject the
payment.
3. YES
- Sec. 306 of the Insurance Code provides that any
insurance company that delivers a policy to its agent
is deemed to have authorized such agent to receive
payment of premium on its behalf. It is a well-known
principle under the law of agency that payment to an
authorized agent is equivalent to payment to the
principal himself. MICOs acknowledgement of Adora
as its agent thus defeats its contention that he was
not authorized to receive payments on its behalf.
4. NO
- In absence of fraud, the amount of the loss may be
determined on the basis of such proof offered by the
insured. Here. The certification of the Integrated
National Police as the extent of the loss should
suffice.
Disposition petition is DENIED

MANUFACTURERS LIFE INSURANCE CO. v. MEER
89 PHIL 351
BENGZON, June 29, 1951

NATURE
APPEAL from a judgment of the Court of First
Instance of Manila

FACTS
(this is a tax case. Whats really important here is
the definition of CASH SURRENDER VALUE).
- Manufacturers Life Insurance Company is a duly
organized corporation which has its head office at
Toronto. It is duly registered and licensed to engage
in life insurance business in the Philippines, and,
maintains a branch office in Manila. It was engaged
in such business in the Philippines for more than five
years before and including the year 1941. But due to
the exigencies of the war It closed the branch office
at Manila during 1942 up to September 1945.
- Plaintiff issued a number of life insurance policies in
the Philippines containing stipulations referred to as
NONFORFEITURE CLAUSES
5

- From January 1, 1942 to December 31, 1946,
Plaintiff head office at Toronto applied the provisions
of the automatic premium loan clauses upon the
nonpayment of the corresponding premiums by the
people who subscribed to the insurance. The net
amount of premiums advanced (by the company) or
loaned (to the insured) as payment for the premium
due totaled P1,069,254.98.
- Meer, the Collector of the National Internal
Revenue assessed the net amount of premium at
P17,917.12 pursuant to SEC.255, National Internal
Revenue Code
6

- Company protested the assessment, but paid the
taxes anyway. Then they filed a complaint to recover
money paid under protest for taxes
- CFI: Dissmiss complaint
- PLAINTIFFs MAIN CONTENTION: when it made
premium loans or premium advances by virtue of the
non-forfeiture clauses, it did not collect premiums
within the meaning of the above sections of the law,
and therefore it is not amenable to the tax therein
provided.

ISSUES
1. WON premium advances made by plaintiff-
appellant under the automatic premium loan clause
of its policies are premiums collected' by the
Company subject to tax
2. WON, in the application of the automatic premium
loan clause of plaintiff-appellant's policies, there is
'payment in money, notes, credits, or any
substitutes for money

HONP& #:%+,/%0. Q$1,0:, R+/-&SG702 Q+50.< 27/55 -+% 5/321 6+$ -+-S3/<,1-% +6 /-< 3$1,0:, /6%1$ 0% 7/2 411- %7$11 6:55 <1/$2 0- 6+$.18 0%8 /% %71 9:1
9/%1 +6 2:.7 3$1,0:,8 %71 */27 T/5:1 +6 %702 Q+50.< /-9 +6 /-< 4+-:2 /990%0+-2 /-9 90F091-92 516% +- /..:,:5/%0+- ?/6%1$ 919:.%0-; /-<
0-914%19-122 %+ %71 .+,3/-< /-9 %71 0-%1$12% /..$:19 %71$1+-@ 27/55 1J.119 %71 /,+:-% +6 2/09 3$1,0:,& U- =70.7 1F1-% %71 .+,3/-< =0558 =0%7+:%
6:$%71$ $1B:12%8 %$1/% %71 3$1,0:, %71- 9:1 /2 3/098 /-9 %71 /,+:-% +6 2:.7 3$1,0:,8 =0%7 0-%1$12% 6$+, 0%2 /.%:/5 9:1 9/%1 /% 20J 31$ .1-% 31$
/--:,8 .+,3+:-919 <1/$5<8 /-9 +-1 31$ .1-%8 .+,3+:-919 <1/$5<8 6+$ 1J31-2128 27/55 41 / 60$2% 501- +- %702 Q+50.< 0- %71 *+,3/-<N2 6/F+:$ 0-
3$0+$0%< %+ %71 .5/0, +6 /-< /220;-11 +$ /-< +%71$ 31$2+-& G71 /..:,:5/%19 501- ,/< /% /-< %0,18 =7051 %71 Q+50.< 02 0- 6+$.18 41 3/09 0- =7+51 +$ 0-
3/$%&
NV71- %71 3$1,0:, 6/552 9:1 /-9 02 -+% 3/09 0- ./27 =0%70- %71 ,+-%7N2 ;$/.18 06 %71 */27 T/5:1 +6 %702 3+50.< /-9 +6 /-< 4+-:2 /990%0+-2 /-9
90F091-92 516% +- /..:,:5/%0+- ?/6%1$ 919:.%0-; /-< /..:,:5/%19 0-914%19-122@ 41 5122 %7/- %71 3$1,0:, %71- 9:18 %71 *+,3/-< =0558 =0%7+:% 6:$%71$
$1B:12%28 .+-%0-:1 %702 0-2:$/-.1 0- 6+$.1 6+$ / 31$0+9 W W W&
N!L& */27 /-9 Q/09SX3 U-2:$/-.1 T/5:12&S#% %71 1-9 +6 %71 %70$9 3+50.< <1/$ +$ %71$1/6%1$8 :3+- %71 51;/5 2:$$1-91$ +6 %702 Q+50.< %+ %71 *+,3/-<
=7051 %71$1 02 -+ 916/:5% 0- 3$1,0:, 3/<,1-%2 +$ =0%70- %=+ ,+-%72 /6%1$ %71 9:1 9/%1 +6 %71 3$1,0:, 0- 916/:5%8 %71 *+,3/-< =055 ?!@ ;$/-% / ./27
F/5:1 /2 231.06019 0- *+5:,- ?#@ 0-.$1/219 4< %71 ./27 F/5:1 +6 /-< 4+-:2 /990%0+-2 /-9 90F091-92 516% +- /..:,:5/%0+-8 =70.7 7/F1 411- /55+%19 %+
%702 Q+50.<8 5122 /55 0-914%19-122 %+ %71 *+,3/-< +- %702 Q+551< /- %71 9/%1 +6 +:.7 2:$$1-91$8 +$ ?E@ 1-9+$21 %702 Q+50.< /2 / Y+-SQ/$%0.03/%0-; Q/09S
:3 Q+551< 6+$ %71 /,+:-% /2 231.06019 U- *+5:,- ?Z@ +6 %71 G/451 +6 [:/$/-%119 T/5:12 W W W&
N!!& \J%1-919 U-2:$/-.1S#6%1$ %71 3$1,0:,2 6+$ %7$11 +$ ,+$1 6:55 <1/$2 7/F1 411- 3/09 71$1:-91$ 0- ./278 06 /-< 2:421B:1-% 3$1,0:, 02 -+% 3/09
=71- 9:18 /-9 %71$1 02 -+ 0-914%19-122 %+ %71 *+,3/-< +- %71 =$0%%1- $1B:12% +6 %71 0-2:$19 W W W&O
]O^\*& EHH& G/J12 +- 0-2:$/-.1 3$1,0:,2&SG71$1 27/55 41 .+551.%19 6$+, 1F1$< 31$2+-8 .+,3/-<8 +$ .+$3+$/%0+- ?1J.13% 3:$15< .++31$/%0F1
.+,3/-012 +$ /22+.0/%0+-2@ 9+0-; 0-2:$/-.1 4:20-122 +6 /-< 2+$% 0- %71 Q7050330-12 / %/J +6 +-1 31$ .1-%:, +6 %71 %+%/5 3$1,0:,2 .+551.%19 W W W
=71%71$ 2:.7 31$,0:,2 /$1 3/09 0- ,+-1<8 -+%128 .$190%28 +$ /-< 2:42%0%:%1 6+$ ,+-1< 4:% 3$1,0:,2 $16:-919 =0%70- 20J ,+-%72 /6%1$ 3/<,1-% +-
/..+:-% +6 $1_1.%0+- +6 $02C +$ $1%:$-19 6+$ +%71$ $1/2+- %+ 31$2+- 0-2:$19 27/55 -+% 41 0-.5:919 0- %71 %/J/451 $1.103%2 W W W&O
!"#$%&"'( *+,- 17

3. WON the collection of the alleged deficiency
premium taxes constitutes double taxation
4. WON the making of premium advances, granting
for the sake of argument that it amounted to
collection of premiums, were done in Toronto,
Canada
5. WON the fact that plaintiff-appellant was not
doing business in the Philippines during the period
from January 1, 1942 to September 30, 1945,
inclusive, exempts it from payment of premium
taxes corresponding to said period


HELD
NOTE (example given by the plaintiff):
"Suppose that 'A', 30 years of age, secures a 20-
year endowment policy for P5,000 from plaintiff-
appellant Company and pays an annual premium of
P250. 'A' pays the first ten yearly premiums
amounting to P2,500 and on this amount plaintiff-
appellant pays the corresponding taxes under section
255 of the National Internal Revenue Code. Suppose
also that the cash value of said policy after the
payment of the 10th annual premium amounts to
P1,000." When on the eleventh year the annual
premium fell due and the insured remitted no money
within the mouth grace, the insurer treated the
premium then over due as paid from the cash value,
the amount being a loan to the policyholder1
who could discharge it at any time with interest at 6
per cent. The insurance contract, therefore,
continued in force for the eleventh year.
1. YES
- Based on the example given by the plaintiff, the
insurer collected the amount of P250 as the annual
premium for the eleventh year on the said policy
when it loaned to A the sum of P250. The insurer
became a creditor of the loan, but not of the
premium that had already been paid (advanced by
the insurer). The insurer is entitled to collect interest
on the loan, not on the premium. "A" paid the
premium for the eleventh year; but in turn he
became a debtor of the company for the sum of
P250. This debt he could repay either by later
remitting the money to the insurer or by letting the
cash value compensate for it. The debt may also be
deducted from the amount of the policy should "A"
die thereafter during the continuance of the policy.
- ON ARGUMENT THAT THE ASSETS OF THE INSURER
REMAINED THE SAME AFTER THE APPLICATION OF THE
AUTOMATIC PREMIUM LOAN CLAUSE: there was an
increase in assets in the form of CREDIT for the
advances made (in the example, the P250 for the
11
th
year).
- ON ARGUMENT THAT IF THE CREDIT IS PAID OUT OF THE
CASH SURRENDER VALUE, THERE WERE NO NEW FUNDS ADDED
TO THE COMPANY'S ASSETS: Cash surrender value "as
applied to a life insurance policy, is the amount of
money the company agrees to pay to the holder of
the policy if he surrenders it and releases his claims
upon it. The more premiums the insured has paid the
greater will be the surrender value; but the
surrender value is always a lesser sum than the total
amount of premiums paid." (Cyclopedia Law
Dictionary 3d. ed. 1077.) The cash value or cash
surrender value is therefore an amount which the
insurance company holds In trust for the insured to
be delivered to him upon demand. It is therefore a
liability of the company to the insured. Now then,
when the company's credit for advances is paid out
of the cash value or cash surrender value, that value
and the company's liability is thereby diminished pro
tanto.
2. YES
- the insurer agreed to consider the premium paid on
the strength of the automatic loan. The premium was
therefore paid by means of a "note" or "credit" or
"other substitute for money" and the tax is due
because section 255 above quoted levies taxes
according to the total premiums collected by the
insurer "whether such premiums are paid in money,
notes, credits or any substitute for money.
3. NO
- No constitutional prohibition against double
taxation.
4. NO
- The loans are made to policyholders in the
Philippines, who in turn pay therewith the premium
to the insurer thru the Manila branch. Approval of
appellant's position will enable foreign insurers to
evade the tax by contriving to require that premium
payments shall be made at their head offices. What
is important, the law does not contemplate
premiums collected in the Philippines. It is enough
that the insurer is doing insurance business in the
Philippines, irrespective of the place of its
organization or establishment.
5. NO
- Although during those years the appellant was not
open for new business because its branch office was
closed, still it was practically and legally, operating in
this country by collecting premiums on its
outstanding policies, incurring the risks and/or
enjoying the benefits consequent thereto, without
having previously taken any steps indicating
withdrawal in good faith from this field of economic
activity.
Disposition finding no prejudicial error in the
appealed decision, we hereby affirm it with costs.








ANDRES v. CROWN LIFE INSURANCE
102 Phil. 919
REYES, J.B.L., Jan.28, 1958

NATURE
Appeal from judgment of CFI

FACTS
- Feb. 13, 1950: For the sum of P5,000, defendant-
appellee Crown Life issued an insurance policy in the
name of plaintiff-appellant Rufino and his wife, with
the stipulation that the premiums are to be paid
semi-annually.
- The premiums for the 1
st
and 2
nd
semester of the
1
st
year, in the amount of P165.15 were paid by
Rufino but the premium for the third semester, in
the same amount, was not paid.
- Jan. 6, 1951, Crown Life, through its branch
secretary, wrote to Mr. and Mrs. Andres advising
them that their insurance policy lapsed on Dec. 26,
1950 and the amount of P165.15 was overdue,
giving them 60 days from the date of lapse to file an
application for reinstatement. Crown Life later sent
another letter telling the spouses Andres that their
insurance policy was no longer in force.
- Feb. 1951: Plaintiff and his wife executed a
Statement of Health and application for
reinstatement of the aforesaid policy.
- Feb. 20, 1951: Plaintiff wrote a letter to the
defendant, enclosed with a money order for P100.
Upon acceptance, defendant advised Rufino that its
main office had approved the application and that
the reinstatement of the lapsed policy was subject to
the payment of the remaining premium balance of
P65.15.
- May 3, 1951: Severa Andres died of dystocia,
contracted pelvis.
!"#$%&"'( *+,- 1.

- May 5, 1951: Plaintiff sent a letter enclosed with a
money order in the amount of P65, for the remaining
balance due.
- May 15, 1951: Defendant sent a letter with official
receipt of the P165.15 paid by Rufino as well as a
Certificate of Reinstatement.
- June 7, 1951: Rufino presented a death claim as
survivor-beneficiary of his deceased wife. Payment
was denied by the defendant.
- April 1952: Rufino filed a complaint in CFI against
Crown Life for the recovery of the amount of P5,000
as the face value of a joint 20-year endowment
insurance policy issued by defendant in favor of
plaintiff and his wife, on Feb. 13, 1950. In its
answer, Crown Life disclaimed liability and set forth
the special defense that the aforementioned policy
had already lapsed.
- Aug. 5, 1954: CFI rendered a decision absolving
the defendant company from any liability on the
ground that the policy had lapsed and it was not
reinstated at the time of the plaintiffs wifes death.
Plaintiff later appealed to the CA but the same was
certified by the CA to the SC for having no question
of fact.

ISSUE
WON the insurance policy, which has been in a state
of lapse before May 3, 1951, has been validly and
completely reinstated after said date (Was there a
perfected contract of reinstatement after the policy
lapsed due to non-payment of premiums?)

HELD
NO
Ratio The stipulation in a life insurance policy giving
the insured the privilege to reinstate it upon written
application does not give the insured absolute right
to such reinstatement by the mere filing of an
application. The Company has the right to deny the
reinstatement if it is not satisfied as to the
insurability of the insured and if the latter does not
pay all overdue premiums and all other indebtedness
to the Company. After the death of the insured the
insurance Company cannot be compelled to entertain
an application for reinstatement of the policy
because the conditions precedent to reinstatement
can no longer be determined and satisfied.
Reasoning
- The stipulations of facts render it undisputable that
the original policy lapsed for non-payment of
premiums on Dec. 26, 1950, upon expiration of the
31-day grace period.
- As found by the lower court, the conditions set
forth in the policy for reinstatement as provided in
the contract itself are the following: (A) application
shall be made within 3 years from the date of lapse;
(B) there should be a production of evidence of the
good health of the insured; (C) if the rate of
premium depends upon the age of the Beneficiary,
there should likewise be a production of evidence of
his or her good health; (D) there should be
presented such other evidence of insurability at the
date of application for reinstatement; (E) there
should be no change which has taken place in such
good health and insurability subsequent to the date
of such application and before the policy is
reinstated; and (F) all overdue premiums and other
indebtedness in respect of the policy, together with
interest at 6%, compounded annually, should first be
paid.
- The plaintiff did not comply with the last condition;
for he only paid P100 before his wifes death; and
despite the Companys reminders, he only remitted
the balance of P65.15 two days after his wife died.
On the face of such facts, the Company had the right
to treat the contract as lapsed and refuse payment
of the policy.
- Rufino contends that the condition regarding
payment of the premium was waived by the
insurance Company through its letters, wherein it
made statements such as: If you are unable to pay
the full amount immediately, send as large amount
as possible and advise us how soon you expect to be
able to pay the balance; we will work out an
adjustment most beneficial to you. The Court found
the statements to be too vague and indefinite to
indicate an intention on the insurers part to waive
the full payment as prerequisite to the reinstatement
of the lapsed policy. The Court reiterated the rule
that a waiver must be clear and positive, the intent
to waive shown clearly and convincingly. On the
other hand, It found subsequent letters sent by
defendant indicating that they insisted on full
payment of the premium before the policy was
reinstated and that defendant did not consider partial
payment as sufficient consideration for the
reinstatement. Plaintiff-Appellants failure to remit
the balance before the death of his wife operated to
deprive him of any right to waive the policy and
recover the face value thereof.
Disposition Judgment appealed from is affirmed.

VALENZUELA v. CA (PHILIPPINE AMERICAN
GENERAL INSURANCE COMPANY, INC.)
191 SCRA 1
GUTIERREZ; October 19, 1990

NATURE
Petition for review of the decision of theca.

FACTS
- Petitioner Arturo P. Valenzuela is a General Agent
of private respondent Philippine American General
Insurance Company, Inc. (Philamgen for short) since
1965. As such, he was authorized to solicit and sell
in behalf of Philamgen all kinds of non-life insurance,
and in consideration of services rendered was
entitled to receive the full agent's commission of
32.5% from Philamgen under the scheduled
commission rates.
- From 1973 to 1975, Valenzuela solicited marine
insurance from one of his clients, the Delta Motors,
Inc. (Division of Electronics Airconditioning and
Refrigeration) in the amount of P4.4 Million from
which he was entitled to a commission of 32%.
However, Valenzuela did not receive his full
commission which amounted to P1.6 Million from the
P4.4 Million insurance coverage of the Delta Motors.
During the period 1976 to 1978, premium payments
amounting to P1,946,886.00 were paid directly to
Philamgen and Valenzuela's commission to which he
is entitled amounted to P632,737.00.
- In 1977, Philamgen started to become interested in
and expressed its intent to share in the commission
due Valenzuela on a fifty-fifty basis. Valenzuela
refused.
- Because of the refusal of Valenzuela, Philamgen
and its officers took drastic action against
Valenzuela. They: (a) reversed the commission due
him by not crediting in his account the commission
earned from the Delta Motors, Inc. insurance ; (b)
placed agency transactions on a cash-and-carry
basis; (c) threatened the cancellation of policies
issued by his agency; and (d) started to leak out
news that Valenzuela has a substantial account with
Philamgen. All of these acts resulted in the decline of
his business as insurance agent.
- Then on December 27, 1978, Philamgen terminated
the General Agency Agreement of Valenzuela.
- Lower court: the termination of Valenzuela as
General Agent was improper because the record will
show the principal cause of the termination of the
plaintiff as General Agent of defendant Philamgen
was his refusal to share his Delta commission.
- CA: In any event the principal's power to revoke an
agency at will is so pervasive, that the Supreme
!"#$%&"'( *+,- 1/

Court has consistently held that termination may be
effected even if the principal acts in bad faith,
subject only to the principal's liability for damages.
(CA ordered Valenzuela to pay Philamgen the
amount of One Million Nine Hundred Thirty-Two
Thousand Five Hundred Thirty-Two and 17/100
Pesos (P1,932,532.17) with legal interest)

ISSUES
1. WON whether or not Philamgen and/or its officers
can be held liable for damages due to the
termination of the General Agency Agreement it
entered into with the petitioners
2. WON petitioners are liable to Philamgen for the
unpaid and uncollected premiums

HELD
1. YES
- If a principal acts in bad faith and with abuse of
right in terminating the agency, then he is liable in
damages.
- There is an exception to the principle that an
agency is revocable at will and that is when the
agency has been given not only for the interest of
the principal but for the interest of third persons or
for the mutual interest of the principal and the
agent. In these cases, it is evident that the agency
ceases to be freely revocable by the sole will of the
principal
(PROCEDURAL: Where the findings of the Court of
Appeals and the trial court are contrary to each
other, this Court may scrutinize the evidence on
record
- After a painstaking review of the entire records of
the case and the findings of facts of both the court a
quo and respondent appellate court, the Court
affirmed the trial courts findings.)
- The principal cause of the termination of Valenzuela
as General Agent of Philamgen arose from his refusal
to share his Delta commission. The records sustain
the conclusions of the trial court on the apparent bad
faith of the private respondents in terminating the
General Agency Agreement of petitioners.
- It is also evident from the records that the agency
involving petitioner and private respondent is one
"coupled with an interest," and, therefore, should not
be freely revocable at the unilateral will of the latter.
- The private respondents by the simple expedient of
terminating the General Agency Agreement
appropriated the entire insurance business of
Valenzuela. With the termination of the General
Agency Agreement, Valenzuela would no longer be
entitled to commission on the renewal of insurance
policies of clients sourced from his agency. Worse,
despite the termination of the agency, Philamgen
continued to hold Valenzuela jointly and severally
liable with the insured for unpaid premiums. Under
these circumstances, it is clear that Valenzuela had
an interest in the continuation of the agency when it
was unceremoniously terminated not only because of
the commissions he should continue to receive from
the insurance business he has solicited and procured
but also for the fact that by the very acts of the
respondents, he was made liable to Philamgen in the
event the insured fail to pay the premiums due. They
are estopped by their own positive averments and
claims for damages.
- "The principal may not defeat the agent's right to
indemnification by a termination of the contract of
agency (Erskine v. Chevrolet Motors Co. 185 NC 479,
117 SE 706, 32 ALR 196).
- For the pivotal factor rendering Philamgen and the
other private respondents liable in damages is that
the termination by them of the General Agency
Agreement was tainted with bad faith. This is in
accordance with the precepts in Human Relations
enshrined in our Civil Code.
2. NO. The respondent court erred in holding
Valenzuela liable. There was no factual and legal
basis for the award. Under Section 77 of the
Insurance Code, the remedy for the non-payment
of premiums is to put an end to and render the
insurance policy not binding - "Sec. 77 . . .
[N]otwithstanding any agreement to the contrary, no
policy or contract of insurance is valid and binding
unless and until the premiums thereof have been
paid except in the case of a life or industrial life
policy whenever the grace period provision applies
(P.D. 612, as amended otherwise known as the
Insurance Code of 1974)
- This is buttressed by Section 776 of the
Insurance Code (Presidential Decree No. 612,
promulgated on December 18, 1974), which now
provides that no contract of Insurance by an
insurance company is valid and binding unless and
until the premium thereof has been paid,
notwithstanding any agreement to the contrary."
Disposition Petition is GRANTED. CA decision SET
ASIDE. The decision of the TC REINSTATED with the
MODIFICATIONS. And that the contractual
relationship between Arturo P. Valenzuela and
Philippine American General Insurance Company
shall be deemed terminated upon the satisfaction of
the judgment as modified.

CHAPTER V THE POLICY, PARTIES THERETO,
& RIGHTS THEREON

DE LIM v. SUN LIFE ASSURANCE COMPANY OF
CANADA
41 PHIL 263
MALCOLM; November 29, 1920

NATURE
Appeal from an order of the CFI of Zamboanga
sustaining a demurrer to plaintiff's complaint upon
the ground that it fails to state a cause of action.

FACTS
- On July 6, 1917, Luis Lim of Zamboanga made
application to the Sun Life Assurance Company of
Canada for a policy of insurance on his life in the
sum of P5,000. In his application Lim designated his
wife, Pilar de Lim, the plaintiff herein, as the
beneficiary. The first premium of P433 was paid by
Lim, and upon such payment the company issued
what was called a ''provisional policy." Luis Lim died
on August 23, 1917, after the issuance of the
provisional policy but before approval of the
application by the home office of the insurance
company. Pilar de Lim brought an action to recover
from the Sun Life sum of P5,000, the amount named
in the provisional policy.
- The "provisional policy" reads: "Received (subject
to the following stipulations and agreements) the
sum of P433, being the amount of the first year's
premium for a Life Assurance Policy on the life of Mr.
Luis D. Lim of Zamboanga for P5,000, for which an
application dated the 6th day of July, 1917, has been
made to the Sun Life Assurance Company of Canada.
- The above-mentioned life is to be assured in
accordance with the terms and conditions contained
or inserted by the Company in the policy which may
be granted by it in this particular case for four
months only from the date of the application,
provided that the Company shall confirm this
agreement by issuing a policy on said application
when the same shall be submitted to the Head Office
in Montreal. Should the Company not issue such a
policy, then this agreement shall be null and void ab
initio, and the Company shall be held not to have
been on the risk at all, but in such case the amount
herein acknowledged shall be returned.

ISSUE
!"#$%&"'( *+,- 10

WON the contract of insurance between Luis Lim and
Sun Life Assurance Company of Canada was
perfected

HELD
NO.
- The document it is to be a provisional policy "for
four months only from the date of this application."
Immediately following the words fixing the four
months period comes the word "provided" which has
the meaning of "if." Otherwise stated, the policy for
four months is expressly made subject to the
affirmative condition that the company shall confirm
this agreement by issuing a policy on said application
when the same shall be submitted to the head office
in Montreal. To re-enforce the same there follows the
negative condition - "Should the company not issue
such a policy, then this agreement shall be null and
void ab initio, and the company shall be held not to
have been on the risks." Certainly language could
hardly be used which would more clearly stipulate
that the agreement should not go into effect until the
home office of the company should confirm it by
issuing a policy. As we read and understand the so-
called provisional policy, it amounts to nothing but
an acknowledgment on behalf of the company, that
it has received from the person named therein the
sum of money agreed upon as the first year's
premium upon a policy to be issued upon the
application, if the application is accepted by the
company.
- It is of course a primary rule that a contract of
insurance, like other contracts, must be assented to
by both parties either in person or by their agents.
So long as an application for insurance has not been
either accepted or rejected, it is merely an offer or
proposal to make a contract. The contract, to be
binding from the date of the application must have
been a completed contract, one that leaves nothing
to be done, nothing to be completed, nothing to be
passed upon, or determined, before it shall take
effect. There can be no contract of insurance unless
the minds of the parties have met in agreement. Our
view is, that a contract of insurance was not here
consummated by the parties.
- The trial court committed no error in sustaining the
demurrer and dismissing the case. It is to be noted,
however that counsel for appellee admits the liability
of the company for the return of the first premium to
the estate of the deceased.

GREAT PACIFIC LIFE v. CA (LEUTERIO)
316 SCRA 677
QUISUMBING; October 13, 1999

NATURE
Petition for Review of CA decision

FACTS
- A contract of group life insurance was executed
between petitioner Great Pacific Life Assurance
Corporation (hereinafter Grepalife) and Development
Bank of the Philippines (hereinafter DBP). Grepalife
agreed to insure the lives of eligible housing loan
mortgagors of DBP.
- In Nov. 1983, Dr. Wilfredo Leuterio, a physician
and a housing debtor of DBP applied for membership
in the group life insurance plan. In an application
form, Dr. Leuterio answered Qs concerning his health
condition as follows:
Q: Have you ever had, or consulted, a physician for
a heart condition, high blood pressure, cancer,
diabetes, lung, kidney or stomach disorder or any
other physical impairment? No.
Q: Are you now, to the best of your knowledge, in
good health? Yes.
- Grepalife issued an insurance coverage of Dr.
Leuterio, to the extent of his DBP mortgage
indebtedness of P86,200.00. In Aug. 1984, Dr.
Leuterio died due to "massive cerebral hemorrhage."
DBP submitted a death claim to Grepalife. Grepalife
denied the claim because Dr. Leuterio was not
physically healthy when he applied for an insurance.
Grepalife insisted that Dr. Leuterio did not disclose
he had been suffering from hypertension, which
caused his death. Allegedly, such non-disclosure
constituted concealment that justified the denial of
the claim.
- Herein respondent Medarda Leuterio, widow, filed a
complaint with RTC against Grepalife for "Specific
Performance with Damages." Dr. Mejia, who issued
the death certificate, testified that Dr. Leuterio
complained of headaches presumably due to high
blood pressure. The inference was not conclusive
because Dr. Leuterio was not autopsied, hence,
other causes were not ruled out.
- RTC ruled in favor of respondent widow and against
Grepalife. CA sustained the RTC decision. Hence, the
present petition.



ISSUES
1. WON CA erred in holding petitioner liable to DBP
as beneficiary in a group life insurance contract from
a complaint filed by the widow of the
decedent/mortgagor
2. WON CA erred in not finding that Dr. Leuterio
concealed that he had hypertension, which would
vitiate the insurance contract
3. WON CA erred in holding Grepalife liable for
P86,200.00 without proof of the actual outstanding
mortgage payable by the mortgagor to DBP

HELD
1. NO
Ratio Insured, being the person with whom the
contract was made, is primarily the proper person to
bring suit. Subject to some exceptions, insured may
thus sue, although the policy is taken wholly or in
part for the benefit of another person named or
unnamed, and although it is expressly made payable
to another as his interest may appear or otherwise.
Although a policy issued to a mortgagor is taken out
for the benefit of the mortgagee and is made
payable to him, yet the mortgagor may sue thereon
in his own name, especially where the mortgagee's
interest is less than the full amount recoverable
under the policy. (See Sec. 8, Insurance Code)
Reasoning
[a] The insured private respondent did not cede to
the mortgagee all his rights or interests in the
insurance, the policy stating that: In the event of
the debtor's death before his indebtedness with the
Creditor (DBP) shall have been fully paid, an amount
to pay the outstanding indebtedness shall first be
paid to the creditor and the balance of sum assured,
if there is any, shall then be paid to the
beneficiary/ies designated by the debtor. When DBP
submitted the insurance claim against Grepalife, the
latter denied payment thereof, interposing the
defense of concealment committed by the insured.
Thereafter, DBP collected the debt from the
mortgagor and took the necessary action of
foreclosure on the residential lot of private
respondent.
[b] Since a policy of insurance upon life or health
may pass by transfer, will or succession to any
person, whether he has an insurable interest or not,
and such person may recover it whatever the insured
might have recovered, the widow of the decedent Dr.
Leuterio may file the suit against the insurer,
Grepalife.
2. NO
!"#$%&"'( *+,- 11

Ratio The fraudulent intent on the part of the
insured must be established to entitle the insurer to
rescind the contract. Misrepresentation as a defense
of the insurer to avoid liability is an affirmative
defense and the duty to establish such defense by
satisfactory and convincing evidence rests upon the
insurer. In the case at bar, the petitioner failed to
clearly and satisfactorily establish its defense, and is
therefore liable to pay the proceeds of the insurance.
Reasoning
[a] The insured, Dr. Leuterio, had answered in his
insurance application that he was in good health and
that he had not consulted a doctor or any of the
enumerated ailments, including hypertension; when
he died the attending physician had certified in the
death certificate that the former died of cerebral
hemorrhage, probably secondary to hypertension.
From this report, petitioner Grepalife refused to pay
the insurance claim. It alleged that the insured had
concealed the fact that he had hypertension.
[b] Contrary to Grepalifes allegations, there was no
sufficient proof that the insured had suffered from
hypertension. Aside from the statement of the
insured's widow who was not even sure if the
medicines taken by Dr. Leuterio were for
hypertension, the appellant had not proven nor
produced any witness who could attest to Dr.
Leuterio's medical history.
[c] Grepalife had failed to establish that there was
concealment made by the insured, hence, it cannot
refuse payment of the claim.
3. NO
- Considering the supervening event that DBP
foreclosed in 1995 their residential lot, in satisfaction
of mortgagor's outstanding loan, the insurance
proceeds shall inure to the benefit of the heirs of the
deceased person or his beneficiaries. Equity dictates
that DBP should not unjustly enrich itself at the
expense of another. Hence, it cannot collect the
insurance proceeds, after it already foreclosed on the
mortgage. The proceeds now rightly belong to Dr.
Leuterio's heirs represented by his widow, herein
private respondent.
- The Court ruled this issue based on the clear
provisions of the policy. The mortgagor paid the
premium according to the coverage of his insurance,
which states that: "The policy states that upon
receipt of due proof of the Debtor's death during the
terms of this insurance, a death benefit in the
amount of P86,200.00 shall be paid In the event of
the debtor's death before his indebtedness with the
creditor shall have been fully paid, an amount to pay
the outstanding indebtedness shall first be paid to
the Creditor and the balance of the Sum Assured, if
there is any shall then be paid to the beneficiary/ies
designated by the debtor." From this, it is clear that
Grepalife is liable and that Dr. Leuterios heirs must
get the proceeds.
Disposition Petition DENIED. CA Decision AFFIRMED
with modification.

PACIFIC TIMBER EXPORT CORPORATION v. CA
(WORKMENS INSURANCE CO)
112 SCRA 199
DE CASTRO; February 25, 1982

FACTS
- March 19, 1963 - the plaintiff secured temporary
insurance from the defendant for its exportation of
1,250,000 board feet of Philippine Lauan and Apitong
logs to be shipped from the Diapitan Bay, Quezon to
Okinawa and Tokyo, Japan. The defendant issued on
said date Cover Note No. 1010, insuring the said
cargo of the plaintiff "Subject to the Terms and
Conditions of the WORKMEN'S INSURANCE
COMPANY, INC. printed Marine Policy form as filed
with and approved by the Office of the Insurance
Commissioner.
- April 2, 1963 - The two (2) regular marine cargo
policies were issued by the defendant in favor of the
plaintiff. The total cargo insured under the two
marine policies accordingly consisted of 1,395 logs,
or the equivalent of 1,195,498 bd. ft.
- After the issuance of cover note but before the
issuance of the two marine policies some of the logs
intended to be exported were lost during loading
operations in the Diapitan Bay due to bad weather.
- April 4, 1963 - The plaintiff informed the defendant
about the loss of 'approximately 32 pieces of logs'
during loading through a letter.
- The plaintiff subsequently submitted a 'Claim
Statement' demanding payment of the loss under
the second marine cargo policy.
- July 17, 1963 - the defendant requested the First
Philippine Adjustment Corporation to inspect the loss
and assess the damage.
- August 23, 1963 - the adjuster reported that 'the
loss of 30 pieces of logs is not covered by the two
policies inasmuch as said policies covered the actual
number of logs loaded on board. But it is covered by
Cover Note.
- On January 13, 1964 - the defendant wrote the
plaintiff denying the latter's claim, on the ground
that defendant's investigation revealed that the
entire shipment of logs covered by the two marines
policies were received in good order at their point of
destination. It was further stated that the said loss
may not be considered as covered under cover note
because the said note had become 'null and void by
virtue of the issuance of two marine policies.
- The CFI of Manila ruled in favour of the petitioner.
- The Court of Appeals reversed the decision of the
CFI.

ISSUES
1. WON the cover note is null and void for lack of
valuable consideration because no separate
premiums are collected by private respondent on all
its cover notes
2. WON the court of appeals erred in holding that
private respondent was released from liability under
the cover note due to unreasonable delay in giving
notice of loss because the court disregarded the
proven fact that private respondent did not promptly
and specifically object to the claim on the ground of
delay in giving notice of loss and, consequently,
objections on that ground are waived under section
84 of the insurance act

HELD
1. NO
Ratio Cover note is issued with a consideration
when, by express stipulation, the cover note is made
subject to the terms and conditions of the marine
policies, and the payment of premiums is one of the
terms of the policies.
Reasoning
a. the cover note in question is subject to the terms
and conditions of the marine policies
b. Nature of the Cover Note: The fact that no
separate premium was paid on the Cover Note
before the loss insured against occurred, does not
militate against the validity of petitioner's
contention, for no such premium could have been
paid, since by the nature of the Cover Note, it did
not contain, as all Cover Notes do not contain
particulars of the shipment that would serve as basis
for the computation of the premiums. As a logical
consequence, no separate premiums are intended or
required to be paid on a Cover Note.
c. The petitioner paid in full all the premiums as
called for by the statement issued by private
respondent after the issuance of the two regular
marine insurance policies, thereby leaving no
account unpaid by petitioner due on the insurance
coverage, which must be deemed to include the
!"#$%&"'( *+,- 12

Cover Note. If the Note is to be treated as a separate
policy instead of integrating it to the regular policies
subsequently issued, the purpose and function of the
Cover Note would be set at naught or rendered
meaningless, for it is in a real sense a contract, not a
mere application for insurance which is a mere offer.
Had all the logs been lost during the loading
operations, but after the issuance of the Cover Note,
liability on the note would have already arisen even
before payment of premium. This is how the cover
note as a "binder" should legally operate; otherwise,
it would serve no practical purpose in the realm of
commerce, and is supported by the doctrine that
where a policy is delivered without requiring
payment of the premium, the presumption is that a
credit was intended and policy is valid.
2. NO
- The private respondent company never raised this
ground in the proceedings. It must be because it did
not find any delay, as this Court fails to find a real
and substantial sign thereof. But even on the
assumption that there was delay, this Court is
satisfied and convinced that as expressly provided by
law, waiver can successfully be raised against private
respondent. Thus Section 84 of the Insurance Act
provides:
"Section 84. - Delay in the presentation to an
insurer of notice or proof of loss is waived if
caused by any act of his or if he omits to take
objection promptly and specifically upon that
ground."
- From what has been said, We find duly
substantiated petitioner's assignments of error.
Disposition The appealed decision is set aside and
the decision of the Court of First Instance is
reinstated in toto with the affirmance of this Court.

DEVELOPMENT INSURANCE v. IAC (PHIL
UNION REALTY DEVELOPMENT CORP)
143 SCRA 62
CRUZ; July 16, 1986

FACTS
- A fire occurred in the building of the private
respondent and it sued for recovery of damages from
the petitioner on the basis of an insurance contract
between them. The petitioner allegedly failed to
answer on time and was declared in default by TC. A
judgment of default was rendered on the strength of
the evidence submitted ex parte by the private
respondent, which was allowed full recovery of its
claimed damages.
- On learning of this decision, the petitioner moved
to lift the order of default, invoking excusable
neglect, and to vacate the judgment by default. Its
motion was denied.
- On appeal, IAC affirmed the TC decision in toto.

ISSUE
1. WON default of petitioner is based on excusable
neglect
2. What is the amount of indemnity due to the
private respondent under its insurance contract?
WON CFI was correct in interpreting the contract

HELD
1. NO
- Summons was served through its vice-president.
There were even several extensions to the original
period to answer. As a consequence, the TC, on
motion of the private respondent filed declared the
petitioner in default. This was done almost one
month later. Even so, the petitioner made no move
at all for two months thereafter. It was only more
than one month after the judgment of default was
rendered by the TC that it filed a motion to lift the
order of default and vacate the judgment by default.
- There is a pattern of inexcusable neglect.
2. The policy is an open policy which means that the
actual loss, as determined, will represent the total
indemnity due the insured from the insurer except
only that the total indemnity shall not exceed the
face value of the policy.
- The petitioner argues that since at the time of the
fire the building insured was worth P5,800,000.00,
the private respondent should be considered its own
insurer for the difference between that amount and
the face value of the policy and should share pro rata
in the loss sustained. Accordingly, the private
respondent is entitled to an indemnity of only
P67,629.31, the rest of the loss to be shouldered by
it alone. The petitioner cites Condition 17 of the
policy, which provides:
"If the property hereby insured shall, at the
breaking out of any fire, be collectively of greater
value than the sum insured thereon then the
insured shall be considered as being his own
insurer for the difference, and shall bear a ratable
proportion of the loss accordingly. Every item, if
more than one, of the policy shall be separately
subject to this condition."
- However, there is no evidence on record that the
building was worth P5,800,000.00 at the time of the
loss. On the contrary, the building was insured at
P2,500,000.00, and this must be considered, by
agreement, the actual value of the property insured
on the day the fire occurred. This valuation becomes
even more believable if it is remembered that at the
time the building was burned it was still under
construction and not yet completed.
- The Court notes that the policy in this case is an
open policy and is subject to the express condition
that:
"Open Policy.
This is an open policy as defined in Sec57 of the
Insurance Act. In the event of loss, whether total
or partial, it is understood that the amount of the
loss shall be subject to appraisal and the liability of
the company, if established, shall be limited to the
actual loss, subject to the applicable terms,
conditions, warranties and clauses of this Policy,
and in no case shall exceed the amount of the
policy."
- As defined in the aforestated provision, which is
now Sec60 of the Insurance Code, "an open policy is
one in which the value of the thing insured is not
agreed upon but is left to be ascertained in case of
loss.".
- The actual loss has been ascertained in this case
and the Court will respect such factual determination
in the absence of proof that it was arrived at
arbitrarily. There is no such showing. Hence,
applying the open policy clause as expressly agreed
upon by the parties in their contract, we hold that
the private respondent is entitled to the payment of
indemnity under the said contract in the total
amount of P508,867.00.
- The refusal of its vice-president to receive the
private respondent's complaint, as reported in the
sheriff's return, was the first indication of the
petitioner's intention to prolong this case and
postpone the discharge of its obligation to the
private respondent under this agreement. That
intention was revealed further in its subsequent acts
---- or inaction ---- which indeed enabled it to avoid
payment for more than five years from the filing of
the claim against it in 1980.
Disposition The appealed decision is affirmed in
full, with costs against the petitioner.

HARDING v. COMMERCIAL UNION ASSURANCE
38 PHIL 464
FISHER; August 10, 1918

FACTS
!"#$%&"'( *+,- 13

- Mrs. Harding was the owner of a Studebaker
automobile; in consideration of the payment to the
defendant of the premium of P150, by said plaintiff,
Mrs. Henry E. Harding, with the consent of her
husband, the defendant by its duly authorized agent,
Smith, Bell & Company (limited), made its policy of
insurance in writing upon said automobile was set
forth in said policy to be P3,000 that the value of
said automobile was set forth in said policy to be
P3,000; that on March 24, 1916, said automobile
was totally destroyed by fire; that the loss thereby to
plaintiffs was the sum of P3,000.
- The defendants version is that by request of Mrs.
Harding, it issued the policy of insurance on an
automobile alleged by the said plaintiff to be her
property. It was made by means of a proposal in
writing signed and delivered by said plaintiff to the
defendant, guaranteeing the truth of the statements
contained therein which said proposal is referred to
in the said policy of insurance made a part thereof;
that certain of the statements and representations
contained in said proposal and warranted by said
plaintiff to be true, to wit: (a) the price paid by the
proposer for the said automobile; (b) the value of
said automobile at the time of the execution and
delivery of the said proposal and (c) the ownership
of said automobile, were false and known to be false
by the said plaintiff at the time of signing and
delivering the said proposal and were made for the
purpose of misleading and deceiving the defendant,
and inducing the defendant, relying upon the
warranties, statements, and representations
contained in the said proposal and believing the
same to be true, issued the said policy of insurance.
- The evidence shows that Hermanos, the Manila
agents for the Studebaker automobile, sold the
automobile to Canson for P3,200 (testimony of Mr.
Diehl); who sold the said automobile to Henry
Harding for the sum of P1,500. Harding sold the said
automobile to J. Brannigan for the sum of P2,000
who sold the said automobile Henry Harding for the
sum of P2,800; Henry Harding gave the said
automobile to his wife as a present; that said
automobile was repaired and repainted at the Luneta
Garage at a cost of some P900; that while the said
automobile was at the Luneta Garage; the latter
solicited of Mrs. Harding the insurance of said
automobile by the Company; that a proposal was
filled out by the said agent and signed by the plaintiff
Mrs. Henry E. Harding, and in said proposal under
the heading "Price paid by proposer," is the amount
of "3,500" and under another heading "Present
value" is the amount of "3,000".
- After the said proposal was made a representative
of the Manila agent of defendant went to the Luneta
Garage and examined said automobile and Mr.
Server, the General Manager of the Luneta Garage,
an experienced automobile mechanic, testified that
at the time this automobile was insured it was worth
about P3,000, and the defendant, by and through its
said agent Smith, Bell & Company (limited),
thereafter issued a policy of insurance upon proposal
in which policy the said automobile was described as
of the "present value" of P3,000 and the said
defendant charged the said plaintiff Mrs. Henry E.
Harding as premium on said policy the sum of P150,
or 5 per cent of the then estimated value of P3,000.
- The "Schedule" in said policy of insurance describes
the automobile here in question, and provides in part
of follows:
"That during the period above set forth and during
any period for which the company may agree to
renew this policy the company will subject to the
exception and conditions contained herein or
endorsed hereon indemnify the insured against loss
of or damage to any motor car described in the
schedule hereto (including accessories) by whatever
cause such loss or damage may be occasioned and
will further indemnify the insured up to the value of
the car or P3,000 whichever is the greater against
any claim at common law made by any person (not
being a person in the said motor car nor in the
insured's service) for loss of life or for accidental
bodily injury or damage to property caused by the
said motor car including law costs payable in
connection with such claim when incurred with the
consent of the company."
- On March 24, 1916, the said automobile was totally
destroyed by fire, and that the iron and steel
portions of said automobile which did not burn were
taken into the possession of the defendant by and
through its agent Smith, Bell & Company (limited),
and sold by it for a small sum, which had never been
tendered to the plaintiff prior to the trial of this case,
but in open court during the trial the sum of P10 as
the proceeds of such sale was tendered to plaintiff
and refused.
- Trial judge decided that there was no proof of fraud
on the part of plaintiff in her statement of the value
of the automobile, or with respect to its ownership;
that she had an insurable interest therein; and that
defendant, having agreed to the estimated value,
P3,000, and having insured the automobile for that
amount, upon the basis of which the premium was
paid, is bound by it and must pay the loss in
accordance with the stipulated insured value.

ISSUE
1. WON Mrs. Harding was not the owner of the
automobile at the time of the issuance of the policy,
and, therefore, had no insurable interest in it
2. WON the statement regarding the cost of the
automobile was a warranty, that the statement was
false, and that, therefore, the policy never attached
to the risk

HELD
1. NO
- Article 1334 of the Civil Code which provides that
"All gifts between spouses during the marriage shall
be void. Moderate gifts which the spouses bestow on
each other on festive days of the family are not
included in this rule."
- Even assuming that defendant might have invoked
article 1334 as a defense, the burden would be upon
it to show that the gift in question does not fall
within the exception therein established. We cannot
say, as a matter of law, that the gift of an
automobile by a husband to his wife is not a
moderate one. Whether it is or is not would depend
upon the circumstances of the parties, as to which
nothing is disclosed by the record.
- We are of the opinion that it would be unfair to
hold the policy void simply because the outlay
represented by the automobile was made by the
plaintiff's husband and not by his wife, to whom he
had given the automobile. It cannot be assumed that
defendant should not have issued the policy unless it
were strictly true that the price representing the cost
of the machine had been paid by the insured and by
no other person ? that it would no event insure an
automobile acquired by gift, inheritance, exchange,
or any other title not requiring the owner to make a
specific cash outlay for its acquisition.
2. NO
- It has not been shown by the evidence that the
statement was false; on the contrary we believe that
it shows that the automobile had in fact cost more
than the amount mentioned. The court below found,
and the evidence shows, that the automobile was
bought by plaintiff's husband a few weeks before the
issuance of the policy in question for the sum of
P2,800, and that between that time and the issuance
of the policy some P900 was spent upon it in repairs
and repainting.
!"#$%&"'( *+,- 14

- The witness Server, an expert automobile
mechanic, testified that the automobile was
practically as good as new at the time the insurance
was effected. The form of proposal upon which the
policy was issued does not call for a statement
regarding the value of the automobile at the time of
its acquisition by the applicant for the insurance, but
merely a statement of its cost. The amount stated
was less than the actual outlay which the automobile
represented to Mr. Harding, including repairs, when
the insurance policy was issued.
- The court below found and the evidence shows,
without dispute, that the proposal upon which the
policy in question was issued was made out by
defendant's agent by whom the insurance was
solicited, and that appellee simply signed the same.
It also appears that an examiner employed by the
defendant made an inspection of the automobile
before the acceptance of the risk, and that the sum
after this examination. The trial court found that Mrs.
Harding, in fixing the value of the automobile at
P3,000, acted upon information given her by her
husband and by Mr. Server, the manager of the
Luneta Garage. She merely repeated the information
which had been given her by her husband, and at
the same time disclosed to defendant's agent the
source of her information. There is no evidence to
sustain the contention that this communication was
made in bad faith. We do not think that the facts
stated in the proposal can be held as a warranty of
the insured, even if it should have been shown that
they were incorrect in the absence of proof of willful
misstatement. Under such circumstance, the
proposal is to be regarded as the act of the insurer
and not of the insured.
Disposition Plaintiff was the owner of the
automobile in question and had an insurable interest
therein; that there was no fraud on her part in
procuring the insurance; that the valuation of the
automobile, for the purposes of the insurance, is
binding upon the defendant corporation, and that the
judgment of the court below is, therefore, correct
and must be affirmed, with interest, the costs of this
appeal to be paid by the appellant.

WHITE GOLD MARINE SERVICES v. PIONEER
INSURANCE
464 SCRA 448
QUISUMBING; July 28, 2005

NATURE
This petition for review assails the Decision of the
Court of Appeals, affirming the Decision of the
Insurance Commission. Both decisions held that
there was no violation of the Insurance Code and the
respondents do not need license as insurer and
insurance agent/broker.

FACTS
- White Gold procured a protection and indemnity
coverage for its vessels from Steamship Mutual
through Pioneer Insurance. Subsequently, White
Gold was issued a Certificate of Entry and
Acceptance. Pioneer also issued receipts evidencing
payments for the coverage. When White Gold failed
to fully pay its accounts, Steamship Mutual refused
to renew the coverage.
- Steamship Mutual thereafter filed a case against
White Gold for collection of sum of money to recover
the latters unpaid balance. White Gold on the other
hand, filed a complaint before the Insurance
Commission claiming that Steamship Mutual violated
Sections 186 and 187 of the Insurance Code, while
Pioneer violated Sections 299, 300 and 301 in
relation to Sections 302 and 303, thereof.
- The Insurance Commission dismissed the
complaint. It said that there was no need for
Steamship Mutual to secure a license because it was
not engaged in the insurance business. It explained
that Steamship Mutual was a Protection and
Indemnity Club (P & I Club). Likewise, Pioneer need
not obtain another license as insurance agent and/or
a broker for Steamship Mutual because Steamship
Mutual was not engaged in the insurance business.
Moreover, Pioneer was already licensed, hence, a
separate license solely as agent/broker of Steamship
Mutual was already superfluous.
- The Court of Appeals affirmed the decision of the
Insurance Commissioner. In its decision, the
appellate court distinguished between P & I Clubs
vis--vis conventional insurance. The appellate court
also held that Pioneer merely acted as a collection
agent of Steamship Mutual.

ISSUES
1. WON Steamship Mutual, a P & I Club, is engaged
in the insurance business in the Philippines
2. WON Pioneer needs a license as an insurance
agent/broker for Steamship Mutual

HELD
1. YES
- The test to determine if a contract is an insurance
contract or not, depends on the nature of the
promise, the act required to be performed, and the
exact nature of the agreement in the light of the
occurrence, contingency, or circumstances under
which the performance becomes requisite. It is not
by what it is called. Basically, an insurance contract
is a contract of indemnity. In it, one undertakes for
a consideration to indemnify another against loss,
damage or liability arising from an unknown or
contingent event.
- In particular, a marine insurance undertakes to
indemnify the assured against marine losses, such as
the losses incident to a marine adventure. Section 99
of the Insurance Code enumerates the coverage of
marine insurance.
- Relatedly, a mutual insurance company is a
cooperative enterprise where the members are both
the insurer and insured. In it, the members all
contribute, by a system of premiums or
assessments, to the creation of a fund from which all
losses and liabilities are paid, and where the profits
are divided among themselves, in proportion to their
interest. Additionally, mutual insurance associations,
or clubs, provide three types of coverage, namely,
protection and indemnity, war risks, and defense
costs.
- A P & I Club is a form of insurance against third
party liability, where the third party is anyone other
than the P & I Club and the members. By definition
then, Steamship Mutual as a P & I Club is a mutual
insurance association engaged in the marine
insurance business.
- The records reveal Steamship Mutual is doing
business in the country albeit without the requisite
certificate of authority mandated by Section 187 of
the Insurance Code. It maintains a resident agent in
the Philippines to solicit insurance and to collect
payments in its behalf. We note that Steamship
Mutual even renewed its P & I Club cover until it was
cancelled due to non-payment of the calls. Thus, to
continue doing business here, Steamship Mutual or
through its agent Pioneer, must secure a license
from the Insurance Commission.
- Since a contract of insurance involves public
interest, regulation by the State is necessary. Thus,
no insurer or insurance company is allowed to
engage in the insurance business without a license or
a certificate of authority from the Insurance
Commission.
2. YES
- SEC. 299 . . .
!"#$%&"'( *+,- 15

- No person shall act as an insurance agent or as an
insurance broker in the solicitation or procurement of
applications for insurance, or receive for services in
obtaining insurance, any commission or other
compensation from any insurance company doing
business in the Philippines or any agent thereof,
without first procuring a license so to act from the
Commissioner, which must be renewed annually on
the first day of January, or within six months
thereafter.
Disposition The petition is PARTIALLY GRANTED.
The Decision dated July 30, 2002 of the Court of
Appeals affirming the Decision dated May 3, 2000 of
the Insurance Commission is hereby REVERSED AND
SET ASIDE. The Steamship Mutual Underwriting
Association (Bermuda) Ltd., and Pioneer Insurance
and Surety Corporation are ORDERED to obtain
licenses and to secure proper authorizations to do
business as insurer and insurance agent,
respectively. The petitioners prayer for the
revocation of Pioneers Certificate of Authority and
removal of its directors and officers, is DENIED.

PANDIMAN v. MARINE MANNING MNGT CORP.
460 SCRA 418
GARCIA; June 21, 2005

NATURE
Petition for certiorari to review CA decision

FACTS
- Benito Singhid was hired as chief cook on board the
vessel MV Sun Richie Five for a term of one year by
Fullwin Maritime Limited through its Philippine agent,
Marine Manning and Management Corporation. While
the said vessel was on its way to Shanghai from Ho
Chih Minh City, Benito suffered a heart attack and
subsequently died on June 24, 1997.
- Apparently, the vessel and the crew were insured
with Ocean Marine Mutual Insurance Association
Limited (OMMIAL), a Protective and Indemnity Club
of which Sun Richie Five Bulkers S.A. is a member.
Pandiman Philippines, the petitioner, is the local
correspondent of OMMIAL.
- Benitos widow, Rosita, filed a claim for death
benefits with Marine which referred her to Pandiman.
After her submission of the required documentation,
Pandiman recommended payment of the death
benefits amounting to $79,000. However, payment
has not been made.
- Rosita filed a complaint with the Labor Arbiter
naming Marine, Pandiman, OMMIAL, and Fullwin as
respondents. The Arbiter ordered all the
respondents, except Pandiman, to jointly and
severally pay the widow the death benefits plus legal
fees. The NLRC, on appeal by Marine, limited the
liable parties to Pandiman and OMMIAL but
maintained the money award. The CA sustained the
decision of the NLRC. Hence this appeal.


ISSUE
1. WON Pandiman may be held liable for the death
benefits
2. WON Marine and its foreign principal, Fullwin,
should be absolved from the death claim liabilities

HELD
1. NO
- Pandiman is not an insurance agent as defined by
Section 300
7
of the Insurance Code. In this case,
there was no showing that Pndiman in fact
negotiated the insurance contract between Sun
Richie Five and the insurer OMMIAL. Even, if
Pandiman were an agent, payment for claims arising
from peril insured against, to which the insurer is
liable, is definitely not one of the liabilities of an
insurance agent. Thus, there is no legal basis
whatsoever for holding petitioner solidarily liable with
insurer OMMIAL for the widows claim for death
benefits. Also, Pandiman is not a party to the
insurance contract and hence under Article 1311 of
the Civil Code, it is not liable for the obligation
arising out of the insurance contract.
2. NO
- Fullwin, as Benitos principal employer is liable
under the employment contract. Marine is also
bound by its undertaking pursuant to the Rules and
Regulations Governing Overseas Employment that it
shall assume joint and solidary liability with the
employer for all the claims and liabilities which may
arise in connection with the implementation of the
contract, including but not limited to the payment of
wages, heath and disability compensation and
repatriation. In other words, both Fullwin and
Marine should be held liable for whatever death
benefits the widow of Benito may be entitled to.
Disposition The petition is granted and the CA
decision is reversed and set aside.

'
^1.%0+- (LL& #-< 31$2+- =7+ 6+$ .+,31-2/%0+- 2+50.0%2 +$ +4%/0-2 +- 417/56 +6 /-< 0-2:$/-.1
.+,3/-< %$/-2,0%2 6+$ / 31$2+- +%71$ %7/- 70,2156 /- /3350./%0+- 6+$ / 3+50.< +$ .+-%$/.% +6
0-2:$/-.1 %+ +$ 6$+, 2:.7 .+,3/-< +$ +661$2 +$ /22:,12 %+ /.% 0- %71 -1;+%0/%0-; +6 2:.7 0-2:$/-.1
27/55 41 /- 0-2:$/-.1 /;1-% =0%70- %71 0-%1-% +6 %702 21.%0+- /-9 27/55 %71$14< 41.+,1 50/451 %+ /55
%71 9:%0128 $1B:0$1,1-%28 50/4050%0128 /-9 31-/5%012 %+ =70.7 /- 0-2:$/-.1 /;1-% 02 2:4_1.%&

FILIPINAS COMPANIA DE SEGUROS V
CHRISTERN, HUENEFELD AND CO INC
89 PHIL 54
PARAS; May 25, 1951

FACTS
- October 1, 1941 - Christern Huenefeld, & Co., Inc.,
after payment of corresponding premium, obtained
from the Filipinas Cia. de Seguros a fire policy in the
sum of P1000,000, covering merchandise contained
in No. 711 Roman Street, Binondo Manila.
- February 27, 1942 or during the Japanese military
occupation - building and insured merchandise were
burned. In due time the Huenefeld Co submitted to
the Filipinas Cia its claim under the policy. The
salvage goods were sold at public auction and, after
deducting their value, the total loss suffered by the
respondent was fixed at P92,650.
- Filipinas Cia refused to pay the claim on the ground
that the policy in favor of the respondent had ceased
to be in force on the date the United States declared
war against Germany, the respondent Corporation
(though organized under and by virtue of the laws of
the Philippines) being controlled by the German
subjects and the Filipinas Cia being a company under
American jurisdiction when said policy was issued on
October 1, 1941. Filipinas Cia, however, in
pursuance of the order of the Director of Bureau of
Financing, Philippine Executive Commission, dated
April 9, 1943, paid to the Huenefeld Co the sum of
P92,650 on April 19, 1943.
- August 6, 1946 action filed in CFI Manila to
recover from the Huenefeld Co the sum of P92,650
above mentioned. The theory of the Filipinas Cia is
that the insured merchandise were burned up after
the policy issued in 1941 in favor of Huenefeld Co
has ceased to be effective because of the outbreak of
the war between the United States and Germany on
December 10, 1941, and that the payment made by
the Filipinas Cia to Huenefeld Co during the Japanese
military occupation was under pressure.
- CFI: dismissed the action without pronouncement
as to costs.
- CA: CFI judgment affirmed, with costs. The case is
now before us on appeal by certiorari from the
decision of the Court of Appeals.

ISSUE
WON the policy in question became null and void
upon the declaration of war between United States
and Germany
!"#$%&"'( *+,- 16


HELD
YES
Ratio The Philippine Insurance Law (Act No. 2427,
as amended,) in section 8, provides that "anyone
except a public enemy may be insured." It stands to
reason that an insurance policy ceases to be
allowable as soon as an insured becomes a public
enemy.
> Effect of war, generally. - All intercourse
between citizens of belligerent powers which is
inconsistent with a state of war is prohibited by
the law of nations. Such prohibition includes all
negotiations, commerce, or trading with the
enemy; all acts which will increase, or tend to
increase, its income or resources; all acts of
voluntary submission to it; or receiving its
protection; also all acts concerning the
transmission of money or goods; and all contracts
relating thereto are thereby nullified. It further
prohibits insurance upon trade with or by the
enemy, upon the life or lives of aliens engaged in
service with the enemy; this for the reason that
the subjects of one country cannot be permitted to
lend their assistance to protect by insurance the
commerce or property of belligerent, alien
subjects, or to do anything detrimental too their
country's interest. The purpose of war is to cripple
the power and exhaust the resources of the
enemy, and it is inconsistent that one country
should destroy its enemy's property and repay in
insurance the value of what has been so
destroyed, or that it should in such manner
increase the resources of the enemy, or render it
aid, and the commencement of war determines,
for like reasons, all trading intercourse with the
enemy, which prior thereto may have been lawful.
All individuals therefore, who compose the
belligerent powers, exist, as to each other, in a
state of utter exclusion, and are public enemies. (6
Couch, Cyc. of Ins. Law, pp. 5352-5353.)
> In the case of an ordinary fire policy, which
grants insurance only from year, or for some other
specified term it is plain that when the parties
become alien enemies, the contractual tie is
broken and the contractual rights of the parties, so
far as not vested. lost. (Vance, the Law on
Insurance, Sec. 44, p. 112.)
Reasoning
- The Court of Appeals overruled the contention of
the petitioner that the respondent corporation
became an enemy when the United States declared
war against Germany, relying on English and
American cases which held that a corporation is a
citizen of the country or state by and under the laws
of which it was created or organized. It rejected the
theory that nationality of private corporation is
determined by the character or citizenship of its
controlling stockholders.
- There is no question that majority of the
stockholders of the respondent corporation were
German subjects. Therefore, Huenefeld Co became
an enemy corporation upon the outbreak of the war
between the United States and Germany. The
English and American cases relied upon by the Court
of Appeals have lost their force in view of the latest
decision of the Supreme Court of the United States
in Clark vs. Uebersee Finanz Korporation, decided on
December 8, 1947, in which the controls test has
been adopted. In "Enemy Corporation" by Martin
Domke, a paper presented to the Second
International Conference of the Legal Profession held
at the Hague (Netherlands) in August. 1948 also
discussed this dilemma
> In Clark vs. Uebersee Finanz Korporation, A. G.,
dealing with a Swiss corporation allegedly
controlled by German interest, the Court: "The
property of all foreign interest was placed within
the reach of the vesting power (of the Alien
Property Custodian) not to appropriate friendly or
neutral assets but to reach enemy interest which
masqueraded under those innocent fronts. . . . The
power of seizure and vesting was extended to all
property of any foreign country or national so that
no innocent appearing device could become a
Trojan horse."
- The respondent having become an enemy
corporation on December 10, 1941, the insurance
policy issued in its favor on October 1, 1941, by the
petitioner (a Philippine corporation) had ceased to be
valid and enforcible, and since the insured goods
were burned after December 10, 1941, and during
the war, the respondent was not entitled to any
indemnity under said policy from the petitioner.
However, elementary rules of justice (in the absence
of specific provision in the Insurance Law) require
that the premium paid by the respondent for the
period covered by its policy from December 11,
1941, should be returned by the petitioner.
Disposition the appealed decision is hereby
reversed and the respondent corporation is ordered
to pay to the petitioner the sum of P77,208.33,
Philippine currency, less the amount of the premium,
in Philippine currency, that should be returned by the
petitioner for the unexpired term of the policy in
question, beginning December 11, 1941.







INSULAR LIFE ASSURANCE CO. v. EBRADO
80 SCRA 181
MARTIN; October 28, 1977

NATURE
Appeal from judgment of RTC.

FACTS
- Buenaventura Ebrado obtained a whole-life
insurance policy from Insular, for P5,882.00 with a
rider for accidental death benefits for the same
amount. He designated Carponia Ebrado as the
revocable beneficiary, referring to her as the wife.
- Afterwards, he died as a result of an accident when
he was hit by a falling branch of a tree. Carponia
filed a claim for the proceeds as the designated
beneficiary in the policy, although she admits that
she and Buenaventura were merely living as
husband and wife without the benefit of marriage.
The legal wife, Pascuala Vda De Ebrado, also filed
her claim as the widow of the deceased.
- Insular then filed an interpleader in court (CFI
Rizal) to determine to whom the proceeds should be
paid. CFI declared that Carponia was disqualified
from becoming beneficiary of the insured and
directing the Insular to pay the proceeds to the
estate of Buenaventura.

ISSUE
1. WON a common-law wife named as beneficiary in
the insurance policy of a legally married man claim
the proceeds of the same

HELD
1. NO
Ratio The prohibition that husband and wife cannot
donate to each other applies to common-law
relationships. As the appointment of a beneficiary in
insurance may be considered a donation, one cannot
name as beneficiary his common-law wife.
Reasoning
!"#$%&"'( *+,- 27

- It is quite unfortunate that the Insurance Code
does not contain any specific provision grossly
resolutory of the prime question at hand.
- Rather, general rules of civil law should be applied
to resolve the issue. Art.2011, CC states: The
contract of insurance is governed by special laws.
Matters not expressly provided for in such special
laws shall be regulated by this Code. Thus, when
not otherwise specifically provided for by the
Insurance Law, the contract of life insurance is
governed by the general rules of the civil law
regulating contracts.
- Also, Art.2012 any person who is forbidden from
receiving any donation under Article 739 cannot be
named beneficiary of a life insurance policy by the
person who cannot make a donation to him.
Common-law spouses are, definitely, barred from
receiving donations from each other.
- Art.739, CC: The following donations shall be void:
1. Those made between persons who were guilty of
adultery or concubinage at the time of donation;
- In essence, a life insurance policy is no different
from a civil donation insofar as the beneficiary is
concerned. Both are founded upon the same
consideration: liberality. A beneficiary is like a
donee, because from the premiums of the policy
which the insured pays out of liberality, the
beneficiary will receive the proceeds or profits of said
insurance. As a consequence, the proscription in
Art.739 CC should equally operate in life insurance
contracts. The mandate of Art.2012 cannot be laid
aside: any person who cannot receive a donation
cannot be named as beneficiary in the life insurance
policy of the person who cannot make the donation.
- Policy considerations and dictates of morality
rightly justify the institution of a barrier between
common-law spouses in regard to property relations
since such relationship ultimately encroaches upon
the nuptial and filial rights of the legitimate family.
There is every reason to hold that the bar in
donations between legitimate spouses and those
between illegitimate ones should be enforced in life
insurance policies since the same are based on
similar consideration.
- So long as marriage remains the threshold of
family laws, reason and morality dictate that the
impediments imposed upon married couple should
likewise be imposed upon extra-marital relationship.
If legitimate relationship is circumscribed by these
legal disabilities, with more reason should an illicit
relationship be restricted by these disabilities.
Disposition Decision AFFIRMED.
CONSUEGRA v. GSIS
37 SCRA 315
ZALDIVAR; January 30, 1971

NATURE
Appeal from the decision of the Court of First
Instance of Surigao del Norte awarding the 8/16 part
of the proceeds of the deceased Consuegras
retirement benefits to Rosario Diaz.

FACTS
- The late Jose Consuegra, at the time of his death,
was employed as a shop foreman of the office of the
District Engineer in the province of Surigao del
Norte. In his lifetime, Consuegra contracted two
marriages, the first with herein respondent Rosario
Diaz, solemnized in the parish church of San Nicolas
de Tolentino, Surigao, Surigao, on July 15, 1937, out
of which marriage were born two children, namely,
Jose Consuegra, Jr. and Pedro Consuegra, but both
predeceased their father; and the second, which was
contracted in good faith while the first marriage was
subsisting, with herein petitioner Basilia Berdin, on
May 1, 1957 in the same parish and municipality, out
of which marriage were born seven children, namely,
Juliana, Pacita, Maria Lourdes, Jose, Rodrigo, Lenida
and Luz, all surnamed Consuegra.
- Being a member of the Government Service
Insurance System (GSIS, for short) when Consuegra
died on September 26, 1965, the proceeds of his life
insurance under policy No. 601801 were paid by the
GSIS to petitioner Basilia Berdin and her children
who were the beneficiaries named in the policy.
- However, Consuegra did not designate any
beneficiary who would receive the retirement
insurance benefits due to him. Respondent Rosario
Diaz, the widow by the first marriage, filed a claim
with the GSIS asking that the retirement insurance
benefits be paid to her as the only legal heir of
Consuegra, considering that the deceased did not
designate any beneficiary with respect to his
retirement insurance benefits. Petitioner Basilia
Berdin and her children, likewise, filed a similar claim
with the GSIS, asserting that being the beneficiaries
named in the life insurance policy of Consuegra, they
are the only ones entitled to receive the retirement
insurance benefits due the deceased Consuegra.
Resolving the conflicting claims, the GSIS ruled that
the legal heirs of the late Jose Consuegra were
Rosario Diaz, his widow by his first marriage who is
entitled to one-half, or 8/16, of the retirement
insurance benefits, on the one hand; and Basilia
Berdin, his widow by the second marriage and their
seven children, on the other hand, who are entitled
to the remaining one-half, or 8/16, each of them to
receive an equal share of 1/16.
- Dissatisfied with the foregoing ruling and
apportionment made by the GSIS, Basilia Berdin and
her children filed on October 10, 1966 a petition for
mandamus with preliminary injunction in the Court
of First Instance of Surigao.
- The CFI of Surigao ruled in favor of respondent
Rosario Diaz and upheld the ruling of GSIS in all
aspect. Thus, Basilia Berdin and her children
appealed said decision to the Supreme Court.

ISSUE
WON GSIS was correct in awarding half of the
retirement benefit of the deceased to Rosario Diaz,
the first wife, notwithstanding the fact that the
petitioners were named as beneficiaries of the life
insurance

HELD
YES
- The GSIS offers two separate and distinct systems
of benefits to its members, one is the life insurance
and the other is the retirement insurance. These two
distinct systems of benefits are paid out from two
distinct and separate funds that are maintained by
the GSIS. Thus, it doesnt necessarily mean that the
beneficiaries in the life insurance are also the
beneficiaries in the retirement insurance.
- Consuegra started in the government service
sometime during the early part of 1943, or before
1943. In 1943 Com. Act 186 was not yet amended,
and the only benefits then provided for in said Com.
Act 186 were those that proceed from a life
insurance. Upon entering the government service
Consuegra became a compulsory member of the
GSIS, being automatically insured on his life,
pursuant to the provisions of Com. Act 186 which
was in force at the time. During 1943 the operation
of the Government Service Insurance System was
suspended because of the war, and the operation
was resumed sometime in 1946. When Consuegra
designated his beneficiaries in his life insurance he
could not have intended those beneficiaries of his life
insurance as also the beneficiaries of his retirement
insurance because the provisions on retirement
insurance under the GSIS came about only when
Com. Act 186 was amended by Rep. Act 660 on June
16, 1951. Hence, it cannot be said that because
herein appellants were designated beneficiaries in
!"#$%&"'( *+,- 2.

Consuegra's life insurance they automatically
became the beneficiaries also of his retirement
insurance.
- The provisions of subsection (b) of Section 11 of
Commonwealth Act 186, as amended by Rep. Act
660, clearly indicate that there is need for the
employee to file an application for retirement
insurance benefits when he becomes a member of
the GSIS, and he should state in his application the
beneficiary of his retirement insurance. Hence, the
beneficiary named in the life insurance does not
automatically become the beneficiary in the
retirement insurance unless the same beneficiary in
the life insurance is so designated in the application
for retirement insurance.
- In the case of the proceeds of a life insurance, the
same are paid to whoever is named the beneficiary
in the life insurance policy. As in the case of a life
insurance provided for in the Insurance Act, the
beneficiary in a life insurance under the GSIS may
not necessarily be an heir of the insured. The
insured in a life insurance may designate any
person as beneficiary unless disqualified to be
so under the provisions of the Civil Code. And in
the absence of any beneficiary named in the life
insurance policy, the proceeds of the insurance will
go to the estate of the insured.
- On the other hand, the beneficiary of the
retirement insurance can only claim the proceeds of
the retirement insurance if the employee dies before
retirement. If the employee failed or overlooked to
state the beneficiary of his retirement insurance, the
retirement benefits will accrue to his estate and will
be given to his legal heirs in accordance with law, as
in the case of a life insurance if no beneficiary is
named in the insurance policy.
Disposition Petition Denied. It is Our view,
therefore, that the respondent GSIS had correctly
acted when it ruled that the proceeds of the
retirement insurance of the late Jose Consuegra
should be divided equally between his first living wife
Rosario Diaz, on the one hand, and his second wife
Basilia Berdin and his children by her.

SSS v. DAVAC
17 SCRA 863
BARRERA: July 30, 1966

NATURE
APPEAL from a resolution Of the Social Security
Commission.

FACTS
- Petronilo Davac, became a member of the Social
Security System (SSS for short) on September 1,
1957. In the Member's Record he designated
respondent, Candelaria Davac as his beneficiary and
indicated his relationship to her as that of "wife".
- He died on April 5, 1959. It appears that the
deceased contracted two marriages, the first, with
Lourdes Tuplano on August 29, 1946, who bore him
a child, Romeo Davac, and the second, with
Candelaria Davac on January 18, 1949, with whom
he had a minor daughter, Elizabeth Davac. Both filed
their claims for death benefit with the SSS.
- Social Security Commission issued the resolution
declaring respondent Candelaria Davac as the person
entitled to receive the death benefits payable for the
death of Petronilo Davac.

ISSUES
1. WON the Social Security Commission Candelaria
Davac is entitled to receive the death benefits
2. WON a beneficiary under the Social Security
System partakes of the nature of a beneficiary in a
life insurance policy and, therefore the designation
made in the person DAVAC as bigamous wife is null
and void, because it contravenes the provisions of
the Civil Code
3. WON the benefits accruing from membership with
SSS forms part of the conjugal property thus the
resolution deprives the lawful wife of her share in the
conjugal property as well as of her own and her
child's legitime in the inheritance

HELD
1. YES
- Section 13, RA1161 provides that the beneficiary
"as recorded" by the employee's employer is the one
entitled to the death benefits.
- Section 13, Republic Act No. 1161, as amended by
Republic Act No. 1792, in force at the time of
Petronilo Davac's death provides: Upon the covered
employee's death or total and permanent disability
under such conditions as the Commission may
define, before becoming eligible for retirement and if
either such death or disability is not compensable
under the Workmen's Compensation Act, he or. in
case of his death, his beneficiaries, as recorded by
his employer shall be entitled to the following
benefit:
- In Tecson vs. Social Security System. Section 13
was construed:"it may be true that the purpose of
the coverage under the Social Security System is
protection of the employee as well as of his family,
but this purpose or intention of the law cannot be
enforced to the extent of contradicting the very
provisions of said law contained in Section 13,
thereof
- When the provisions of a law are clear and explicit,
the courts can do nothing but apply its clear and
explicit provisions (Velasco vs. Lopez)
2. NO
- The disqualification mentioned in Article 739 is not
applicable to herein appellee Candelaria Davac
because she was not guilty of concubinage, there
being no proof that she had knowledge of the
previous marriage of her husband Petronilo.
ART. 2012. Any person who is forbidden from
receiving any donation under Article 739 cannot be
named beneficiary of a life insurance policy by the
person who cannot make any donation to him
according to said article.
ART. 739. The following donations shall be void:
(1) Those made between persons who were guilty
of adultery or concubinage at the time of the
donation; (the court did not decide whether this
partakes the nature of a life insurance policy)
3. NO
- The benefit receivable under the Act is in the
nature of a special privilege or an arrangement
secured by the law pursuant to the policy of the
State to provide social security to the workingmen.
The amounts that may thus be received cannot be
considered as property earned by the member
during his lifetime. His contribution to the fund
constitutes only an insignificant portion thereof.
Then, the benefits are specifically declared not
transferable, and exempted from tax, legal
processes, and lien. Furthermore, in the settlement
of claims thereunder, the procedure to be observed
is governed not by the general provisions of law, but
by rules and regulations promulgated by the
Commission. Thus, if the money is payable to the
estate of a deceased member, it is the Commission,
not the probate or regular court that determines the
person or persons to whom it is payable.
- They are disbursed from a public special fund
created by Congress.The sources of this special fund
are the covered employee's contribution (equal to 2-
1/2 per cent of the employee's monthly
compensation) ; the employer's 'Contribution
(equivalent to 3-1/2 per cent of the monthly
compensation of the covered employee) ;and the
Government contribution which consists in yearly
appropriation of public funds to assure the
!"#$%&"'( *+,- 2/

maintenance of an adequate working balance of the
funds of the System. Additionally, Section 21 of the
Social Security Actprovides that the benefits
prescribed in this Act shall not be diminished and the
Government of the Republic of the Philippines
accepts general responsibility for the solvency of the
System.
- The benefits under the Social Security Act are not
intended by the lawmaking body to form part of the
estate of the covered members.
- Social Security Act is not a law of succession.
Disposition Resolution of the Social Security
Commission appealed is affirmed

FRANCISCO DEL VAL v. ANDRES DEL VAL
29 PHIL 534
MORELAND; February 16, 1915

NATURE
Appeal from a judgment of the Court of First
Instance of the city of Manila dismissing the
complaint with costs.

FACTS
- Plaintiffs and defendant are brothers and sisters;
that they are the only heirs at law and next of kin of
Gregorio Nacianceno del Val, who died in Manila on
August 4, 1910, intestate
- During the lifetime of the deceased he took out
insurance on his life for the sum of P40,000 and
made it payable to the defendant ANDRES DEL VAL
as sole beneficiary. After his death the defendant
collected the face of the policy. From said policy he
paid the sum of P18,365.20 to redeem certain real
estate which the decedent had sold to third persons
with a right to repurchase.
- The redemption of said premises was made by the
attorney of defendant ANDRES in the name of the
plaintiffs and the defendant as heirs of the deceased
vendor. It further appears from the pleadings that
the defendant, on the death of the deceased, took
possession of most of his personal property, which
he still has in his possession, and that he has also
the balance on said insurance policy amounting to
P21,634.80.
- Plaintiffs contend that the amount of the insurance
policy belonged to the estate of the deceased and
not to the defendant personally; that, therefore, they
are entitled to a partition not only of the real and
personal property, but also of the P40,000 life
insurance. The complaint prays a partition of all the
property, both real and personal, left by the
deceased; that the defendant account for
P21,634.80, and that the sum be divided equally
among the plaintiffs and defendant along with the
other property of deceased.
- The defendant denies the material allegations of
the complaint and sets up as special defense and
counterclaim that the redemption of the real estate
sold by his father was made in the name of the
plaintiffs and himself instead of in his name alone
without his knowledge or consent. Andres contends
that it was not his intention to use the proceeds of
the insurance policy for the benefit of any person but
himself, he alleging that he was and is the sole
owner thereof and that it is his individual property.
He, therefore, asks that he be declared the owner of
the real estate redeemed by the payment of the
P18,365.20, the owner of the remaining P21,634.80,
the balance of the insurance policy, and that the
plaintiffs account for the use and occupation of the
premises so redeemed since the date of the
redemption.
- The trial court refused to give relief to either party
and dismissed the action. In this appeal, it is claimed
by the attorney for the plaintiffs that insurance
provisions in the Code of Commerce are
subordinated to the provisions of the Civil Code as
found in article 1035. This article reads:
"An heir by force of law surviving with others of the
same character to a succession must bring into the
hereditary estate the property or securities he may
have received from the deceased during the life of
the same, by way of dowry, gift, or for any good
consideration, in order to compute it in fixing the
legal portions and in the account of the division."
- Counsel also claims that the proceeds of the
insurance policy were a donation or gift made by the
father during his lifetime to the defendant and that,
as such, its ultimate destination is determined by
those provisions of the Civil Code which relate to
donations, especially article 819. This article provides
that "gifts made to children which are not
betterments shall be considered as part of their legal
portion."

ISSUES
1. WON the insurance belongs to the defendant and
not to the decedents estate
2. WON the Civil code provisions on succession
prevail over any other law with respect to the
insurance

HELD
1. YES
- The SC agreed with the finding of the trial court
that the proceeds of the life-insurance policy belong
exclusively to the defendant as his individual and
separate property, we agree. That the proceeds of
an insurance policy belong exclusively to the
beneficiary and not to the estate of the person
whose life was insured, and that such proceeds are
the separate and individual property of the
beneficiary, and not of the heirs of the person whose
life was insured, is the doctrine in America. We
believe that the same doctrine obtains in these
Islands by virtue of section 428 of the Code of
Commerce, which reads:
"The amounts which the underwriter must deliver
to the person insured, in fulfillment of the
contract, shall be the property of the latter, even
against the claims of the legitimate heirs or
creditors of any kind whatsoever of the person
who effected the insurance in favor of the former."
2. NO
- The contract of life insurance is a special contract
and the destination of the proceeds thereof is
determined by special laws which deal exclusively
with that subject. The Civil Code has no provisions
which relate directly and specifically to life-insurance
contracts or to the destination of life insurance
proceeds. That subject is regulated exclusively by
the Code of Commerce which provides for the terms
of the contract, the relations of the parties and the
destination of the proceeds of the policy.
- Assuming that the proceeds of the life-insurance
policy being the exclusive property of the defendant
and he having used a portion thereof in the
repurchase of the real estate sold by the decedent
prior to his death with right to repurchase, and such
repurchase having been made and the conveyance
taken in the names of all of the heirs instead of the
defendant alone, plaintiffs claim that the property
belongs to the heirs in common and not to the
defendant alone.
- The Court rejected this contention unless the fact
appear or be shown that the defendant acted as he
did with the intention that the other heirs should
enjoy with him the ownership of the estate ---- in
other words, that he proposed, in effect, to make a
gift of the real estate to the other heirs. If it is
established by the evidence that was his intention
and that the real estate was delivered to the
plaintiffs with that understanding, then it is probable
that their contention is correct and that they are
entitled to share equally with the defendant therein.
!"#$%&"'( *+,- 20

If, however, it appears from the evidence in the case
that the conveyances were taken in the name of the
plaintiffs without his knowledge or consent, or that it
was not his intention to make a gift to them of the
real estate, then it belongs to him. If the facts are as
stated, he has two remedies. The one is to compel
the plaintiffs to reconvey to him and the other is to
let the title stand with them and to recover from
them the sum he paid on their behalf.
- For the complete and proper determination of the
questions at issue in this case, the Court was of the
opinion that the cause should be returned to the trial
court with instructions to permit the parties to frame
such issues as will permit the settlement of all the
questions involved and to introduce such evidence as
may be necessary for the full determination of the
issues framed. Upon such issues and evidence taken
thereunder the court will decide the questions
involved according to the evidence, subordinating his
conclusions of law to the rules laid down in this
opinion. REMANDED.

GERCIO v. SUN LIFE ASSURANCE OF CANADA
48 PHIL 53
MALCOLM; September 28, 1925

NATURE
Mandamus to compel Sun Life Assurance Co. of
Canada to change the beneficiary in the policy issued
by the defendant company on the life of the plaintiff
Hilario Gercio

FACTS
- On January 29, 1910, the Sun Life Assurance Co. of
Canada issued an insurance policy on the life of
Hilario Gercio. The policy was what is known as a 20-
year endowment policy. By its terms, the insurance
company agreed to insure the life of Hilario Gercio
for the sum of P2,000, to be paid him on February 1,
1930, or if the insured should die before said date,
then to his wife, Mrs. Andrea Zialcita, should she
survive him; otherwise to the executors,
administrators, or assigns of the insured. The policy
did not include any provision reserving to the insured
the right to change the beneficiary.
- On the date the policy was issued, Andrea Zialcita
was the lawful wife of Hilario Gercio. Towards the
end of the year 1919, she was convicted of the crime
of adultery. On September 4, 1920, a decree of
divorce was issued in civil case no. 17955, which had
the effect of completely dissolving their bonds of
matrimony
- On March 4, 1922, Hilario Gercio formally notified
the Sun Life that he had revoked his donation in
favor of Andrea Zialcita, and that he had designated
in her stead his present wife, Adela Garcia de Gercio,
as the beneficiary of the policy. Gercio requested the
insurance company to eliminate Andrea Zialcita as
beneficiary. This, the insurance company has refused
and still refuses to do.

ISSUES
1. (Preliminary) WON the provisions of the Code of
Commerce and the Civil Code shall be in force in
1910, or the provisions of the Insurance Act now in
force, or the general principles of law, guide the
court in its decision
2. WON the insured, the husband, has the power to
change the beneficiary, the former wife, and to name
instead his actual wife, where the insured and the
beneficiary have been divorced and where the policy
of insurance does not expressly reserve to the
insured the right to change the beneficiary

HELD
1. Whether the case be considered in the light of the
Code of Commerce, the Civil Code, or the Insurance
Act, the deficiencies in the law will have to be
supplemented by the general principles prevailing on
the subject. To that end, we have gathered the rules
which follow from the best considered American
authorities. In adopting these rules, we do so with
the purpose of having the Philippine Law of
Insurance conform as nearly as possible to the
modern Law of Insurance as found in the United
States proper.
- Courts first duty is to determine what law should
be applied to the facts. The insurance policy was
taken out in 1910, that the Insurance Act. No. 2427,
became effective in 1914, and that the effort to
change the beneficiary was made in 1922.
- Code of Commerce- there can be found in it no
provision either permitting or prohibiting the insured
to change the beneficiary.
- Civil Code- it would be most difficult, if indeed it is
practicable, to test a life insurance policy by its
provisions. In the case of Del Val vs. Del Val, it
declined to consider the proceeds of the insurance
policy as a donation or gift, saying "the contract of
life insurance is a special contract and the
destination of the proceeds thereof is determined by
special laws which deal exclusively with that subject.
The Civil Code has no provisions which relate directly
and specifically to life-insurance contracts or to the
destination of life-insurance proceeds. . . ."
- Insurance Act- there is likewise no provision
either permitting or prohibiting the insured to change
the beneficiary.
2. NO
Ratio The wife has an insurable interest in the life of
her husband. The beneficiary has an absolute vested
interest in the policy from the date of its issuance
and delivery. So when a policy of life insurance is
taken out by the husband in which the wife is named
as beneficiary, she has a subsisting interest in the
policy. And this applies to a policy to which there are
attached the incidents of a loan value, cash
surrender value, an automatic extension by
premiums paid, and to an endowment policy, as well
as to an ordinary life insurance policy. If the husband
wishes to retain to himself the control and ownership
of the policy he may so provide in the policy. But if
the policy contains no provision authorizing a change
of beneficiary without the beneficiary's consent, the
insured cannot make such change. Accordingly, it is
held that a life insurance policy of a husband made
payable to the wife as beneficiary, is the separate
property of the beneficiary and beyond the control of
the husband.
- Unlike the statutes of a few jurisdictions, there is
no provision in the Philippine Law permitting the
beneficiary in a policy for the benefit of the wife of
the husband to be changed after a divorce. It must
follow, therefore, in the absence of a statute to the
contrary, that if a policy is taken out upon a
husband's life the wife is named as beneficiary
therein, a subsequent divorce does not destroy her
rights under the policy.
Reasoning
- Yore vs. Booth
. . . It seems to be the settled doctrine, with but
slight dissent in the courts of this country, that a
person who procures a policy upon his own life,
payable to a designated beneficiary, although he
pays the premiums himself, and keeps the policy
in his exclusive possession, has no power to
change the beneficiary, unless the policy itself, or
the charter of the insurance company, so provides.
In policy, although he has parted with nothing,
and is simply the object of another's bounty, has
acquired a vested and irrevocable interest in the
policy, which he may keep alive for his own benefit
by paying the premiums or assessments if the
person who effected the insurance fails or refuses
to do so.
!"#$%&"'( *+,- 21

- Connecticut Mutual Life Insurance Company
vs Schaefer
We do not hesitate to say, however, that a policy
taken out in good faith and valid at its inception, is
not avoided by the cessation of the insurable
interest, unless such be the necessary effect of the
provisions of the policy itself.. . . .In our judgment
of life policy, originally valid, does not cease to be
so by the cessation of the assured party's interest
in the life insured.
- Central National Bank of Washington City vs.
Hume
It is indeed the general rule that a policy, and the
money to become due under it, belong, the
moment it is issued, to the person or persons
named in it as the beneficiary or beneficiaries, and
that there is no power in the person procuring the
insurance, by any act of his, by deed or by will, to
transfer to any other person the interest of the
person named.
- In re Dreuil & Co.
In so far as the law of Louisiana is concerned, it
may also be considered settled that where a policy
is of the semitontine variety, as in this case, the
beneficiary has a vested right in the policy, of
which she cannot be deprived without her consent
- Wallace vs Mutual Benefit Life Insurance Co.
As soon as the policy was issued Mrs. Wallace
acquired a vested interest therein, of which she
could not be deprived without her consent, except
under the terms of the contract with the insurance
company. No right to change the beneficiary was
reserved. Her interest in the policy was her
individual property, subject to be divested only by
her death, the lapse of time, or by the failure of
the insured to pay the premiums. She could keep
the policy alive by paying the premiums, if the
insured did not do so. It was contingent upon
these events, but it was free from the control of
her husband. He had no interest in her property in
this policy, contingent or otherwise. Her interest
was free from any claim on the part of the insured
or his creditors. He could deprive her of her
interest absolutely in but one way, by living more
than twenty years.
- Filley vs. Illinois Life Insurance Company
The benefit accruing from a policy of life
insurance upon the life of a married man, payable
upon his death to his wife, naming her, is payable
to the surviving beneficiary named, although she
may have years thereafter secured a divorce from
her husband, and he was thereafter again married
to one who sustained the relation of wife to him at
the time of his death.
The rights of a beneficiary in an ordinary life
insurance policy become vested upon the issuance
of the policy, and can thereafter, during the life of
the beneficiary, be defeated only as provided by
the terms of the policy.
- On the admitted facts and the authorities
supporting the nearly universally accepted principles
of insurance, we are irresistibly led to the conclusion
that the question at issue must be answered in the
negative
Disposition The judgment appealed from will be
reversed and the complaint ordered dismissed as to
the appellant.

SEPARATE OPINION

JOHNSON [concur]
- I agree with the majority of the court, that the
judgment of the lower court should be revoked, but
for a different reason. The purpose of the petition is
to have declared the rights of certain persons in an
insurance policy which is not yet due and payable. It
may never become due and payable. The premiums
may not be paid, thereby rendering the contract of
insurance of non effect, and many other things may
occur, before the policy becomes due, which would
render it non effective. The plaintiff and the other
parties who are claiming an interest in said policy
should wait until there is something due them under
the same. For the courts to declare now who are the
persons entitled to receive the amounts due, if they
ever become due and payable, is impossible, for the
reason that nothing may ever become payable under
the contract of insurance, and for many reasons such
persons may never have a right to receive anything
when the policy does become due and payable. In
my judgment, the action is premature and should
have been dismissed.

PHIL. AMERICAN LIFE INSURANCE v. PINEDA
175 SCRA 416
PARAS; July 19, 1989

NATURE
Petition for review on certiorari the orders of CFI
Judge Pineda

FACTS
- In 1968, Private Respondent Rodolfo Dimayuga
procured an ordinary life insurance policy from the
petitioner company and designated his wife and
children as irrevocable beneficiaries. On Feb. 22,
1980, Dimayuga filed with the CFI a petition to
amend the designation of the beneficiaries in his life
policy from irrevocable to revocable. Petitioner filed
an Urgent Motion to reset hearing as well as its
comment and/or Opposition to the respondents
petition.
- Respondent Judge denied petitioners Urgent
Motion, thus allowing private respondent to adduce
evidence, the consequence of which was the
issuance of the questioned Order granting the
petition. Petitioner then filed a MFR which was also
denied hence this petition.

ISSUE
1. WON the designation of the irrevocable
beneficiaries could be changed or amended without
the consent of all the irrevocable beneficiaries
2. WON the irrevocable beneficiaries herein, one of
whom is already deceased while the others are all
minors could validly give consent to the change or
amendment in the designation of the irrevocable
beneficiaries

HELD
1. NO
- Based on the provision of their contract and the law
applicable, it is only with the consent of all the
beneficiaries that any change or amendment in the
policy concerning the irrevocable beneficiaries may
be legally and validly effected. Both the law and the
Policy do not provide for any other exception.
Reasoning
- Since the policy was procured in 1968, the
applicable law in this case is the Insurance Act and
under that law, the beneficiary designated in a life
insurance contract cannot be changed without the
consent of the beneficiary because he has a vested
interest in the policy.
- The Beneficiary Designation Indorsement in the
policy in the name of Dimayuga states that the
designation of the beneficiaries is irrevocable: no
right or privilege under the Policy may be exercised,
or agreement made with the Company to any change
in or amendment to the Policy, without the consent
of the said beneficiary/beneficiaries.
- Contracts which are the private laws of the
contracting parties should be fulfilled according to
the literal sense of their stipulations, if their terms
are clear and leave no room for doubt as to the
intention of the contracting parties, for contracts are
!"#$%&"'( *+,- 22

obligatory, no matter in what form they may be,
whenever the essential requisites for their validity
are present.
- Finally, the fact that the contract of insurance does
not contain a contingency when the change in the
designation of beneficiaries could be validly effected
means that it was never within the contemplation of
the parties.
2. NO
- The parent-insured cannot exercise rights and/or
privileges pertaining to the insurance contract, for
otherwise, the vested rights of the irrevocable
beneficiaries would be rendered inconsequential. The
alleged acquiescence of the 6 children beneficiaries
cannot be considered an effective ratification to the
change of the beneficiaries from irrevocable to
revocable. They were minors at the time, and could
not validly give consent. Neither could they act
through their father-insured since their interests are
quite divergent from one another.
Disposition questioned Orders of respondent judge
are nullified and set aside.

SUN LIFE ASSURANCE v. INGERSOLL
41 PHIL 331
STREET; November 8, 1921

NATURE
Action of interpleader

FACTS
- April 16, 1918, Sun Life Assurance Company of
Canada (Sun Life), in consideration of the payment
of a stipulated annual premium during the period of
the policy, or until the premiums had been
completely paid for twenty years, issued a policy of
insurance on the life of Dy Poco for US$12,500,
payable to the said assured or his assigns on the
21st day of February, 1938, and if he should die
before that date then to his legal representatives.
- June 23, 1919, the assured, Dy Poco, was
adjudged an involuntary insolvent by the CFI Manila,
and Frank B. Ingersoll was appointed assignee of his
estate.
- July 10, 1919, Dy Poco died, and on August 21,
1919, Tan Sit, was duly appointed as the
administratrix of his intestate estate.
- By the terms of the policy it was provided that after
the payment of three full premiums, the assured
could surrender the policy to the company for a
"cash surrender value," indicated in an annexed
table; but inasmuch as no more than two premiums
had been paid upon the policy now in question up to
the time of the death of the assured, this provision
had not become effective; and it does not appear
that the company would in accordance with its own
usage or otherwise have made any concession to the
assured in the event he had desired, before his
death, to surrender the policy. It must therefore be
accepted that this policy had no cash surrender
value, at the time of the assured's death, either by
contract or by convention practice of the company in
such cases.
- Both Ingersoll, as assignee, and Tan Sit, as
administratix of Dy Poco's estate, asserted claims to
the proceeds of the policy. The lower court found
that Ingersoll had a better right and ordered Sun Life
to pay the insurance proceeds to him.

ISSUE
WON Ingersoll, as assignee, has a right to the
proceeds of the insurance

HELD
NO
On the Philippine Insolvency Law (Act No. 1956)
- The property and interests of the insolvent which
become vested in the assignee of the insolvent are
specified in section 32 of the Insolvency Law which
reads as follows:
"SEC. 32. As soon as an assignee is elected
or appointed and qualified, the clerk of the court
shall, by an instrument under his hand and seal of
the court, assign and convey to the assignee all the
real and personal property, estate, and effects of the
debtor with all his deeds, books, and papers relating
thereto, and such assignment shall relate back to the
commencement of the proceedings in insolvency,
and shall relate back to the acts upon which the
adjudication was founded, and by operation of law
shall vest the title to all such property, estate, and
effects in the assignee, although the same is then
attached on mesne process, as the property of the
debtor. Such assignment shall operate to vest in the
assignee all of the estate of the insolvent debtor not
exempt by law from execution."
- the Insolvency Law is in great part a copy of the
Insolvency Act of California, enacted in 1895, though
it contains a few provisions from the American
Bankruptcy Law of 1898
- Under each of said laws the assignee acquires all
the real and personal property, estate, and effects of
the debtor, not exempt by law from execution, with
all deeds, books and papers relating thereto; and
while this language is broad, it nevertheless lacks
the comprehensiveness of section 70 (a) of the
American Bankruptcy Law of 1898 in at least two
particulars; for under subsection 3 of section 70 (a)
of the last mentioned law, the trustee in bankruptcy
acquires the right to exercise any powers which the
insolvent might have exercised for his own benefit,
and under subsection 5 the trustee acquires any
property of the insolvent which the latter could by
any means have assigned to another. The Insolvency
Law here in force, in common with the predecessor
laws above-mentioned, contains nothing similar to
these provisions.
On the applicability of the Insolvency Law
- Sec 32 of the Insolvency Law among other things,
declares that the assignment to be made by the clerk
of the court "shall operate to vest in the assignee all
of the estate of the insolvent debtor not exempt by
law from execution." Moreover, by section 24, the
court is required, upon making an order adjudicating
any person insolvent, to stay any civil proceedings
pending against him; and it is declared in section 60
that no creditor whose debt is provable under the Act
shall be allowed, after the commencement of
proceedings in insolvency, to prosecute to final
judgment any action therefor against the debtor. In
connection with the foregoing may be mentioned
subsections 1 and 2 of section 36, as well as the
opening words of section 33, to the effect that the
assignee shall have the right and power to recover
and to take into his possession, all of the estate,
assets, and claims belonging to the insolvent, except
such as are exempt by law from execution.
- These provisions clearly evince an intention to vest
in the assignee, for the benefit of all the creditors of
the insolvent, such elements of property and
property right as could be reached and subjected by
process of law by any single creditor suing alone.
And this is exactly as it should be: for it cannot be
supposed that the Legislature would suppress the
right of action of every individual creditor upon the
adjudication of insolvency, and at the same time
allow the insolvent debtor to retain anything subject
to the payment of his debts in a normal state of
solvency.
- "leviable assets" and "assets in insolvency" are
practically coextensive terms. Hence, in determining
what elements of value constitute assets in
insolvency, SC is at liberty to consider what
elements of value are subject to be taken upon
execution, and vice versa.
!"#$%&"'( *+,- 23

On whether a policy of insurance having no cash
surrender value, but payable to insured or his legal
representative, is property that may be taken upon
execution against him.
- Philippine laws declare no exemption with respect
to insurance policies; and this species of property is
not enumerated, in section 48 of the Insolvency Law,
among items from the ownership of which the
assignee is excluded. Moreover, all life insurance
policies are declared by law to be assignable,
regardless of whether the assignee has an insurable
interest in the life of the insured or not (Insurance
Act No. 2427, sec. 166).
- SC has held that insurance policies having a
present cash surrender value are subject to be taken
upon execution. (Misut Garcia vs. West Coast San
Francisco Life Ins. Co.)
- a policy devoid of a cash surrender value cannot be
either "leviable assets" or "assets in insolvency."
- the assignee in insolvency acquired no beneficial
interest in the policy of insurance in question; that
its proceeds are not liable for any of the debts
provable against the insolvent in the pending
proceedings, and that said proceeds should therefore
be delivered to his administratrix.
On applicable US case
- In re McKinney: no beneficial interest in the policy
had ever passed to the assignee over and beyond
what constituted the surrender value, and that the
legal title to the policy was vested in the assignee
merely in order to make the surrender value-
available to him. The assignee should surrender the
policy upon the payment to him of said value, as he
was in fact directed to do. The assignee in
bankruptcy had no right to keep the estate unsettled
for an indefinite period, for the mere purpose of
speculating upon the chances of the bankrupt's
death. As regards everything beyond the surrender
value, the assignee in bankruptcy would, after the
discharge of the bankrupt, have no insurable interest
in the life of the bankrupt.
- surrender value of a policy "arises from the fact
that the fixed annual premiums is much in excess of
the annual risk during the earlier years of the policy,
an excess made necessary in order to balance the
deficiency of the same premium to meet the annual
risk during the latter years of the policy. This excess
in the premium paid over the annual cost of
insurance, with accumulations of interest, constitutes
the surrender value. Though this excess of premiums
paid is legally the sole property of the company, still
in practical effect, though not in law, it is moneys of
the assured deposited with the company in advance
to make up the deficiency in later premiums to cover
the annual cost of insurance, instead of being
retained by the assured and paid by him to the
company in the shape of greatly-increased
premiums, when the risk is greatest. It is the 'net
reserve' required by law to be kept by the company
for the benefit of the assured, and to be maintained
to the credit of the policy. So long as the policy
remains in force the company has not practically any
beneficial interest in it, except as its custodian, with
the obligation to maintain it unimpaired and suitably
invested for the benefit of the insured. This is the
practical, though not the legal, relation of the
company to this fund. "Upon the surrender of the
policy before the death of the assured, the company,
to be relieved from all responsibility for the increased
risk, which is represented by this accumulating
reserve, could well afford to surrender a considerable
part of it to the assured, or his representative. A
return of a part in some form or other is now Usually
made." (In re McKinney)
- the stipulation providing for a cash surrender value
is a comparatively recent innovation in life insurance.
Formerly the contracts provided as they still
commonly do in the policies issued by fraternal
organizations and benefit societies for the
payment of a premium sufficient to keep the
estimated risk covered; and in case of a lapse the
policy-holder received nothing. Furthermore, the
practice is common among insurance companies
even now to concede nothing in the character of
cash surrender value, until three full premiums have
been paid, as in this case.
- CONLUSION (from this case and other English and
American cases cited following the same opinion):
the assignee acquires no beneficial interest in
insurance effected on the life of the insolvent, except
to the extent that such insurance contains assets
which can be realized upon as of the date when the
petition of insolvency is filed. The explanation is to
be found in the consideration that the destruction of
a contract of life insurance is not only highly
prejudicial to the insured and those dependent upon
him, but is inimical to the interests of society.
Insurance is a species of property that should be
conserved and not dissipated. As is well known, life
insurance is increasingly difficult to obtain with
advancing years, and even when procurable after the
age of fifty, the cost is then so great as to be
practically prohibitive to many. Insolvency is a
disaster likely to overtake men in mature life; and
one who has gone through the process of bankruptcy
usually finds himself in his declining years with the
accumulated savings of years swept away and
earning power diminished. The courts are therefore
practically unanimous in refusing to permit the
assignee in insolvency to wrest from the insolvent a
policy of insurance which contains in it no present
realizable assets.
On the applicability of the Insolvency Law
- Sec 32 of the Insolvency Law among other things,
declares that the assignment to be made by the clerk
of the court "shall operate to vest in the assignee all
of the estate of the insolvent debtor not exempt by
law from execution." Moreover, by section 24, the
court is required, upon making an order adjudicating
any person insolvent, to stay any civil proceedings
pending against him; and it is declared in section 60
that no creditor whose debt is provable under the Act
shall be allowed, after the commencement of
proceedings in insolvency, to prosecute to final
judgment any action therefor against the debtor. In
connection with the foregoing may be mentioned
subsections 1 and 2 of section 36, as well as the
opening words of section 33, to the effect that the
assignee shall have the right and power to recover
and to take into his possession, all of the estate,
assets, and claims belonging to the insolvent, except
such as are exempt by law from execution.
- These provisions clearly evince an intention to vest
in the assignee, for the benefit of all the creditors of
the insolvent, such elements of property and
property right as could be reached and subjected by
process of law by any single creditor suing alone.
And this is exactly as it should be: for it cannot be
supposed that the Legislature would suppress the
right of action of every individual creditor upon the
adjudication of insolvency, and at the same time
allow the insolvent debtor to retain anything subject
to the payment of his debts in a normal state of
solvency.
- "leviable assets" and "assets in insolvency" are
practically coextensive terms. Hence, in determining
what elements of value constitute assets in
insolvency, SC is at liberty to consider what
elements of value are subject to be taken upon
execution, and vice versa.
On whether a policy of insurance having no cash
surrender value, but payable to the insured or his
legal representative, is property that may be taken
upon execution against him.
- Philippine laws declare no exemption with respect
to insurance policies; and this species of property is
!"#$%&"'( *+,- 24

not enumerated, in section 48 of the Insolvency Law,
among items from the ownership of which the
assignee is excluded. Moreover, all life insurance
policies are declared by law to be assignable,
regardless of whether the assignee has an insurable
interest in the life of the insured or not (Insurance
Act No. 2427, sec. 166).
- SC has held that insurance policies having a
present cash surrender value are subject to be taken
upon execution. (Misut Garcia vs. West Coast San
Francisco Life Ins. Co., 41 Phil., 258.)
- a policy devoid of a cash surrender value cannot be
either "leviable assets" or "assets in insolvency."
- the assignee in insolvency acquired no beneficial
interest in the policy of insurance in question; that
its proceeds are not liable for any of the debts
provable against the insolvent in the pending
proceedings, and that said proceeds should therefore
be delivered to his administratrix.
Disposition Judgment reversed. Sun Life is directed
to pay the proceeds of the policy to Tan Sit.

CHAPTER VI RESCISSION OF INSURANCE
CONTRACTS: CONCEALMENT,
MISREPRESENTATION, & BREACH OF
WARRANTIES

NG v. ASIAN CRUSADER LIFE ASSURANCE CORP
122 SCRA 461
ESCOLIN; May 30, 1983

FACTS
- On May 12, 1962, Kwong Nam applied for a 20-
year endowment insurance on his life for the sum of
P20,000, with his wife, Ng Gan Zee, as beneficiary.
- He died on Dec 1963 of cancer of the liver with
metastasis. All premiums had been paid at the time
of his death.
- Ng presented a claim for payment of the face value
of the policy. Appellant (Asian Crusader) denied the
claim on the ground that the answers given by the
insured to the questions appearing in his application
for life insurance were untrue.
-Appellant: the insured was guilty of
misrepresentation when
1) he answered "No" to the question (in the
application) of "Has any life insurance company ever
refused your application for insurance or for
reinstatement of a lapsed policy or offered you a
policy different from that applied for?" when in fact,
Insular Life denied his application for reinstatement
of his lapsed life insurance policy
2) he gave the appellant's medical examiner false
and misleading information as to his ailment and
previous operation when he said he was
operated on for a Tumor [mayoma] of the
stomach associated with ulcer of stomach. Tumor
taken out was hard and of a hen's egg size.
Operation was two years ago in Chinese General
Hospital by Dr. Yap. Claims he is completely
recovered. Medical report show that insured was
operated on for "peptic ulcer", involving the excision
of a portion of the stomach, not tumor.

ISSUE
WON there was concealment (Was appellant,
because of insured's aforesaid representation, misled
or deceived into entering the contract or in accepting
the risk at the rate of premium agreed upon?)

HELD
NO
-"concealment exists where the assured had
knowledge of a fact material to the risk, and
honesty, good faith, and fair dealing requires that he
should communicate it to the assurer, but he
designedly and intentionally withholds the same."
- It has also been held "that the concealment must,
in the absence of inquiries, be not only material, but
fraudulent, or the fact must have been intentionally
withheld."
Reasoning
1) The evidence shows that the Insular Life
Assurance Co., Ltd. approved Kwong Nam's request
for reinstatement and amendment of his lapsed
insurance policy on April 24, 1962. It results,
therefore, that when on May 12, 1962 Kwong Nam
answered `No' to the question whether any life
insurance company ever refused his application for
reinstatement of a lapsed policy he did not
misrepresent any fact.
2) Assuming that the aforesaid answer given by the
insured is false, Sec. 27
8
of the Insurance Law
nevertheless requires that fraudulent intent on the
part of the insured be established to entitle the
insurer to rescind the contract. And as correctly
observed by the lower court, "misrepresentation as a
defense of the insurer to avoid liability is an
`affirmative defense. The duty to establish such a
defense by satisfactory and convincing evidence

P
O^1.& E'& ^:.7 3/$%< %+ / .+-%$/.% +6 0-2:$/-.1 ,:2% .+,,:-0./%1 %+ %71 +%71$8 0- ;++9 6/0%78 /55
6/.%2 =0%70- 702 C-+=519;1 =70.7 /$1 ,/%1$0/5 %+ %71 .+-%$/.%8 /-9 =70.7 %71 +%71$ 7/2 -+% %71
,1/-2 +6 /2.1$%/0-0-;8 /-9 /2 %+ =70.7 71 ,/C12 -+ =/$$/-%<&O

rests upon the defendant. The evidence before the
Court does not clearly and satisfactorily establish
that defense."
-Kwong Nam had informed the appellant's medical
examiner that the tumor for which he was operated
on was ''associated with ulcer of the stomach." In
the absence of evidence that the insured had
sufficient medical knowledge as to enable him to
distinguish between "peptic ulcer" and "a tumor", his
statement that said tumor was "associated with ulcer
of the stomach" should be construed as an
expression made in good faith of his belief as to the
nature of his ailment and operation. Indeed, such
statement must be presumed to have been made by
him without knowledge of its incorrectness and
without any deliberate intent on his part to mislead
the appellant.
3) Waiver:
While it may be conceded that, from the viewpoint of
a medical expert, the information communicated was
imperfect, the same was nevertheless sufficient to
have induced appellant to make further inquiries
about the ailment and operation of the insured.
Section 32 of Insurance Law [Act No. 2427]
provides:
The right to information of material facts may be
waived either by the terms of insurance or by
neglect to make inquiries as to such facts where they
are distinctly implied in other facts of which
information is communicated.
It has been held that where, "upon the face of the
application, a question appears to be not answered
at all or to be imperfectly answered, and the insurers
issue a policy without any further inquiry, they waive
the imperfection of the answer and render the
omission to answer more fully immaterial.
Disposition the judgment appealed from is hereby
affirmed, with costs against appellant

CANILANG v. CA (GREAT PACIFIC LIFE
ASSURANCE CORP.)
223 SCRA 443
FELICIANO; June 17, 1993

NATURE
Petition for review on certiorari of the decision of the
Court of Appeals

FACTS
- June 18, 1982 Jaime Canilang was diagnosed by
Dr. Claudio to have sinus tachycardia. He was
!"#$%&"'( *+,- 25

directed by the doctor to take a tranquilizer
(Trazepam) and a beta-blocker drug (Aptin).
- August 3, 1982 Jaime consulted Dr. Claudio again
and was diagnosed to have acute bronchitis.
- August 4, 1982 Jaime applied for a nonmedical
insurance policy with Great Pacific Life Assurance
Company. He named his wife Thelma as his
beneficiary. He was issue the policy with a face
value of P19,700 effective August 9, 1982.
- August 5, 1983 Jaime died of congestive heart
failure, anemia and chronic anemia. Thelma filed her
claim but the insurance company refused to grant it
on the ground that Jaime had concealed information.
- Thelma filed a complaint against Great Pacific to
recover the insurance proceeds. She testified that
she was not aware of her husbands ailments and
that she thought he had died from a kidney disorder.
- Great Pacific presented as witness Dr. Quismorio
who testified that Jaimes insurance application was
the basis of his medical declaration and she
explained that an applicant was required to undergo
medical examination only if the applicant had
disclosed that he had previously been consulted with
a doctor and had been hospitalized.
- The Insurance Commissioner ordered Great Pacific
to pay Thelma the insurance proceeds, including
attorneys fees, holding that Jaimes illness was not
that serious as to Great Pacifics decision to insure
him and that there was no concealment on the part
of Jaime with regard to his illness.
Petitioners Claim:
> Thelma argues that the non-disclosure of Jaime
did not amount to fraud.
> She also argues that the CA erred in not holding
that the issue in the case agreed upon between the
parties before the Insurance Commission is whether
or not Jaime 'intentionally' made material
concealment in stating his state of health;
Respondents Comments:
> The CA reversed the Insurance Commissioners
decision, holding that the use of the word
'intentionally" by the Insurance Commissioner in
defining and resolving the issue agreed upon by the
parties at pre-trial before the Insurance
Commissioner was not supported by the evidence
and that the issue agreed upon by the parties had
been whether Jaime made a material concealment as
to the state of his health at the time of the filing of
insurance application, justifying the denial of the
claim.
> It also found that the failure of Jaime to disclose
previous medical consultation and treatment
constituted material information which should have
been communicated to Great Pacific to enable the
latter to make proper inquiries.

ISSUES
1. WON Jaime intentionally withheld information
from Great Pacific
2. WON the information withheld would have been
material to Great Pacifics decision to grant Jaime the
insurance policy

HELD
1. YES
Ratio Section 27 of the Insurance Code of 1978 is
properly read as referring to "any concealment
without regard to whether such concealment is
intentional or unintentional. The restoration in 1985
by B.P. Blg. 874 of the phrase "whether intentional
or unintentional" merely underscored the fact that all
throughout (from 1914 to 1985), the statute did not
require proof that concealment must be "intentional"
in order to authorize rescission by the injured party.
Reasoning
- Art. 27 of the 1978 Insurance Code reads that a
concealment entitles the injured party to rescind a
contract of insurance, which does not include the
words whether intentional or unintentional from
the previous statutes. The Insurance Commissioner
relied on this deletion in arguing that the statute
intended to limit the kinds of concealment which
generate a right to rescind on the part of the injured
party to "intentional concealments."
- In the case at bar, the nature of the facts not
conveyed to the insurer was such that the failure to
communicate must have been intentional rather than
merely inadvertent.
> Jaime could not have been unaware that his
heart beat would at times rise to high and
alarming levels and that he had consulted a doctor
twice two months before applying for non-medical
insurance.
> The last medical consultation took place just the
day before the insurance application was filed.
2. YES
Ratio Materiality relates rather to the "probable and
reasonable influence of the facts" upon the party to
whom the communication should have been made, in
assessing the risk involved in making or omitting to
make further inquiries and in accepting the
application for insurance; that "probable and
reasonable influence of the farts" concealed must, of
course, be determined objectively, by the judge
ultimately.
Reasoning
- The information which Jaime failed to disclose was
material to the ability of Great Pacific to estimate the
probable risk he presented as a subject of life
insurance.
- Had Canilang disclosed his visits to his doctor, the
diagnosis made and the medicines prescribed by
such doctor, in the insurance application, it may be
reasonably assumed that Great Pacific would have
made further inquiries and would have probably
refused to issue a non-medical insurance policy or,
at the very least, required a higher premium for the
same coverage.
- As held in the case of Saturnino vs. Philippine-
American Life Insurance, the waiver of medical
examination in a non-medical insurance contract
renders even more material the information inquired
of the applicant concerning previous condition of
health and diseases suffered, for such information
necessarily constitutes an important factor which the
insurer takes into consideration in deciding whether
to issue the policy or not.
Disposition the Petition for Review is DENIED for
lack of merit and the Decision of the Court of
Appeals dated 16 October 1989 in C.A.-G.R. SP No.
08696 is hereby AFFIRMED.

YU PANG CHENG v. CA
105 PHIL 930
BAUTISTA ANGELO; May 29, 1959

FACTS
- September 5, 1950: Yu Pang Eng submitted parts
II and III of his application for insurance consisting
of the medical declaration made by him to the
medical examiner of defendant and the medical
examiner's report
- September 7: he submitted part I of his application
which is the declaration made by him to an agent of
defendant
- September 8: defendant issued to the insured
Policy No. 812858
- December 27, 1950: the insured entered St.
Luke's Hospital for medical treatment but he died on
February 27, 1951.
- According to the death certificate, he died of
"infiltrating medullary carcinoma, Grade 4, advanced
cardiac and of lesser curvature, stomach metastases
spleen."
!"#$%&"'( *+,- 26

- Plaintiff, brother and beneficiary of the insured,
demanded from defendant the payment of the
proceeds of the insurance policy and when the
demand was refused, he brought the present action.
- The insured, in his application for insurance,
particularly in his declarations to the examining
physician, stated the following in answering the
questions propounded to him:
14. Have you ever had any of the following diseases
or symtoms? Each question must be read and
answered "Yes" or "No.".
"Gastritis, Ulcer of the Stomach or any disease of
that organ? No.
"Vertigo, Dizziness, Fainting-spells or
Unconsciouness? No.
"Cancer, Tumors or Ulcers of any kind? No.
- 15. Have you ever consulted any physician riot
included in any of the above answers? Give names
and address or physicians list ailments or accidents
and date. No."
- It appears that the insured entered the Chinese
General Hospital for medical treatment on January
29, 1950 having stayed there up to February 11,
1950.
- An X-ray picture of his stomach was taken and the
diagnosis made of him by his doctors showed that
his illness was "peptic ulcer, bleeding."

ISSUE
WON the insured is guilty of concealment of some
facts material to the risk insured against which has
the effect of avoiding the policy as found by
respondent court.

HELD
- It should be noted that the insured's confinement
in the Chinese General Hospital took place from
January 29, 1950 to February 11, 1950, whereas his
application for insurance wherein he stated his
answers to the questions propounded to him by the
examining physician of defendant was submitted to
defendant on September 5, 1950.
- It is apparent that when the insured gave his
answers regarding his previous ailment, particularly
with regard to "Gastritis, Ulcer of the Stomach or any
disease of that organ" and "Vertigo, Dizziness,
Fainting-spells or Unconsciousness", he concealed
the ailment of which he was treated in the Chinese
General Hospital which precisely has direct
connection with the subject of the questions
propounded.
- The negative answers given by the insured
regarding his previous ailment, or his concealment of
the fact that he was hospitalized and treated for
sometime of peptic ulcer and had suffered from
"dizziness, anemia, abdominal pains and tarry
stools", deprived defendant of the opportunity to
make the necessary inquiry as to the nature of his
past illness so that it may form its estimate relative
to the approval of his application.
- Had defendant been given such opportunity,
considering the previous illness of the insured as
disclosed by the records of the Chinese General
Hospital, defendant would probably had never
consented to the issuance of the policy in question.
In fact, according to the death certificate, the
insured died of "infiltrating medullary carcinoma,
Grade, 4, advanced cardiac and of lesser curvature,
stomach metastases spleen", which may have a
direct connection with his previous illness.
- Our Insurance Law provides that "A neglect to
communicate that which a party knows and ought to
communicate, is called concealment" (Section 25,
Act No. 2427). Whether intentional or unintentional,
the concealment entitles the insurer to rescind the
contract of insurance (Section 26).
- Our law even requires the insured to communicate
to the insurer all facts within his knowledge which
are material to the contract and which the other
party has not the means of ascertaining (Section
27), and the materiality is to be determined not by
the event but solely by the probable and reasonable
influence of the facts upon the party to whom the
communication is due (Section 30).
- Argente vs. West Coast Life Insurance Co.: "One
ground for the rescission of a contract of insurance
under the Insurance Act is 'a concealment', which in
section 25 is defined 'A neglect to communicate that
which a party knows and ought to communicate.'
Appellant argues that the concealment was
immaterial and insufficient to avoid the policy. We
cannot agree. In an action on a life insurance policy
where the evidence conclusively shows that the
answers to questions concerning diseases were
untrue, the truth or falsity of the answers become
the determining factor. If the policy was procured by
fraudulent representations, the contract of insurance
apparently set forth therein was never legally
existent. It can fairly be assumed that had the true
facts been disclosed by the assured, the insurance
would never have been granted."
Disposition Decision affirmed.






GREAT PACIFIC LIFE v. CA (supra p.34)

PACIFIC BANKING CORP v. CA (ORIENTAL
ASSURANCE CORPORATION)
168 SCRA 1
PARAS; November 28, 1988

NATURE
Petition for review on certiorari of the CA decision,
which set aside the decision of CFI Manila, which had
in turn granted the complaint for a sum of money in
civil case filed by Pacific Banking against Oriental
Assurance.

FACTS
- October 21,1963: an open Fire Policy was issued to
the Paramount Shirt Manufacturing Co. (insured), by
which Oriental Assurance Corporation bound itself to
indemnify the insured for any loss or damage, not
exceeding P61,000.00, caused by fire to its property
consisting of stocks, materials and supplies usual to
a shirt factory, including furniture, fixtures,
machinery and equipment while contained in the
ground, second and third floors of the building
situated at number 256 Jaboneros St., San Nicolas,
Manila, for a period of one year commencing from
that date to October 21, 1964.
- Insured was at the time of the issuance of the
policy and is up to this time, a debtor of Pacific
Banking in the amount of not less P800,000.00 and
the goods described in the policy were held in trust
by the insured for the Pacific Banking under thrust
receipts.
- Said policy was duly endorsed to Pacific Banking as
mortgagee/trustor of the properties insured, with the
knowledge and consent of Oriental Assurance to the
effect that "loss if any under this policy is payable to
the Pacific Banking Corporation".
- While the aforesaid policy was in full force and
effect, a fire broke out on the subject premises
destroying the goods contained in its ground and
second floors. Counsel for the Pacific Banking sent a
letter of demand to Oriental Assurance for indemnity
due to the loss of property by fire. Oriental
Assurance informed counsel that it was not yet ready
to accede to the latter's demand as the former is
!"#$%&"'( *+,- 37

awaiting the final report of the insurance adjuster,
H.H. Bayne Adjustment Company.
- Said insurance adjuster notified counsel for the
Pacific Banking that the insured under the policy had
not filed any claim with it, nor submitted proof of
loss which is a clear violation of Policy Condition
No.11, and for which reason, determination of the
liability of Oriental Assurance could not be had.
Pacific Banking's counsel replied asking the insurance
adjuster to verify from the records of the Bureau of
Customs the entries of merchandise taken into the
customs bonded warehouse razed by fire as a
reliable proof of loss.
- For failure of the insurance company to pay the
loss as demanded, Pacific Banking field before CFI an
action for a sum of money against the Oriental
Assurance, in the principal sum of P61,000.00 issued
in favor of Paramount Shirt Manufacturing Co.
Oriental Assurance defenses
(a) lack of formal claim by insured over the loss and
(b) premature filing of the suit as neither plaintiff nor
insured had submitted any proof of loss on the basis
of which defendant would determine its liability and
the amount thereof, either to the Oriental Assurance
or its adjuster H.H. Bayne Adjustment Co.
Pacific Banking
> presented evidence that insured has undeclared
co-insurances with the following: P30,000.00 with
Wellington Insurance; P25,000. 00 with Empire
Surety and P250,000.00 with Asian Surety;
undertaken by insured Paramount on the same
property covered by its policy with Oriental
Assurance whereas the only co-insurances declared
in the subject policy are those of P30,000.00 with
Malayan, P50,000.00 with South Sea, and
P25.000.00 with Victory
- NOTE: the defense of fraud and/or violation of non-
declaration of co-insurances was not pleaded in the
answer, also not pleaded in the Motion to Dismiss.
- CFI denied Oriental Assurance's motion on the
ground that since the defense was raised for the first
time, it must be deemed to have waived the
requirement of proof of loss. Case was submitted for
decision. But upon MR, Oriental Asurance was
allowed to present additional evidence, "in order to
prove that 'insured has committed a violation of
condition No. 3 of the policy in relation to the other
Insurance Clause.' " CFI eventually adjudged
Oriental Assurance liable to the Pacific Banking under
the said contract of insurance.
- Court of Appeals reversed. Pacific Banking's MR
denied.

ISSUES
1. WON insured is guilty of fraud
2. WON mortgagee/assignee can still claim from the
insurance

HELD
1. YES
- The crux of the controversy centers on two points:
(a) unrevealed co-insurances which violated policy
conditions No. 3; and (b) failure of the insured to file
the required proof of loss prior to court action.
- Policy Condition No. 3 explicitly provides: The
Insured shall give notice to the Company of any
insurance already effected, or which may
subsequently be effected, covering any of the
property hereby insured, and unless such notice be
given and the particulars of such insurance or
insurances be stated in or endorsed on this Policy by
or on behalf of the Company before the occurrence
of any loss or damage, all benefit under this policy
shall be forfeited.
- It is not disputed that the insured failed to reveal
before the loss three other insurances. By reason of
said unrevealed insurances, the insured had been
guilty of a false declaration; a clear
misrepresentation and a vital one because where the
insured had been asked to reveal but did not, that
was deception. Otherwise stated, had the insurer
known that there were many co-insurances, it could
have hesitated or plainly desisted from entering into
such contract. Hence, the insured was guilty of clear
fraud.
- Pacific Banking's contention that the allegation of
fraud is but a mere inference or suspicion is
untenable. Concrete evidence of fraud or false
declaration by the insured was furnished by the
Pacific Banking itself when the facts alleged in the
policy under clauses "Co-Insurances Declared" and
"Other Insurance Clause" are materially different
from the actual number of co-insurances taken over
the subject property. Consequently, the whole
foundation of the contract fails, the risk does not
attach and the policy never becomes a contract
between the parties. Representations of facts are the
foundation of the contract and if the foundation does
not exist, the superstructure does not arise.
Falsehood in such representations is not shown to
vary or add to the contract, or to terminate a
contract which has once been made, but to show
that no contract has ever existed (Tolentino). A void
or inexistent contract is one which has no force and
effect from the very beginning, as if it had never
been entered into, and which cannot be validated
either by time or by ratification.
- As the insurance policy against fire expressly
required that notice should be given by the insured
of other insurance upon the same property, the total
absence of such notice nullifies the policy.
- Argument that notice of co-insurances may be
made orally is preposterous and negates policy
condition No. 20 which requires every notice and
other communications to the insurer to be written or
printed.
2. NO
- Subject mortgage clause pecifically provides: Loss,
if any, under this policy, shall be payable to the
PACIFIC BANKING CORPORATION Manila
mortgagee/trustor as its interest may appear, it
being hereby understood and agreed that this
insurance as to the interest of the mortgagee/trustor
only herein, shall not be invalidated by any act or
neglect except fraud or misrepresentation, or arson
of the mortgagor or owner/trustee of the property
insured; provided, that in case the mortgagor or
owner/ trustee neglects or refuses to pay any
premium, the mortgagee/ trustor shall, on demand
pay the same.
- The paragraph clearly states the exceptions to the
general rule that insurance as to the interest of the
mortgagee, cannot be invalidated; namely: fraud, or
misrepresentation or arson.
- Concealment of the aforecited co-insurances can
easily be fraud, or in the very least,
misrepresentation. It is but fair and just that where
the insured who is primarily entitled to receive the
proceeds of the policy has by its fraud and/or
misrepresentation, forfeited said right, with more
reason Pacific Banking which is merely claiming as
indorsee of said insured, cannot be entitled to such
proceeds.
- The fact of fraud was tried by express or at least
implied consent of the parties. Pacific Banking did
not only object to the introduction of evidence but on
the contrary, presented the very evidence that
proved its existence.
- Be that as it may, SC has ample authority to give
beyond the pleadings where in the interest of justice
and the promotion of public policy, there is a need to
make its own finding to support its conclusion.
Otherwise stated, the Court can consider a fact
which surfaced only after trial proper.
- Generally, the cause of action on the policy accrues
when the loss occurs, but when the policy provides
!"#$%&"'( *+,- 3.

that no action shall be brought unless the claim is
first presented extrajudicially in the manner provided
in the policy, the cause of action will accrue from the
time the insurer finally rejects the claim for
payment.
- In the case at bar, policy condition No. 11
specifically provides that the insured shall on the
happening of any loss or damage give notice to the
company and shall within fifteen (15) days after such
loss or damage deliver to the Oriental Assurance (a)
a claim in writing giving particular account as to the
articles or goods destroyed and the amount of the
loss or damage and (b) particulars of all other
insurances, if any. Likewise, insured was required "at
his own expense to produce, procure and give to the
company all such further particulars, plans,
specifications, books, vouchers, invoices, duplicates
or copies thereof, documents, proofs and information
with respect to the claim".
- Evidence adduced shows that 24 days after the
fire, Pacific Banking merely wrote letters to Oriental
Assurance to serve as a notice of loss, thereafter,
the former did not furnish the latter whatever
pertinent documents were necessary to prove and
estimate its loss. Instead, Pacific Banking shifted
upon Oriental Assurance the burden of fishing out
the necessary information to ascertain the particular
account of the articles destroyed by fire as well as
the amount of loss.
- Oriental Assurance and its adjuster notified Pacific
Banking that insured had not yet filed a written claim
nor submitted the supporting documents in
compliance with the requirements set forth in the
policy. Despite the notice, the latter remained
unheedful. Since the required claim by insured,
together with the preliminary submittal of relevant
documents had not been complied with, it follows
that Oriental Assurance could not be deemed to have
finally rejected Pacific Banking's claim and therefore
the latter's cause of action had not yet arisen.
Compliance with condition No. 11 is a requirement
sine qua non to the right to maintain an action as
prior thereto no violation of Pacific Banking's right
can be attributable to Oriental Assurance. As before
such final rejection, there was no real necessity for
bringing suit. Pacific Banking should have
endeavored to file the formal claim and procure all
the documents, papers, inventory needed by Oriental
Assurance or its adjuster to ascertain the amount of
loss and after compliance await the final rejection of
its claim. Indeed, the law does not encourage
unnecessary litigation.
- Pacific Banking prematurely filed the civil case and
dismissal thereof was warranted under the
circumstances. While it is a cardinal principle of
insurance law that a policy or contract of insurance is
to be construed liberally in favor of the insured and
strictly as against the insurer company yet, contracts
of insurance, like other contracts, are to be
construed according to the sense and meaning of the
terms which the parties themselves have used. If
such terms are clear and unambiguous, they must be
taken and understood in their plain, ordinary and
popular sense.
- Contracts of insurance are contracts of indemnity
upon the terms and conditions specified in the policy.
The parties have a right to impose such reasonable
conditions at the time of the making of the contract
as they may deem wise and necessary. The
agreement has the force of law between the parties.
The terms of the policy constitute the measure of the
insurer's liability, and in order to recover, the insured
must show himself within those terms. The
compliance of the insured with the terms of the
policy is a condition precedent to the light of
recovery.
- It appearing that insured has violated or failed to
perform the conditions under No. 3 and 11 of the
contract, and such violation or want of performance
has not been waived by the insurer, the insured
cannot recover, much less the herein Pacific Banking.
Courts are not permitted to make contracts for the
parties; the function and duty of the courts is simply
to enforce and carry out the contracts actually made.
Disposition Petition dismissed. CA affirmed.

SUNLIFE ASSURANCE COMPANY v. CA (SPS.
BACANI)
245 SCRA 268
QUIASON; June 22, 1995

NATURE
A petition for review on certiorari.

FACTS
- April 15, 1986: Robert John B. Bacani procured a
life insurance contract for himself from SUNLIFE
(petitioner) valued at P100K. The designated
beneficiary was his mother, Bernarda Bacani
(respondent).
- June 26, 1987: the insured died in a plane crash.
Bernarda Bacani filed a claim with Sunlife, seeking
the benefits of the insurance policy taken by her son.
Petitioner conducted an investigation and its findings
prompted it to reject the claim on the ground that
the insured did not disclose facts material to the
issuance of the policy. The insured gave false
statements in the application when he answered in
the negative to the question have you ever had or
sought advice for urine, kidney, bladder
disorder?
- Sunlife discovered that two weeks prior to the
issuance, insured was diagnosed with renal failure,
was confined, and underwent tests.
- November 17, 1988: Bacani and her husband filed
for specific performance against Sunlife. RTC granted
the plea on the ground that that the facts concealed
by the insured were made in good faith and under
the belief that they need not be disclosed, and that
the disclosure was not material since the policy was
non-medical.
- Sunlife appealed to the CA, but the latter denied
the appeal on the ground that the cause of death
was unrelated to the facts concealed by the insured.

Petitioners Claim
> The insured did not disclose facts relevant to the
issuance of the policy, thus rescission of the contract
may be invoked by the insurance company.
Respondents Comments
> The actual cause of death was not relevant to the
concealed information, and the policy was entered
into by the insured in good faith.

ISSUE
WON the concealment renders the insurance policy
rescissible

HELD
YES
Ratio The terms of the contract are clear. The
insured is specifically required to disclose to the
insurer matters relating to his health.
Reasoning
SEC. 26 (IC)
A neglect to communicate that which a party
knows and ought to communicate, is called a
concealment.
SEC. 31 (IC)
Materiality is to be determined not by the event,
but solely by the probable and reasonable influence
of the facts upon the party to whom communication
is due, in forming his estimate of the disadvantages
of the proposed contract or in making his inquiries
- The information which the insured failed to disclose
was material and relevant to the approval and the
!"#$%&"'( *+,- 3/

issuance of the insurance policy. The matters
concealed would have definitely affected petitioner's
action on his application, either by approving it with
the corresponding adjustment for a higher premium
or rejecting the same.
- Good faith is no defense in concealment. It appears
that such concealment was deliberate on the part of
the insured.
- The waiver of a medical examination [in a non-
medical insurance contract] renders even more
material the information required of the applicant
concerning previous condition of health and diseases
suffered, for such information necessarily constitutes
an important factor which the insurer takes into
consideration in deciding whether to issue the policy
or not.
- Anent the finding that the facts concealed had no
bearing to the cause of death of the insured, it is
well settled that the insured need not die of the
disease he had failed to disclose to the insurer. It is
sufficient that his non-disclosure misled the insurer
in forming his estimates of the risks of the proposed
insurance policy or in making inquiries
Disposition Petition is granted and the decision of
CA is reversed and set aside.

EGUARAS v. GREAT EASTERN
33 PHIL. 263
TORRES.; January 24, 1916

NATURE
Appeal filed through bill of exceptions from the
judgment of the CFI

FACTS
- Francisca Eguaras filed a written complaint in court,
alleging as a cause of action that her son-in-law
Dominador Albay had applied in writing to the
defendant insurance company to insure his life for
the sum of P5,000, naming as the beneficiary in case
of his death the plaintiff Francisca Eguaras; that after
compliance with the requisites and the investigation
carried on by the defendant company, it accepted
the application for insurance and issued the policy;
that, said policy being in force, the insured died, and
despite the fact that the beneficiary submitted
satisfactory proofs of his death and that the
defendant company investigated the event, still it
refused and continues to refuse to pay to the plaintiff
the value of the policy.
- Defendant set forth in special defense that the
insurance policy issued in the name of Dominador
Albay had been obtained through fraud and deceit
known and consented to by the interested parties
and is therefore completely illegal, void, and
ineffective.
- A criminal case for frustrated estafa was filed by
defendant against Ponciano Remigio, Castor Garcia
and Francisca Eguaras. They were acquitted, and
claim that the judgment produces the effect of res
judicata in the present suit.

ISSUE
WON the life insurance obtained by Dominador Albay
was issued through fraud and deceit

HELD
YES
Ratio In a contract where one of the contracting
parties may have given his consent through error,
violence, intimidation, or deceit, and in any of such
cases the contract is void, even though, despite this
nullity, no crime was committed. There may not
have been estafa in the case at bar, but it was
conclusively demonstrated by the trial that deceit
entered into the insurance contract, fulfillment
whereof is claimed, and therefore the conclusions
reached by the court in the judgment it rendered in
the criminal proceedings for estafa do not affect this
suit, nor can they produce in the present suit the
force of res adjudicata.
Reasoning
- It is proven that the signatures on the insurance
applications reading "Dominado Albay" are false and
forged; that the person who presented himself to Dr.
Vidal to be examined was not the real Dominador
Albay, but Castor Garcia who was positively
identified by Dr. Vidal; that at the time of the
application for insurance and the issuance of the
policy which is the subject matter of this suit the real
Dominador Albay was informed of all those
machinations, wherefore it is plain that the insurance
contract between the defendant and Dominador
Albay is null and void because it is false, fraudulent
and illegal.
Disposition The judgment appealed from is
reversed and the defendant absolved from the
complaint without special finding as to the costs.

QUA CHEE GAN v. LAW UNION AND ROCK
98 PHIL 85
REYES; December 17, 1955

FACTS
- Qua Chee Gan insured 4 of his bodegas with Law
Union & Rock Insurance Co in 1937. These bodegas
were used for the storage of stocks of copra and of
hemp, baled and loose.
- Fire of undetermined origin that broke out in the
early morning of July 21, 1940, and lasted almost
one week, gutted and completely destroyed Bodegas
Nos. 1, 2 and 4, with the merchandise stored
therein.
- Qua Chee Gan informed the insurance company of
the fire. Fire adjusters of the company conducted an
extensive investigation. Qua Chee Gan submitted the
corresponding fire claims, totaling P398,562.81 (but
reduced to the full amount of the insurance,
P370,000), the Insurance Company resisted
payment, claiming violation of warranties and
conditions, filing of fraudulent claims, and that the
fire had been deliberately caused by the insured or
by other persons in connivance with him.
- Qua Chee Gan, his brother and his employees were
tried for arson, where counsel of the insurance
company acted as a private prosecutor. They were
acquitted.
- This civil suit was then instituted to claim against
the insurance company. The CFI ruled in favor of
Qua Chee Gan and ordered Law Union Rock Co. to
pay.

ISSUES
1. WON there was a breach of the fire hydrant
warranty
2. WON the insured violated the Hemp warranty
3. WON Qua Chee Gan is guilty of overvaluation
4. WON Qua Chee Gan caused the fire
5. WON there was an error in the amount of copra
and hemp lost
6. WON the claims contained false and fraudulent
statements

HELD
1. NO
- It is argued that he should have 11 fire hydrants in
the compound, but he only had 2. We are in
agreement with the trial Court that the appellant is
barred by waiver (or rather estoppel) to claim
violation of the so-called fire hydrants warranty, for
the reason that knowing fully all that the number of
hydrants demanded therein never existed from the
very beginning, the appellant nevertheless issued
the policies in question subject to such warranty, and
received the corresponding premiums.
2. NO
!"#$%&"'( *+,- 30

- The insurance company avers that the insured
violated the hemp warranty when it admitted that it
had 36 cans of gasoline in the building. It is well to
note that gasoline is not specifically mentioned
among the prohibited articles listed in the so-called
"hemp warranty." The cause relied upon by the
insurer speaks of "oils (animal and/or vegetable
and/or mineral and/or their liquid products having a
flash point below 300o Fahrenheit", and is decidedly
ambiguous and uncertain; for in ordinary parlance,
"Oils" mean "lubricants" and not gasoline or
kerosene. And how many insured, it may well be
wondered, are in a position to understand or
determine "flash point below 003o Fahrenheit. Here,
again, by reason of the exclusive control of the
insurance company over the terms and phraseology
of the contract, the ambiguity must be held strictly
against the insurer and liberally in favor of the
insured, especially to avoid a forfeiture
- Another point that is in favor of the insured is that
the gasoline kept in Bodega No. 2 was only incidental
to his business, being no more than a customary 2
day's supply for the five or six motor vehicles used
for transporting of the stored merchandise). "It is
well settled that the keeping of inflammable oils on
the premises though prohibited by the policy does
not void it if such keeping is incidental to the
business." (Bachrach vs. British American Ass. Co.,
17 Phil. 555, 560)
3. NO
- The charge that the insured failed or refused to
submit to the examiners of the insurer the books,
vouchers, etc. demanded by them was found
unsubstantiated by the trial Court, and no reason
has been shown to alter this finding.
- In view of the discrepancy in the valuations
between the insured and the adjuster Stewart for the
insurer, the Court referred the controversy to a
government auditor, Apolonio Ramos; but the latter
reached a different result from the other two. Not
only that, but Ramos reported two different
valuations that could be reached according to the
methods employed. Clearly then, the charge of
fraudulent overvaluation cannot be seriously
entertained.
4. NO
- This defense is predicted on the assumption that
the insured was in financial difficulties and set the
fire to defraud the insurance company, presumably
in order to pay off the Philippine National Bank, to
which most of the insured hemp and copra was
pledged. This defense is fatally undermined by the
established fact that, notwithstanding the insurer's
refusal to pay the value of the policies the extensive
resources of the insured enabled him to pay off the
National Bank in a short time; and if he was able to
do so, no motive appears for attempt to defraud the
insurer. While the acquittal of the insured in the
arson case is not res judicata on the present civil
action, the insurer's evidence, to judge from the
decision in the criminal case, is practically identical in
both cases and must lead to the same result, since
the proof to establish the defense of connivance at
the fire in order to defraud the insurer "cannot be
materially less convincing than that required in order
to convict the insured of the crime of arson.
5. NO
- As to the defense that the burned bodegas could
not possibly have contained the quantities of copra
and hemp stated in the fire claims, the insurer's case
rests almost exclusively on the estimates, inferences
and conclusions of its adjuster investigator,
Alexander D. Stewart, who examined the premises
during and after the fire. His testimony, however,
was based on inferences from the photographs and
traces found after the fire, and must yield to the
contradictory testimony of engineer Andres Bolinas,
and specially of the then Chief of the Loan
Department of the National Bank's Legaspi branch,
Porfirio Barrios, and of Bank Appraiser Loreto
Samson, who actually saw the contents of the
bodegas shortly before the fire, while inspecting
them for the mortgagee Bank
6. NO
- Appellant insurance company also contends that
the claims filed by the insured contained false and
fraudulent statements that avoided the insurance
policy. But the trial Court found that the
discrepancies were a result of the insured's
erroneous interpretation of the provisions of the
insurance policies and claim forms, caused by his
imperfect knowledge of English, and that the
misstatements were innocently made and without
intent to defraud. The trial courts ruling must be
upheld.
- For example, the occurrence of previous fires in the
premises insured in 1939, altho omitted in the
claims, Exhibits EE and FF, were nevertheless
revealed by the insured in his claims Exhibits Q (filed
simultaneously with them), KK and WW. Considering
that all these claims were submitted to the smae
agent, and that this same agent had paid the loss
caused by the 1939 fire, we find no error in the trial
Court's acceptance of the insured's explanation that
the omission in Exhibits EE and FF was due to
inadvertance, for the insured could hardly expect
under such circumstances, that the 1939 would pass
unnoticed by the insurance agents. Similarly, the 20
per cent overclaim on 70 per cent of the hemo stock,
was explained by the insured as caused by his belief
that he was entitled to include in the claim his
expected profit on the 70 per cent of the hemp,
because the same was already contracted for and
sold to other parties before the fire occurred.
Compared with other cases of over-valuation
recorded in our judicial annals, the 20 per cent
excess in the case of the insured is not by itself
sufficient to establish fraudulent intent. Certainly,
the insured's overclaim of 20 per cent in the case at
bar, duly explained by him to the Court a quo,
appears puny by comparison (compared to other
cases cited by the court), and can not be regarded
as "more than misstatement, more than
inadvertence of mistake, more than a mere error in
opinion, more than a slight exaggeration" that would
entitle the insurer to avoid the policy. It is well to
note that the overcharge of 20 per cent was claimed
only on a part (70 per cent) of the hemp stock; had
the insured acted with fraudulent intent, nothing
prevented him from increasing the value of all of his
copra, hemp and buildings in the same proportion.
This also applies to the alleged fraudulent claim for
burned empty sacks, that was likewise explained to
our satisfaction and that of the trial Court. The rule is
that to avoid a policy, the false swearing must be
willful and with intent to defraud which was not the
cause. Of course, the lack of fraudulent intent would
not authorize the collection of the expected profit
under the terms of the polices, and the trial Court
correctly deducted the same from its award.
Disposition Decision affirmed

ARGENTE v. WEST COAST LIFE
51 PHIL 725
MALCOLM; March 19, 1928

FACTS
- This is an action upon a joint life insurance policy
for P15,000 issued by the West Coast Life Insurance
Co., on May 15, 1925, in favor of Bernardo Argente,
and his wife, Vicenta de Ocampo, the latter having
died on November 18, 1925. Fraud in obtaining the
policy was pleaded by way of special defense. On the
issue thus suggested, the court adopted the theory
of the defendant, and held the insurance policy null
!"#$%&"'( *+,- 31

and void, with the result that the complaint was
dismissed, with costs.
-Bernardo Argente signed an application for joint
insurance with his wife in the sum of P2,000. The
wife, Vicenta de Ocampo, signed a like application
for the same policy.
- Bernardo Argente and his wife was examined by
Dr. Cesareo Sta. Ana, a medical examiner for the
West Coast Life Insurance Co. which did not show
previous and existing health problems.
- A temporary policy for P15,000 was issued to
Bernardo Argente and his wife as of May 15, 1925.
In view of the fact that more than thirty days had
elapsed since the applicants were examined by the
company's physician, each of them was required to
file a certificate of health before the policy was
delivered to them.
- On November 18, 1925, Vicenta de Ocampo died of
cerebral apoplexy. Thereafter Bernardo Argente
presented a claim. Following investigation conducted
by the Manager of the Manila office of the insurance
company, it was apparently disclosed that the
answers given by the insured in their medical
examinations with regard to their health and
previous illnesses and medical attendance were
untrue. West Coast Life Insurance Co. refused to pay
the claim of Bernardo Argente, and wrote him to the
effect that the claim was rejected because the
insurance was obtained through fraud and
misrepresentation.
- It is admitted that it appears in the Medical
Examiner's Report that Bernardo Argente gave false
responses. As well as with the Medical Examiner's
Report that Vicenta de Ocampo. It is, however, not
disputed that Vicenta de Ocampo was taken by a
patrolman, at the request of her husband, Bernardo
Argente, on May 19, 1924, to the Meisic police
station, and from there was transferred to the San
Lazaro Hospital. In San Lazaro Hospital, her case
was diagnosed by the admitting physician as
"alcoholism," but later Doctor Domingo made a
diagnosis of probable "manic-depressive psychosis,"
and still, later in Mary Chiles Hospital, made a final
diagnosis of "phycho-neurosis."
- Bernardo Argente, while readily conceding most of
the facts herein narrated, yet alleges that both he
and his wife revealed to the company's physician,
Doctor Sta. Ana, all the facts concerning their
previous illnesses and medical attendance, but that
Doctor Sta. Ana, presumably acting in collusion with
the insurance agent, Jose Geronimo del Rosario,
failed to record them in the medical reports. The
evidence on these points consists of the testimony of
the plaintiff and his subordinate clerk, Apolonio
Espiritu, on the one hand, and of the testimony of
Doctor Sta. Ana and Jose Geronimo del Rosario on
the other. This was rejected by the Trial Court. Trial
judge found with the insurance company with regard
to the question of fact. SC agrees. There appears no
motive whatever on the part of Doctor Sta. Ana to
falsify the Medical Examiner's Reports and thereby
not only jeopardize his career as a physician, but
also gravely implicate himself criminally.

ISSUE
WON the contract of insurance may be rescinded



HELD
YES
- Bernardo Argente and his wife applications were
false with respect to their state of health during the
period of five years preceding the date of such
applications and that they knew the representations
made by them in their applications were false. The
question arises as to the state of the law in relation
thereto.
- One ground for the rescission of a contract of
insurance under the Insurance Act is "a
concealment," which in section 25 is defined as "A
neglect to communicate that which a party knows
and ought to communicate." In an action on a life
insurance policy where the evidence conclusively
shows that the answers to questions concerning
diseases were untrue, the truth or falsity of the
answers become the determining factor. If the policy
was procured by fraudulent representations, the
contract of insurance apparently set forth therein
was never legally existent. It can fairly be assumed
that had the true facts been disclosed by the
assured, the insurance would never have been
granted.
- In Joyce, The Law of Insurance, second edition,
volume 3, Chapter LV, is found the following:
"The basis of the rule vitiating the contract in
cases of concealment is that it misleads or
deceives the insurer into accepting the risk, or
accepting it at the rate of premium agreed upon;
The insurer, relying upon the belief that the
assured will disclose every material fact within his
actual or presumed knowledge, is misled into a
belief that the circumstance withheld does not
exist, and he is thereby induced to estimate the
risk upon a false basis that it does not exist. The
principal question, therefore, must be, Was the
assurer misled or deceived into entering a contract
obligation or in fixing the premium of insurance by
a withholding of material information or facts
within the assured's knowledge or presumed
knowledge?
"It therefore follows that the assurer in assuming a
risk is entitled to know every material fact of which
the assured has exclusive or peculiar knowledge,
as well as all material facts which directly tend to
increase the hazard or risk which are known by the
assured, or which ought to be or are presumed to
be known by him. And a concealment of such facts
vitiates the policy. 'It does not seem to be
necessary . . . that the . . . suppression of the
truth should have been willful.' If it were but an
inadvertent omission, yet if it were material to the
risk and such as the plaintiff should have known to
be so, it would render the policy void. But it is held
that if untrue or false answers are given in
response to inquiries and they relate to material
facts the policy is avoided without regard to the
knowledge or fraud of assured, although under the
statute statements are representations which must
be fraudulent to avoid the policy. So under certain
codes the important inquiries are whether the
concealment was willful and related to a matter
material to the risk.
xxx xxx xxx
"If the assured has exclusive knowledge of
material facts, he should fully and fairly disclose
the same, whether he believes them material or
not. But notwithstanding this general rule it will
not infrequently happen, especially in life risks,
that the assured may have a knowledge actual or
presumed of material facts, and yet entertain an
honest belief that they are not material. . . . The
determination of the point whether there has or
has not been a material concealment must rest
largely in all cases upon the form of the questions
propounded and the exact terms of the contract.
Thus, where in addition to specifically named
diseases the insured was asked whether he had
had any sickness within ten years, to which he
answered 'No,' and it was proven that within that
period he had had a slight attack of pharyngitis, it
was held a question properly for the jury whether
such an inflammation of the throat was a 'sickness'
within the intent of the inquiry, and the court
remarked on the appeal decision that if it could be
held as a matter of law that the policy was thereby
!"#$%&"'( *+,- 32

avoided, then it was a mere device on the part of
insurance companies to obtain money without
rendering themselves liable under the policy. . . .
" . . . The question should be left to the jury
whether the assured truly represented the state of
his health so as not to mislead or deceive the
insurer; and if he did not deal in good faith with
the insurer in that matter, then the inquiry should
be made, Did he know the state of his health so as
to be able to furnish a proper answer to such
questions as are propounded? A Massachusetts
case, if construed as it is frequently cited, would
be opposed to the above conclusion; but, on the
contrary, it sustains it, for the reason that
symptoms of consumption had so far developed
themselves within a few months prior to effecting
the insurance as to induce a reasonable belief that
the applicant had that fatal disease, and we should
further construe this case as establishing the rule
that such a matter cannot rest alone upon the
assured's belief irrespective of what is a
reasonable belief, but that it ought to be judged by
the criterion whether the belief is one fairly
warranted by the circumstances. A case in
Indiana, however, holds that if the assured has
some affection or ailment of one or more of the
organs inquired about so well defined and marked
as to materially derange for a time the functions of
such organ, as in the case of Bright's disease, the
policy will be avoided by a nondisclosure,
irrespective of the fact whether the assured knew
of such ailment or not. . . ."
- Lastly, appellant contends that even if the
insurance company had a right to rescind the
contract, such right cannot now be enforced in view
of the provisions of section 47 of the Insurance Act
providing "Whenever a right to rescind a contract of
insurance is given to the insurer by any provision of
this chapter, such right must be exercised previous
to the commencement of an action on the contract."
This section was derived from section 2583 of the
California Civil Code, but in contrast thereto, makes
use of the imperative "must" instead of the
permissive "may." Nevertheless, there are two
answers to the problem as propounded. The first is
that the California law as construed by the code
examiners, at whose recommendation it was
adopted, conceded that "A failure to exercise the
right (of rescission), cannot, of course, prejudice any
defense to the action which the concealment may
furnish." (Codes of California Annotated; Tan Chay
Heng vs. West Coast Life Insurance Company
[1927], p. 80, ante.) The second answer is that the
insurance company more than one month previous
to the commencement of the present action wrote
the plaintiff and informed him that the insurance
contract was void because it had been procured
through fraudulent representations, and offered to
refund to the plaintiff the premium which the latter
had paid upon the return of the policy for
cancellation. As held in California as to a fire
insurance policy, where any of the material
representations are false, the insurer's tender of the
premium and notice that the policy is canceled,
before the commencement of suit thereon, operate
to rescind the contract of insurance. (Rankin vs.
Amazon Insurance Co. [1891], 89 Cal., 203.)
Disposition Judgment affirmed, with the costs of
this instance against the appellant.

GREAT PACIFIC LIFE v. CA (NGO HING)
89 SCRA 543
DE CASTRO, J; April 30, 1979

NATURE
Petition for certiorari

FACTS
- On March 14, 1957, private respondent Ngo Hing
filed an application with the Great Pacific Life
Assurance Co. (Pacific Life) for a 20 year endowment
policy of P50k on the life of his 1 year old daughter,
Helen. Ngo Hing supplied the essetntial data which
petitioner Mondragon, branch manager of the Pacific
Life in Cebu, wrote on the corresponding form in his
own handwriting, later typing the data on an
application form signed by Ngo Hing. The latter paid
the P1077.75 annual premium but retained P1,317
as commission as he was also a duly authorized
agent of Pacific Life. The binding deposit receipt was
then issued to Ngo Hing; Mondragon handwrote his
strong recommendation for the approval of the
application on the back of the form.
- On April 30, Mondragon received a letter from
Pacific Life which stated that the 20 year endowment
plan was not available for minors below 7, but that
Pacific Life could consider the same under the
Juvenile Triple Action Plan, advising that if the offer
was acceptable, the Juvenile Non-Medical Declaration
be sent to the company.
-Mondragon allegedly failed to inform Ngo Hing of
the non-acceptance of the insurance plan, instead
writing Pacific Life again, recommending the
approval of the endowment plan to children since
customers had been asking for such coverage since
1954.
-On May 28, 1957, Helen died of influenza. Ngo Hing
sought the payment of the proceeds of the
insurance, but having failed to do so, filed an action
for recovery with the CFI of Cebu. The Court ordered
Pacific Life to pay P50k with 6% interest, hence this
petition.

ISSUE
WON the binding deposit receipt constituted a
temporary contract of the life insurance in question

HELD
NO
- The binding deposit receipt is merely a provisional
contract and only upon compliance with the ff
conditions: (1) that the company be satisfied that
the applicant was insurable on standard rates (2)
that if the company does not accept the application
and offers a different policy, the insurance contract
shall not be binding until the applicant accepts the
new policy (3) that if the applicant is not found to be
insurable on standard rates and the application is
disapproved, the insurance shall not be in force at
any time and the premium be returned to the
applicant.
-This implies the receipt is merely an
acknowledgement, on behalf of the company, that
the Cebu branch of Pacific Life had received the
premium and had accepted the application subject to
processing by the insurance company, which will
approve or reject it depending on whether the
applicant is insurable on standard rates. As such, the
receipt was never in forceit does not insure
outright. No liability attaches until the principal
approves the risk and a receipt is given by the
agent; because private respondent failed to accept
Pacific Lifes offer for the Juvenile Triple Action plan,
there was no meeting of the minds and thus no
contract. Also, being an authorized agent of Pacific
Life, Ngo Hing must have known the company did
not offer the insurance applied for and merely took a
chance on Mondragons recommendation.
Disposition the decision appealed from is set aside,
absolving Pacific Life from their civil liabilities

EDILLON v. MANILA BANKERS LIFE
117 SCRA 187
VASQUEZ; September 30, 1982

NATURE
!"#$%&"'( *+,- 33

Appeal from a decision of the CFI

FACTS
- Sometime in April 1969, Carmen O, Lapuz applied
with respondent insurance corporation for insurance
coverage against accident and injuries. In the
application form which was dated April 15, 1969, she
gave the date of her birth as July 11, 1904. On the
same date, she paid the sum of P20.00 representing
the premium for which she was issued the
corresponding receipt signed by an authorized agent
of the respondent insurance corporation. Upon the
filing of said application and the payment of the
premium on the policy applied for, the respondent
insurance corporation issued to Carmen O. Lapuz its
Certificate of Insurance. The policy was to be
effective for a period of 90 days.
- On May 31, 1969 or during the effectivity of the
Insurance, Carmen O. Lapuz died in a vehicular
accident.
- On June 7, 1969, petitioner Regina L. Edillon, a
sister of the insured and who was the named
beneficiary in the policy, filed her claim for the
proceeds of the insurance, submitting all the
necessary papers and other requisites with the
private respondent. Her claim having been denied,
Regina L. Edillon instituted this action in the Court of
First Instance of Rizal.
- In resisting the claim of the petitioner, the
respondent insurance corporation relies on a
provision contained in the Certificate of Insurance,
excluding its liability to pay claims under the policy
in behalf of "persons who are under the age of
sixteen (16) years of age or over the age of sixty
(60) years ..." It is pointed out that the insured
being over sixty (60) years of age when she applied
for the insurance coverage, the policy was null and
void, and no risk on the part of the respondent
insurance corporation had arisen therefrom.
- RTC dismissed the complaint.

ISSUE
WON the acceptance by the private respondent
insurance corporation of the premium and the
issuance of the corresponding certificate of insurance
should be deemed a waiver of the exclusionary
condition of overage stated in the said certificate of
insurance

HELD
YES
- The age of the insured Carmen 0. Lapuz was not
concealed to the insurance company. Her application
for insurance coverage which was on a printed form
furnished by private respondent and which contained
very few items of information clearly indicated her
age of the time of filing the same to be almost 65
years of age. Despite such information which could
hardly be overlooked in the application form,
considering its prominence thereon and its
materiality to the coverage applied for, the
respondent insurance corporation received her
payment of premium and issued the corresponding
certificate of insurance without question. The
accident which resulted in the death of the insured, a
risk covered by the policy, occurred on May 31, 1969
or FORTY-FIVE (45) DAYS after the insurance
coverage was applied for. There was sufficient time
for the private respondent to process the application
and to notice that the applicant was over 60 years of
age and thereby cancel the policy on that ground if it
was minded to do so. If the private respondent failed
to act, it is either because it was willing to waive
such disqualification; or, through the negligence or
incompetence of its employees for which it has only
itself to blame, it simply overlooked such fact. Under
the circumstances, the insurance corporation is
already deemed in estoppel. Its inaction to revoke
the policy despite a departure from the exclusionary
condition contained in the said policy constituted a
waiver of such condition.
Disposition Judgment appealed from is REVERSED
and SET ASIDE and respondent insurance
corporation is ordered to pay to the petitioner the
proceeds of Insurance

HARDING v. COMMERCIAL UNION (supra p.36)

TAN v. CA (PHILIPPINE AMERICAN LIFE
INSURANCE COMPANY)
174 SCRA 403
GUTIERREZ; June 29, 1989

NATURE
Review on certiorari of the decision of the Court of
Appeals affirming the decision of the Insurance
Commissioner

FACTS
- On September 23,1973, Tan Lee Siong, father of
herein petitioners, applied for life insurance in the
amount of P 80,000.00 with respondent company.
Said application was approved and was issued
effective November 6, 1973
- On April 26,1975, Tan Lee Siong died of hepatoma
(Exhibit B). Petitioners then filed with respondent
company their claim for the proceeds of the life
insurance policy
-respondent company denied petitioners' claim and
rescinded the policy by reason of the alleged
misrepresentation and concealment of material facts
made by the deceased Tan Lee Siong in his
application for insurance. The premiums paid on the
policy were thereupon refunded
- Petitioners filed on November 27, 1975, a
complaint against the former with the Office of the
Insurance Commissioner. Commissioner denied
petition. CA affirmed Commissioners decision

ISSUE
WON according to Sec. 48 of the Insurance Code,
insurance company is barred from rescinding
contract

HELD
- Section 48. Whenever a right to rescind a contract
of insurance is given to the insurer by any provision
of this chapter, such right must be exercised
previous to the commencement of an action on the
contract.
After a policy of life insurance made payable on the
death of the insured shall have been in force during
the lifetime of the insured for a period of two years
from the date of its issue or of its last reinstatement,
the insurer cannot prove that the policy is void ab
initio or is rescindable by reason of the fraudulent
concealment or misrepresentation of the insured or
his agent.
- According to the petitioners, the Insurance Law
was amended and the second paragraph of Section
48 added to prevent the insurance company from
exercising a right to rescind after the death of the
insured
- The so-called "incontestability clause" precludes the
insurer from raising the defenses of false
representations or concealment of material facts
insofar as health and previous diseases are
concerned if the insurance has been in force for at
least two years during the insured's lifetime. The
phrase "during the lifetime" found in Section 48
simply means that the policy is no longer considered
in force after the insured has died. The key phrase in
the second paragraph of Section 48 is "for a period
of two years."
!"#$%&"'( *+,- 34

- The policy was issued on November 6,1973 and the
insured died on April 26,1975. The policy was thus in
force for a period of only one year and five months.
Considering that the insured died before the two-
year period had lapsed, respondent company is not,
therefore, barred from proving that the policy is void
ab initio by reason of the insured's fraudulent
concealment or misrepresentation.
- The petitioners contend that there could have been
no concealment or misrepresentation by their late
father because Tan Lee Siong did not have to buy
insurance. He was only pressured by insistent
salesmen to do so
-The legislative answer to the arguments posed by
the petitioners is the "incontestability clause" added
by the second paragraph of Section 48. The insurer
has two years from the date of issuance of the
insurance contract or of its last reinstatement within
which to contest the policy, whether or not, the
insured still lives within such period. After two years,
the defenses of concealment or misrepresentation,
no matter how patent or well founded, no longer lie

TAN CHAY HENG v. WEST COAST LIFE
INSURANCE
51 PHIL 80
JOHNS; November 21, 1927

FACTS
- Plaintiff alleges that defendant accepted and
approved a life insurance policy of for the sum of
P10,000 in which the plaintiff was the sole
beneficiary; that the policy was issued upon the
payment by the said Tan Ceang of the first year's
premium amounting to P936; that in and by its
terms, the defendant agreed to pay the plaintiff as
beneficiary the amount of the policy upon the receipt
of the proofs of the death of the insured while the
policy was in force; that without any premium being
due or unpaid, Tan Ceang died on May 10, 1925;
that in June, 1925, plaintiff submitted the proofs of
the death of Tan Ceang with a claim for the payment
of the policy which the defendant refused to pay, for
which he prays for a corresponding judgment, with
legal interest from the date of the policy, and costs.
- Defendant alleges that the insurance policy on the
life of Tan Ceang, upon which plaintiff's action is
based, was obtained by the plaintiff in confabulation
with one Go Chulian, of Bacolod, Negros Occidental;
Francisco Sanchez of the same place; and Dr. V. S.
Locsin, of La Carlota, Negros Occidental, thru fraud
and deceit perpetrated against this defendant in the
following manner, to wit:
1. Go, Sanchez and Locsin, caused Tan Caeng to
sign an application for insurance with the
defendant in the sum of P10,000, in which it was
said that Tan Ceang was single and was a
merchant, and that the plaintiff Tan Chai Heng,
the beneficiary, was his nephew, whereas in
truth and in fact and as the plaintiff and his said
coconspirators well knew, the said Tan Ceang
was not single but was married and had several
children; and was not a merchant but a mere
employee of Tan Quina from whom he received
only a meager salary, and that plaintiff was not
a nephew of the said Tan Ceang.
2. Tan Ceang was seriously ill, suffering from
pulmonary tuberculosis of about three years'
duration, which illness was incurable and was
well known to the plaintiff and his said
coconspirators.
3. Locsin, in his capacity as medical examiner for
the defendant, prepared and falsified the
necessary medical certificate, in which it was
made to appear, among other things, that Tan
Ceang had never used morphine, cocaine or any
other drug; that he was then in good health and
had never consulted any physician; that he had
never spit blood; and that there was no sign of
either present or past disease of his lungs;
whereas in truth and in fact, plaintiff and
coconspirators well knew, Tan Ceang was
addicted to morphine, cocaine, and opium and
had been convicted and imprisoned therefor,
and for about three year prior thereto had been
suffering from pulmonary tuberculosis.
4. Plaintiff caused a confidential report to the
defendant insurance company to be signed by
one V. Sy Yock Kian, who was an employee of
Go Chulian, in which it was falsely represented
that Tan Ceang was worth about P40,000, had
an annual income of from eight to ten thousand
pesos net, had the appearance of good health,
and never had tuberculosis.
5. After said application for insurance, medical
certificate and confidential report had been
prepared and falsified, plaintiff and
coconspirators caused the same to be forwarded
to the defendant at its office in Manila, the
medical certificate thru the said Dr. V. S. Locsin
as medical examiner, and said application for
insurance and confidential report thru the said
Francisco Sanchez in his capacity as one of the
agents of the defendant insurance company in
the Province of Occidental Negros; that the
defendant, believing that the representations
made in said document were true, and relying
thereon, provisionally accepted the said
application for insurance on the life of Tan
Ceang in the sum of P10,000 and issued a
temporary policy pending the final approval or
disapproval of said application by defendant's
home-office in San Francisco, California, where
in case of approval a permanent policy was to be
issued; that such permanent policy was never
delivered to the plaintiff because defendant
discovered the fraud before its delivery.
6. That the first agreed annual premium on the
insurance in question of P936.50 not having
been paid within 60 days after medical
examination of the applicant as required by the
regulations of the defendant insurance company,
plaintiff and coconspirators caused Tan Ceang to
sign a health certificate for reinstatement; that
the said temporary policy was delivered by
defendant to the insured on April 10, 1925, in
the belief that said statements and
representations were true and in reliance
thereon.
7. 2 months after the supposed medical
examination above referred to, and exactly 1
month after the date of the health certificate for
reinstatement above set forth, Tan Ceang died
in Valladolid, Occidental Negros, of pulmonary
tuberculosis, the same illness from which
suffering at the time it is supposed he was
examined by Dr. V. S. Locsin, but that the
plaintiff coconspirators, pursuant to their
conspiracy, caused the said Dr. V. S. Locsin to
state falsely in the certificate of death that the
said Tan Ceang had died of cerebral
hemorrhage.
- Defendant also alleges that plaintiff was, like V. Sy
Yock Kian, an employee of Go Chulian; that the
latter was the ringleader of a gang of malefactors,
who, during, and for some years previous to the
dates above mentioned, were engaged in the illicit
enterprise of procuring fraudulent life insurances
from the present defendant, similar to the one in
question, and which enterprise was capitalized by
him by furnishing the funds with which to pay the
premium on said fraudulent insurance; that the said
Go Chulian was the one who furnished the money
with which to pay the first and only annual premium
on the insurance here in question, amounting to
!"#$%&"'( *+,- 35

P936.50; that the said Go Chulian, on August 28,
1926, was convicted by the Court of First Instance of
the City of Manila, in criminal case No. 31425 of that
court, of the crime of falsification of private
documents in connection with an fraudulent
insurance, similar to the present, committed against
this defendant in the month of September, 1924;
that in the same case the said Francisco Sanchez
was one of the coaccused of the said Go Chulian but
was discharged from the complaint, because he
offered himself and was utilized as a state's witness;
that there is another civil action now pending against
Go Chulian and Sanchez in the Court of First
Instance of Manila (civil case No. 28680), in which
the present defendant is the plaintiff, for the
recovery of the amounts of two insurance policies
aggregating P19,000, fraudulently obtained by the
said Go Chulian and Sanchez.
- To this, plaintiff filed a demurrer which was
granted.

ISSUE
WON defense is barred by Art. 47

HELD
NO
Ratio The word "rescind" has a well defined legal
meaning, and as applied to contracts, it presupposes
the existence of a contract to rescind.
Reasoning
- Plaintiff vigorously contends that section 47 of the
Insurance Act should be applied, and that when so
applied, defendant is barred and estopped to plead
and set forth the matters alleged in its special
defense. That section is as follows:
Whenever a right to rescind a contract of
insurance is given to the insurer by any provision
of this chapter, such right must be exercised
previous to the commencement of an action on the
contract.
- It will be noted that defendant does not seek to
have the alleged insurance contract rescinded. It
denies that it ever made any contract of insurance
on the life of Tan Ceang or that any such a contract
ever existed, and that is the question which it seeks
to have litigated by its special defense. In the very
nature of things, if the defendant never made or
entered into the contract in question, there is no
contract to rescind, and, hence, section 47 upon
which the lower based its decision in sustaining the
demurrer does not apply. As stated, an action to
rescind a contract is founded upon and presupposes
the existence of the contract which is sought to be
rescinded. If all of the material matters set forth and
alleged in the defendant's special plea are true, there
was no valid contract of insurance, for the simple
reason that the minds of the parties never met and
never agreed upon the terms and conditions of the
contract. We are clearly of the opinion that, if such
matters are known to exist by a preponderance of
the evidence, they would constitute a valid defense
to plaintiff's cause of action. Upon the question as to
whether or not they or are not true, we do not at this
time have or express any opinion, but we are clear
that section 47 does not apply to the allegations
made in the answer, and that the trial court erred in
sustaining the demurrer.
Disposition The judgment of the lower court is
reversed and the case is remanded for such other
and further proceedings as are not inconsistent with
this opinion, with costs against the plaintiff.

PIONEER INSURANCE AND SURETY
CORPORATION v. YAP
61 SCRA 426
FERNANDEZ; December 19, 1974

NATURE
Appeal by certiorari from CA decision affirming a CFI
decision which declared plaintiff Yap entitled to
recover from defendant Pioneer Insurance and
Surety Corp, the full amount of the damage inquired
in Policy No. 4219

FACTS
- Yap owned a store in a 2 storey building, where
she sold shopping bags and footwear. Her son-in-law
was in charge of the store
- April 19, 1962- Yap took out Fire Insurance Policy
No. 4216 from Pioneer with a face value of P25,000
covering her stocks, office furniture, fixtures, etc.
- among the conditions set forth:
The Insured shall give notice to the
Company of any insurance or insurances already
effected, or which may subsequently be
effected, covering any of the property hereby
insured, and unless such notice be given and
the particulars of such insurance or insurances
be stated in, or endorsed on this Policy by or on
behalf of the Company before the occurrence of
any loss or damage, all benefits under this
Policy shall be forfeited. (emphasis supplied)
It is understood that, except as may be
stated on the face of this policy there is no other
insurance on the property hereby covered and
no other insurance is allowed except by the
consent of the Company endorsed hereon. Any
false declaration or breach or this condition will
render this policy null and void.
- At the time of insurance of Policy 4219(April 19,
1962), an insurance policy for P20,000 issued by the
Great American Insurance Company covering the
same properties was noted on said policy as co-
insurance.
- August 29, 1962 : parties executed an
endorsement on Policy 4219 stating:
It is hereby declared and agreed that the co-
insurance existing at present under this policy is
as follows: P20,000.00 Northwest Ins., and not
as originally stated. (emphasis supplied)
Except as varied by this endorsement, all other
terms and conditions remain unchanged.
- September 26, 1962: Yap took out another fire
insurance policy for P20,000 covering the same
properties, from Federal Insurance Company. This
policy was procured without notice to and the written
consent of Pioneer, and was therefore not noted as a
co-insurance in Policy 4219.
- December 19, 1962: Fire burned Yaps store

ISSUE
WON petitioner should be absolved from liability on
Fire insurance Policy No. 4219 on account of any
violation by respondent Yap of the co-insurance
clause therein

HELD
YES
- The petitioner should be absolved.
Reasoning
- There was a violation by Yap of the co-insurance
clause contained in Policy No. 4219 which resulted in
the avoidance of the petitioners liability.
- By the plain terms of the policy, other insurance
without the consent of petitioner would ipso facto
avoid the contract. It required no affirmative act of
election on the part of the company to make
operative the clause avoiding the contract, wherever
the specified conditions should occur. Its obligations
ceased, unless, being informed of the fact, it
consented to the additional insurance.
- The obvious purpose of the aforesaid requirement
in the policy is to prevent over-insurance and thus
avert the perpetration of fraud. The public, as well as
the insurer, is interested in preventing the situation
in which a fire would be profitable to the insured.
!"#$%&"'( *+,- 36

According to Justice Story: "The insured has no right
to complain, for he assents to comply with all the
stipulation on his side, in order to entitle himself to
the benefit of the contract, which, upon reason or
principle, he has no right to ask the court to
dispense with the performance of his own part of the
agreement, and yet to bind the other party to
obligations, which, but for those stipulation would
not have been entered into."
Disposition the appealed judgment of the Court of
Appeals is reversed and set aside, and the petitioner
absolved from all liability under the policy.

NEW LIFE ENTERPRISES v. CA
207 SCRA 609
REGALADO; March 31, 1992

FACTS
- Julian Sy and Jose Sy Bang are partners engaged
in the business of selling construction materials
under the business name New Life Enterprises.
Julian Sy insured against fire the stocks in trade of
New Life Enterprises with Western Guaranty
Corporation, Reliance Surety and Insurance Co. Inc.,
and Equitable Insurance Corporation in the
aggregate amount of P1,550,000.00. When the
building where New Life Enterprises was located,
along with the stocks in trade therein, were gutted
by fire, petitioners filed an insurance claim against
the three companies. The insurance companies all
denied Julian Sys claim on the ground of breach of
policy condition, (i.e., the other insurance clause
which required New Life Enterprises to inform each
of the insurance companies in case the former
insures with another company the same property
already insured by each of the insurance
companies).
- Because of the denial of their claims for payment
by the 3 insurance companies, petitioners filed
separate civil actions against the former before the
Regional Trial Court of Lucena City, which cases were
consolidated for trial. The trial court ruled in favor of
petitioner. However, the Court of Appeals reversed
the trial courts decision, found petitioner to have
violated Clauses 3 and 27 of the separate insurance
policies issued by the 3 companies, and exonerated
the insurance companies from liability.

ISSUE
WON petitioners violated the Other Insurance
Clause of the insurance policies

HELD
YES
- Petitioners admit that the respective insurance
policies issued by private respondents did not state
or endorse thereon the other insurance coverage
obtained or subsequently effected on the same
stocks in trade for the loss of which compensation is
claimed by petitioners. It is further admitted by
petitioners that Equitable's policy stated "nil" in the
space thereon requiring indication of any co-
insurance although there were 3 policies subsisting
on the same stocks in trade at the time of the loss,
namely, that of Western in the amount of
P350,000.00 and two 2 policies of Reliance in the
total amount of P1,000,000.00.
- The coverage by other insurance or co-insurance
effected or subsequently arranged by petitioners
were neither stated nor endorsed in the policies of
the 3 private respondents, warranting forfeiture of all
benefits thereunder if we are to follow the express
stipulation in Policy Condition No. 3.
- The terms of the contract are clear and
unambiguous. The insured is specifically required to
disclose to the insurer any other insurance and its
particulars which he may have effected on the same
subject matter. The knowledge of such insurance by
the insurer's agents, even assuming the acquisition
thereof by the former, is not the "notice" that would
stop the insurers from denying the claim. Besides,
the so-called theory of imputed knowledge, that is,
knowledge of the agent is knowledge of the principal,
aside from being of dubious applicability here has
likewise been roundly refuted by respondent court
whose factual findings we find acceptable. The mere
fact that Yap Kam Chuan was an agent for both
Reliance and Equitable does not justify the allegation
that the two are sister companies. Availment of the
services of the same agents and adjusters by
different companies is a common practice in the
insurance business and such facts do not warrant the
speculative conclusion of the trial court.
- Considering the terms of the policy which required
the insured to declare other insurances, the
statement in question must be deemed to be a
statement (warranty) binding on both insurer and
insured, that there were no other insurance on the
property. The annotation then, must be deemed to
be a warranty that the property was not insured by
any other policy. Violation thereof entitled the
insurer to rescind.
- The obvious purpose of the aforesaid requirement
in the policy is to prevent over-insurance and thus
avert the perpetration of fraud. The public, as well as
the insurer, is interested in preventing the situation
in which a fire would be profitable to the insured.
The insured has no right to complain, for he assents
to comply with all the stipulations on his side, in
order to entitle himself to the benefit of the contract,
which, upon reason or principle, he has no right to
ask the court to dispense with the performance of his
own part of the agreement, and yet to bind the other
party to obligations, which, but for those
stipulations, would not nave been entered into.
- It is not disputed that the insured failed to reveal
before the loss three other insurances. By reason of
said unrevealed insurances, the insured had been
guilty of a false declaration; a clear
misrepresentation and a vital one because where the
insured had been asked to reveal but did not, that
was deception. Otherwise stated, had the insurer
known that there were many co-insurances, it could
have hesitated or plainly desisted from entering into
such contract. Hence, the insured was guilty of clear
fraud.
- As the insurance policy against fire expressly
required that notice should be given by the insured
of other insurance upon the same property, the total
absence of such notice nullifies the policy.
- Additionally, insofar as the liability of respondent
Reliance is concerned, it is not denied that the
complaint for recovery was filed in court by
petitioners only on January 31, 1984, or after more
than one (1) year had elapsed from petitioners'
receipt of the insurers' letter of denial on November
29, 1982.
- The condition contained in an insurance policy that
claims must be presented within one year after
rejection is not merely a procedural requirement but
an important matter essential to a prompt
settlement of claims against insurance companies as
it demands that insurance suits be brought by the
insured while the evidence as to the origin and cause
of destruction have not yet disappeared.

QUA CHEE GAN v. LAW UNION (supra p.48)







YOUNG v. MIDLAND TEXTILE INSURANCE CO.
30 PHIL 617
!"#$%&"'( *+,- 47

JOHNSON; March 31, 1915

FACTS
- K.S. Young had a candy and fruit store on the
Escolta, Manila, and occupied a building at 321 Calle
Claveria, as a residence and bodega. The Midland
Textile Insurance Co. in consideration of the
payment of a premium of P60, entered into a
contract of insurance with Young by the terms of
which the company, upon certain conditions,
promised to pay Young the sum of P3,000 in case
said residence and bodega and contents should be
destroyed by fire.
- One of the conditions of the contract is: "Warranty
B It is hereby declared and agreed that during the
pendency of this policy no hazardous goods be
stored or kept for sale, and no hazardous trade or
process be carried on, in the building to which this
insurance applies, or in any building connected
therewith."
- Young placed in the residence and bodega three
boxes filled with fireworks. Said residence and
bodega and the contents thereof were partially
destroyed by fire.
- The fireworks had been given to Young by the
former owner of the Luneta Candy Store. He
intended to use them in the celebration of the
Chinese New Year. However, the authorities of the
city of Manila had prohibited the use of fireworks on
said occasion, so Young then placed them in the
bodega where they remained from the 4th or 5th of
February, 1913 until after the fire of March 18, 1913.
- Both parties agree that the fireworks come within
the phrase "hazardous goods," mentioned in
"Warranty B" of the policy; that the fireworks were
found in a part of the building not destroyed by the
fire and that they in no way contributed to the fire,
or to the loss that resulted.
- The lower court rendered a judgment in favor of
Young for the sum of P2,708.78, and costs.

ISSUE
1. WON the placing of the fireworks in the building
insured, they being "hazardous goods," was a
violation of the terms of the contract of insurance
and especially of "Warranty B."

HELD
1. YES.
Reasoning It is admitted by both parties that the
fireworks are hazardous goods. The defendant
alleged that they were "stored." The plaintiff
contends that under all the facts and circumstances
of the case, they were not stored in said building,
and that the placing of them in the building was not
a violation of the terms of the contract.
- Whether a particular article is "stored" or not must,
in some degree, depend upon the intention of the
parties. Nearly all of the cases cited by the lower
court are cases where the article was being put to
some reasonable and actual use, which might easily
have been permitted by the terms of the policy, and
within the intention of the parties, and excepted
from the operation of the warranty, like the present.
- (1) Where merchants have had or kept the
"hazardous" articles in small quantities, and for
actual daily use, for sale, such as gasoline,
gunpowder, etc.; (2) Where such articles have been
brought on the premises for actual use thereon, and
in small quantities, such as oil, paints, etc; and (3)
Where such articles or goods were used for lighting
purposes, and in small quantities.
- In the present case no claim is made that the
"hazardous goods" were placed in the bodega for
present or daily use. It is admitted that they were
placed in the bodega "for future use," or for future
consumption, or for safe keeping. It seems clear to
us that the "hazardous goods" in question were
"stored" in the bodega, as that word is generally
defined. That being true, suppose the defendant had
made an examination of the premises, even in the
absence of a fire, and had found the "hazardous
goods" there, would it not have been justified in
declaring the policy null and of no effect by reason of
a violation of its terms? If it might, then may it not
repudiate its liability, even after the fire? If the
"warranty" is a term of the contract, will not its
violation cause a breach and justify noncompliance
or repudiation?
- Contracts of insurance are contracts of indemnity,
upon the terms and conditions specified therein.
Parties have a right to impose such reasonable
conditions at the time of the making of the contract
as they deem wise and necessary. The rate of
premium is measured by the character of the risk
assumed. The insurer, for a comparatively small
consideration, undertakes to guarantee the insured
against loss or damage, upon the terms and
conditions agreed upon, and upon no other. When
the insurer is called upon to pay, in case of loss, he
may justly insist upon a fulfillment of the terms of
the contract. If the insured, cannot bring himself
within the terms and conditions of the contract, he is
not entitled to recover for any loss suffered. The
terms of the contract constitute the measure of the
insurer's liability. If the contract has been
terminated, by a violation of its terms on the part of
the insured, there can be no recovery. Compliance
with the terms of the contract is a condition
precedent to the right of recovery.
- Young argues that since the "storing" of the
fireworks on the premises did not contribute in any
way to the damage occasioned by the fire, he should
be permitted to recover. That argument, however, is
beside the question, if the "storing" was a violation
of the terms of the contract. The violation of the
terms of the contract, by virtue of the provisions of
the policy itself, terminated, at the election of either
party, the contractual relations.
- Young paid a premium based upon the risk at the
time the policy was issued. Certainly, the placing of
the firecrackers in the building insured increased the
risk. Young had not paid a premium based upon the
increased risk, neither had the defendant issued a
policy upon the theory of a different risk. He was
enjoying, if his contention may be allowed, the
benefits of an insurance policy upon one risk,
whereas, as a matter of fact, it was issued upon an
entirely different risk. The defendant had neither
been paid nor had issued a policy to cover the
increased risk. An increase of risk which is
substantial and which is continued for a considerable
period of time, is a direct and certain injury to the
insurer, and changes the basis upon which the
contract of insurance rests.
Disposition Decision of the lower court is
REVERSED.

TAN v. CA (supra p.51)

AREOLA v. CA (supra p.26)

TAN CHAY v. WEST COAST (supra p.51)

FILIPINAS LIFE ASSURANCE v. NAVA
17 SCRA 210
BAUTISTA ANGELO; May 20, 1966

NATURE
Petition for review of a decision of the Court of
Appeals

FACTS
- Before the war, Nava entered into a contract of
insurance with Insular Life Assurance Co., Ltd. (face
value of P5k), and 17 separate contracts of life
!"#$%&"'( *+,- 4.

insurance with Filipinas Life Assurance Co. (total face
value of P90k). Each and everyone of the 18 policies
issued by defendants to plaintiff contains a loan
clause of the following tenor:
Policy loans. After three full years'
premiums have been paid upon this Policy, if no
premium payment is in default, the Company,
subject to its then existing rules, will advance on
proper assignment and delivery of this Policy and
on the sole security thereof a sum equal to, or at
the option of the owner less than, the cash value
specified in the Schedule of Policy Values, less any
existing indebtedness on or secured by this Policy
and any unpaid balance of the premium for the
current policy-year; provided interest at six per
centum per annum on the whole amount of the
loan is paid in advance to the end of the current
policy-year. At the end of the current policy-year
interest at the same rate for one year in advance
will be due and payable, and annually thereafter,
and if not so paid will be added to the principal and
bear the same rate of interest. Failure to repay
any such loan or interest shall not avoid this Policy
unless the total indebtedness shall equal or exceed
the full amount of the loan value available
hereunder.
Any indebtedness on this Policy shall first be
deducted from any money payable or in any
settlement under this Policy.
- Nava had so far paid to Insular a total of P2,574;
and to Filipinas Life, a total of P32,072.60.
- April 28, 1948: Nava applied to the companies for a
P5k loan in line with the loan clause, but they
refused to grant it because certain regulations issued
by the Insurance Commissioner required the
insurance companies to withhold the payments on
premiums made during the Japanese occupation
because the same shall be subject to future
adjustments " as soon as debtor-creditor relationship
is established" and because of such process of
"withholding" plaintiff was not entitled to borrow any
amount until such adjustment has been made.
- Sept 30, 1948: Nava called the attention of the
insurance companies to the SC decision (Haw Pia v.
China Banking Corporation) establishing and
recognizing the relationship of debtor and creditor
with respect to payments in fiat currency made
during the Japanese occupation on pre-war
obligations.
- Companies still refused saying that the SC decision
was not applicable to transactions undertaken during
Japanese occupation when they relate to life
insurance policies.
- Feb 4, 1949: Nava was again refused even if the
total amount of the cash surrender values of the 18
policies reached the sum of P9,468.29.
- Feb 10, 1949: Nava brought case to the CFI Manila
praying for the rescission of the abovementioned 18
policies and for the refund to him of all the premiums
so far paid by him to defendants in the amount of
P31,633.80, plus 6% interest thereon as damages
- Nov 28, 1951: companies passed a resolution
which was approved by the Insurance Commissioner,
giving full credit to all premium payments made by
their policyholders in fiat currency during the
Japanese occupation on account of pre-war policies
for which reason they filed an amended answer
offering to pay plaintiff the amount of P9,468.29
which represents the aggregate cash surrender
values of all the policies in question as of February
10, 1949, but apparently this offer was refused.
- CFI: (1) rescinded the insurance contracts; (2)
ordered defendant Filipinas Life Assurance Co. to pay
plaintiff the amount of P32,072.60; and (3) ordered
defendant Insular Life Assurance Co., Ltd. to pay
plaintiff the amount of P2,574.00
- CA affirmed.

ISSUES
1. WON CA erred in ruling that as a consequence of
the decision in the Haw Pia case petitioners violated
the loan clause contained in the insurance policies
thereby entitling respondent to their rescission
2. WON CA erred in ruling that by virtue of Article
1295 of the old Civil Code petitioners should refund
to defendant all the premiums paid on his insurance
policies as a consequence of their rescission
3. WON CA erred in not ruling that, even if
respondent is entitled to the rescission of said
insurance policies, he can only recover their cash
surrender value at the time the complaint was filed

HELD
1. NO.
- Even assuming the validity of the Insurance
Commissioners regulations, the fact however is that
such requirement has already lost its legal effect and
value when our Supreme Court rendered its decision
in the Haw Pia case wherein it was declared, among
others, that all payments made in fiat currency
during the Japanese occupation in relation with any
contractual obligation executed before the war were
valid to all intents and purposes, and yet petitioners
apparently did not give any importance to such
decision for in their opinion it does not have any
application to transactions which have any relation to
payment of premiums on life insurance policies.
- It cannot be denied that a life insurance policy
involves a contractual obligation wherein the insured
becomes duty bound to pay the premiums agreed
upon, lest he runs the risk of having his insurance
policy lapse if he fails to pay such premiums.
- The fact that if the insured had paid in full the
premiums corresponding to the first 3 years of the
life of his policy he cannot be considered delinquent
that would cause the lapse of his policy if the same
contains an automatic premium payment clause
cannot divest such policy of its contractual nature,
for the result of such failure would only be for him to
pay later the premium plus the corresponding
interest depending upon the condition of the policy.
But certainly it does not cease to be a contractual
liability insofar as the payment of that premium is
concerned for whether he likes it or not that
premium has to be paid lest he allows the lapse of
his policy. Consequently, the payment of premiums
on the life insurance policies made by Nava before
and during the war up to the time he applied for the
loan in question with petitioners should be
considered likewise as valid payments upon the
theory that such insurance policies are in the nature
of a contractual obligation within the meaning of the
civil law. In effect, therefore, those payments were
made by a debtor to a creditor within the meaning of
the requirement of the regulations of the Insurance
Commissioner and as such they can offer no excuse
to petitioners for refusing to grant the loan as
contemplated in the loan clause embodied in the
policies in question.
- It is clear from the foregoing that the petitioners
violated the loan clause embodied in each of the 18
life insurance policies issued to respondent to rescind
all said policies under Section 69 of the Insurance
Act, which provides: "The violation of a material
warranty, or other material provision of a policy, on
the part of either party thereto, entitles the other to
rescind."
- "The general rule is that a breach of the agreement
to make the loan does not entitle the insured to
rescind the contract," is not controlling in this
jurisdiction. Firstly, it was not shown that the
insurance laws in the states where said ruling
prevails contain a provision identical to Section 69 of
our Insurance Law we quoted above, and secondly,
the rule cited by Vance is not a rule uniformly
!"#$%&"'( *+,- 4/

followed by all states in the US, for on this matter
there is a marked divergence of opinion.
2. NO
- Considering that our Insurance Law does not
contain an express provision as to what the court
should do in cases of rescission of an insurance
policy under Section 69, the provision that should
apply is that embodied in Article 1225 of the old Civil
Code, as postulated in Article 16 of the same Code,
which provides that on matters which are not
governed by special laws the provisions of said Code
shall supplement its deficiency. And said Article 1295
provides:
ART. 1295. Rescission makes necessary the return
of the things which were the subject-matter of the
contract, with their fruits, and of the price paid,
with interest thereon. ...xxx
- Said the petitioners: "Recovery of the full amount
of the premium after the insurer has sustained for
sometime the risk of the insurance and the insured
has enjoyed the benefit of protection is obviously
unjust and is so recognized by the better
authorities." The ruling above quoted merely
represents the minority rule in the US, the majority
rule being that the insured can recover all premiums
paid, in some cases with interest in case of wrongful
cancellation, repudiation, termination or rescission of
the contract of life insurance.
- Contention that because respondent cannot restore
to petitioners the "value of the benefit of protection"
which he might have received under the 18 life
insurance policies in question he is not entitled to
rescind them under the provision of Article 1295 of
the old Civil Code, is untenable because said article
only contemplates a transaction whether material
things are involved, and do not refer to intangible
ones which cannot be the subject of restoration, for
to interpret it otherwise would be to defeat the law
itself with the result that rescission can never be had
under Section 69 of our Insurance Law.
- It cannot be denied that petitioners had in turn
already derived material benefits from the use of
premiums paid to them by respondent before, during
and after the last war from which they must have
realized huge profits, and in this light alone
petitioners cannot claim prejudice or unfairness if
they are ordered to refund the premiums paid by
respondents.
3. NO.
- Issue is corollary to preceding issue. No need to
refute.
Disposition Decision appealed from is AFFIRMED.
Costs against petitioners

CHAPTER VII. RISKS AND COVERAGES

VDA. DE BATACLAN v. MEDINA
102 PHIL 181
MONTEMAYOR; October 22, 1957

FACTS
- Juan Bataclan rode Bus 30 of Medina Transport,
driven by Saylon, shortly after midnight. The bus
was running very fast. One of the front tires burst.
Bus fell into canal and turned turtle. 4 passengers
couldnt get out, including Bataclan. Gasoline began
to leak from the overturned bus. 10 men came to
help. 1 carried a torch and when he approached the
bus, fire started, killing the trapped passengers.
- TC opined that proximate cause of Bataclans
death was not the overturning of bus but the fire. At
the time fire started, Bataclan, though injured, was
still alive and damages were awarded, not for his
death, but for physical injuries suffered.

ISSUE/S
WON the proximate cause is the overturning of the
bus or the fire

HELD
- The proximate cause is the overturning of the bus.
- Ordinarily, when a bus overturns and pins down
passenger, merely causing him injuries. If through
some event, unexpected and extraordinary, the bus
is set on fire, and passenger is burned to death, one
might contend that the proximate cause was the fire
and not the overturning of the vehicle.
- But here, the proximate cause of Bataclans death
is the overturning of the bus, this for the reason that
when the vehicle turned not only on its side but
completely on its back, leaking of gasoline from the
tank was not unnatural or unexpected.
- The coming of the men with the torch was in
response to the call for help, made only not by the
passengers but even the driver and conductor, and
because it was very dark, about 2:30 am, rescuers
had to carry a light with them. Coming as they did
from rural area where lanterns and flashlights were
not available, they had to use a torch. What was
more natural than that said rescuers should
innocently approach the overturned vehicle to extend
aid.
- The coming of the men with the torch was to be
expected, and was a natural sequence of the
overturning of the bus, the trapping of some of its
passengers and the call for outside help.
- The burning of bus can also in part be attributed to
negligence of carrier, through its driver and
conductor. They, or at least the driver, should have
known that in the position in which the overturned
bus was, gasoline could and must have leaked from
the gasoline tank and soaked the area in and around
the bus. Gasoline can be smelt and detected even
from a distance, and yet neither the driver nor the
conductor would appear to have cautioned or taken
steps to warn rescuers not to bring the lighted torch
too near the bus.

FINMAN GENERAL ASSURANCE CORPORATION
v. CA (SURPOSA)
213 SCRA 493
NOCON; September 2, 1992

NATURE
Certiorari

FACTS
- Oct. 22, 1986: Carlie Surposa was insured with
Finman General Assurance Corporation under Finman
General Teachers Protection Plan Master Policy No.
2005 and Individual Policy No. 08924 with his
parents, spouses Julia and Carlos Surposa, and
brothers Christopher, Charles, Chester and Clifton,
all surnamed, Surposa, as beneficiaries.
- While said insurance policy was in full force and
effect, the insured, Carlie Surposa, died on October
18, 1988 as a result of a stab wound inflicted by one
of the 3 unidentified men without provocation and
warning on the part of the former as he and his
cousin, Winston Surposa, were waiting for a ride on
their way home after attending the celebration of the
"Maskarra Annual Festival."
- Thereafter, Julia Surposa and the other
beneficiaries of said insurance policy filed a written
notice of claim with the FINMAN Corp which denied
said claim contending that murder and assault are
not within the scope of the coverage of the insurance
policy.
- Feb. 24, 1989: Surposa filed a complaint with the
Insurance Commission which subsequently ordered
FINMAN to pay Surposa the proceeds of the policy
with interest.
- CA affirmed said decision.

!"#$%&"'( *+,- 40

ISSUE
WON CA committed GAD in applying the principle of
"expresso unius exclusio alterius" in a personal
accident insurance policy (since death resulting from
murder and/or assault are impliedly excluded in said
insurance policy considering that the cause of death
of the insured was not accidental but rather a
deliberate and intentional act of the assailant in
killing the former as indicated by the location of the
lone stab wound on the insured) [TF they cannot be
made to indemnify the Surposa heirs]

HELD
NO
- The record is barren of any circumstance showing
how the stab wound was inflicted. While the act may
not exempt the unknown perpetrator from criminal
liability, the fact remains that the happening was a
pure accident on the part of the victim. The insured
died from an event that took place without his
foresight or expectation, an event that proceeded
from an unusual effect of a known cause and,
therefore, not expected.
Reasoning
- De la Cruz vs. Capital Insurance & Surety Co., Inc
(1966)~ The terms "accident" and "accidental" as
used in insurance contracts have not acquired any
technical meaning, and are construed by the courts
in their ordinary and common acceptation. Thus, the
terms have been taken to mean that which happen
by chance or fortuitously, without intention and
design, and which is unexpected, unusual, and
unforeseen. An accident is an event that takes place
without one's foresight or expectation an event that
proceeds from an unknown cause, or is an unusual
effect of a known cause and, therefore, not
expected.
Ratio The generally accepted rule is that, death or
injury does not result from accident or accidental
means within the terms of an accident-policy if it is
the natural result of the insured's voluntary act,
unaccompanied by anything unforeseen except the
death or injury. There is no accident when a
deliberate act is performed unless some additional,
unexpected, independent, and unforeseen happening
occurs which produces or brings about the result of
injury or death. In other words, where the death or
injury is not the natural or probable result of the
insured's voluntary act, or if something unforeseen
occurs in the doing of the act which produces the
injury, the resulting death is within the protection of
the policies insuring against death or injury from
accident.
- The personal accident insurance policy involved
herein specifically enumerated only 10 circumstances
wherein no liability attaches to FINMAN for any
injury, disability or loss suffered by the insured as a
result of any of the stimulated causes.
-The principle of " expresso unius exclusio alterius"
the mention of one thing implies the exclusion of
another thing is therefore applicable in the instant
case since murder and assault, not having been
expressly included in the enumeration of the
circumstances that would negate liability in said
insurance policy: the failure of the FINMAN to include
death resulting from murder or assault among the
prohibited risks leads inevitably to the conclusion
that it did not intend to limit or exempt itself from
liability for such death.
- A1377 NCC: The interpretation of obscure words
or stipulations in a contract shall not favor the party
who caused the obscurity.
- NPC vs. CA [1986]~ It is well settled that
contracts of insurance are to be construed liberally in
favor of the insured and strictly against the insurer.
Thus ambiguity in the words of an insurance contract
should be interpreted in favor of its beneficiary.
Disposition DENIED for lack of merit.

CALANOC v. CA
98 PHIL 79
BAUTISTA; December 16, 1955

FACTS
- Basilio was a watchman of the Manila Auto Supply
located at the corner of Avenida Rizal and Zurbaran.
He secured a life insurance policy from the Philippine
American Life Insurance Company in the amount of
P2,000 to which was attached a supplementary
contract covering death by accident. On January 25,
1951, he died of a gunshot wound on the occasion of
a robbery committed in the house of Atty. Ojeda at
the corner of Oroquieta and Zurbaran streets.
Calanoc, the widow, was paid the sum of P2,000,
face value of the policy, but when she demanded the
payment of the additional sum of P2,000
representing the value of the supplemental policy,
the company refused alleging, as main defense, that
the deceased died because he was murdered by a
person who took part in the commission of the
robbery and while making an arrest as an officer of
the law which contingencies were expressly excluded
in the contract and have the effect of exempting the
company from liability.
- It is contended in behalf of the company that
Basilio was killed which "making an arrest as an
officer of the law" or as a result of an "assault or
murder" committed in the place and therefore his
death was caused by one of the risks excluded by
the supplementary contract which exempts the
company from liability. This contention was upheld
by the Court of Appeals. Hence, this petition.

ISSUE
WON the death of the victim comes within the
purview of the exception clause of the
supplementary policy and, hence, exempts the
company from liability

HELD
NO
- Basilio was a watchman of the Manila Auto Supply
which was a block away from the house of Atty.
Ojeda where something suspicious was happening
which caused the latter to ask for help. While at first
he declined the invitation of Atty. Ojeda to go with
him to his residence to inquire into what was going
on because he was not a regular policeman, he later
agreed to come along when prompted by the traffic
policeman, and upon approaching the gate of the
residence he was shot and died. The circumstance
that he was a mere watchman and had no duty to
heed the call of Atty. Ojeda should not be taken as a
capricious desire on his part to expose his life to
danger considering the fact that the place he was in
duty-bound to guard was only a block away. In
volunteering to extend help under the situation, he
might have thought, rightly or wrongly, that to know
the truth was in the interest of his employer it being
a matter that affects the security of the
neighborhood. No doubt there was some risk coming
to him in pursuing that errand, but that risk always
existed it being inherent in the position he was
holding. He cannot therefore be blamed solely for
doing what he believed was in keeping with his duty
as a watchman and as a citizen. And he cannot be
considered as making an arrest as an officer of the
law, as contended, simply because he went with the
traffic policeman, for certainly he did not go there for
that purpose nor was he asked to do so by the
policeman.
- Much less can it be pretended that Basilio died in
the course of an assault or murder considering the
very nature of these crimes. In the first place, there
!"#$%&"'( *+,- 41

is no proof that the death of Basilio is the result of
either crime for the record is barren of any
circumstance showing how the fatal shot was fired.
Perhaps this may be clarified in the criminal case
now pending in court as regards the incident but
before that is done anything that might be said on
the point would be a mere conjecture. Nor can it be
said that the killing was intentional for there is the
possibility that the malefactor had fired the shot
merely to scare away the people around for his own
protection and not necessarily to kill or hit the
victim. In any event, while the act may not exempt
the triggerman from liability for the damage done,
the fact remains that the happening was a pure
accident on the part of the victim. The victim could
have been either the policeman or Atty. Ojeda for it
cannot be pretended that the malefactor aimed at
the deceased precisely because he wanted to take
his life.
Disposition Decision set aside

BIAGTAN v. THE INSULAR LIFE ASSURANCE
COMPANY, LTD.
44 SCRA 58
MAKALINTAL; March 29, 1972

NATURE
Appeal from decision of CFI Pangasinan.

FACTS
- Juan Biagtan was insured with Insular for P5k and
a supplementary contract Accidental Death Benefit
clause for another P5k if "the death of the Insured
resulted directly from bodily injury effected solely
through external and violent means sustained in an
accident . . . and independently of all other causes."
The clause, however, expressly provided that it
would not apply where death resulted from an injury
"intentionally inflicted by a third party."
- One night, a band of robbers entered their house.
Juan went out of his room and he was met with 9
knife stabs. He died. The robbers were convicted of
robbery with homicide.
- The family was claiming the additional P5k from
Insular, under the Accidental Death Benefit clause.
Insular refused on the ground that the death
resulted from injuries intentionally inflicted by 3
rd

parties and was therefore not covered. Biagtans filed
against Insular. CFI ruled in favor of Biagtans.

ISSUE
WON the injuries were intentionally inflicted

HELD
YES
- Whether the robbers had the intent to kill or merely
to scare the victim or to ward off any defense he
might offer, it cannot be denied that the act itself of
inflicting the injuries was intentional.
- The exception in the accidental benefit clause
invoked by the appellant does not speak of the
purpose whether homicidal or not of a third
party in causing the injuries, but only of the fact that
such injuries have been "intentionally" inflicted
this obviously to distinguish them from injuries
which, although received at the hands of a third
party, are purely accidental.
- Examples of unintentional:
>> A gun which discharges while being cleaned and
kills a bystander;
>> a hunter who shoots at his prey and hits a
person instead;
>> an athlete in a competitive game involving
physical effort who collides with an opponent and
fatally injures him as a result.
- In Calanoc vs. CA: Where a shot was fired and it
turned out afterwards that the watchman was hit in
the abdomen, the wound causing his death, the
Court held that it could not be said that the killing
was intentional for there was the possibility that the
malefactor had fired the shot to scare the people
around for his own protection and not necessarily to
kill or hit the victim. A similar possibility is clearly
ruled out by the facts in this case. For while a single
shot fired from a distance, and by a person who was
not even seen aiming at the victim, could indeed
have been fired without intent to kill or injure, nine
wounds inflicted with bladed weapons at close range
cannot conceivably be considered as innocent insofar
as such intent is concerned.
- In Hucthcraft's Ex'r vs. Travelers' Ins. Co. (US
case): where the insured was waylaid and
assassinated for the purpose of robbery, the court
rendered judgment for the insurance company and
held that while the assassination of the insured was
as to him an unforeseen event and therefore
accidental, "the clause of the proviso that excludes
the (insurer's) liability, in case death or injury is
intentionally inflicted by any other person, applies to
this case."
Disposition CFI decision reversed.

SEPARATE OPINION

TEEHANKEE [dissent]
- Calanoc v. CA is controlling in this case because
the insurance company wasnt able to prove that the
killing was intentional. (Burden of proof is with the
insurance company)
- Insurance, being contracts of adhesion, must be
construed strictly against insurance company in
cases of ambiguity.
- The supplementary contract enumerated
exceptions. The only exception which is not
susceptible of classification is that provided in
paragraph 5(e), the very exception herein involved,
which would also except injuries "inflicted
intentionally by a third party, either with or without
provocation on the part of the insured, and whether
or not the attack or the defense by the third party
was caused by a violation of the law by the insured."
- This ambiguous clause conflicts with all the other
four exceptions in the same paragraph 5 particularly
that immediately preceding it in item (d) which
excepts injuries received where the insured has
violated the law or provoked the injury, while this
clause, construed as the insurance company now
claims, would seemingly except also all other
injuries, intentionally inflicted by a third party,
regardless of any violation of law or provocation by
the insured, and defeat the very purpose of the
policy of giving the insured double indemnity in case
of accidental death by "external and violent means"
in the very language of the policy.'
- It is obvious from the very classification of the
exceptions and applying the rule of noscitus a sociis,
that the double-indemnity policy covers the insured
against accidental death, whether caused by fault,
negligence or intent of a third party which is
unforeseen and unexpected by the insured. All the
associated words and concepts in the policy plainly
exclude the accidental death from the coverage of
the policy only where the injuries are self-inflicted or
attended by some proscribed act of the insured or
are incurred in some expressly excluded calamity
such as riot, war or atomic explosion.
- The untenability of insurer's claim that the
insured's death fell within the exception is further
heightened by the stipulated fact that two other
insurance companies which likewise covered the
insured for much larger sums under similar
accidental death benefit clauses promptly paid the
benefits thereof to plaintiffs beneficiaries.

SUN INSURANCE v. CA (LIM)
211 SCRA 554
!"#$%&"'( *+,- 42

CRUZ; July 17, 1992

FACTS
- The petitioner issued Personal Accident Policy to
Felix Lim, Jr. with a face value of P200,000.00. Two
months later, he was dead with a bullet wound in his
head. As beneficiary, his wife Nerissa Lim sought
payment on the policy but her claim was rejected.
The petitioner agreed that there was no suicide. It
argued, however, that there was no accident either.
Pilar Nalagon, Lim's secretary, was the only
eyewitness to his death. According to Nalagon, Lim
was in a happy mood (but not drunk) and was
playing with his handgun, from which he had
previously removed the magazine. As she watched
the television, he stood in front of her and pointed
the gun at her. She pushed it aside and said it might
be loaded. He assured her it was not and then
pointed it to his temple. The next moment there was
an explosion and Lim slumped to the floor. He was
dead before he fell.
- The term "accident" has been defined as follows:
The words "accident" and "accidental" have never
acquired any technical signification in law, and
when used in an insurance contract are to be
construed and considered according to the
ordinary understanding and common usage and
speech of people generally. In substance, the
courts are practically agreed that the words
"accident" and "accidental" mean that which
happens by change or fortuitously, without
intention or design, and which is unexpected,
unusual, and unforeseen. The definition that has
usually been adopted by the courts is that an
accident is an event that takes place without one's
foresight or expectation an event that proceeds
from an unknown cause, or is an unusual effect of
a known case, and therefore not expected.
- An accident is an event which happens without any
human agency or, if happening through human
agency, an event which, under the circumstances, is
unusual to and not expected by the person to whom
it happens. It has also been defined as an injury
which happens by reason of some violence or
casualty to the insured without his design, consent,
or voluntary co-operation.

ISSUE
WON what happened was an accident

HELD
YES
- The petitioner, however, cites one of the four
exceptions provided for in the insurance contract and
contends that the private petitioner's claim is barred
by such provision. It is there stated:
Exceptions The company shall not be liable in
respect of.
1. Bodily injury.
xxx xxx xxx
b. consequent upon.
i) The insured persons attempting to commit
suicide or wilfully exposing himself to needless peril
except in an attempt to save human life.
- To repeat, the parties agree that Lim did not
commit suicide. Nevertheless, the petitioner
contends that the insured willfully exposed himself to
needless peril and thus removed himself from the
coverage of the insurance policy. That posture is
arguable. But what is not is that, as the secretary
testified, Lim had removed the magazine from the
gun and believed it was no longer dangerous. He
expressed assured her that the gun was not loaded.
It is submitted that Lim did not willfully expose
himself to needless peril when he pointed the gun to
his temple because the fact is that he thought it was
not unsafe to do so. The act was precisely intended
to assure Nalagon that the gun was indeed harmless.
Disposition CA Affirmed

DE LA CRUZ v. CAPITAL INSURANCE
17 SCRA 554
BARRERA; June 30, 1966

NATURE
Appeal from the decision of the CFI of Pangasinan

FACTS
- Eduardo de la Cruz, employed in the Itogon-Suyoc
Mines, Inc., was the holder of an accident insurance
policy underwritten by the Capital Insurance &
Surety Co., Inc., for the period beginning November
13, 1956 to November 12, 1957.
- On January 1, 1957, the Itogon-Suyoc Mines, Inc.
sponsored a boxing contest wherein the insured
Eduardo de la Cruz participated.
- In the course of his bout, Eduardo slipped and was
hit by his opponent on the left part of the back of the
head, causing Eduardo to fall, with his head hitting
the rope of the ring.
- He was brought to the Baguio General Hospital, but
he died as a result of hemorrhage, intracranial, left.
- Simon de la Cruz, the father and named beneficiary
of the insured, filed a claim with the insurance
company for payment of the indemnity, but it was
denied.
- He instituted the action in the CFI of Pangasinan for
specific performance.
- Defendant insurer set up the defense that the
death of the insured, caused by his participation in a
boxing contest, was not accidental and, therefore,
not covered by insurance
- The court rendered the decision in favor of the
plaintiff, hence, the present appeal.

ISSUE
WON the death of the insured was not accidental
and, therefore, not covered by insurance
HELD
NO
- The terms "accident" and "accidental", as used in
insurance contracts, have not acquired any technical
meaning, and are construed by the courts in their
ordinary and common acceptation. Thus, the terms
have been taken to mean that which happen by
chance or fortuitously, without intention and design,
and which is unexpected, unusual, and unforeseen.
An accident is an event that takes place without
one's foresight or expectation, an event that
proceeds from an unknown cause, or is an unusual
effect of a known cause and, therefore, not
expected.
- The generally accepted rule is that, death or injury
does not result from accident or accidental means
within the terms of an accident-policy if it is the
natural result of the insured's voluntary act,
unaccompanied by anything unforeseen except the
death or injury. There is no accident when a
deliberate act is performed unless some additional,
unexpected, independent, and unforeseen happening
occurs which produces or brings about the result of
injury or death. In other words, where the death or
injury is not the natural or probable result of the
insured's voluntary act, or if something unforeseen
occurs in the doing of the act which produces the
injury, the resulting death is within the protection of
policies insuring against death or injury from
accident.
- In the present case, while the participation of the
insured in the boxing contest is voluntary, the injury
was sustained when he slid, giving occasion to the
infliction by his opponent of the blow that threw him
to the ropes of the ring.
- The fact that boxing is attended with some risks of
external injuries does not make any injuries received
in the course of the game not accidental. In boxing
!"#$%&"'( *+,- 43

as in other equally physically rigorous sports, such as
basketball or baseball, death is not ordinarily
anticipated to result. If, therefore, it ever does, the
injury or death can only be accidental or produced by
some unforeseen happening or event as what
occurred in this case.
- Furthermore, the policy involved herein specifically
excluded from its coverage:
(e) Death or disablement consequent upon the
Insured engaging in football, hunting, pigsticking,
steeplechasing, polo-playing, racing of any kind,
mountaineering, or motorcycling.
- Death or disablement resulting from engagement in
boxing contests was not declared outside of the
protection of the insurance contract. Failure of the
defendant insurance company to include death
resulting from a boxing match or other sports among
the prohibitive risks leads inevitably to the
conclusion that it did not intend to limit or exempt
itself from liability for such death.
Disposition The decision appealed from is affirmed

FORTUNE INSURANCE v. CA (supra p.7)

PHIL HOME ASSURANCE CORP v. CA (EASTERN
SHIPPING)
257 SCRA 468
KAPUNAN; June 20, 1996

NATURE
- Eastern Shipping Lines, Inc. loaded on board SS
Eastern Explorer in Kobe, Japan, the following
shipment for carriage to Manila and Cebu, freight
pre-paid and in good order and condition: (a) 2
boxes internal combustion engine parts, consigned to
William Lines, Inc.; (b) 10 metric tons (334 bags)
ammonium chloride, consigned to Orca's Company;
(c) 200 bags Glue 300, consigned to Pan Oriental
Match Company; and (d) garments, consigned to
Ding Velayo. All consignations were made by virtue
of a Bill of Lading.
- While the vessel was off Okinawa, a small flame
was detected on the acetylene cylinder located in the
accommodation area near the engine room. As the
crew was trying to extinguish the fire, the cylinder
suddenly exploded, thus causing death and severe
injuries to the crew and instantly setting fire to the
whole vessel.
- SS Eastern Explorer was then found to be a
constructive total loss and its voyage was declared
abandoned.
- After the fire was extinguished, the cargoes which
were saved were loaded to another vessel for
delivery to their original ports of destination. ESLI
charged the consignees several amounts
corresponding to additional freight and salvage
charges.
- The charges were all paid by Philippine Home
Assurance Corporation (PHAC) under protest for and
in behalf of the consignees. PHAC, as subrogee of
the consignees, then filed a complaint before the
RTC of Manila, against ESLI to recover the sum paid
under protest on the ground that the same were
actually damages directly brought about by the fault,
negligence, illegal act and/or breach of contract of
ESLI.
- ESLI contended that it exercised the diligence
required by law in the handling, custody and carriage
of the shipment; that the fire was caused by an
unforeseen event; that the additional freight charges
are due and demandable pursuant to the Bill of
Lading; and that salvage charges are properly
collectible under Act No. 2616, known as the Salvage
Law.
- RTC: dismissed PHAC's complaint and ruled in
favor of ESLI.
- The burning of the vessel was not the fault or
negligence of defendant but a natural disaster or
calamity. Salvage operations conducted by Fukuda
Salvage Company was perfectly a legal operation
and charges made on the goods recovered were
legitimate charges. Section 1
9
of Act No. 2616, the
Salvage Law is applicable. With respect to the
additional freight charged by defendant from the
consignees of the goods, the same are also validly
demandable.
- The burning of "EASTERN EXPLORER" while off
Okinawa rendered it physically impossible for
defendant to comply with its obligation of delivering
the goods to their port of destination pursuant to the
contract of carriage. Under Article 1266 of the Civil
Code, the physical impossibility of the prestation
extinguished defendant's obligation.
- Note: The goods subject of the present
controversy were neither lost nor damaged in transit
by the fire that razed the carrier. In fact, these were
all delivered to the consignees, even if the
transshipment took longer than necessary. What is
at issue therefore is not whether or not the carrier is

`
!"#62./ %$ V71- 0- ./21 +6 2703=$1.C8 %71 F12215 +$ 0%2 ./$;+ 27/55 41 41<+-9 %71 .+-%$+5 +6 %71
.$1=8 +$ 27/55 7/F1 411- /4/-9+-19 4< %71,8 /-9 30.C19 :3 /-9 .+-F1<19 %+ / 2/61 35/.1 4<
+%71$ 31$2+-28 %71 5/%%1$ 27/55 41 1-%0%519 %+ / $1=/$9 6+$ %71 2/5F/;1&
G7+21 =7+8 -+% 410-; 0-.5:919 0- %71 /4+F1 3/$/;$/378 /2202% 0- 2/F0-; / F12215 +$ 0%2 ./$;+ 6$+,
2703=$1.C8 27/55 41 1-%0%519 %+ 50C1 $1=/$9&

liable for the loss, damage, or deterioration of the
goods transported by them but who, among the
carrier, consignee or insurer of the goods, is liable
for the additional charges or expenses incurred by
the owner of the ship in the salvage operations and
in the transshipment of the goods via a different
carrier. In absolving respondent carrier of any
liability, CA sustained the trial court's finding that the
fire that gutted the ship was a natural disaster or
calamity.

ISSUE
WON the burning of the SS Eastern Explorer
rendering it a constructive total loss was a natural
disaster or calamity

HELD
NO
Ratio In our jurisprudence, fire may not be
considered a natural disaster or calamity since it
almost always arises from some act of man or by
human means. It cannot be an act of God unless
caused by lightning or a natural disaster or casualty
not attributable to human agency.
Reasoning
- There was no showing, and none was alleged by
the parties, that the fire was caused by a natural
disaster. On the contrary, there is strong evidence
indicating that the acetylene cylinder caught fire
because of the fault and negligence of respondent
ESLI, its captain and its crew:
(1) The acetylene cylinder which was fully loaded
should not have been stored near the engine room
where the heat generated therefrom could cause the
acetylene cylinder to explode by reason of
spontaneous combustion. ESLI should have easily
foreseen that the acetylene cylinder, containing
highly inflammable material, was in a real danger of
exploding.
(2) ESLI should have known that by storing the
acetylene cylinder in the accommodation area
supposed to be reserved for passengers, it
unnecessarily exposed its passengers to grave
danger and injury.
(3) The fact that the acetylene cylinder was checked,
tested and examined and subsequently certified as
having complied with the safety measures and
standards by qualified experts before it was loaded
in the vessel only shows to a great extent that
negligence was present in the handling of the
acetylene cylinder after it was loaded and while it
was on board the ship.
!"#$%&"'( *+,- 44

- From the foregoing premises, it indubitably follows
that the cargo consignees cannot be made liable to
respondent carrier for additional freight and salvage
charges.
Disposition Judgment appealed from is REVERSED
and SET ASIDE. Respondent Eastern Shipping Lines,
Inc. is ORDERED to return to petitioner Philippine
Home Assurance Corporation the amount it paid
under protest in behalf of the consignees herein.








PHILIPPINE AMERICAN LIFE INSURANCE
COMPANY v. THE AUDITOR GENERAL
22 SCRA 135
SANCHEZ, JANUARY 18, 1968

NATURE
PETITION FOR REVIEW of a ruling of the Auditor
General.

FACTS
- Philamlife, a domestic life insurance corp., and
American International Reinsurance Company
(Airco), a corporation organized under the laws of
the Republic of Panama, entered into a
REINSURANCE TREATY wherein Philamlife agrees to
reinsure with Airco on January 1950. Philamlife
agreed to pay premiums for all reinsurances on an
annual premium basis.
- In July 16, 1959, the Margin Law was approved
and became effective, which exempts certain
obligations from payment of margin fees,
particularly contractual obligations calling for
payment of foreign exchange issued, approved and
outstanding as of the date this Act takes place.
- Central Bank of the Philippines collected
P268,747.48 as foreign exchange margin on
Philamlife remittances to Airco purportedly totalling
$610,998.63 and made subsequent to July 16, 1959.
Philamlife filed a claim for refund on the ground that
the reinsurance premiums remitted were paid in
pursuant to the January 1950 reinsurance treaty,
and therefore exempted.
- Monetary Board exempted Philamlife from payment
of margin fee. However, Auditor of CB refused to
pass in audit Philamlifes claim for refund. Philamlife
sought reconsideration but was denied, saying
reinsurance treaty NOT EXEMPTED.

ISSUES
1. WON the premia remitted were in pursuance of
the reinsurance treaty between Philamlife and Airco
of January 1959, a contract antedating the Margin
Law, and therefore, Philamlife exempted from paying
margin fee
2. WON Margin Law impairs the obligation of contract
3. WON reinsurance contracts abroad would be made
impractical by the imposition of the 25% margin fee

HELD
1. NO
- For an exemption to come into play, there must be
a reinsurance policy or, as in the reinsurance treaty
provided, a "reinsurance cession" which may be
automatic or facultative.
Ratio A reinsurance policy is thus a contract of
indemnity one insurer makes with another to protect
the first insurer from a risk it has already assumed. .
. . In contradistinction, a reinsurance treaty is
merely an agreement between two insurance
companies whereby one agrees to cede and the
other to accept reinsurance business pursuant to
provisions specified in the treaty. The practice of
issuing policies by insurance companies includes,
among other things, the issuance of reinsurance
policies on standard risks and also on substandard
risks under special arrangements. The lumping of the
different agreements under a contract has resulted in
the term known to the insurance world as 'treaties.'
Such a treaty is, in fact, an agreement between
insurance companies to cover the different situations
described. Reinsurance treaties and reinsurance
policies are not synonymous. Treaties are
contracts for insurance; reinsurance policies or
cessions are contracts of insurance.
Reasoning
- Even if reinsurance treaty preceded the Margin Law
by over nine years, nothing in the treaty obligates
Philamlife to remit to Airco a fixed, certain, and
obligatory sum by way of reinsurance premiums. The
reinsurance treaty per se cannot give rise to a
contractual obligation for the payment of foreign
exchange. Philamlifes obligation to remit reinsurance
premiums becomes fixed and definite upon the
execution of the reinsurance cession. It is only
after a reinsurance cession is made that payment of
reinsurance premium may be exacted, as it is only
after Philamlife seeks to remit that reinsurance
premium that the obligation to pay the margin fee
arises.
2. NO
Ratio. Existing laws form part of the contract "as the
measure of the obligation to perform them by the
one party and the right acquired by the other. If the
obligation does not inhere and subsist in the
contract itself, propio vigore, but in the law
applicable to the contract.



Reasoning
- . When petitioner entered into the reinsurance
treaty of January 1, 1950 with Airco, it did so with
the understanding that the municipal laws of the
Philippines at the time said treaty was executed,
became an unwritten condition thereof. Such
municipal laws constitute part of the obligation of
contract.
-Rationale of Margin Law: to reduce the excessive
demand on and prevent further decline of our
international reserves; to provide the Central Bank
with an additional instrument for effectively coping
with the problem and achieving domestic and
international stability of our currency; to reduce the
excessive demand-for foreign exchange.
- implementation of Margin Law in accordance with
police power
3. NO
Reasoning
- First, there is no concrete evidence that such
imposition of the 25% margin fee is unreasonable,
Second, if really continuance of the existing
reinsurance treaty becomes unbearable, that
contract itself provides that petitioner may
potestatively write finis thereto on ninety days'
written notice. Petitioner is not forced to continue
its reinsurance treaty indefinitely with Airco.
Disposition For the reasons given, the petition for
review is hereby denied, and the ruling of the
Auditor General of October 24, 1961 denying refund
is hereby affirmed.
Costs against petitioner. So ordered.

FIELDMEN'S INSURANCE CO INC v. ASIAN
SURETY & INSURANCE CO INC
34 SCRA 36
MAKALINTAL; July 31, 1970

FACTS
!"#$%&"'( *+,- 45

- On various dates between April 11, 1960 and Jan.
9, 1961 the Asian Surety & Insurance Company,
Inc. and the Fieldmen's Insurance Company, Inc.
entered into 7 reinsurance agreements under which
the former, as the ceding company undertook to
cede to the latter, as the reinsuring company, a
specified portion of the amount of insurance
underwritten by ASIAN upon payment to
FIELDMEN'S of a proportionate share of the gross
rate of the premium applicable with respect to each
cession after deducting a commission. Said
agreements were to take effect from certain specific
dates and were to be in force until cancelled by
either party upon previous notice of at least 3
months by registered mail to the other party, the
cancellation to take effect as of Dec. 31 of the year
in which the notice was given.
- On Sep. 19, 1961 FIELDMEN'S, by means of
registered mail, served notice to ASIAN of the
former's desire to be relieved from all participation in
its various agreements with the latter effective Dec.
31, 1961. This communication, although admittedly
received by ASIAN on Sep. 25, 1961, did not elicit
any reply from ASIAN.
- On Dec. 7, 1961 FIELDMEN'S sent another letter to
ASIAN expressing regrets at alleged violations
committed by the latter with respect to the various
agreements between them and reiterated its position
that it would consider itself "no longer at risk for any
reinsurance and/or cession" given by ASIAN which
might be in force on Dec. 31, 1961. Not having
received any formal reply from ASIAN, FIELDMEN'S
sent a new a letter on Feb. 17, 1962 reminding
ASIAN of the cancellation of all the reinsurance
treaties and cessions as of Dec. 31, 1961 and
requested ASIAN to submit its final accounting of all
cessions made to the former for the preceding
months when the reinsurance agreements were in
force.
- Meanwhile one of the risks reinsured with
FIELDMEN'S issued in favor of the GSIS became a
liability when the insured property was burned on
February 16, 1962. Since the policy was issued on
July 1, 1961, it was supposed to expire on July 1,
1962. 2 The next day, Feb. 17, ASIAN immediately
notified FIELDMEN'S of said fire loss.
- FIELDMEN'S, relying on the sufficiency of its notice
of termination dated September 19, 1961 and
obviously bent on avoiding its liability under the
reinsurance agreements with ASIAN, filed a petition
for declaratory relief with the CFI of Manila to seek a
declaration that all the reinsurance contracts entered
into between them had terminated as of December
31, 1961 and to obtain an order directing ASIAN to
render final accounting of the transactions between
them with respect to said reinsurance treaties as of
the cut-off date.
- In its answer below ASIAN denied having received
FIELDMEN'S letter dated Sep 19, 1961, and argued
that even assuming it did, FIELDMEN'S could not
have terminated the reinsurance treaties as of Dec
31, 1961 because the letter was merely an
expression of FIELDMEN'S desire to cancel the
treaties and not a formal notice of cancellation as
contemplated in their reinsurance agreements. By
way of special defense Asian contended that even if
the Sep. 19 letter were considered sufficient notice
of cancellation thereby rendering the reinsurance
agreements terminated as of December 31, 1961
the liability of FIELDMEN'S with respect to policies or
cessions issued under two of the said agreements
prior to their cancellation continued to have full force
and effect until the stated expiry dates of such
policies or cessions.
- On Dec. 4, 1962, the trial court declared 6 of the 7
reinsurance agreements in question cancelled as of
Dec 31, 1961. At the same time, it upheld ASIAN'S
position that all cessions of reinsurance made by it to
FIELDMEN'S prior to the cancellation of the
reinsurance treaties continued in full force and effect
until expiry dates and ordered FIELDMEN'S to make
an accounting of its business transactions with
ASIAN within 30 days.
- On appeal to the CA, the decision of the trial court
was substantially affirmed, with the slight
modification that the order for accounting was
eliminated, without prejudice to the filing of a proper
action between the parties for that purpose.

ISSUE
WON the cancellation as of Dec. 31, 1961 of the
reinsurance treaties had the effect of terminating
also the liability of FIELDMEN'S as reinsurer with
respect to policies or cessions issued prior to the
termination of the principal reinsurance contracts or
treaties

HELD
NO to the 2 reinsurance contracts
- Of the 6 reinsurance contracts, 2 contain
provisions, which clearly and expressly recognize the
continuing effectivity of policies ceded under them
for reinsurance notwithstanding the cancellation of
the contracts themselves. The said treaties provide
"that in the event of termination of this Agreement .
. ., the liability of the Fieldmen's under current
cessions shall continue in full force and effect until
their natural expiry . . .;" and the 4th paragraph of
Article VI of the Personal Accident Reinsurance
Treaty states:
"4. On the termination of this Agreement from
any cause whatever, the liability of the
REINSURER (Fieldmen's) under any current
cession including any amounts due to be ceded
under the terms of this Agreement and which are
not cancelled in the ordinary course of business
shall continue in full force until their expiry unless
the COMPANY (Asian) shall, prior to the thirty-first
December next following such notice, elect to
withdraw the existing cessions . . ."
- Thus, insofar as the two reinsurance agreements
are concerned, there is clearly no merit in
FIELDMEN'S claim that their cancellation carried with
it ipso facto the termination of all reinsurance
cessions thereunder. Such cessions continued to be
in force until their respective dates of expiration.
Since it was under one of said agreements that the
reinsurance cession corresponding to the GSIS policy
had been made, FIELDMEN'S cannot avoid liability
which arose by reason of the burning of the insured
property.
- With respect to the other 4 agreements, it would
seem that the petition for declaratory relief is moot,
and that no useful purpose would be served by
defining the respective rights and obligations of the
parties thereunder. The said agreements have been
cancelled, and it does not appear that any claim by
or liability in favor of the insured has actually arisen
under any of the reinsurance cessions made prior to
such cancellation. Future conflicts of the same nature
as those which have motivated the present action
can of course be obviated by using more precise and
definite terminology in the reinsurance agreements
which the parties may enter into henceforth.

EQUITABLE INSURANCE v. RURAL INSURANCE
4 SCRA 343
BARRERA; January 31, 1962

FACTS
- Plaintiff Equitable Insurance file a complaint with
the CFI of Manila against defendant Rural Insurance
alleging, as first cause of action, that they entered
into a reciprocal facultative reinsurance agreement,
wherein they agreed to cede to each other. Pursuant
to said agreement, plaintiff reinsured for P2k with
!"#$%&"'( *+,- 46

defendant the stock covered by fire insurance Policy
No. 5880 issued by plaintiff which was later burned;
the share of the loss of defendant as per insurance
agreement was computed at P2,024 for which
plaintiff sent to defendant a statement of account for
payment by the latter. Despite repeated demands
by plaintiff, defendant refused to pay.
- On the second cause of action, plaintiff reinsured
for P2k with defendant stock covered by fire
insurance Policy No. 6062 which also burned. Again,
defendant refused to pay its share of the loss of
P1,334 hence said complaint.
- Defendant filed a motion to dismiss on the ground
that it states no cause of action, as pursuant to Art
VIII of the Reinsurance Agreement between the
parties, before a court action can be brought, the
parties agreed to submit all disputes to a board of
arbitrators. The Court denied the motion and
required defendant to answer.
- Defendant filed its answer, alleging that the nature
of the agreement is self-liquidating between the
parties, the reinsurer becoming the reinsured and
vice versa; and that said agreement has not yet
been abrogated so the liability of either to the other
is not yet known. Defendant prayed that the
complaint be dismissed and plaintiff filed a motion
for judgment on the pleadings which the court
denied.
- Instead of going into a formal hearing, the parties
submitted their case for decision stipulating the ff
facts: defendant admits the allegations of the
complaint and that plaintiff admits that the issues of
the complaint were not submitted to a Board of
Arbitrators as provided in par VIII of the complaint,
but instead referred it to the Insurance
Commissioner. The CFI rendered judgment in favor
of plaintiff. Hence this appeal.

ISSUES
1. WON Equitable had no cause of action as the
matter was not referred to the decision of arbitrators
2. WON in a facultative obligation the right to choose
an alternative remedy lies only with the debtor (here
the defendant) under Art 1206

HELD
1. NO
- The requirement of submission for decision to 2
arbitrators or an umpire the matter of losses by fire
or the liability of the parties thereto under Art VIII of
the agreement arises only if the same is disputed by
one of the parties. In the instant case, there is no
dispute between the parties; in the stipulation of
facts defendant admitted that plaintiff had paid its
liability and defendant likewise admitted that it
ignored plaintiffs demands for reimbursement for
defendants failure to pay its share as reinsurer. As
held in Maligad v United Assurance Co., if in the
course of the settlement of a loss, the action of the
company or its agents amounts to refusal to pay, the
company will be deemed to have waived the
condition precedent with reference to arbitration and
a suit upon the policy will lie.
2. NO
- There is no connection between Art 1206 NCC and
the agreement of this action. The term facultative
is used in reinsurance contracts, and it is so used in
this particular case, merely to define the right of the
reinsurer to accept or not to accept participation in
the risk insured. But once the share is accepted, as it
was in the case at bar, the obligation is absolute and
the liability assumed thereunder can be discharged
by only one waythe payment of the share of the
losses.
Disposition judgment appealed from the TC is
affirmed

ARTEX DEVELOPMENT CO INC v. WELLINGTON
INSURANCE CO INC
51 SCRA 352
TEEHANKEE; June 27, 1973

FACTS
- Wellington Insurance Co. Inc. insured for
P24,346,509.00 the buildings, stocks and machinery
of plaintiff Artex Development Co. Inc. against loss
or damage by fire or lighting upon payment of the
plaintiff of the corresponding premiums; that said
properties were insured for an additional sum of
P883,034.00; that defendant insured plaintiff against
business interruption (use and occupancy) for
P5,200,000.00; Wellington entered into a contract
of reinsurance with Alexander and Alexander, Inc. of
New York. USA.
- The buildings, stocks and machineries of plaintiffs
spinning department were burned.
- Notice of the loss and damage was given the
defendant; that as per report of the adjusters, the
total property loss of the plaintiff was the sum of
P10,106,554.40 and the total business interruption
loss was P3,000,000.00;
- That defendant has paid to the plaintiff the sum of
P6,481,870.07 of the property loss suffered by
plaintiff and P1,864,134.08 on its business
interruption loss, leaving a balance of P3,624,683.43
and P1,748,460.00, respectively.
- The counsel for Artex filed a Manifestation saying
that in view of the Deeds of Discharge and Collateral
Agreement, the only remaining liability subject of
litigation shall be the proportion of the loss reinsured
with or through Alexander and Alexander, Inc. of
New York, USA, namely, P397,813.00.
- The document recited further that Artex
acknowledges receipt of the sum of P3.6M paid by
the insurer in full and final settlement of all or any
claims of Artex against its insurer. It discharges its
insurer from all actions, proceedings, claims,
demands, costs and expenses in respect thereof.
- With regard the balance unpaid, Wellington
contends that Artex should have been directed
against the reinsurers to cover the liability and not
against Wellington.

ISSUE
WON the insured (Artex) has a cause of action
against the reinsurer

HELD
NO
- Unless there is a specific grant in, or assignment
of, the reinsurance contract in favor of the insured or
a manifest intention of the contracting parties to the
reinsurance contract to grant such benefit or favor to
the insured, the insured, not being privy to the
reinsurance contract, has no cause of action against
the reinsurer. It is expressly provided in Section 91
the Insurance Act 1 that "(T)he original insured has
no interest in a contract of insurance."

PERLA COMPANIA DE SEGUROS v. CA(LIM)
208 SCRA 487
NOCON; May 7, 1992

NATURE
Petition for certiorari by Perla Compania de Seguros
and FOC Credit Corporation seeking to annul and set
aside CA decision revering the RTC decision for
replevin and damages.

FACTS
- Private respondents spouses Herminio and Evelyn
Lim executed a promissory note in favor of
Supercars, Inc. in the sum of P77,940.00, payable in
monthly installments according to the schedule of
payment indicated in said note, and secured by a
chattel mortgage over a brand new red Ford Laser,
!"#$%&"'( *+,- 57

which is registered under the name of private
respondent Herminio Lim and insured with the
petitioner Perla Compania de Seguros, Inc. (Perla for
brevity) for comprehensive coverage.
- On the same date, Supercars, Inc., with notice to
private respondents spouses, assigned to petitioner
FCP Credit Corporation (FCP for brevity) its rights,
title and interest on said promissory note and chattel
mortgage as shown by the Deed of Assignment.
- At around 2:30pm Nov9, 1982, said vehicle was
carnapped while parked at the back of Broadway
Centrum. Evelyn Lim, who was driving said car
before it was carnapped, immediately called up the
Anti-Carnapping Unit of the Philippine Constabulary
to report said incident and thereafter, went to the
nearest police substation to make a police report
regarding said incident.
- On Nov10, 1982, Evelyn Lim reported said incident
to the LTO in compliance with the insurance
requirement. She also filed a complaint with the
Headquarters. Constabulary Highway Patrol Group.
- On Nov11, 1982, private respondent filed a claim
for loss with the petitioner Perla but said claim was
denied on Nov18, 1982 on the ground that Evelyn
Lim, who was using the vehicle before it was
carnapped, was in possession of an expired driver's
license at the time of the loss of said vehicle which is
in violation of the authorized driver clause of the
insurance policy, which states, to wit:
"AUTHORIZED DRIVER:
Any of the following: (a) The Insured (b) Any
person driving on the Insured's order, or with his
permission. Provided that the person driving is
permitted, in accordance with the licensing or
other laws or regulations, to drive the Scheduled
Vehicle, or has been permitted and is not
disqualified by order of a Court of Law or by
reason of any enactment or regulation in that
behalf."
- On Nov17, 1982, private respondents requested
from petitioner FCP for a suspension of payment on
the monthly amortization agreed upon due to the
loss of the vehicle and, since the carnapped vehicle
was insured with petitioner Perla, said insurance
company should be made to pay the remaining
balance of the promissory note and the chattel
mortgage contract.
- Perla, however, denied private respondents' claim.
Consequently, petitioner FCP demanded that private
respondents pay the whole balance of the
promissory note or to return the vehicle but the
latter refused.
- On July25, 1983, petitioner FCP filed a complaint
against private respondents, who in turn filed an
amended third party complaint against petitioner
Perla on Dec8, 1983. After trial on the merits, TC
ordered sps Lim to pay jointly and severally, plaintiff
the sum of P55,055.93 plus interest thereon at the
rate of 24% per annum from July 2, 1983 until fully
paid; as well as the cost of suit. It also ordered the
dismissal of the Third party complaint against Third-
Party Defendant.
- Upon appeal, CA reversed said decision
- After petitioners' separate MFRs were denied by
CA, petitioners filed these separate petitions for
review on certiorari.
ISSUE
1. WON there was grave abuse of discretion on the
part of the appellate court in holding that private
respondents did not violate the insurance contract
because the authorized driver clause is not
applicable to the "Theft" clause of said Contract
2. WON the loss of the collateral exempted the
debtor from his admitted obligations under the
promissory note particularly the payment of interest,
litigation expenses and attorney's fees

HELD
1. NO
- The comprehensive insurance policy issued by
petitioner Perla undertook to indemnify the private
respondents against loss or damages to the car (a)
by accidental collision or overturning, or collision or
overturning consequent upon mechanical breakdown
or consequent upon wear and tear; (b) by fire,
external explosion, self-ignition or lightning or
burglary, housebreaking or theft; and (c) by
malicious act.
- Where a car is unlawfully and wrongfully taken
without the owner's consent or knowledge, such
taking constitutes theft, and, therefore, it is the
"THEFT" clause, and not the "AUTHORIZED DRIVER"
clause, that should apply.The risk against accident is
distinct from the risk against theft. The 'authorized
driver clause' in a typical insurance policy as in
contemplation or anticipation of accident in the legal
sense in which it should be understood, and not in
contemplation or anticipation of an event such as
theft. The distinction often seized upon by insurance
companies in resisting claims from their assureds
between death occurring as a result of accident and
death occurring as a result of intent may apply to the
case at bar.
- If the insured vehicle had figured in an accident at
the time she drove it with an expired license, then,
appellee Perla Compania could properly resist
appellants' claim for indemnification for the loss or
destruction of the vehicle resulting from the
accident. But in the present case, the loss of the
insured vehicle did not result from an accident where
intent was involved; the loss in the present case was
caused by theft, the commission of which was
attended by intent."
- There is no causal connection between the
possession of a valid driver's license and the loss of a
vehicle. To rule otherwise would render car insurance
practically a sham since an insurance company can
easily escape liability by citing restrictions which are
not applicable or germane to the claim, thereby
reducing indemnity to a shadow.
2. The court agrees with FCP that Lim spouses are
not relieved of their obligation to pay the former the
installments due on the promissory note on account
of the loss of the automobile. The chattel mortgage
constituted over the automobile is merely an
accessory contract to the promissory note. Being the
principal contract, the promissory note is unaffected
by whatever befalls the subject matter of the
accessory contract.
- The unpaid balance on the promissory note should
be paid, and not just the installments due and
payable before the automobile was carnapped, as
erronously held by the CA
- However, this does not mean that private
respondents are bound to pay the interest, litigation
expenses and attorney's fees stipulated in the
promissory note. Because of the peculiar relationship
between the three contracts in this case, i. e., the
promissory note, the chattel mortgage contract and
the insurance policy, the Court is compelled to
construe all three contracts as intimately interrelated
to each other, despite the fact that at first glance
there is no relationship whatsoever between the
parties thereto.
- Under the promissory note, Lim spouses are
obliged to pay Supercars, Inc. the amount stated
therein in accordance with the schedule provided for.
To secure said promissory note, private respondents
constituted a chattel mortgage in favor of Supercars,
Inc. over the automobile the former purchased from
the latter. The chattel mortgage, in turn, required
private respondents to insure the automobile and to
make the proceeds thereof payable to Supercars,
Inc. The promissory note and chattel mortgage were
assigned by Supercars, Inc. to petitioner FCP, with
!"#$%&"'( *+,- 5.

the knowledge of private respondents. Private
respondents were able to secure an insurance policy
from petitioner Perla, and the same was made
specifically payable to petitioner FCP.
- From the abovementioned provision that upon the
loss of the insured vehicle, the insurance company
Perla undertakes to pay directly to the mortgagor or
to their assignee, FCP, the outstanding balance of
the mortgage at the time of said loss under the
mortgage contract. If the claim on the insurance
policy had been approved by petitioner Perla, it
would have paid the proceeds thereof directly to
petitioner FCP, and this would have had the effect of
extinguishing private respondents' obligation to
petitioner FCP. Therefore, private respondents were
justified in asking petitioner FCP to demand the
unpaid installments from petitioner Perla.
- Because petitioner Perla had unreasonably denied
their valid claim, private respondents should not be
made to pay the interest, liquidated damages and
attorney's fees as stipulated in the promissory note.
As mentioned above, the contract of indemnity was
procured to insure the return of the money loaned
from petitioner FCP, and the unjustified refusal of
petitioner Perla to recognize the valid claim of the
private respondents should not in any way prejudice
the latter.
- Private respondents can not be said to have unduly
enriched themselves at the expense of FCP since
they will be required to pay the latter the unpaid
balance of its obligation under the promissory note.
- In view of the foregoing discussion, We hold that
the Court of Appeals did not err in requiring
petitioner Perla to indemnify private respondents for
the loss of their insured vehicle. However, the latter
should be ordered to pay petitioner FCP the amount
of P55,055.93, representing the unpaid installments
from December 30, 1982 up to July 1, 1983, as
shown in the statement of account prepared by
petitioner FCP, 18 plus legal interest from July 2,
1983 until fully paid.
- As to the award of moral damages, exemplary
damages and attorney's fees, private respondents
are legally entitled to the same since Perla had acted
in bad faith by unreasonably refusing to honor the
insurance claim of the private respondents. Besides,
awards for moral and exemplary damages, as well as
attorney's fees are left to the sound discretion of the
Court. Such discretion, if well exercised, will not be
disturbed on appeal.
Disposition the assailed decision of the CA is
hereby MODIFIED to require private respondents to
pay petitioner FCP the amount of P55,055.93, with
legal interest from July 2, 1983 until fully paid. The
decision appealed from is hereby affirmed as to all
other respects. No pronouncement as to costs.

SHAFER v. JUDGE
167 SCRA 386
PADILLA; November 14, 1988

NATURE
Petition for review on certiorari

FACTS
- Sherman Shafer obtained a private car policy over
his Ford Laser from Makati Insurance Company, Inc.,
for third party liability. During the effectivity of the
policy, an information for reckless imprudence
resulting in damage to property and serious physical
injuries was filed against shafer. The information
said that on or about the 17th day of May 1985, in
the City of Olongapo. Shafer hit and bumped a
Volkswagen car owned and driven by Felino llano y
Legaspi, thereby causing damage in the total amount
of P12,345.00 and as a result thereof one Jovencio
Poblete, Sr. who was on board of the said
Volkswagen car sustained physical injuries which
injuries causing deformity on the face. The owner of
the damaged Volkswagen car filed a separate civil
action against petitioner for damages, while Jovencio
Poblete, Sr., who was a passenger in the Volkswagen
car, did not reserve his right to file a separate civil
action for damages. Instead, in the course of the
trial in the criminal case, Poblete, Sr. testified on his
claim for damages for the serious physical injuries
which he claimed to have sustained as a result of the
accident.
- The court issued an order dismissing the third party
complaint on the ground that it was premature,
based on the premise that unless the accused
(herein petitioner) is found guilty and sentenced to
pay the offended party (Poblete Sr.) indemnity or
damages, the third party complaint is without cause
of action. The court further stated that the better
procedure is for the accused (petitioner) to wait for
the outcome of the criminal aspect of the case to
determine whether or not the accused, also the third
party plaintiff, has a cause of action against the third
party defendant for the enforcement of its third party
liability (TPL) under the insurance contract. 6
Petitioner moved for reconsideration of said order,
but the motion was denied; hence, this petition.

ISSUE
WON the court a quo erred in dismissing petitioner's
third party complaint on the ground that petitioner
had no cause of action yet against the insurance
company

HELD
YES
- There is no need on the part of the insured to wait
for the decision of the trial court finding him guilty of
reckless imprudence. The occurrence of the injury to
the third party immediately gave rise to the liability
of the insurer under its policy. Respondent
insurance company's contention that the third party
complaint involves extraneous matter which will only
clutter, complicate and delay the criminal case is
without merit. The civil aspect of the offense
charged, i.e., serious physical injuries allegedly
suffered by Jovencio Poblete, Sr., was impliedly
instituted with the criminal case. Petitioner may thus
raise all defenses available to him insofar as the
criminal and civil aspects of the case are concerned.
The claim of petitioner for payment of indemnity to
the injured third party, under the insurance policy,
for the alleged bodily injuries caused to said third
party, arose from the offense charged in the criminal
case, from which the injured (Jovencio Poblete, Sr.)
has sought to recover civil damages. Hence, such
claim of petitioner against the insurance company
cannot be regarded as not related to the criminal
action.
- A third party complaint is a device allowed by the
rules of procedure by which the defendant can bring
into the original suit a party against whom he will
have a claim for indemnity or remuneration as a
result of a liability established against him in the
original suit.
13
Third party complaints are allowed to
minimize the number of lawsuits and avoid the
necessity of bringing two (2) or more actions
involving the same subject matter. They are
predicated on the need for expediency and the
avoidance of unnecessary lawsuits. If it appears
probable that a second action will result if the
plaintiff prevails, and that this result can be avoided
by allowing the third party complaint to remain, then
the motion to dismiss the third party complaint
should be denied.
- Compulsory Motor Vehicle Liability Insurance (third
party liability, or TPL) is primarily intended to
provide compensation for the death or bodily injuries
suffered by innocent third parties or passengers as a
result of a negligent operation and use of motor
!"#$%&"'( *+,- 5/

vehicles. The victims and/or their dependents are
assured of immediate financial assistance, regardless
of the financial capacity of motor vehicle owners.
- The liability of the insurance company under the
Compulsory Motor Vehicle Liability Insurance is for
loss or damage. Where an insurance policy insures
directly against liability, the insurer's liability accrues
immediately upon the occurrence of the injury or
event upon which the liability depends, and does not
depend on the recovery of judgment by the injured
party against the insured.
- The injured for whom the contract of insurance is
intended can sue directly the insurer. The general
purpose of statutes enabling an injured person to
proceed directly against the insurer is to protect
injured persons against the insolvency of the insured
who causes such injury, and to give such injured
person a certain beneficial interest in the proceeds of
the policy, and statutes are to be liberally construed
so that their intended purpose may be accomplished.
It has even been held that such a provision creates a
contractual relation which inures to the benefit of
any and every person who may be negligently
injured by the named insured as if such injured
person were specifically named in the policy.
- In the event that the injured fails or refuses to
include the insurer as party defendant in his claim
for indemnity against the insured, the latter is not
prevented by law to avail of the procedural rules
intended to avoid multiplicity of suits. Not even a "no
action" clause under the policy-which requires that a
final judgment be first obtained against the insured
and that only thereafter can the person insured
recover on the policy can prevail over the Rules of
Court provisions aimed at avoiding multiplicity of
suits.
Disposition instant petition is GRANTED. The
questioned order dated 24 April 1987 is SET ASIDE
and a new one entered admitting petitioner's third
party complaint against the private respondent
Makati Insurance Company, Inc.

VDA DE MAGLANA v. CONSOLACION
212 SCRA 268
ROMERO; August 6, 1992

NATURE
Petition for certiorari

FACTS
- Lope Maglana was an employee of the Bureau of
Customs whose work station was at Lasa, here in
Davao City. One day, when he was on his way to his
work, he met an accident that resulted in his death.
He died on the spot.
- The PUJ jeep that bumped the deceased was driven
by Pepito Into, operated and owned by defendant
Destrajo. From the investigation conducted by the
traffic investigator, the PUJ jeep was overtaking
another passenger jeep that was going towards the
city poblacion. While overtaking, the PUJ jeep of
defendant Destrajo running abreast with the
overtaken jeep, bumped the motorcycle driven by
the deceased. The point of impact was on the lane of
the motorcycle and the deceased was thrown from
the road and met his untimely death.
- Heirs of Lope Maglana, Sr. filed an action for
damages and attorney's fees against operator
Patricio Destrajo and the Afisco Insurance
Corporation (AFISCO). An information for homicide
thru reckless imprudence was also filed against
Pepito Into.
- During the pendency of the civil case, Into was
sentenced to suffer an indeterminate penalty, with
all the accessory penalties provided by law, and to
indemnify the heirs of Lope Maglana, Sr. in the
amount of twelve thousand pesos with subsidiary
imprisonment in case of insolvency, plus five
thousand pesos in the concept of moral and
exemplary damages with costs. No appeal was
interposed by accused who later applied for
probation.
- The lower court rendered a decision finding that
Destrajo had not exercised sufficient diligence as the
operator of the jeepney ordering him to pay plaintiffs
the sum for loss of income; funeral and burial
expenses of the deceased; moral damages, and
attorney's fees and costs of suit. The defendant
insurance company is ordered to reimburse
defendant Destrajo whatever amounts the latter
shall have paid only up to the extent of its insurance
coverage.
- Petitioners filed a motion for the reconsideration of
the second paragraph of the decision contending that
AFISCO should not merely be held secondarily liable
because the Insurance Code provides that the
insurer's liability is "direct and primary and/or jointly
and severally with the operator of the vehicle,
although only up to the extent of the insurance
coverage." Hence, they argued that the P20,000.00
coverage of the insurance policy issued by AFISCO,
should have been awarded in their favor.
- AFISCO argued that since the Insurance Code
does not expressly provide for a solidary obligation,
the presumption is that the obligation is joint.
- The lower court denied the motion for
reconsideration ruling that since the insurance
contract "is in the nature of suretyship, then the
liability of the insurer is secondary only up to the
extent of the insurance coverage."
- Petitioners filed a second motion for
reconsideration reiterating that the liability of the
insurer is direct, primary and solidary with the
jeepney operator because the petitioners became
direct beneficiaries under the provision of the policy
which, in effect, is a stipulation pour autrui. This
motion was likewise denied for lack of merit.

ISSUE
WON AFISCO can be held directly liable

HELD
YES
- As this Court ruled in Shafer vs. Judge, RTC of
Olongapo City, Br. 75, "[w]here an insurance policy
insures directly against liability, the insurer's liability
accrues immediately upon the occurrence of the
injury or even upon which the liability depends, and
does not depend on the recovery of judgment by the
injured party against the insured."
- The underlying reason behind the third party
liability (TPL) of the Compulsory Motor Vehicle
Liability Insurance is "to protect injured persons
against the insolvency of the insured who causes
such injury, and to give such injured person a certain
beneficial interest in the proceeds of the policy . . ."
Since petitioners had received from AFISCO the sum
of P5,000.00 under the no-fault clause, AFISCO's
liability is now limited to P15,000.00.
- However, we cannot agree that AFISCO is likewise
solidarily liable with Destrajo. In Malayan Insurance
Co., Inc. v. Court of Appeals, this Court had the
opportunity to resolve the issue as to the nature of
the liability of the insurer and the insured vis-a-vis
the third party injured in an accident. We
categorically ruled thus: While it is true that where
the insurance contract provides for indemnity against
liability to third persons, such third persons can
directly sue the insurer, however, the direct liability
of the insurer under indemnity contracts against
third party liability does not mean that the insurer
can be held solidarily liable with the insured and/or
the other parties found at fault. The liability of the
insurer is based on contract; that of the insured is
!"#$%&"'( *+,- 50

based on tort. In the case at bar, petitioner as
insurer of Sio Choy, is liable to respondent Vallejos
(the injured third party), but it cannot, as incorrectly
held by the trial court, be made "solidarily" liable
with the two principal tortfeasors, namely
respondents Sio Choy and San Leon Rice Mill, Inc.
For if petitioner-insurer were solidarily liable with
said, two (2) respondents by reason of the indemnity
contract against third party liability under which an
insurer can be directly sued by a third party this will
result in a violation of the principles underlying
solidary obligation and insurance contracts.
- While in solidary obligations, the creditor may
enforce the entire obligation against one of the
solidary debtors, in an insurance contract, the
insurer undertakes for a consideration to indemnify
the insured against loss, damage or liability arising
from an unknown or contingent event.
- Similarly, petitioners herein cannot validly claim
that AFISCO, whose liability under the insurance
policy is also P20,000.00, can be held solidarily liable
with Destrajo for the total amount of P53,901.70 in
accordance with the decision of the lower court.
Since under both the law and the insurance policy,
AFISCO's liability is only up to P20,000.00, the
second paragraph of the dispositive portion of the
decision in question may have unwittingly sown
confusion among the petitioners and their counsel.
What should have been clearly stressed as to leave
no room for doubt was the liability of AFISCO under
the explicit terms of the insurance contract.
Disposition present petition is hereby GRANTED.
The award of P28,800.00 representing loss of income
is INCREASED to P192,000.00 and the death
indemnity of P12,000.00 to P50,000.00.






FAR EASTERN SURETY v. MISA
25 SCRA 663
REYES; October 26, 1968

NATURE
Appeal by petition for review from a CA judgment

FACTS
- Socorro Dancel Vda.de Misa and Araceli Pinto hired
a taxi cab operated by La Mallorca on September 3,
1957. The taxi they were riding in collided with a
gravel and sand truck resulting to injuries to both
Misa and Pinto.
- The two passengers instituted a suit for damages
against La Mallorca who, while denying
responsibility, instituted a third party complaint
against Far Eastern Surety to recoup from the latter
any award for damages that might be recovered by
the passengers.
- It would appear from the case that a sticker was
placed in all the taxis of La Mallorca stating that
passengers of the taxis were insured against
accidents. This was done to entice the public into
patronizing La Mallorca.
- The trial court awarded to Misa and Pinto actual,
moral and exemplary damages, and attorneys fees
payable by La Mallorca and sentenced Far Eastern to
pay La Mallorca P10,000. on its third party liability
insurance.
- On appeal, the CA, while holding that the collision
was due to the fault of the driver of the gravel and
sand truck, found the taxi company liable for
damages to the passengers on the strength of its
representation contained in the sticker above noted
that the passengers were insured against accidents.
In so ruling, the CA overruled the defense of the
insurance company to the effect that it was
responsible only if the insured, La Mallorca, was
involved in accidents caused by, or arising out of, the
use of the motor vehicle. A motion for
reconsideration was filed in and dismissed by the CA.

ISSUE
WON Far Eastern Surety is liable to the insured on its
insurance policy

HELD
NO
- The award for damages made to the passengers
was exclusively predicated on the representation
made by La Mallorca that its passengers were
insured against accidents and not because it was at
fault in causing the accident.
Reasoning
- In this case, the findings of the CA and the trial
court that the causative factor of the mishap was the
negligence of the gravel and truck driver would have
been sufficient to relieve the taxi company of any
liability arising from the accident. However, in view
of the sticker in all of its taxicabs, La Mallorca has
insured its passengers against accidents, whether it
was at fault or not. In other words, La Mallorca
accepted the responsibility for damages or injuries to
passengers even if it had no fault at all.
- In the case of the insurance company, the SC ruled
that it neither authorized nor consented to the
representations made by the taxi company to its
passengers. As such, the liability of the said
insurance company based on its insurance contract
is limited to the recovery by the insured of all sums,
cost and expenses which the insured shall become
legally liable. The insurance company therefore
cannot be held liable for the award.
- The taxi company is adjudged to be the sole party
responsible for the award.
Disposition The decision of the CA is modified by
eliminating the award against Far Eastern.

PEZA v. ALIKPALA
160 SCRA 31
NARVASA; April 15, 1988

NATURE
Motion praying that Judge Alikpala be declared guilty
of contempt of court for having decided the case on
the merits despite the pendency in this Court of the
certiorari action instituted by the plaintiffs

FACTS
- vehicular accident with 2 children running across
the path of a Chevrolet "Carry-All", belonging to a
partnership known as Diman & Company driven by
its driver, Perfecto Amar, as it was passing a
national highway at barrio Makiling Calamba,
Laguna. They were killed. It was insured with the
Empire Insurance Co., Inc. under a so-called
'comprehensive coverage" policy, loss by theft
excluded. The policy was in force at the time of the
accident.
- Placida Peza, the managing partner of Diman & Co.
filed a claim with Empire, for payment of
compensation to the family of the 2 children who
died as a result of the accident. Empire refused to
pay on the ground that the driver had no authority to
operate the vehicle, a fact which it expressly
excepted from liability under the policy. What Peza
did was to negotiate directly with the deceased
children father for an out-of-court settlement. The
father agreed to accept P6,200.00 in fun settlement
of the liability of the vehicles owner and driver, and
Peza paid him this sum.
- Peza thereafter sued Empire to recover this sum of
P6,200.00 as actual damages, as well as P20,000.00
as moral damages, P10,000.00 as exemplary
!"#$%&"'( *+,- 51

damages, and P10,000.00 as attorney's fees. She
amended her complaint shortly thereafter to include
Diman & Co. as alternative party plaintiff.
- Empire's basic defense to the suit was anchored
on the explicit requirement in the policy limiting the
operation of the insured vehicle to the "authorized
driver" therein defined, namely, (a) the insured, or
(b) any person driving on the insured order or with
his permission, provided that-
... that the person driving is permited in
accordance with the licensing or other laws or
regulations to drive the Motor vehicle or has been
so permitted and is not disqualified by order of the
Court of Law of by reason of any enactment or
regulation in that behalf from driving such Motor
Vehicle.-
- driver Perfecto Amar, only having a temporary
operator's permit (TVR) [already expired] his drivers
license having earlier been confiscated by an agent
of the Land Transportation Commission for an
alleged violation of Land Transportation and Traffic
Rules, was not permitted by law and was in truth
disqualified to operate any motor vehicle; Peza
attempted to neutralize that fact by(1) the issuance
of the TVR by the LTC officer to Amar; in proof of the
proposition that there was no reason for confiscation
of Amar's license (2) Amar's license had not expired,
but had been renewed.
- Judge Alikpala did not admit such evidence

ISSUES
1. WON Judge Alikapala committed grave abuse of
discretion in not admitting evidence
2. WON confiscation of license and expiration of TVR
of the driver would serve as bar for Peza in
recovering from Empire

HELD
1. NO
- Even positing error in the Judge's analysis of the
evidence attempted to be introduced and his
rejection thereof, it is clear that it was at most an
error of judgment, not such an error as may be
branded a grave abuse of discretion, i.e., such
capricious and whimsical exercise of judgment as is
equivalent to lack of jurisdiction, against which the
writ of certiorari will lie. In any event, the
established principle is "that ruling of the trial court
on procedural questions and on admissibility of
evidence during the course of the trial are
interlocutory in nature and may not be the subject of
separate appeal or review on certiorari, but are to be
assigned as errors and reviewed in the appeal
properly taken from the decision rendered by the
trial court on the merits of the case.
- In the meantime, Judge Alikpala rendered
judgment on the merits, since the case was then
already ripe for adjudication. The judgment ordered
dismissal of the case for failure on the part of the
plaintiff to prove their cause of action against
Empire. Notice of the judgment was served on the
parties in due course.
2. YES
- It would seem fairly obvious that whether the LTC
agent was correct or not in his opinion that driver
Amar had violated some traffic regulation warranting
confiscation of his license and issuance of a TVR in
lieu thereof, this would not alter the undisputed fact
that Amar's licence had indeed been confiscated and
a TVR issued to him, and the TVR had already
expired at the time that the vehicle being operated
by him killed two children by accident. Neither would
proof of the renewal of Amar's license change the
fact that it had really been earlier confiscated by the
LTC agent.
Disposition petition is DISMISSED for lack of merit








PERLA COMPANIA DE SEGUROS v. ANCHETA
164 SCRA 144
CORTES; August 8, 1988

NATURE
Petition for certiorari and prohibition with prelim
injunction to review orders of CFI Camarines Norte

FACTS
- Perla was the insurer of a Superlines bus which
figured in a collision with a III Scout (its a kind of
vehicle). Injured passengers of the latter (and
respondents in this case) filed a complaint for
damages against Superlines, the bus driver, and
Perla (as insurer of the bus). CFI Judge Ancheta
ordered that Perla should pay the respondents
immediately the P5000 under the no fault clause as
provided in Sec. 378.
Sec. 378: Any claim for death or injury to any
passenger or 3
rd
party pursuant to the provisions of
this chapter shall be paid without the necessity of
proving fault or negligence of any kind. Provided,
That for purposes of this section
(i) The indemnity in respect of any one person shall
not exceed P5,000;
(ii) The following proofs of loss, when submitted
under oath, shall be sufficient evidence to
substantiate the claim:
(a) Police report of accident, and
(b) Death certificate and evidence sufficient to
establish, the proper payee, or
(c) Medical report and evidence of medical or
hospital disbursement in respect of which refund is
claimed;
(iii) Claim may be made against one motor vehicle
only. In the case of an occupant of a vehicle, claim
shall lie against the insurer of the vehicle in which
the occupant is riding, mounting or dismounting
from. In any other case, claim shall lie against the
insurer of the directly offending vehicle. In all cases,
the right of the party paying the claim to recover
against the owner of the vehicle responsible for the
accident shall be maintained.
Perla denied its liability under the above provision
and said that the insurer of the vehicle that the
respondents were riding (Malayan Insurance in this
case) should be liable. Its 2 MFRs denied, Perla filed
this action

ISSUE
WON Perla is the insurer liable to indemnify under
Sec. 378

HELD
NO
Ratio The law is very clear the claim shall lie
against the insurer of the vehicle in which the
occupant xxx is riding, and no other. The claimant
is not free to choose from which insurer he will claim
the no fault indemnity, as the law, by using the
word shall, makes it mandatory that the claim be
made against the insurer of the vehicle in which the
occupant is riding, mounting or dismounting from.
Reasoning
- the rules on claims under the no fault indemnity
provision, where proof of fault or negligence is not
necessary for payment of any claim for death or
injury to a passenger or to a 3
rd
party, are
established:
1. A claim may be made against one motor vehicle
only.
!"#$%&"'( *+,- 52

2. If the victim is an occupant of a vehicle, the claim
shall lie against the insurer of the vehicle in which he
is riding, mounting or dismounting from.
3. In any other case (i.e. if the victim is not an
occupant of a vehicle), the claim shall lie against the
insurer of the directly offending vehicle.
4. In all cases, the right of the party paying the claim
to recover against the owner of the vehicle
responsible for the accident shall be maintained.
-That the vehicle ridden might not be the one that
caused the accident is of no moment since the law
itself provides that the party paying the claim under
Sec. 378 may recover against the owner of the
vehicle responsible for the accident. This is precisely
the essence of no fault indemnity insurance which
was introduced to and made part of our laws in order
to provide victims of vehicular accidents or their
heirs immediate compensation, although in a limited
amount, pending final determination of who is
responsible for the accident and liable for the
victims' injuries or death. In turn, the no fault
indemnity provision is part and parcel of the Code
provisions on compulsory motor vehicle liability
insurance and should be read together with the
requirement for compulsory passenger and/or 3
rd

party liability insurance (Sec. 377) which was
mandated in order to ensure ready compensation for
victims of vehicular accidents.
-Irrespective of whether or not fault or negligence
lies with the driver of the Superlines bus, as
respondents were not occupants of the bus, they
cannot claim the no fault indemnity provided in
Sec. 378 from Perla. The claim should be made
against the insurer of the vehicle they were riding.
Disposition Petition GRANTED. Orders of CFI
ordering Perla to pay respondents immediately
P5000 ANNULLED and SET ASIDE

WESTERN GUARANTY CORPORATION v. CA
(RODRIGUEZ & DE DIOS TRANS)
185 SCRA 652
FELICIANO; July 20, 1990

NATURE
Petition for review the decision of CA affirming in
toto the damages awarded to private respondent by
the trial court.

FACTS
- Respondent Priscilla E. Rodriguez was struck by a
De Dios passenger bus owned by respondent De Dios
Transportation Co., Inc. Priscilla was thrown to the
ground, hitting her forehead. She was treated at the
Protacio Emergency Hospital and later on
hospitalized at the San Juan De Dios Hospital. Her
face was permanently disfigured, causing her serious
anxiety and moral distress. Respondent bus
company was insured with petitioner Western
Guaranty Corporation ("Western") under its Master
Policy which provided, among other things, for
protection against third party liability, the relevant
section reading as follows:
Section 1. Liability to the Public ? Company will,
subject to the Limits of Liability, pay all sums
necessary to discharge liability of the insured in
respect of ?
(a) death of or bodily injury to or damage to
property of any passenger as defined herein.
(b) death of or bodily injury or damage to property
of any THIRD PARTY as defined herein in any
accident caused by or arising out of the use of the
Schedule Vehicle, provided that the liability shall
have first been determined. In no case, however,
shall the Company's total payment under both
Section I and Section 11 combined exceed the
Limits of Liability set forth herein. With respect to
death of or bodily injury to any third party or
passenger, the company's payment per victim in
any one accident shall not exceed the limits
indicated in the Schedule of indemnities provided
for in this policy excluding the cost of additional
medicines, and such other burial and funeral
expenses that might have been incurred.
- Respondent Priscilla Rodriguez filed a complaint for
damages before the Regional Trial Court of Makati
against De Dios Transportation Co. and Walter A.
Saga Respondent De Dios Transportation Co., in
turn, filed a third-party complaint against its
insurance carrier, petitioner Western. On 6 August
1985, the trial court rendered a decision in favor of
respondent Priscilla E. Rodriguez, awarding moral
damages, lossof earning and attorney's fees among
others.
- On appeal, the Court of Appeals affirmed in toto
the decision of the trial court.
- Petitioner contends that it cannot be held liable for
loss of earnings, moral damages and attorney's fees
because these items are not among those included in
the Schedule of Indemnities set forth in the
insurance policy.

ISSUE
WON petitioner can be held liable for loss of
earnings, moral damages and attorney's fees

HELD
YES
- The Schedule of Indemnities does not purport to
restrict the kinds of damages that may be awarded
against Western once liability has arisen. It was
merely meant to set limits to the amounts the
movant would be liable for in cases of claims for
death, bodily injuries of, professional services and
hospital charges, for services rendered to traffic
accident victims,' and not necessarily exclude claims
against the insurance policy for other kinds of
damages, such as those in question.
- It will be seen that the above quoted Schedule of
Indemnities establishes monetary limits which
Western may invoke in case of occurrence of the
particular kinds of physical injury there listed.
- It must be stressed, however, that the Schedule of
Indemnities does not purport to limit, or to
enumerate exhaustively, the species of bodily injury
occurrence of which generate liability for petitioner
Western. A car accident may, for instance, result in
injury to internal organs of a passenger or third
party, without any accompanying amputation or loss
of an external member (e.g., a foot or an arm or an
eye). But such internal injuries are surely covered by
Section I of the Master Policy, since they certainly
constitute bodily injuries.
- The Schedule of Indemnities does not purport to
restrict the kinds of damages that may be awarded
against Western once liability has arisen. Section 1,
quoted above, does refer to certain "Limits of
Liability" which in the case of the third party liability
section of the Master Policy, is apparently
P50,000.00 per person per accident. Within this
over-all quantitative limit, all kinds of damages
allowable by law" actual or compensatory
damages"; "moral damages'; "nominal damages";
"temperate or moderate damages"; "liquidated
damages"; and "exemplary damages" ? may be
awarded by a competent court against the insurer
once liability is shown to have arisen, and the
essential requisites or conditions for grant of each
species of damages are present. It appears to us
self-evident that the Schedule of Indemnities was
not intended to be an enumeration, much less a
closed enumeration, of the specific kinds of damages
which may be awarded under the Master Policy
Western has issued.
- The reading urged by Western of the Schedule of
Indemnities comes too close to working fraud upon
both the insured and the third party beneficiary of
!"#$%&"'( *+,- 53

Section 1, quoted above. For Western's reading
would drastically and without warning limit the
otherwise unlimited and comprehensive scope of
liability assumed by the insurer Western under
Section 1: "all sums necessary to discharge liability
of the insured in respect of [bodily injury to a third
party]". This result- which is not essentially different
from taking away with the left hand what had been
given with the right hand we must avoid as obviously
repugnant to public policy. If what Western now
urges is what Western intended to achieve by its
Schedule of Indemnities, it was incumbent upon
Western to use language far more specific and
precise than that used in fact by Western, so that
the insured, and potential purchasers of its Master
Policy, and the Office of the Insurance
Commissioner, may be properly informed and act
accordingly.
- Moreover, an insurance contract is a contract of
adhesion. The rule is well entrenched in our
jurisprudence that the terms of such contract are to
be construed strictly against the party which
prepared the contract, which in this case happens to
be petitioner Western.
Disposition Petition denied.

SUMMIT GUARANTY & INSURANCE COMPANY v.
ARNALDO
158 SCRA 332
GANCAYCO; February 29, 1988

NATURE
PETITION to review the order of the Insurance
Commissioner.

FACTS
- On Nov. 26, 1976, a Ford Pick-up truck owned by
Marcos Olasco was bumped by a cargo truck owned
by Floralde.
FGU Insurance Corporation (FG U) by reason of
Motor Vehicle Insurance Policy No. IC-VF-07185 paid
Olaso the sum of P2,817.50 as its share in the repair
cost of the said Ford Pick-up. Having been
subrogated to the rights and causes of action of
Olaso in the said amount FGU formally demanded
payment of said amount from Floralde and
attempted to verify Floralde's insurance carrier but
failed to do so. In 1978 FGU was able to ascertain
the identity of Floralde's insurance carrier to be the
Summit Guaranty and Insurance Company, Inc.
(Summit) and thus requested the insurance
commissioner for a conference with Summit and
demanded from Summit through counsel on
February 28, 1978 the payment of the damages
sustained by the car of Olaso but to no avail.
- Hence on May 22, 1978 FGU filed a case in the
Insurance Commissioner's Office against Summit for
recovery of said amount.
- Summit filed a motion to dismiss on the ground of
prescription under Section 384 of PD No. 612.
Averring that the accident happened on November
26, 1976 while the complaint was filed on May 22,
1978 beyond the one-year period from the time of
the accident provided for by the said provision.

ISSUE
WON the action must be dismissed on the ground of
prescription under Section 384 of PD No. 612

HELD
NO
- The case do not fall within the meaning of proper
cases' as contemplated in Section 384 of the
Insurance Code.


Reasoning
- Section 384 of PD 612 (Insurance Code)
Any person having any claim upon the policy
issued pursuant to this chapter shall, without any
unnecessary delay, present to the insurance
company concerned a written notice of claim
setting forth the amount of his loss, and/or the
nature, extent and duration of the injuries
sustained as certified by a duly licensed physician.
Notice of claim must be filed within six months
from date of the accident, otherwise, the claim
shall be deemed waived. Action or suit for
recovery of damage due to loss or injury must be
brought, in proper cases, with the Commissioner
or the Courts within one year from date of
accident, otherwise, the claimant's right of action
shall prescribe.
- It is very clear that the one-year period is only
required In proper cases. Had the lawmakers
intended it to be the way Petitioner Company
assumes it to be, then the phrase 'in proper cases'
would not have been inserted.
- in Aisporna. vs. Court of Appeals:
'Legislative intent must be ascertained from a
consideration of the statute as a whole. The
particular words, clauses and phrases should not
be studied as detached and isolated expressions,
but the whole and every part of the statute must
be considered in fixing the meaning of any of its
parts and in order to produce a harmonious whole.
A statute must be so construed as to harmonize
and give effect to all its provisions whenever
possible.'
- Petitioner company is trying to use Section 384 of
the Insurance Code as a cloak to hide itself from its
liabilities. The facts of these cases evidently reflect
the deliberate efforts of petitioner company to
prevent the filing of a formal action against it.
Bearing in mind that if it succeeds in doing so until
one year lapses from the date of the accident it could
set up the defense of prescription, petitioner
company made private respondents believe that
their claims would be settled in order that the latter
will not find it necessary to immediately bring suit. In
violation of its duties to adopt and implement
reasonable standards for the prompt investigation of
claims and to effectuate prompt, fair and equitable
settlement of claims, and with manifest bad faith,
petitioner company devised means and ways of
stalling the settlement proceedings. In G.R. No. L-
50997, no steps were taken to process the claim and
no rejection of said claim was ever made even if
private respondent had already complied with all the
requirements.
- In G.R. No. L-48758-petitioner company even
provided legal assistance to one of the private
respondents in the criminal case filed against him
leading Private respondents to believe that it was
ready to pay. In the same case, petitioner company
admits that it took no final action or adjudication of
the claim. Worse still, in G.R. No. L-48679,
assurances of payment were constantly given and
petitioner company even said that a check was ready
for release. This Court has made the observation
that some insurance companies have been inventing
excuses to avoid their just obligations and it is only
the State that can give the protection which the
insuring public needs from possible abuses of the
insurers. In view of the foregoing,
- It is not denied that an extrajudicial demand for
payment was made by respondent FGU on petitioner
but petitioner failed to respond to the same.
Nevertheless the complaint was filed even before a
denial of the claim was made by petitioner. For all
legal purposes, the one-year prescriptive period
provided for in Section 384 of the Insurance Code
has not begun to run.The cause of action arises only
and starts to run upon the denial of the claim by the
insurance company.The court takes note of the
dilatory tactics employed by petitioner in this as in
!"#$%&"'( *+,- 54

the several cases aforecited to avoid payment of its
liabilities.

VILLACORTA v. THE INSURANCE COMMISSION
100 SCRA 467
TEEHANKEE; October 30, 1980

FACTS
- JEWEL VILLACORTA was the owner of a Colt
Lancer, Model 1976, insured with respondent
company for P35,000.00 - Own Damage; P30,000.00
- Theft; and P30,000.00 - Third Party Liability,
effective May 16, 1977 to May 16, 1978.
- On May 9, 1978, the vehicle was brought to the
Sunday Machine Works, Inc., for general check-up
and repairs. On May 11, 1978, while it was in the
custody of the Sunday Machine Works, the car was
allegedly taken by six (6) persons and driven out to
Montalban, Rizal. While travelling along Mabini St.,
Sitio Palyasan, Barrio Burgos, going North at
Montalban, Rizal, the car figured in an accident,
hitting and bumping a gravel and sand truck parked
at the right side of the road going south. As a
consequence, the gravel and sand truck veered to
the right side of the pavement going south and the
car veered to the right side of the pavement going
north. The driver, Benito Mabasa, and one of the
passengers died and the other four sustained
physical injuries. The car, as well, suffered extensive
damage. Complainant, thereafter, filed a claim for
total loss with the respondent company but claim
was denied. Hence, complainant was compelled to
institute the present action."
- The comprehensive motor car insurance policy for
P35,000.00 issued by respondent Empire Insurance
Company admittedly undertook to indemnify the
petitioner-insured against loss or damage to the car
(a) by accidental collision or overturning, or collision
or overturning consequent upon mechanical
breakdown or consequent upon wear and tear; (b)
by fire, external explosion, self-ignition or lightning
or burglary, housebreaking or theft; and (c) by
malicious act.
- Respondent insurance commission, however,
dismissed petitioner's complaint for recovery of the
total loss of the vehicle against private respondent,
sustaining respondent insurer's contention that the
accident did not fall within the provisions of the
policy either for the Own Damage or Theft coverage,
invoking the policy provision on "Authorized Driver"
clause, which clause limits the use of the insured
vehicle to two (2) persons only, namely: the insured
himself or any person on his (insured's) permission.
Apparently, the Insurance commission sees the
unauthorized taking of the vehicle for a joyride as a
violation of the 'Authorized Driver' clause of the
policy."
- Respondent commission likewise upheld private
respondent's assertion that the car was not stolen
and therefore not covered by the Theft clause, ruling
that "(T)he element of 'taking' in Article 308 of the
Revised Penal Code means that the act of depriving
another of the possession and dominion of a
movable thing is coupled . . . with the intention, at
the time of the 'taking', of withholding it with the
character of permanency

ISSUE
WON the Insurance commissions findings are in
accord with law

HELD
NO
- First, respondent commission's ruling that the
person who drove the vehicle in the person of Benito
Mabasa, who, according to its own finding, was one
of the residents of the Sunday Machine Works, Inc.
to whom the car had been entrusted for general
check-up and repairs was not an "authorized driver"
of petitioner-complainant is too restrictive and
contrary to the established principle that insurance
contracts, being contracts of adhesion where the
only participation of the other party is the signing of
his signature or his "adhesion" thereto, "obviously
call for greater strictness and vigilance on the part of
courts of justice with a view of protecting the weaker
party from abuse and imposition, and prevent their
becoming traps for the unwary."
- The main purpose of the "authorized driver" clause,
as may be seen from its text, supra, is that a person
other than the insured owner, who drives the car on
the insured's order, such as his regular driver, or
with his permission, such as a friend or member of
the family or the employees of a car service or repair
shop must be duly licensed drivers and have no
disqualification to drive a motor vehicle. A car owner
who entrusts his car to an established car service
and repair shop necessarily entrusts his car key to
the shop owner and employees who are presumed to
have the insured's permission to drive the car for
legitimate purposes of checking or road-testing the
car. The mere happenstance that the employee(s) of
the shop owner diverts the use of the car to his own
illicit or unauthorized purpose in violation of the trust
reposed in the shop by the insured car owner does
not mean that the "authorized driver" clause has
been violated such as to bar recovery, provided that
such employee is duly qualified to drive under a valid
driver's license.
- Secondly, and independently of the foregoing
(since when a car is unlawfully taken, it is the theft
clause, not the "authorized driver" clause, that
applies), where a car is admittedly as in this case
unlawfully and wrongfully taken by some people, be
they employees of the car shop or not to whom it
had been entrusted, and taken on a long trip to
Montalban without the owner's consent or
knowledge, such taking constitutes or partakes of
the nature of theft as defined in Article 308 of the
Revised Penal Code.
- The Court rejects respondent commission's
premise that there must be an intent on the part of
the taker of the car "permanently to deprive the
insured of his car" and that since the taking here was
for a "joy ride" and "merely temporary in nature," a
"temporary taking is held not a taking insured
against."
- The insurer must therefore indemnify the petitioner
owner for the total loss of the insured car in the sum
of P35,000.00 under the theft clause of the policy,
subject to the filing of such claim for reimbursement
or payment as it may have as subrogee against the
Sunday Machine Works, Inc.







CHAPTER VIII. MARINE INSURANCE

MAGSAYSAY INC v. AGAN
96 PHIL 504
REYES; January 31, 1955

FACTS
- The S S "San Antonio", vessel owned and operated
by plaintiff, left Manila on October 6, 1949, bound for
Basco, Batanes, vis Aparri, Cagayan, with general
cargo belonging to different shippers, among them
the defendant. The vessel reached Aparri, but while
still in the port, it ran aground at the mouth of the
Cagayan river, and, attempts to refloat it under its
own power having failed, plaintiff had it refloated by
the Luzon Stevedoring Co. at an agreed
!"#$%&"'( *+,- 55

compensation. Once afloat the vessel returned to
Manila to refuel and then proceeded to Basco, the
port of destination. There the cargoes were delivered
to their respective owners or consignees, who, with
the exception of defendant, made a deposit or signed
a bond to answer for their contribution to the
average.
- On the theory that the expenses incurred in
floating the vessel constitute general average to
which both ship and cargo should contribute, plaintiff
brought the present action in the CFI of Manila to
make defendant pay his contribution, which, as
determined by the average adjuster, amounts to
P841.40.
- Defendant denies liability to his amount, alleging,
among other things, that the stranding of the vessel
was due to the fault, negligence and lack of skill of
its master, that the expenses incurred in putting it
afloat did not constitute general average, and that
the liquidation of the average was not made in
accordance with law.
- The lower court found for plaintiff

ISSUE
WON the expenses incurred in floating a vessel so
stranded should be considered general average and
shared by the cargo owners

HELD
NO
Ratio The law on averages is contained in the Code
of Commerce. Under that law, averages are classified
into simple or particular and general or gross.
Generally speaking, simple or particular averages
include all expenses and damages caused to the
vessel or cargo which have not inured to the
common benefit (Art. 809), and are, therefore, to be
borne only by the owner of the property gave rise to
same (Art. 810); while general or gross averages
include "all the damages and expenses which are
deliberately caused in order to save the vessel, its
cargo, or both at the same time, from a real and
known risk" (Art. 811). Being for the common
benefit, gross averages are to be borne by the
owners of the articles saved (Art. 812).
Reasoning
- the stranding of plaintiff's vessel was due to the
sudden shifting of the sandbars at the mouth of the
river which the port pilot did not anticipate. The
standing may, therefore, be regarded as accidental.
- Tolentino, in his commentaries on the Code of
Commerce, gives the following requisites for
general average:
First, there must be a common danger. This means,
that both the ship and the cargo, after has been
loaded, are subject to the same danger, whether
during the voyage, or in the port of loading or
unloading; that the danger arises from the accidents
of the sea, dispositions of the authority, or faults of
men, provided that the circumstances producing the
peril should be ascertained and imminent or may
rationally be said to be certain and imminent. This
last requirement exclude measures undertaken
against a distant peril.
Second, that for the common safety part of the
vessel or of the cargo or both is sacrificed
deliberately.
Third, that from the expenses or damages caused
follows the successful saving of the vessel and cargo.
Fourth, that the expenses or damages should have
been incurred or inflicted after taking proper legal
steps and authority.
- With respect to the first requisite, the evidence
does not disclose that the expenses sought to be
recovered from defendant were incurred to save
vessel and cargo from a common danger...it is the
safety of the property, and not of the voyage, which
constitutes the true foundation of the general
average.
- As to the second requisite, we need only repeat
that the expenses in question were not incurred for
the common safety of vessel and cargo, since they,
or at least the cargo, were not in imminent peril.
- With respect to the third requisite, the salvage
operation, it is true, was a success. But as the
sacrifice was for the benefit of the vessel to enable it
to proceed to destination and not for the purpose of
saving the cargo, the cargo owners are not in law
bound to contribute to the expenses.
- The final requisite has not been proved, for it does
not appear that the expenses here in question were
incurred after following the procedure laid down in
article 813.
Disposition Wherefore, the decision appealed from
is reversed.

JARQUE v. SMITH, BELL & CO.
56 PHIL 758
OSTRAND; November11, 1932

NATURE
Appeal from judgment of the lower court

FACTS
- Plaintiffs motorboat, Pandan was insured on a
marine insurance policy with National Union Fire
Insurance Company (NUFIC) for P45K. According to
the provisions of a rider attached to the policy, the
insurance was against the absolute total loss of the
vessel only. On Oct. 31, 1928, the ship ran into
very heavy sea and it became necessary to jettison a
portion of the cargo. As a result of the jettison, the
NUFIC was assessed P2,610.86 as its contribution to
the general average.
- The insurance company, insisting that its obligation
did not extend beyond the insurance of the absolute
total loss of the vessel only, and to pay proportionate
salvage of the declared value, refused to contribute
to the settlement of the gen. ave. The present action
was thereupon instituted, and after trial the court
below rendered judgment in favor of the plaintiff and
ordered the defendant to pay the plaintiff P2,610.86
as its part of the indemnity for the gen. ave. brought
about by the jettison of cargo. The insurance
company then appealed to the SC.
- The insurance contract is printed in the English
common form of marine policies. One of the clauses
of the document originally read as follows:
Touching the Adventures and Perils which the said
NUFIC is content to bear, and to take upon them
in this Voyage; they are of the Seas, Men-of-War,
Fire, Pirates, Thieves, Jettison, Letters of Mart and
Countermart, Surprisals, and Takings at Sea.
Arrests, Restraints and Detainments, of all Kings,
Princes and People of what Nation, Condition or
Quality soever; Barratry of the Master and
Marines, and of all other Perils, Losses and
Misfortunes, that have or shall come to the Hurt,
Detriment, or Damage of the said Vessel or any
part thereof; and in case of any Loss or
Misfortunes, it shall be lawful for the Assured, his
or their Factors, Servants, or assigns, to sue,
labour and travel for, in and about the Defence.
Safeguard, and recovery of the said Vessel or any
part thereof, without Prejudice to this Insurance;
to the Charges whereof the said Company, will
contribute, according to the rate and quantity of
the sum herein assured...
- Attached to the policy over and above the said
clause is a rider containing typewritten provisions,
among which appears in capitalized type the
following clause:
!"#$%&"'( *+,- 56

AGAINST THE ABSOLUTE TOTAL LOSS OF THE
VESSEL ONLY, AND TO PAY PROPORTIONATE
SALVAGE CHARGES OF THE DECLARED VALUE.

ISSUES
1. WON the lower court erred in disregarding the
typewritten clause endorsed upon the policy,
expressly limiting insurer's liability thereunder of the
total loss of the wooden vessel Pandan and to
proportionate salvage charges
2. WON lower court erred in concluding that
defendant and appellant, NUFIC is liable to
contribute to the general average resulting from the
jettison of a part of said vessel's cargo

HELD
1. NO
Ratio In case repugnance exists between written
and printed portions of a policy, the written portion
prevails.
Reasoning
- Section 291 of the Code of Civil Procedure provides
that when an instrument consists partly of written
words and partly of a printed form and the two are
inconsistent, the former controls the latter.


2. NO
Ratio The liability for contribution in general average
is not based on the express terms of the policy, but
rests upon the theory that from the relation of the
parties and for their benefit, a quasi contract is
implied by law.
Reasoning
- In the absence of positive legislation to the
contrary, the liability of the defendant insurance
company on its policy would, perhaps, be limited to
absolute loss of the vessel only, and to pay
proportionate salvage of the declared value. But the
policy was executed in this jurisdiction and
warranted to trade within the waters of the
Philippine Archipelago only. Here, Art. 859 of the
Code of Commerce is still in force:
ART. 859. The underwriters of the vessel, of the
freight, and of the cargo shall be obliged to pay for
the indemnity of the gross average in so far as is
required of each one of these objects
respectively.
- The article is mandatory in its terms, and the
insurers (whether for the vessel or for the freight or
for the cargo) are bound to contribute to the
indemnity of the general average. The provision
simply places the insurer on the same footing as
other persons who have an interest in the vessel, or
the cargo therein, at the time of the occurrence of
the general average and who are compelled to
contribute (Art. 812, Code of Commerce).
- In the present case it is not disputed that the ship
was in grave peril and that the jettison of part of the
cargo was necessary. If the cargo was in peril to the
extent of call for general average, the ship must also
have been in great danger, possibly sufficient to
cause its absolute loss. The jettison was therefore as
much to the benefit of the underwriter as to the
owner of the cargo. The latter was compelled to
contribute to the indemnity; why should not the
insurer be required to do likewise? If no jettison had
taken place and if the ship by reason thereof had
foundered, the underwriter's loss would have been
many times as large as the contribution now
demanded.
Disposition Appealed judgment is affirmed

GO TIACO v. UNION INSURANCE
40 PHIL 40
STREET; September 1, 1919

FACTS
- Union Insurance Society of Canton, Ltd., issued a
marine insurance policy upon a cargo of rice
belonging to the Go Tiaoco Brothers, which was
transported in the early days of May, 1915, on the
steamship Hondagua from the port of Saigon to
Cebu.
- On discharging the rice from one of the
compartments in the after hold, upon arrival at
Cebu, it was discovered that 1473 sacks had been
damaged by sea water. The loss was P3,875.25.
- The trial court found that the inflow of the sea
water during the voyage was due to a defect in one
of the drain pipes of the ship and concluded that the
loss was not covered by the policy of insurance. The
trial court made the ff findings:
The drain pipe which served as a discharge
from the water closet passed down through
the compartment where the rice in question
was stowed and thence out to sea through the
wall of the compartment, which was a part of
the wall of the ship. The joint or elbow where
the pipe changed its direction was of cast
iron; and in course of time it had become
corroded and abraded until a longitudinal
opening had appeared in the pipe about one
inch in length. This hole had been in existence
before the voyage was begun, and an attempt
had been made to repair it by filling with
cement and bolting over it a strip of iron. The
effect of loading the boat was to submerge
the vent, or orifice, of the pipe until it was
about 18 inches or 2 feet below the level of
the sea. As a consequence the sea water rose
in the pipe. Navigation under these conditions
resulted in the washing out of the cement-
filling from the action of the sea water, thus
permitting the continued flow of the salt water
into the compartment of rice.
- The court found in effect that the opening above
described had resulted in course of time from
ordinary wear and tear and not from the straining of
the ship in rough weather on that voyage. The court
also found that the repairs that had been made on
the pipe were slovenly and defective and that, by
reason of the condition of this pipe, the ship was not
properly equipped to receive the rice at the time the
voyage was begun. For this reason the court held
that the ship was unseaworthy.
- The policy purports to insure the cargo from the
following among other risks: "Perils . . . of the seas,
men, of war, fire, enemies, pirates, rovers, thieves,
.jettisons, . . . barratry of the master and mariners,
and of all other perils, losses, and misfortunes that
have or shall come to the hurt, detriment, or
damage of the said goods and merchandise or any
part thereof."


ISSUE
WON Union Insurance is liable for the loss of the Go
Tiaco Brothers

HELD
NO
- the words "all other perils, losses, and misfortunes"
are to be interpreted as covering risks which are of
like kind (ejusdem generis) with the particular risks
which are enumerated in the preceding part of the
same clause of the contract. ''According to the
ordinary rules of construction, these words must be
interpreted with reference to the words which
immediately precede them. They were no doubt
inserted in order to prevent disputes founded on nice
distinctions. X x x For example, if the expression
'perils of the seas' is given its widest sense the
general words have little or no effect as applied to
that case. If on the other hand that expression is to
receive a limited construction, as apparently it did in
!"#$%&"'( *+,- 67

Cullen vs. Butler (5 M. & S., 461), and loss by perils
of the seas is to be confined to loss ex marine
tempestatis discrimine, the general words become
most important. X x x" (Thames and Mersey Marine
Insurance Co. vs. Hamilton, Fraser & Co.)
- a loss which, in the ordinary course of events,
results from the natural and inevitable action of the
sea, from the ordinary wear and tear of the ship, or
from the negligent failure of the ship's owner to
provide the vessel with proper equipment to convey
the cargo under ordinary conditions, is not a peril of
the sea. Such a loss is rather due to what has been
aptly called the "peril of the ship." The insurer
undertakes to insure against perils of the sea and
similar perils, not against perils of the ship. There
must, in order to make the insurer liable, be "some
casualty, something which could not be foreseen as
one of the necessary incidents of the adventure. The
purpose of the policy is to secure an indemnity
against accidents which may happen, not against
events which must happen." (Wilson, Sons & Co. vs.
Owners of Cargo per the Xantho)
- In the present case the entrance of the sea water
into the ship's hold through the defective pipe
already described was not due to any accident which
happened during the voyage, but to the failure of the
ship's owner properly to repair a defect of the
existence of which he was apprised. The loss was
therefore more analogous to that which directly
results from simple unseaworthiness than to that
which results from perils of the sea.
- there is no room to doubt the liability of the
shipowner for such a loss as occurred in this case. By
parity of reasoning the insurer is not liable; for,
generally speaking, the shipowner excepts the perils
of the sea from his engagement under the bill of
lading, while this is the very peril against which the
insurer intends to give protection. As applied to the
present case it results that the owners of the
damaged rice must look to the shipowner for redress
and not to the insurer.
The same conclusion must be reached if the question
be discussed with reference to the seaworthiness of
the ship. It is universally accepted that in every
contract of insurance upon anything which is the
subject of marine insurance, a warranty is implied
that the ship shall be seaworthy at the time of the
inception of the voyage. This rule is accepted in our
own Insurance Law (Act No. 2427, sec. 106). It is
also well settled that a ship which is seaworthy for
the purpose of insurance upon the ship may yet be
unseaworthy for the purpose of insurance upon the
cargo (Act No. 2427, sec. 106).
Disposition Decision of trial court is affirmed

CATHAY INSURANCE CO. v. CA (REMINGTON
INDUSTRIAL SALES CORP.)
151 SCRA 710
PARAS; June 30 1987

FACTS
- Remington Industrial Sales Corp insured its
shipment of seamless steel pipes. It incurred losses
and damages (I gather the steel pipes rusted during
the voyage from Japan to the Phils. on board vessel
SS "Eastern Mariner) and filed complaint against
Cathay Insurance Co seeking collection of the sum of
P868,339.15
- TC decided for Remington. Cathay filed MR, which
was denied. CA affirmed.
- CA said (among other things): 1. Coverage of
private respondent's loss under the insurance policy
issued by petitioner is unmistakable;
2. Alleged contractual limitations contained in
insurance policies are regarded with extreme caution
by courts and are to be strictly construed against the
insurer; obscure phrases and exceptions should not
be allowed to defeat the very purpose for which the
policy was procured;
3. Rust is not an inherent vice of the seamless steel
pipes without interference of external factors
- Cathay contend (among other things): 1. private
respondent has admitted that the questioned
shipment is not covered by a "square provision of
the contract," but private respondent claims implied
coverage from the phrase "perils of the sea"
mentioned in the opening sentence of the policy; 2.
The insistence of private respondent that rusting is a
peril of the sea is erroneous; 3. Rusting is not a
risk insured against, since a risk to be insured
against should be a casualty or some casualty,
something which could not be foreseen as one of the
necessary incidents of adventure; 4. A fact capable
of unquestionable demonstration or of public
knowledge needs no evidence. This fact of
unquestionable demonstration or of public knowledge
is that heavy rusting of steel or iron pipes cannot
occur within a period of a seven (7) day voyage.
Besides, petitioner had introduced the clear cargo
receipts or tally sheets indicating that there was no
damage on the steel pipes during the voyage.

ISSUE
WON rusting is a peril of the sea

HELD
YES
- There is no question that the rusting of steel pipes
in the course of a voyage is a "peril of the sea" in
view of the toll on the cargo of wind, water, and salt
conditions. At any rate if the insurer cannot be held
accountable therefor, We would fail to observe a
cardinal rule in the interpretation of contracts,
namely, that any ambiguity therein should be
construed against the maker/issuer/drafter thereof,
namely, the insurer. Besides the precise purpose of
insuring cargo during a voyage would be rendered
fruitless.
Disposition WHEREFORE, this petition is hereby
DENIED, and the assailed decision of the Court of
Appeals is hereby AFFIRMED.

ROQUE v. IAC (PIONEER INSURANCE AND
SURETY CORP.)
139 SCRA 596
GUTIERREZ; November 11, 1985

NATURE
Petition for certiorari to review the decision of the
IAC

FACTS
- February 19, 1972 Common carrier Manila Bay
Lighterage Corp. entered into a contract with Roque
Timber Enterprises and Chiong. The contract stated
that Manila Bay would carry 422.18 cu. meters of
logs on its vessel Mable 10 from Malampaya Sound,
Palawan to Manila North Harbor. Roque insured the
logs with Pioneer Insurance for P100,000.
- February 29, 1972 811 logs were loaded in
Malampaya but en route to Manila, Mable 10 sank.
- March 8,1972 Roque and Chiong wrote a letter to
Manila Bay, demanding payment of P150,000.00 for
the loss of the shipment plus P100,000.00 as
unrealized profits but the latter ignored the demand.
- A letter was also sent to Pioneer, claiming the full
amount of P100,000.00 under the insurance policy
but Pioneer refused to pay on the ground that its
liability depended upon the "Total Loss by Total Loss
of Vessel only".
- After hearing, the trial court favored Roque.
Pioneer and Manila Bay were ordered to pay Roque
P100,000. Pioneer appealed the decision.
- January 30, 1984 Pioneer was absolved from
liability after finding that there was a breach of
!"#$%&"'( *+,- 6.

implied warranty of seaworthiness on the part of the
petitioners and that the loss of the insured cargo was
caused by the "perils of the ship" and not by the
"perils of the sea". It ruled that the loss is not
covered by the marine insurance policy.
- It was alleged that Mable 10 was not seaworthy
and that it developed a leak
- The IAC found that one of the hatches was left
open, causing water to enter the barge and because
the barge was not provided with the necessary cover
or tarpaulin, the splash of sea waves brought more
water inside the barge.
- Petitioners contend that the implied warranty of
seaworthiness provided for in the Insurance Code
refers only to the responsibility of the shipowner who
must see to it that his ship is reasonably fit to make
in safety the contemplated voyage.
- The petitioners state that a mere shipper of cargo,
having no control over the ship, has nothing to do
with its seaworthiness. They argue that a cargo
owner has no control over the structure of the ship,
its cables, anchors, fuel and provisions, the manner
of loading his cargo and the cargo of other shippers,
and the hiring of a sufficient number of competent
officers and seamen.
ISSUE
WON the loss should have been covered by the
marine insurance policy

HELD
NO
Ratio It is universally accepted that in every
contract of insurance upon anything which is the
subject of marine insurance, a warranty is implied
that the ship shall be seaworthy at the time of the
inception of the voyage. In marine insurance, the
risks insured against are classified as 'perils of the
sea, which includes such losses that are of
extraordinary nature, or arise from some
overwhelming power, which cannot be guarded
against by the ordinary exertion of human skill and
prudence.
Reasoning
- Based on Sec. 113 and Sec. 99 of the Insurance
Code, the term "cargo" can be the subject of marine
insurance and that once it is so made, the implied
warranty of seaworthiness immediately attaches to
whoever is insuring the cargo whether he be the
shipowner or not.
- The fact that the un-seaworthiness of the ship was
unknown to the insured is immaterial in ordinary
marine insurance and may not be used by him as a
defense in order to recover on the marine insurance
policy.
- Since the law provides for an implied warranty of
seaworthiness in every contract of ordinary marine
insurance, it becomes the obligation of a cargo
owner to look for a reliable common carrier which
keeps its vessels in seaworthy condition. The shipper
of cargo my have no control over the vessel but he
has full control in the choice of the common carrier
that will transport his goods.
- In marine cases, the risks insured against are
'perils of the sea. The term extends only to losses
caused by sea damage, or by the violence of the
elements, and does not embrace all losses
happening at sea.
- It is quite unmistakable that the loss of the cargo
was due to the perils of the ship rather than the
perils of the sea.
- Loss which, in the ordinary course of events,
results from the natural and inevitable action of the
sea, from the ordinary wear and tear of the ship, or
from the negligent failure of the ship's owner to
provide the vessel with proper equipment to convey
the cargo under ordinary conditions, is not a peril of
the sea but is called peril of the ship.
Disposition Decision appealed from is affirmed.

LA RAZON v. UNION INSURANCE SOCIETY OF
CANTON, LTD.
40 PHIL 40
STREET; September 1, 1919

FACTS
- This is an action on a policy of marine insurance
issued by the Union Insurance Society of Canton,
Ltd., upon a cargo of rice belonging to the plaintiffs,
Go Tiaoco Brothers, which was transported on the
steamship Hondagua from the port of Saigon to
Cebu.
- On discharging the rice from one of the
compartments in the after hold, upon arrival at
Cebu, it was discovered that 1,473 sacks had been
damaged by sea water.
- The loss so resulting to the owners of rice, after
proper deduction had been made for the portion
saved, was P3,875.25.
- The trial court found that the inflow of the sea
water during the voyage was due to a defect in one
of the drain pipes of the ship and concluded that the
loss was not covered by the policy of insurance.
Judgment was accordingly entered in favor of the
defendant and the plaintiffs appealed.
- The court found in effect that the opening above
described had resulted in course of time from
ordinary wear and tear and not from the straining of
the ship in rough weather on that voyage. The court
also found that the repairs that had been made on
the pipe were slovenly and defective and that, by
reason of the condition of this pipe, the ship was not
properly equipped to receive the rice at the time the
voyage was begun. For this reason the court held
that the ship was unseaworthy.

ISSUE
WON the insurer is liable

HELD
- The question whether the insurer is liable on this
policy for the loss caused in the manner above
stated presents two phases which are in a manner
involved with each other. One has reference to the
meaning of the expression "perils of the seas and all
other perils, losses, and misfortunes," as used in the
policy; the other has reference to the implied
warranty, on the part of the insured, as to the
seaworthiness of the ship.
- The meaning of the expression "perils * * * of the
seas * * * and all other perils, losses, and
misfortunes," used in describing the risks covered by
policies of marine insurance, has been the subject of
frequent discussion; and certain propositions relative
thereto are now so generally accepted as to be
considered definitely settled.
- The words "all other perils, losses, and
misfortunes" are to be interpreted as covering risks
which are of like kind (ejusdem generis) with the
particular risks which are enumerated in the
preceding part of the same clause of the contract.
- A loss which, in the ordinary course of events,
results from the natural and inevitable action of the
sea, from the ordinary wear and tear of the ship, or
from the negligent failure of the ship's owner to
provide the vessel with proper equipment to convey
the cargo under ordinary conditions, is not a peril of
the sea. Such a loss is rather due to what has been
aptly called the "peril of the ship." The insurer
undertakes to insure against perils of the sea and
similar perils, not against perils of the ship.
- As was said by Lord Herschell in Wilson, Sons & Co.
vs. Owners of Cargo per the Xantho, there must, in
order to make the insurer liable, be "some casualty,
something which could not be foreseen as one of the
necessary incidents of the adventure. The purpose of
the policy is to secure an indemnity against accidents
!"#$%&"'( *+,- 6/

which may happen, not against events which must
happen."
- In the present case the entrance of the sea water
into the ship's hold through the defective pipe
already described was not due to any accident which
happened during the voyage, but to the failure of the
ship's owner properly to repair a defect of the
existence of which, he was apprised. The loss was
therefore more analogous to that which directly
results from simple unseaworthiness than to that
which results from perils of the sea.
- It is universally accepted that in every contract of
insurance upon anything which is the subject of
marine insurance, a warranty is implied that the ship
shall be seaworthy at the time of the inception of the
voyage. This rule is accepted in our own Insurance
Law (Act No. 2427, see. 106).
- It is also well settled that a ship which is
seaworthy for the purpose of insurance upon the
ship may yet be unseaworthy for the purpose of
insurance upon the cargo (Act No. 2427, see. 106).
Disposition Jjudgment affirmed.

MALAYAN INSURANCE v. CA (supra p.10)

FILIPINO MERCHANTS INS. CO. v. CA (supra
p.19)

COASTWISE LIGHTERAGE CORP v. CA
(PHILIPPINE GENERAL INSURANCE COMPANY)
245 SCRA 796
FRANCISCO; July 12, 1995

NATURE
Petition for review of CA Decision affirming decision
of RTC Manila holding that Coastwise is liable to pay
PhilGen Insurance the amount of P700thou plus legal
interest thereon, another sum of P100thou as
attorney's fees and the cost of the suit.

FACTS
- Pag-asa Sales, Inc. entered into a contract to
transport molasses from the province of Negros to
Manila with Coastwise, using the latter's dumb
barges. The barges were towed in tandem by the
tugboat MT Marica, also owned by Coastwise. Upon
reaching Manila Bay, while approaching Pier 18, one
of the barges struck an unknown sunken object. The
forward buoyancy compartment was damaged, and
water gushed in through a hole "two inches wide and
twenty-two inches long."
- As a consequence, the molasses at the cargo tanks
were contaminated and rendered unfit for the use it
was intended. This prompted consignee Pag-asa
Sales to reject the shipment of molasses as a total
loss. Thereafter, Pag-asa Sales filed a formal claim
with the insurer of its lost cargo (PhilGen) and
against the carrier (Coastwise). Coastwise denied the
claim and it was PhilGen which paid Pag-asa Sales
the amount of P700k representing the value of the
damaged cargo of molasses.
- PhilGen then filed an action against Coastwise
before the RTC Manila, seeking to recover the P700k
which it paid to Pag-asa Sales for the latter's lost
cargo. PhilGen now claims to be subrogated to all the
contractual rights and claims which the consignee
may have against the carrier, which is presumed to
have violated the contract of carriage.
- RTC awarded the amount prayed for by PhilGen. CA
affirmed. Hence, this petition.

ISSUES
1. WON Coastwise Lighterage was transformed into a
private carrier, by virtue of the contract of
affreightment which it entered into with the
consignee, Pag-asa Sales, Inc. (Corollarily, if it were
in fact transformed into a private carrier, did it
exercise the ordinary diligence to which a private
carrier is in turn bound?)
2. WON the insurer was subrogated into the rights of
the consignee against the carrier, upon payment by
the insurer of the value of the consignee's goods lost
while on board one of the carrier's vessels

HELD
1. NO
- The distinction between the two kinds of charter
parties (i.e. bareboat or demise and contract of
affreightment) is more clearly set out in the case of
Puromines, Inc. vs. Court of Appeals,

wherein SC
ruled:
Under the demise or bareboat charter of the
vessel, the charterer will generally be regarded
as the owner for the voyage or service
stipulated. The charterer mans the vessel with
his own people and becomes the owner pro hac
vice, subject to liability to others for damages
caused by negligence. To create a demise, the
owner of a vessel must completely and
exclusively relinquish possession, command and
navigation thereof to the charterer, anything
short of such a complete transfer is a contract of
affreightment (time or voyage charter party) or
not a charter party at all.
On the other hand a contract of affreightment is
one in which the owner of the vessel leases part
or all of its space to haul goods for others. It is a
contract for special service to be rendered by
the owner of the vessel and under such contract
the general owner retains the possession,
command and navigation of the ship, the
charterer or freighter merely having use of the
space in the vessel in return for his payment of
the charter hire...
An owner who retains possession of the ship
though the hold is the property of the charterer,
remains liable as carrier and must answer for
any breach of duty as to the care, loading and
unloading of the cargo.
- Although a charter party may transform a common
carrier into a private one, the same however is not
true in a contract of affreightment on account of the
aforementioned distinctions between the two.
- SC agrees with Coastwise's admission that the
contract it entered into with the consignee was one
of affreightment.

Pag-asa Sales, Inc. only leased
three of petitioner's vessels, in order to carry cargo
from one point to another, but the possession,
command and navigation of the vessels remained
with Coastwise. As such, Coastwise, by the contract
of affreightment, was not converted into a private
carrier, but remained a common carrier and was still
liable as such.
- Therefore, the mere proof of delivery of goods in
good order to a carrier and the subsequent arrival of
the same goods at the place of destination in bad
order makes for a prima facie case against the
carrier. The presumption of negligence that attaches
to common carriers, once the goods it transports are
lost, destroyed or deteriorated, applies to Coastwise.
This presumption, which is overcome only by proof
of the exercise of extraordinary diligence, remained
unrebutted in this case.
- The damage to the barge which carried the cargo
of molasses was caused by its hitting an unknown
sunken object as it was heading for Pier 18. The
object turned out to be a submerged derelict vessel.
The evidence on record appeared that far from
having rendered service with the greatest skill and
utmost foresight, and being free from fault, the
carrier was culpably remiss in the observance of its
duties.
- Jesus R. Constantino, the patron of the vessel
"Coastwise 9" admitted that he was not licensed. The
!"#$%&"'( *+,- 60

Code of Commerce, which subsidiarily governs
common carriers (which are primarily governed by
the provisions of the Civil Code) provides: Art. 609.
Captains, masters, or patrons of vessels must be
Filipinos, have legal capacity to contract in
accordance with this code, and prove the skill
capacity and qualifications necessary to command
and direct the vessel, as established by marine and
navigation laws, ordinances or regulations, and must
not be disqualified according to the same for the
discharge of the duties of the position.
- Clearly, Coastwise Lighterage's embarking on a
voyage with an unlicensed patron violates this rule.
It cannot safely claim to have exercised
extraordinary diligence, by placing a person whose
navigational skills are questionable, at the helm of
the vessel which eventually met the fateful accident.
It may also logically, follow that a person without
license to navigate, lacks not just the skill to do so,
but also the utmost familiarity with the usual and
safe routes taken by seasoned and legally authorized
ones. Had the patron been licensed, he could be
presumed to have both the skill and the knowledge
that would have prevented the vessel's hitting the
sunken derelict ship that lay on their way to Pier 18.
- As a common carrier, Coastwise is liable for breach
of the contract of carriage, having failed to overcome
the presumption of negligence with the loss and
destruction of goods it transported, by proof of its
exercise of extraordinary diligence.
2. YES
- Coastwise is liable for breach of the contract of
carriage it entered into with Pag-asa Sales, Inc.
However, for the damage sustained by the loss of
the cargo which petitioner-carrier was transporting,
it was not the carrier which paid the value thereof to
Pag-asa Sales, Inc. but the latter's insurer, herein
private respondent PhilGen.
- Article 2207 of the Civil Code: If the plaintiffs
property has been insured, and he has received
indemnity from the insurance company for the injury
or loss arising out of the wrong or breach of contract
complained of, the insurance company shall be
subrogated to the rights of the insured against the
wrongdoer or the person who violated the contract.
- This legal provision is founded on the well-settled
principle of subrogation. If the insured property is
destroyed or damaged through the fault or
negligence of a party other than the assured, then
the insurer, upon payment to the assured will be
subrogated to the rights of the assured to recover
from the wrongdoer to the extent that the insurer
has been obligated to pay. Payment by the insurer to
the assured operated as an equitable assignment to
the former of all remedies which the latter may have
against the third party whose negligence or wrongful
act caused the loss. The right of subrogation is not
dependent upon, nor does it grow out of, any privity
of contract or upon written assignment of claim. It
accrues simply upon payment of the insurance claim
by the insurer.
- Undoubtedly, upon payment by respondent insurer
PhilGen of the amount of P700,000.00 to Pag-asa
Sales, Inc., the consignee of the cargo of molasses
totally damaged while being transported by
petitioner Coastwise Lighterage, the former was
subrogated into all the rights which Pag-asa Sales,
Inc. may have had against the carrier, herein
petitioner Coastwise Lighterage.
Disposition Petition denied. CA affrimed.

THE PHILIPPINE AMERICAN GENERAL
INSURANCE COMPANY INC v. CA (FELMAN
SHIPPING LINES)
273 SCRA 226
BELLOSILLO; June 11, 1997

FACTS
- Coca-Cola Bottlers Philippines, Inc., loaded on
board MV Asilda, a vessel owned and operated by
Felman 7,500 cases of 1-liter Coca-Cola softdrink
bottles to be transported from Zamboanga City to
Cebu for consignee Coca-Cola Bottlers Philippines,
Inc., Cebu.

The shipment was insured with petitioner
Philippine American General under Marine Open
Policy.
- The vessel sank in the waters of Zamboanga del
Norte bringing down her entire cargo with her
including the subject 7,500 cases of 1-liter Coca-Cola
softdrink bottles.
- The consignee filed a claim with respondent
FELMAN for recovery of damages it sustained as a
result of the loss of its softdrink bottles that sank
with MV Asilda. Respondent denied the claim
thus prompting the consignee to file an insurance
claim with PHILAMGEN which paid its claim of
P755,250.00.
- Claiming its right of subrogation PHILAMGEN
sought recourse against respondent FELMAN which
disclaimed any liability for the loss. Consequently,
PHILAMGEN sued the shipowner for sum of money
and damages.
- PHILAMGEN alleged that the sinking and total loss
of MV Asilda and its cargo were due to the vessels
unseaworthiness as she was put to sea in an
unstable condition. It further alleged that the
vessel was improperly manned and that its
officers were grossly negligent in failing to take
appropriate measures to proceed to a nearby port or
beach after the vessel started to list.
- FELMAN filed a motion to dismiss based on the
affirmative defense that no right of subrogation in
favor of PHILAMGEN was transmitted by the shipper,
and that, in any event, FELMAN had abandoned all
its rights, interests and ownership over MV Asilda
together with her freight and appurtenances for the
purpose of limiting and extinguishing its liability
under Art. 587 of the Code of Commerce.
- Trial court dismissed the complaint of PHILAMGEN.
On appeal the Court of Appeals set aside the
dismissal and remanded the case to the lower court
for trial on the merits. FELMAN filed a petition for
certiorari with this Court but it was subsequently
denied on 13 February 1989.
- Trial court rendered judgment in favor of FELMAN.
It ruled that MV Asilda was seaworthy when it left
the port of Zamboanga as confirmed by certificates
issued by the Philippine Coast Guard and the
shipowners surveyor attesting to its seaworthiness.
Thus the loss of the vessel and its entire shipment
could only be attributed to either a fortuitous event,
in which case, no liability should attach unless there
was a stipulation to the contrary, or to the
negligence of the captain and his crew, in which
case, Art. 587 of the Code of Commerce should
apply.
- CA ruled that MV Asilda was unseaworthy for
being top- heavy as 2,500 cases of Coca-Cola
softdrink bottles were improperly stowed on deck.
Nonetheless, the appellate court denied the claim of
PHILAMGEN on the ground that the assureds implied
warranty of seaworthiness was not complied with.
Perfunctorily, PHILAMGEN was not properly
subrogated to the rights and interests of the shipper.
Furthermore, respondent court held that the filing of
notice of abandonment had absolved the
shipowner/agent from liability under the limited
liability rule.

ISSUES
1. WON MV Asilda was seaworthy when it left the
port of Zamboanga
2. WON the limited liability under Art. 587 of the
Code of Commerce should apply
3. WON PHILAMGEN was properly subrogated to the
rights and legal actions which the shipper had
!"#$%&"'( *+,- 61

against FELMAN, the shipowner

HELD
1. YES
- MV Asilda was unseaworthy when it left the port
of Zamboanga. We subscribe to the findings of the
Elite Adjusters, Inc., and the Court of Appeals that
the proximate cause of the sinking of MV Asilda
was its being top-heavy. Contrary to the ship
captains allegations, evidence shows that
approximately 2,500 cases of softdrink bottles were
stowed on deck. Several days after MV Asilda
sank, an estimated 2,500 empty Coca-Cola plastic
cases were recovered near the vicinity of the sinking.
Considering that the ships hatches were properly
secured, the empty Coca-Cola cases recovered could
have come only from the vessels deck cargo. It is
settled that carrying a deck cargo raises the
presumption of unseaworthiness unless it can be
shown that the deck cargo will not interfere with the
proper management of the ship. However, in this
case it was established that MV Asilda was not
designed to carry substantial amount of cargo on
deck. The inordinate loading of cargo deck resulted
in the decrease of the vessels metacentric height
thus making it unstable. The strong winds and
waves encountered by the vessel are but the
ordinary vicissitudes of a sea voyage and as such
merely contributed to its already unstable and
unseaworthy condition.
2. NO
- The ship agent is liable for the negligent acts of the
captain in the care of goods loaded on the vessel.
This liability however can be limited through
abandonment of the vessel, its equipment and
freightage as provided in Art. 587. Nonetheless,
there are exceptional circumstances wherein the
ship agent could still be held answerable despite the
abandonment, as where the loss or injury was due to
the fault of the shipowner and the captain. The
international rule is to the effect that the right of
abandonment of vessels, as a legal limitation of a
shipowners liability, does not apply to cases where
the injury or average was occasioned by the
shipowners own fault.

3. YES
- The doctrine of subrogation has its roots in equity.
It is designed to promote and to accomplish justice
and is the mode which equity adopts to compel the
ultimate payment of a debt by one who in justice,
equity and good conscience ought to pay. Therefore,
the payment made by PHILAMGEN to Coca-Cola
Bottlers Philippines, Inc., gave the former the right
to bring an action as subrogee against FELMAN.
Having failed to rebut the presumption of fault, the
liability of FELMAN for the loss of the 7,500 cases of
1-liter Coca-Cola softdrink bottles is inevitable.
- Sec. 113 of the Insurance Code provides that (i)n
every marine insurance upon a ship or freight, or
freightage, or upon anything which is the subject of
marine insurance, a warranty is implied that the ship
is seaworthy. Under Sec. 114, a ship is seaworthy
when reasonably fit to perform the service, and to
encounter the ordinary perils of the voyage,
contemplated by the parties to the policy. Thus it
becomes the obligation of the cargo owner to look
for a reliable common carrier which keeps its
vessels in seaworthy condition. He may have no
control over the vessel but he has full control in
the selection of the common carrier that will
transport his goods. He also has full discretion in the
choice of assurer that will underwrite a particular
venture.
- In policies where the law will generally imply a
warranty of seaworthiness, it can only be excluded
by terms in writing in the policy in the clearest
language. And where the policy stipulates that the
seaworthiness of the vessel as between the assured
and the assurer is admitted, the question of
seaworthiness cannot be raised by the assurer
without showing concealment or misrepresentation
by the assured.
- PHILAMGENs action against FELMAN is squarely
sanctioned by Art. 2207 of the Civil Code which
provides:
Art. 2207. If the plaintiffs property has been
insured, and he has received indemnity from
the insurance company for the injury or loss
arising out of the wrong or breach of contract
complained of, the insurance company shall
be subrogated to the rights of the insured
against the wrongdoer or the person who has
violated the contract. If the amount paid by
the insurance company does not fully cover
the injury or loss, the aggrieved party shall
be entitled to recover the deficiency from the
person causing the loss or injury.
Disposition Petition is GRANTED. Respondent
FELMAN SHIPPING LINES is ordered to pay petitioner
PHILIPPINE AMERICAN GENERAL INSURANCE CO.,
INC., Seven Hundred Fifty-five Thousand Two
Hundred and Fifty Pesos (P755,250.00) plus legal
interest thereon counted from 29 November 1983,
the date of judicial demand, pursuant to Arts. 2212
and 2213 of the Civil Code.

PHILIPPINE MFTG. CO. v. UNION INSURANCE
SOCIETY OF CANTON
42 PHIL 378
JOHNS; November 22, 1921

FACTS
- The plaintiffs steel tank lighter was insured by
defendant company for absolute total loss. As a
result of a typhoon, the lighter sunk in Manila Bay.
The plaintiff demanded payment from the defendant
insurance company but the latter refused. The
company asked the plaintiff to salvage the ship,
which it was able to do so.
- With the plaintiff able to raise the lighter,
reconstruct it and placed it in commission, the
defendant insurance company claims that it was only
liable for a total absolute loss and that there was no
total destruction of the lighter.
- The trial court decided in favor of the defendant,
saying that the policy only covered an actual total
loss, not a constructive total loss.

ISSUES
1. WON there was an absolute total loss that can be
covered by the policy
2. WON the Marine Law of Great Britain applies

HELD
1. YES
- At the time that the lighter was at the bottom of
the bay, it was of no value to the owner, thus there
was an actual total loss.
- The ship was sunk in July 1, 1918. After several
futile attempts, it was finally raised on Sept. 20,
1918. It is faitr to assume that in its then condition
much further time would be required to make the
necessary repairs and install the new machinery
before it could again be placed in commission.
During that time the owner would be deprived of the
use of its vessel or the interest on its investment.
When those questions are considered the testimony
is conclusive that the cost of salvage, repair and
reconstruction was more than the original cost of the
ship at the time the policy was issued. As found by
the trial court, t is difficult to see how there could
have been a more complete loss of the vessel than
that which actually occurred. Upon the facts shown
here, any other construction would nullify the statute
and as applied to the conditions existing in the
Manila Bay, this kind of policy would be worthless,
!"#$%&"'( *+,- 62

and there would not be any consideration for the
premium.
2. NO
- The defendant argues that the policy contains the
provision that it shall be of as force and effect as
the surest writing or policy of insurance made in
London. However, for such law to apply to our
courts the existence of such law must be proven. It
cannot apply when such proof is lacking.
Nevertheless, in the English practice, a ship is a total
loss when she has sustained such extensive
damages that it would not be reasonably practical to
repair her.
Disposition Decision reversed










CHOA TIEK SENG v. CA (FILIPINO MERCHANTS
INSURANCE)
183 SCRA 223
GANCAYO; March 15, 1990

NATURE
Appeal from a decision of the Court of Appeals

FACTS
- Petitioner imported some lactose crystals from
Holland.
- The importation involved fifteen (15) metric tons
packed in 600 6-ply paper bags with polythelene
inner bags, each bag at 25 kilos net. The goods were
loaded at the port at Rotterdam in sea vans on board
the vessel "MS Benalder' as the mother vessel, and
thereafter aboard the feeder vessel "Wesser Broker
V-25" of respondent Ben Lines Container, Ltd. (Ben
Lines for short). The goods were insured by the
respondent Filipino Merchants' Insurance Co., Inc.
(insurance company for short) for the sum of
P98,882.35, the equivalent of US$8,765.00 plus
50% mark-up or US $13,147.50, against all risks
under the terms of the insurance cargo policy. Upon
arrival at the port of Manila, the cargo was
discharged into the custody of the arrastre operator
respondent E. Razon, Inc. (broker for short), prior to
the delivery to petitioner through his broker. Of the
600 bags delivered to petitioner, 403 were in bad
order. The surveys showed that the bad order bags
suffered spillage and loss later valued at P33,117.63.
Petitioner filed a claim for said loss dated February
16, 1977 against respondent insurance company in
the amount of P33,117.63 as the insured value of
the loss.
- Respondent insurance company rejected the claim
alleging that assuming that spillage took place while
the goods were in transit, petitioner and his agent
failed to avert or minimize the loss by failing to
recover spillage from the sea van, thus violating the
terms of the insurance policy sued upon; and that
assuming that the spillage did not occur while the
cargo was in transit, the said 400 bags were loaded
in bad order, and that in any case, the van did not
carry any evidence of spillage.
- Petitioner filed a complaint in the RTC against the
insurance company seeking payment of the sum of
P33,117.63 as damages plus attorney's fees and
expenses of litigation. Insurance company denied all
the material allegations of the complaint and raised
several special defenses as well as a compulsory
counterclaim. Insurance company filed a third-party
complaint against respondents Ben Lines and broker.
- RTC dismissed the complaint, the counterclaim and
the third-party complaint with costs against the
petitioner. Appealed in CA but denied. MFR was
denied as well.

ISSUE
WON insurance company should be held liable even
if the technical meaning in marine insurance of an
insurance against all risk" is applied

HELD
YES
- In Gloren Inc. vs. Filipinas Cia. de Seguros, 12 it
was held that an all risk insurance policy insures
against all causes of conceivable loss or damage,
except as otherwise excluded in the policy or due to
fraud or intentional misconduct on the part of the
insured. It covers all losses during the voyage
whether arising from a marine peril or not, including
pilferage losses during the war.
- In the present case, the "all risks" clause of the
policy sued upon reads as follows:
"5. This insurance is against all risks of loss or
damage to the subject matter insured but shall in
no case be deemed to extend to cover loss,
damage, or expense proximately caused by delay
or inherent vice or nature of the subject matter
insured. Claims recoverable hereunder shall be
payable irrespective of percentage."
- The terms of the policy are so clear and require no
interpretation. The insurance policy covers all loss or
damage to the cargo except those caused by delay
or inherent vice or nature of the cargo insured. It is
the duty of the respondent insurance company to
establish that said loss or damage falls within the
exceptions provided for by law, otherwise it is liable
therefor.
- An "all risks" provision of a marine policy creates a
special type of insurance which extends coverage to
risks not usually contemplated and avoids putting
upon the insured the burden of establishing that the
loss was due to peril falling within the policy's
coverage. The insurer can avoid coverage upon
demonstrating that a specific provision expressly
excludes the loss from coverage.
- In this case, the damage caused to the cargo has
not been attributed to any of the exceptions provided
for nor is there any pretension to this effect. Thus,
the liability of respondent insurance company is
clear.
Disposition the decision appealed from is hereby
REVERSED AND SET ASIDE and another judgment is
hereby rendered ordering the respondent Filipinas
Merchants Insurance Company, Inc. to pay the sum
of P33,117.63 as damages to petitioner with legal
interest from the filing of the complaint, plus
attorney's fees and expenses of litigation in the
amount of P10,000.00 as well as the costs of the
suit.

FILIPINO MERCHANTS INS. CO. v. CA (supra
p.19)

ABOITIZ SHIPPING v. PHILAMGEN INSURANCE
179 SCRA 357
GANCAYCO; October 5, 1989

NATURE
Petition for review on certiorari

FACTS
- Marinduque Mining Industrial Corporation
(Marinduque) shipped on board SS Arthur Maersk
from Boston, U.S.A. a shipment of 1 skid carton
parts for valves. The shipment was ordered from
Jamesbury, Singapore PTE, LTD., which issued the
cargo's packing list and Invoice number showing the
contents of the carton. The Philippine Consulate in
Singapore issued invoice for the shipment showing
!"#$%&"'( *+,- 63

the contents and its total price of $39,419.60 and
the freight and other charges of $2,791.73. When
the cargo arrived in Manila, it was received and
deposited in the office of Aboitiz Shipping
Corporation (Aboitiz) for transhipment to Nonoc
Island.
- In July 1980, Marinduque, as consignee of the
cargo, made a report that said cargo was pilfered on
July 3, 1980 due to heavy rain at the Aboitiz
terminal and that of the total value of the cargo of
$42,209.33, only $7,412.00 worth remains of the
cargo with the recommendation that the claim be
made against Aboitiz.
- The services of the Manila Adjusters and Surveyors
Co. (Manila Adjusters) were engaged by the Phil-
American General Insurance Co., Inc. (Phil Am)
which came out with the report that the cargo in
question, when inspected, showed that it was
pilfered. A confirmatory report was submitted by the
Manila Adjusters.
- On August 11, 1980 Marinduque then filed a claim
against Aboitiz in the amount of P246,430.80
representing the value of the pilfered cargo. On the
same day Marinduque filed a claim for the same
amount against the Phil-Am on the latter's policy.
Phil-Am paid Marinduque the sum of P246,430.80 as
insurer of the cargo.
- Phil-Am then filed a complaint in RTC Manila
against Aboitiz for recovery of same amount alleging
that it has been subrogated to the rights of
Marinduque. Complaint dismissed and MFR denied.
CA reversed. MFR thereof was denied. Hence, this
petition.

ISSUE
WON petitioner Aboitiz was properly held liable to
the private respondent Phil-Am by the appellate
court

HELD
YES
- The questioned shipment is covered by a
continuing open insurance coverage (which took
effect after Sept. 1, 1975, as contained in Marine
Open Policy No. 100184) from the time it was loaded
aboard the SS Arthur Maersk in Boston, U.S.A. to the
time it was delivered to the possession of petitioner
at its offices at Pier 4 in Manila until it was pilfered
when the great majority of the cargo was lost on July
3, 1980. Hence, petitioner Aboitiz was properly held
liable to Phil-Am.
Reasoning
[a] Records of the case show that Phil-Am executed
a continuous and open insurance coverage covering
goods of Marinduque imported into and exported
from the Philippines which took effect after Sept. 1,
1975, as contained in Marine Open Policy No.
100184.

A similar insurance coverage was also
executed by petitioner in favor of Marinduque for all
its goods shipped or moved within the territorial
limits of the Philippines also effective after Sept. 1,
1975 and contained in Marine Open Policy No.
100185.
[b] TC in dismissing the complaint apparently relied
on Marine Risk Note No. 017545 issued by private
respondent Phil-Am only on July 28, 1980 after the
shipment in question was already pilfered. Obviously
TC mistook said Marine Risk Note as an insurance
policy when it is NOT. It is only an acknowledgment
or declaration of the private respondent confirming
the specific shipment covered by its Marine Open
Policy, the evaluation of the cargo and the
chargeable premium.
[c] The contention of the Aboitiz that it could not be
liable for the pilferage of the cargo as it was stolen
even before it was loaded on its vessel is untenable.
Aboitiz received cargo when it arrived in Manila at its
offices, and it was while in its possession and before
loading it in its vessel that the cargo was pilfered. Its
liability is clear.
Disposition Petition DISMISSED.

ORIENTAL ASSURANCE v. CA (PANAMA SAW
MILL)
200 SCRA 459
MELENCIO-HERRERA; August 9, 1991

NATURE
Petition for review on certiorari

FACTS
- Sometime in January 1986, private respondent
Panama Sawmill Co., Inc. (Panama) bought, in
Palawan, 1,208 pieces of apitong logs, with a total
volume of 2,000 cubic meters. It hired Transpacific
Towage, Inc., to transport the logs by sea to Manila
and insured it against loss for P1-M with petitioner
Oriental Assurance Corporation (Oriental Assurance).
- While the logs were being transported, rough seas
and strong winds caused damage to one of the two
barges resulting in the loss of 497 pieces of logs out
of the 598 pieces loaded thereon.
- Panama demanded payment for the loss but
Oriental Assurance refuse on the ground that its
contracted liability was for "TOTAL LOSS ONLY."
- Unable to convince Oriental Assurance to pay its
claim, Panama filed a Complaint for Damages against
Oriental Assurance before the Regional Trial Court.
- RTC ordered Oriental Assurance to pay Panama
with the view that the insurance contract should be
liberally construed in order to avoid a denial of
substantial justice; and that the logs loaded in the
two barges should be treated separately such that
the loss sustained by the shipment in one of them
may be considered as "constructive total loss" and
correspondingly compensable. CA affirmed in toto.

ISSUE
WON Oriental Assurance can be held liable under its
marine insurance policy based on the theory of a
divisible contract of insurance and, consequently, a
constructive total loss

HELD
NO
- The terms of the contract constitute the measure of
the insurer liability and compliance therewith is a
condition precedent to the insured's right to recovery
from the insurer. Whether a contract is entire or
severable is a question of intention to be determined
by the language employed by the parties. The policy
in question shows that the subject matter insured
was the entire shipment of 2,000 cubic meters of
apitong logs. The fact that the logs were loaded on
two different barges did not make the contract
several and divisible as to the items insured. The
logs on the two barges were not separately valued or
separately insured. Only one premium was paid for
the entire shipment, making for only one cause or
consideration. The insurance contract must,
therefore, be considered indivisible.
- More importantly, the insurer's liability was for
"total loss only." A total loss may be either actual or
constructive (Sec. 129, Insurance Code). An actual
total loss is caused by:
(a) A total destruction of the thing insured;
(b) The irretrievable loss of the thing by sinking, or
by being broken up;
(c) Any damage to the thing which renders it
valueless to the owner for the purpose for which
he held it; or
(d) Any other event which effectively deprives the
owner of the possession, at the port of destination,
!"#$%&"'( *+,- 64

of the thing insured. (Section 130, Insurance
Code).
- A constructive total loss is one which gives to a
person insured a right to abandon, under Section
139 of the Insurance Code. This provision reads:
SECTION 139. A person insured by a contract of
marine insurance may abandon the thing insured,
or any particular portion thereof separately valued
by the policy, or otherwise separately insured, and
recover for a total loss thereof, when the cause of
the loss is a peril injured against,
(a) If more than three-fourths thereof in value is
actually lost, or would have to be expended to
recover it from the peril;
(b) If it is injured to such an extent as to reduce
its value more than three-fourths;
xxx xxx xxx
- The requirements for the application of Section 139
of the Insurance Code, quoted above, have not been
met. The logs involved, although placed in two
barges, were not separately valued by the policy, nor
separately insured. Resultantly, the logs lost in the
damaged barge in relation to the total number of
logs loaded on the same barge cannot be made the
basis for determining constructive total loss. The logs
having been insured as one inseparable unit, the
correct basis for determining the existence of
constructive total loss is the totality of the shipment
of logs. Of the entirety of 1,208, pieces of logs, only
497 pieces thereof were lost or 41.45% of the entire
shipment. Since the cost of those 497 pieces does
not exceed 75% of the value of all 1,208 pieces of
logs, the shipment cannot be said to have sustained
a constructive total loss under Section 139(a) of the
Insurance Code.
Disposition judgment under review is SET ASIDE

PAN MALAYAN INSURANCE v. CA (THE FOOD
AND AGRICULTURAL ORGANIZATION OF THE
UNITED NATIONS)
201 SCRA 382
REGALADO; September 5, 1991

FACTS
- The Food and Agricultural Organization of the
United Nations (hereinafter referred to as FAO),
ntended and made arrangements to send to
Kampuchea 1,500 metric petitions of IR-36 certified
rice seeds to be distributed to the people for seedling
purposes
- LUZTEVECO was to ship the cargo amounting to
US$83,325.92 in respect of one lot of 1,500 metric
petitions winch is the subject of the present action.
The cargo was loaded on board LUZTEVECO Barge
No. LC-3000 and consisted of 34,122 bags of IR-36
certified rice seeds purchased by FAO from the
Bureau of Plant Industry for P4,602,270.00
- FAO secured insurance coverage in the amount of
P5,250,000.00 from petitioner, Pan Malayan
Insurance Corporation
- On June 16, 1980, FAO gave instructions to
LUZTEVECO to leave for Vaung Tau, Vietnam to
deliver the cargo which, by its nature, could not
withstand delay because of the inherent risks of
termination and/or spoilage. On the same date, the
insurance premiums on the shipment was paid by
FAO petitioner
- On June 26, 1980, FAO was advised of the sinking
of the barge in the China Sea, hence it informed
petitioner thereof and, later, formally filed its claim
under the marine insurance policy. On July 29, 1980,
FAO was informed by LUSTEVECO of the recovery of
the lost shipment, for which reason FAO formally
filed its claim with LUZTEVECO for compensation of
damage to its cargo
- LUZTEVECO failed and refused to pay. Pan Malayan
likewise failed to pay for the losses and damages
sustained by FAO by reason of its inability to recover
the value of the shipment from LUZTEVECO
- Pan Malayan claims that part of the cargo was
recovered and thus the claim by FAO was
unwarranted. This is evidenced by two surveys upon
the cargo wherein it was found that only around 78%
was lost.
- FAO filed a civil case against both LUZTEVECO and
Pan Malayan. Trial court found in favor of FAO and
ordered both to pay jointly and severally the full
amount of the claim. This was affirmed by CA

ISSUE
1. WON respondent court committed a reversible
error in holding that the trial court is correct in
holding that there is a total loss of the shipment

HELD
1. NO
- The law classifies loss into either total or partial.
Total loss may be actual or absolute, or it may
otherwise be constructive or technical. Petitioner
submits that respondent court erred in ruling that
there was total loss of the shipment despite the fact
that only 27,922 bags of rice seeds out of 34,122
bags were rendered valueless to FAO and the
shipment sustained only a loss of 78%. - FAO,
however, claims that, for all intents and purposes, it
has practically lost its total or entire shipment in this
case, inclusive of expenses, premium fees, and so
forth, despite the alleged recovery by defendant
LUZTEVECO. As found by the court below and
reproduced with approval by respondent court, FAO
"has never been compensated for this total loss or
damage, a fact which is not denied nor controverted
- If there were some cargoes saved, by LUZTEVECO,
private respondent abandoned it and the same was
sold or used for the benefit of LUZTEVECO or Pan
Malayan Corporation. Under Sections 129 and 130 of
the New Insurance Code, a total loss may either be
actual or constructive. In case of total loss in Marine
Insurance, the assured is entitled to recover from
the underwriter the whole amount of his subscription
- SEC. 130. An actual total loss is caused by: (c) Any
damage to the thing which renders it valueless to the
owner for the purpose for which he held it; or
(d) Any other event which effectively deprives the
owner of the possession, at the port of destination of
the thing insured.
-as said and proven, the seeds were of fragile
nature. And the wetting of said seeds affected the
state of seeds. Thus rendering them useless for FAO.
Although there were bags which were recovered,
these were stained and not in the same condition it
was brought in. in addition to this, FAO did not
receive any compensation for said recovered bags as
the same were distributed by LUZVETECO without
authorization of FAO
- the complete physical destruction of the subject
matter is not essential to constitute an actual total
loss. Such a loss may exist where the form and
specie of the thing is destroyed, although the
materials of which it consisted still exist (Great
Western Ins. Co. vs. Fogarty, N.Y., 19 Wall 640, 22
L. Ed. 216), as where the cargo by the process of
decomposition or other chemical agency no longer
remains the same kind of thing as before (Williams
vs. Cole, 16 Me. 207).
- It is thus clear that FAO suffered actual total loss
under Section 130 of the Insurance Code, specifically
under paragraphs (c) and (d) thereof, recompense
for which it has been denied up to the present
-Section 135 of the Insurance Code explicitly
provides that "(u)pon an actual total loss, a person
insured is entitled to payment without notice of
abandonment." This is a statutory adoption of a long
standing doctrine in maritime insurance law that in
case of actual total loss, the right of the insured to
!"#$%&"'( *+,- 65

claim the whole insurance is absolute, without need
of a notice of abandonment

PHILIPPINE AMERICAN LIFE INSURANCE
COMPANY v. CA (ELIZA PULIDO)
344 SCRA 260
GONZAGA-REYES; November 15, 2000

NATURE
This petition for review on certiorari seeks to reverse
the Decision of the Special Second Division of the
Court of Appeals

FACTS
- On January 9, 1989, petitioner received from one
Florence Pulido an application for life insurance,
dated December 16, 1988, in the amount of
P100,000.00 which designated her sister, herein
private respondent, as its principal beneficiary.
Because the insurance applied for was non-medical,
petitioner did not require a medical examination and
issued a policy on the sole basis of the application on
February 11, 1989. On April 1992, petitioner
received private respondents claim, which declared
that the insured, Florence Pulido, died of acute
pneumonia on September 10, 1991.
- Petitioner withheld payment on the ground that the
policy claimed under was void from the start for
having been procured in fraud. It is petitioners
contention that even before they received private
respondents claim for death benefits, their
investigation concerning the subject policy yielded
the information that the insured, Florence Pulido,
died in 1988, before the application for insurance on
her life was made. While this was communicated to
private respondent in a letter dated April 29, 1992,
private respondent had already filed her claim earlier
that month. In another letter dated July 27, 1992,
however, petitioner confirmed to private respondent
receipt of the claim papers and assured her that her
case was being given preferential attention and
prompt action.
- Following the filing by private respondent of her
claim, petitioner caused another investigation
respecting the subject policy. Pursuant to the
findings of this second investigation, petitioner stood
by its initial decision to treat the policy as void and
not to honor the claim. On November 9, 1992,
private respondent enlisted the services of counsel in
reiterating her claim for death benefits. Petitioner
still refused to make payment and thus, this action.
- Petitioner: the results of its investigations having
indicated that the insured was already dead at the
time the policy was applied for. It also
counterclaimed for attorneys fees. The first report,
prepared by one Dr. Benedicto Briones, was dated
April 1, 1992, and had attached to it a questionnaire,
responded to by one Ramon Piganto, who
represented to be the brother-in-law of the insured
and the barangay chairman of Cardiz, Bagulin, La
Union. To the question Where does [Florence
Pulido] reside now?, Piganto had replied that
Florence Pulido used to live in Cardiz, but was dead
since 1988. Pigantos statement was signed by him,
and witnessed by his wife, Nenita Piganto. This
report was petitioners basis for treating the disputed
policy as void since April 1992, even before receipt
of private respondents claim.

ISSUE
WON there was fraud (whether the insured, Florence
Pulido, was in fact dead before the application for
insurance on her life was made)

HELD
NO
- This the lower courts had effected ruled on, upon a
preponderance of the evidence duly received from
both parties. We see no reversible error in the
finding of both respondent court and the trial court in
favor of the correctness of the entries in Certificate
of Death, duly registered with the Local Civil
Registrar of Bagulin, La Union, which declared that
Florence Pulido died of acute pneumonia on
September 10, 1991. Dr. Irineo Gutierrez, the
Municipal Health Officer of Bagulin, La Union whose
signature appeared in the death certificate, testified
in addition that he ministered to the ailing Florence
Pulido for two days immediately prior to her death.
This fact is likewise noted in the death certificate.
- Death certificates, and notes by a municipal health
officer prepared in the regular performance of his
duties, are prima facie evidence of facts therein
stated. A duly-registered death certificate is
considered a public document and the entries found
therein are presumed correct, unless the party who
contests its accuracy can produce positive evidence
establishing otherwise. Petitioners contention that
the death certificate is suspect because Dr. Gutierrez
was not present when Florence Pulido died, and
knew of Florences death only through Ramon
Piganto, does not merit a conclusion of fraud. No
motive was imputed to Dr. Gutierrez for seeking to
perpetuate a falsity in public records. Petitioner was
likewise unable to make out any clear motive as to
why Ramon Piganto would purposely lie. Mere
allegations of fraud could not substitute for the full
and convincing evidence that is required to prove it.
A failure to do so would leave intact the presumption
of good faith and regularity in the performance of
public duties, which was the basis of both
respondent court and the trial court in finding the
date of Florence Pulidos death to be as plaintiff-
private respondent maintained.
- We cannot likewise give credence to petitioners
submission that the inconsistencies in the
testimonies of the witnesses for plaintiff-private
respondent are in themselves evidence of fraud.
Such alleged inconsistencies are matters of
credibility which had been ably passed upon by the
lower court.
- The absence of fraud, as a factual finding of the
lower court adopted by the Court of Appeals, entirely
consistent with the evidence on record, will not be
reversed and, hence, is final and conclusive upon
this Court.
Disposition The instant petition is DENIED

CHAPTER IX. CLAIMS SETTLEMENT &
SUBROGATION

LONDRES v. NATIONAL LIFE INSURANCE
94 PHIL 627
BAUTISTA ANGELO; March 29, 1954

NATURE
Appeal from a decision of the Court of First Instance
of Manila ordering defendant to pay to plaintiff the
sum of P3,000, Philippine currency, plus legal
interest thereon from the time of the filing of the
complaint until its full payment.

FACTS
- On April 14, 1943, the National Life Insurance
Company of the Philippines issued a policy on the life
of Jose C. Londres whereby it undertook to pay its
beneficiary upon his death the sum of P3,000. All the
premiums due under the policy were actually paid on
their dates of maturity and the policy was in force
when the insured died on February 7, 1945.
Salvacion V. Londres, as beneficiary, demanded from
the company the payment of the proceeds of the
policy, and her demand having been refused, she
instituted the present action against the company in
the Court of First Instance of Manila.
- Defendant in its answer denied, for lack of
!"#$%&"'( *+,- 66

sufficient proof, the allegation that the insured died
on February 7, 1945, and set up the following special
defenses: (a) that plaintiff's claim is covered by the
Moratorium Law; (b) that the policy having been
issued during the Japanese occupation, it is
presumed that its face value should be paid in
Japanese currency, there being no provision in the
policy from which can be inferred that the parties
contemplated payment in any other currency; (c)
that the money paid by the insured as premiums,
together with the money received from other policy-
holders, was all deposited by the defendant in the
Philippine National Bank and said deposit was
declared without value by Executive Order No. 49 of
the President of the Philippines; and (d) that the
policy having been issued under abnormal
circumstances, it should be considered in the light of
equity which does not permit anyone to enrich
himself at the expense of another. Defendant,
however, as a proof of good faith, offered to pay the
value of the policy in accordance with the Ballantyne
scale of values, or the sum of P2,400, Philippine
currency.
- It appears that the deceased took up the policy
under consideration on April 15, 1943 for the sum of
P3,000. All the premiums due under the policy were
actually paid on their dates of maturity and the
policy was in force when the insured died on
February 7, 1945. On said date, the battle of the
liberation of the City of Manila was still raging. While
the northern part may have been liberated, not so
the southern part, as shown from the very affidavits
submitted by appellee wherein it was stated that on
the aforesaid date, the insured, Jose Londres, and
his two sons were taken by the Japanese soldiers
from their house at Singalong Street and were
massacred by their captors. It may therefore be said
that the policy became due when the City of Manila
was still under the yoke of the enemy and became
payable only after liberation which took place on
March 10, 1945 when President Osmena issued
Proclamation No. 6 following the restoration of the
civil government by General Douglas Mac Arthur.
And we say that the policy became payable only
after liberation even if it matured sometime before,
because before that eventuality the insurance
company, appellant herein, was not yet in a position
to pay the value of the policy for the simple reason
that it had not yet reopened.

ISSUE
WON the amount of P3,000 which appellant bound
itself to pay to the insured under the policy upon his
death should be paid in accordance with the present
currency or should be adjusted under the Ballantyne
scale of values

HELD
YES, present currency.
Reasoning
- In the case of Rutter vs. Esteban, 93 Phil., 68, the
Moratorium Law was declared invalid and
unconstitutional.
- During those days of liberation, while the people
were rejoicing because of the happy event, the
banks, the insurance companies, and for that matter
other commercial and business firms, were still
feeling the adverse effects of the sudden fall of
values and were uncertain and apprehensive as to
the manner the readjustment would be made by the
new Government. It is for this reason that the
beneficiary, after realizing the truth about the death
of her husband, and after gathering evidence to
substantiate his death, had difficulty in effecting the
collection of her claim from the insurance company
because at that time it had not yet reopened for
business purposes. Although the record does not
disclose the exact date on which the insurance
company reopened for this purpose, this Court can
take judicial notice that it only did so after liberation.
At that time the legal tender was already the
present currency.
- As final plea, appellant invokes equity in its favor in
view of the nullification of the deposits made by it
with the Philippine National Bank of all fiat money
received from its policyholders, which money was
declared without value by Executive Order No. 49 of
the President of the Philippines. Appellant claims
that, considering the unexpected circumstances that
developed, the indemnity to be paid by it should be
suffered by it under Article 307 of the Code of
Commerce which provides: "When the deposits are
of cash, with a specification of the coins constituting
them, . . . the increase or reduction which their value
may suffer shall be for the account of the depositor."
Appellant, by entering into an insurance
contract, cannot claim, if it suffers loss, that
the beneficiary cannot enrich herself at its
expense. This is a risk attendant to any wagering
contract. One who gambles and loses cannot be
heard to complain of his loss. To appellant, we can
only repeat the following admonition:
"The parties herein gambled and speculated on the
date of the termination of the war and the
liberation of the Philippines by the Americans. This
can be gleaned from the stipulation about
redemption, particularly that portion to the effect
that redemption could be effected not before the
expiration of one year from June 24, 1944. This
kind of agreement is permitted by law. We find
nothing immoral or unlawful in it." (Gomez vs.
Tabia)
Disposition Wherefore, the decision appealed from
is affirmed, with costs against appellant.





VDA. DE FERNANDEZ v. NATIONAL LIFE
INSURANCE CO OF THE PHILS
105 PHIL 59
ENDENCIA; January 27, 1959

NATURE
Appeal from CFI decision applying the Ballantyne
scale of values upon the proceeds of life insurance
taken and maturing during the Japanese occupation
but claimed after liberation

FACTS
- National Life Insurance Company (NLIC) insured J.
Fernandezs life for P10,000 upon his payment of
P444 from July 15, 1944 to July 14, 1945
- The insured died on November 2, 1944, while the
policy was in force
- After more than 7 years, in 1952, Atty de la Torre,
representing the benficiaries of the policy, informed
the company that Fernandez had died in 1944, and
claimed the proceeds of the policy. The company
said that the status of the policies issued during the
Japanese occupation was still pending consideration
before the courts. NLIC said that because the policy
matured upon the insureds death in November,
1944, they should compute the value of their claim
under the Ballantyne scale of values (which would
amount only to P500)
- beneficiaries commenced suit, and the lower court
sustained the stand of the company, dismissed the
complaint.
- beneficiaries maintain that the obligation of the
company to pay accrued not upon the death of
Fernandez, but only upon the receipt and approval
by the company, on proof of death of the insured,
which was in 1954. The policy reads:
National Life Insurance Company of the Philippine
!"#$%&"'( *+,-
.77

hereby agrees to pay at its Home Office, Manila,
Ten Thousand Pesos to Juan D. Fernandez
(hereinafter called the insured) on the 15th day of
July, 1964, if the Insured is living and this Policy is
in force, or upon receipt and approved at its Office
of due proofs of the title of the claimant and of the
prior death of the Insured while this Policy is in
force to Teresa Duat Vda. De Fernandez, Maria T.
and Manuela Fernandez, mother and sisters
respectively of the Insured (Hereinafter called the
Beneficiary) subject to the right of the Insured to
change the beneficiary as stated on the second
page of this Policy.
- The above stipulation is apparently based on Sec.
91-A of the Insurance Law which provides as follows:
The proceeds of a life insurance policy shall be paid
immediately upon maturity of the policy, unless such
proceeds are made payable in installments or a as an
annuity, in which case the installments or annuities
shall be paid as they become due: Provided,
however, That in case of a policy maturing by the
death of the insured, the proceeds thereof shall be
paid within sixty days after presentation of the claim
and filing of the proof of the death of the insured.
Refused to pay the claim within the time prescribed
herein will entitle the beneficiary to collect interest
on the proceeds of the policy for the duration of the
delay at the rate of six per centum per annum,
unless such failure or refusal to pay is based on the
ground that the claim is fraudulent . . . .
- Based on the foregoing provision of law and the
aforequoted stipulation as well as on the allegation
that the filing of proof of death by the beneficiaries is
a condition precedent of the demandability of the
obligation of the insurer to pay the proceeds,
appellants claim that they should be paid P10,000 in
Philippine currency and not under the Ballantyne
scale of values.

ISSUE
WON the policy matured upon the death of the
insured

HELD
YES
Ratio In life insurance, the policy matures either
upon the expiration of the term set forth therein, or
upon his death occuring at any time prior to the
expiration of such stipulated term, in which case, the
proceeds are payable to his beneficiaries within sixty
days after their filing of proof of death.
Reasoning
- The sixty day period fixed by law within which to
pay the proceeds after presentation of proof of death
Is merely procedural in nature, evidently to
determine the exact amount to be paid and the
interest thereon to which the beneficiaries may be
entitled to collect in case of unwarranted refusal of
the company to pay, and also to enable the insurer
to verify or check on the fact of death which it may
even validly waive. It is the happening of the
suspensive condition of death that renders a life
policy matured, and not ht efiling of proof of death
which, as above stated, is merely procedural. The
insured having died during the Japanese occupation,
the proceeds of his policy should be adjusted
accordingly, for The rule is already settled that
where a debtor could have paid his obligation at any
time during the Japanese occupation, payment after
liberation must be adjusted in accordance with the
Ballantyne schedule (De Asis vs. Agdamag, among
other cases). (Collaboration is defined as the acts
of working together in a joint project.
Disposition Judgment affirmed

TIO KHE CHIO v. CA (EASTERN ASSURANCE &
SURETY)
202 SCRA 119
FERNAN; September 30, 1991

FACTS
- Petitioner Tio Khe Chio imported 1,000 bags of
fishmeal valued at $36,000.30 from Agro Impex,
S.A. Dallas, Texas, U.S.A. The goods were insured
with respondent EASCO and shipped on board the
M/V Peskov, a vessel owned by Far Eastern Shipping
Company. When the goods reached Manila, they
were found to have been damaged by sea water
which rendered the fishmeal useless. Petitioner filed
a claim with EASCO and Far Eastern Shipping. Both
refused to pay. Whereupon, petitioner sued them
before the then Court of First Instance of Cebu for
damages. EASCO, as the insurer, filed a
counterclaim against the petitioner for the recovery
of the unpaid insurance premiums.
- The trial court rendered judgment in favor of
petitioner. The judgment became final as to EASCO
but the shipping company appealed to the Court of
Appeals and was absolved from liability by the said
court.
- The trial court, upon motion by petitioner, issued a
writ of execution against EASCO. The sheriff
enforcing the writ reportedly fixed the legal rate of
interest at 12%. Respondent EASCO moved to quash
the writ alleging that the legal interest to be
computed should be 6% per cent per annum in
accordance with Article 2209 of the Civil Code. The
trial court denied EASCO's motion. On appeal, the
Court of Appeals reversed the trial courts denial of
EASCOs motion and ruled that the applicable
interest is 6% per annum. Hence, this petition.

ISSUE
WON the applicable rate of interest is 12% per
annum

HELD
NO
- Sections 243 and 244 of the Insurance Code apply
only when there is an unjustified refusal or
withholding of payment on the insureds claim. In
this case, EASCO's refusal to settle the claim to Tio
Khe Chio was based on some ground which, while
not sufficient to free it from liability under its policy,
nevertheless is sufficient to negate any assertion
that in refusing to pay, it acted unjustifiably. Simply
put, the said provisions of the Insurance Code are
not pertinent to the instant case. They apply only
when the court finds an unreasonable delay or
refusal in the payment of the claims.
- Circular No. 416 of the Central Bank, which raised
the legal rate of interest from 6% to 12% per annum
refers only to loans or forbearances of money, goods
or credits and court judgments thereon but not to
court judgments for damages arising from injury to
persons and loss of property which does not involve
a loan. Clearly, the applicable law is Article 2209 of
the Civil Code.
- And in the light of the fact that the contending
parties did not allege the rate of interest stipulated in
the insurance contract, the legal interest was
properly pegged by the Appellate Court at 6% per
cent.

CATHAY INSURANCE v. CA(LUGAY)
174 SCRA 11
GRINO-AQUINO; June 5, 1989

FACTS
- Petitioners are 6 insurance companies that issued
fire insurance policies for the total sum of
P4,000,000 to the Cebu Filipina Press owned by
Emilia Chan Lugay. The fire policies described the
!"#$%&"'( *+,-
.7.

insured property as "stocks of Printing materials,
papers and general merchandise usual to the
Assured's trade" stored in a one-storey building of
strong materials housing the Cebu Filipina Press
located at UNNO Pres. Quirino cor. Don V. Sotto
Sts., Mabolo, Cebu City. The co-insurers were
indicated in each of the policies. All, except one
policy (Paramount's), were renewals of earlier
policies issued for the same property.
- On December 18, 1981, the Cebu Filipina Press was
razed by electrical fire together with all the stocks
and merchandise stored in the premises. On
January 15, 1982, Lugay submitted sworn
Statements of Loss and Formal Claims to the
insurers, through their adjusters. She claimed a total
loss of P4,595,000.
- After nearly 10 months of waiting, she sued to
collect on December 15, 1982. The insurance
companies denied liability, alleging violation of
certain conditions of the policy, misdeclaration, and
even arson which was not seriously pressed for,
come the pre-trial, the petitioners offered to pay
50% of her claim, but she insisted on full recovery.
- Trial court rendered judgment in her favor ordering
the insurers to pay her a total of P4,000,000 as
indemnity, P48,000 representing expenses of the
plaintiff, a separate amount of 20% of the
P4,000,000 representing fees of counsel, interests at
the rate of twice the ceiling being prescribed by the
Monetary Board starting from the time when the
case was filed, and finally, with costs. CA affirmed.

ISSUES
1. WON the insured's cause of action had already
accrued before she filed her complaint
2. WON sufficient proofs of loss had been presented
by the insured
3. WON the private respondents claim for loss was
inflated
4. WON lower court erred in awarding damages to
the private respondent in the form of interest
equivalent to double the interest ceiling set by the
Monetary Board
5. WON attorney's fees awarded were exorbitant

HELD
1. YES
- As the fire which destroyed the Cebu Filipina Press
occurred on December 19, 1981 and the proofs of
loss were submitted from January 15, 1982 through
June 21, 1982 in compliance with the adjusters'
numerous requests for various documents, payment
should have been made within 90 days thereafter
(Sec 243), or on or before September 21, 1982.
Hence, when the assured filed her complaint on
December 15, 1982, her cause of action had already
accrued.
2. YES
- There is no merit in the petitioners' contention that
the proofs of loss were insufficient because Lugay
failed to comply with the adjuster's request for the
submission of her bank statements. Condition No. 13
of the policy does not require the insured to produce
her bank statements. Therefore, the insured was not
obligated to produce them and the insurers had no
right to ask for them. Condition No. 13 was prepared
by the insurers themselves, hence, it should be
taken most strongly against them.
3. NO
- Both the trial court and the CA noted that the
proofs were ample and more than enough for
defendant insurers to do a just assessment
supporting the 1981 fire claim for an amount
exceeding four million pesos.
4. NO
- The award of double interest on the claim is lawful
and justified under Sections 243 and 244 of the
Insurance Code which provide:
Sec. 243 Refusal or failure to pay the loss or
damage within the time prescribed herein will
entitle the assured to collect interest on the
proceeds of the policy for the duration of the delay
at the rate of twice the ceiling prescribed by the
Monetary Board.
Sec. 244 In case of any litigation for the
enforcement of any policy or contract of insurance,
it shall be the duty of the Commissioner or the
Court, as the case may be, to make a finding as to
whether the payment of the claim of the insured
has been unreasonably denied or withheld; and in
the affirmative case, the insurance company shall
be adjudged to pay damages which shall consist of
attorney's fees and other expenses incurred by the
insured person by reason of such unreasonable
denial or withholding of payment plus interest of
twice the ceiling prescribed by the Monetary Board
of the amount of claim due the insured.
- The petitioners' contention that the charging of
double interest was improper because no
unreasonable delay in the processing of the fire claim
was proven is refuted by the trial court's explicit
finding that "there was a delay that was not
reasonable in processing the claim and doing
payments". Under Section 244, a prima facie
evidence of unreasonable delay in payment of the
claim is created by the failure of the insurer to pay
the claim within the time fixed in both Sec. 242 and
243 of the IC.
- In view of the not insubstantial value of the private
respondent's claims and the considerable time and
effort expended by them and their counsel in
prosecuting these claims for the past 8 years,
attorney's fees were properly awarded to the private
respondents.

5. YES
- An award equivalent to 10% of the proceeds of the
policies would be more reasonable than the 20%
awarded by the trial court and the CA.
Disposition Decision of the CA AFFIRMED with
MODIFICATION.

NODA v. CRUZ
151 SCRA 227
FERNAN; June 22, 1987

NATURE
Petition to review decision of the Insurance
Commissioner

FACTS
- In 1977, Noda obtained from Zenith Insurance
Corporation 2 fire insurance policies: [1] No. F-
03724 with a face value of P30k covering the goods
and stocks in trade in his business establishment at
the market site in Mangagoy, Bislig, Surigao del Sur
and [2] No. F-03734 with a face value in the
aggregate amount of P100k and consisting of Item 1
for P40k on household furniture, fixtures, fittings and
other personal effects, and Item 2 for P60k on stocks
in trade usual to petitioner's retail business situated
in a two-storey building at 039 Barreda St.,
Mangagoy, Bislig, Surigao del Sur.
- While both policies were in force, fire destroyed
petitioner's insured properties at the market site on
September 5, 1977 and at Barreda St. on November
9, 1977.
- When petitioner failed to obtain indemnity on his
claims from Zenith, he filed a complaint with the
Insurance Commission praying that Zenith be
ordered to pay him P130kj representing the value of
the 2 policies insured by respondent with interest at
!"#$%&"'( *+,-
.7/

12% per annum, plus damages, attorney's fees and
other expenses of litigation. ...
- Zenith interposed that petitioner had no cause of
action; that Policy No. F-03724 was not in full force
and effect at the time of the fire because the
premium on the policy was not paid; that Zenith's
liability under Policy No. F-03734, if any, was limited
to P15,472.50 in view of the co-insurance; and that
petitioner failed to substantiate his claim as to the
value of the goods reputedly destroyed by fire.
- While the case was pending, Zenith settled
petitioners fire loss claim under Item 1 of Policy No.
03734 in the amount of P15,472.50.
- Insurance Commissioner allowed petitioner to
recover under said policy and ordered Zenith to pay
him the amount of P20k with legal interest from the
date the complaint was filed, including P1k as
attorney's fees but excluding the actual, moral and
exemplary damages prayed for.

As for petitioner's
claim under Policy No. F-03734, she held that in view
of the payment of P15,472.50 to petitioner, Zenith
had fully discharged its liability under said policy
which covered furniture, fixtures, fittings and other
personal belongings of petitioner.
- In allowing recovery under Policy No. F-03734,
Commissioner placed much weight on the final report
prepared by Dela Merced Adjustment Corporation, an
independent fire, marine and casualty adjuster
contracted by Zenith to investigate the claims of its
various policyholders. Said report concluded that
"the sound value of P26,666.67 represented the
whole loss and damage" incurred by petitioner, but
with the application of the three-fourths loss clause,
Zenith's liability was reduced to P20k.



ISSUES
1. WON Insurance Commissioner erred in denying
petitioner's demand for P60k under Item 2 of Policy
No. F-03734
2. WON Insurance Commissioner erred in not
awarding in favor of petitioner exemplary damages
for Zenith's unjustified and wanton refusal to pay
petitioner's claim under the said two insurance
contracts

HELD
1. YES
- To prove the existence of the stocks in trade
covered by Policy No. F-03734, petitioner offered his
testimony and that of his wife as well as
documentary exhibits. The foregoing evidence for
petitioner preponderantly showed the presence of
some P590k worth of goods in his retail store during
the fire of November 9, 1977.
- While the insurer, and the Insurance Commissioner
for that matter, have the right to reject proofs of loss
if they are unsatisfactory, they may not set up for
themselves an arbitrary standard of satisfaction.
Substantial compliance with the requirements will
always be deemed sufficient.
- Zenith introduced in evidence the final report on
Policy No. F-03734 submitted by its own adjuster,
Dela Merced Adjustment Corporation. Respondent
Commissioner however ignored such report,
reasoning that with regard to Item 2 of Policy No. F-
03734 the claim for loss of the stocks in trade was
not successfully proven in view of petitioner's failure
to present evidence; that the adjuster's report
deserved scant consideration since the allegations
therein were not substantiated, and that said report
did not even make a recommendation for payment.
- A scrutiny of the abovementioned adjuster's report
reveals that together with the formal demand for full
indemnity, petitioner submitted his income tax
return for 1978, purchase invoices, certification from
his suppliers as to his purchases, and other
supporting papers. The report even took into account
the appraisals of the other adjusters and concluded
that the total loss sustained by petitioner in his
household effects and stocks in trade reached
P379,302.12. But after apportioning said amount
among petitioner's six different insurers [the co-
insurance being known to Zenith], the liability of
Zenith was placed at P60,592.10. It therefore
recommended that Zenith pay the petitioner the
amount of P60, 592.10.
- Said document was offered as evidence by Zenith
itself and could very well be considered as an
admission of its liability up to the amount
recommended. Being in the nature of an admission
against interest, it is the best evidence which affords
the greatest certainty of the facts in dispute.
Respondent Commissioner should not have
perfunctorily dismissed that particular evidence as a
worthless piece of paper.
2. NO
- There is no showing that Zenith, in contesting
payment, had acted in a wanton, oppressive or
malevolent manner to warrant the imposition of
corrective damages.
Disposition Zenith Insurance Corporation ordered
to pay petitioner Norman R. Noda the sum of
P60,592.10 with legal interest from the filing of the
complaint until full payment, but deducting
therefrom the amount of P15,472.50 which it had
earlier paid to petitioner.

DELSAN TRANSPORT, INC. v. CA (AMERICAN
HOME ASSURANCE)
369 SCRA 24
DE LEON, JR; November 15, 2001

NATURE
A petition for review on certiorari of the decision of
CA.

FACTS
- Caltex entered into a contract of affreightment with
the petitioner, Delsan Transport Lines, Inc.
(petitioner), for a period of one year whereby the
said common carrier agreed to transport Caltexs
industrial fuel oil from the Batangas-Bataan Refinery
to different parts of the country. Delsan took on
board its vessel, MT Maysun, 2,277.314 kiloliters of
industrial fuel oil of Caltex to be delivered to the
Caltex Oil Terminal in Zamboanga City. The
shipment was insured by American Home Assurance
Corporation (respondent).
- August 14, 1986: MT Maysun set sail from
Batangas for Zamboanga City. The vessel sank in
the early morning of August 16, 1986 near Panay
Gulf in the Visayas taking with it the entire cargo of
fuel oil.
- Respondent paid Caltex P5,096,635.57
representing the insured value of the lost cargo.
Exercising its right of subrogation under Article 2207
of the New Civil Code, the private respondent
demanded of the petitioner the same amount it paid
to Caltex. Delsan refused to pay, forcing American
home to file a case for collection in the RTC.
- RTC found that the vessel, MT Maysun, was
seaworthy to undertake the voyage, and that the
incident was caused by an unexpected inclement
weather condition or force majeure, thus exempting
the common carrier from liability for the loss of its
cargo.
- CA reversed RTC decision on the basis of evidence
from PAG-ASA that there were no 20 ft. waves in the
area. CA ruled that the petitioner is liable on its
obligation as common carrier to respondent
insurance company as subrogee of Caltex.
Petitioners Claim
!"#$%&"'( *+,-
.70

> In every marine insurance upon a ship or freight,
or freightage, or upon any thing which is the subject
of marine insurance there is an implied warranty by
the shipper that the ship is seaworthy.
10
When
private respondent paid Caltex the value of its lost
cargo, the act of the private respondent is equivalent
to a tacit recognition that the ill-fated vessel was
seaworthy.

Respondents Comment
> American Home Assurance is entitled to payment
by its right of subrogation.

ISSUES
1. WON payment made by American Home to Caltex
for the insured value of the lost cargo amounted to
an admission that the vessel was seaworthy, thus
precluding any action for recovery against the
petitioner
2. WON MT Maysun was seaworthy at the time of the
voyage (outline topic)
3. WON non-presentation of the marine insurance
policy bars the complaint for recovery of sum of
money for lack of cause of action

HELD
1. NO
Ratio The fact of payment grants American Home
the subrogatory right which enables it to exercise
legal remedies that would otherwise be available to
Caltex as owner of the lost cargo against the
petitioner common carrier.
Reasoning
Art. 2207. (Civil Code)
If the plaintiffs property has been insured, and he
has received indemnity from the insurance
company for the injury or loss arising out of the
wrong or breach of contract complained of, the
insurance company shall be subrogated to the
rights of the insured against the wrongdoer or the
person who has violated the contract. If the
amount paid by the insurance company does not
fully cover the injury or loss, the aggrieved party
shall be entitled to recover the deficiency from the
person causing the loss or injury.
- The right of subrogation is designed to promote
and to accomplish justice and is the mode which
equity adopts to compel the ultimate payment of a
debt by one who in justice and good conscience

!L
^1.& !!( U-2:$/-.1 *+91
ought to pay. It is not dependent upon, nor does it
grow out of, any privity of contract or upon written
assignment of claim. It accrues simply upon
payment by the insurance company of the insurance
claim.
2. NO
Ratio Seaworthiness relates to a vessels actual
condition. Neither the granting of classification or
the issuance of certificates establishes
seaworthiness.
Reasoning
- Common carriers are bound to observe
extraordinary diligence in the vigilance over the
goods and for the safety of passengers transported
by them, according to all the circumstances of each
case. There is no liability if the loss, destruction or
deterioration is by force majeure.
- The tale of strong winds and big waves by the said
officers of the petitioner however, was effectively
rebutted and belied by the weather report from PAG-
ASA. MT Maysun sank with its entire cargo for the
reason that it was not seaworthy. There was no
squall or bad weather or extremely poor sea
condition in the vicinity when the said vessel sank.
- Petitioner may not escape liability by presenting in
evidence certificates that tend to show that at the
time of dry-docking and inspection by the Philippine
Coast Guard MT Maysun, was fit for voyage. These
pieces of evidence do not necessarily take into
account the actual condition of the vessel at the time
of the commencement of the voyage. At the time of
dry-docking and inspection, the ship may have
appeared fit. The certificates issued, however, do
not negate the presumption of unseaworthiness
triggered by an unexplained sinking.
- Authorities are clear that diligence in securing
certificates of seaworthiness does not satisfy the
vessel owners obligation. Also securing the approval
of the shipper of the cargo, or his surveyor, of the
condition of the vessel or her stowage does not
establish due diligence if the vessel was in fact
unseaworthy, for the cargo owner has no obligation
in relation to seaworthiness.
3. NO
Ratio The presentation in evidence of the marine
insurance policy is not indispensable in this case
before the insurer may recover from the common
carrier the insured value of the lost cargo in the
exercise of its subrogatory right. The subrogation
receipt, by itself, is sufficient to establish not only
the relationship of respondent as insurer and Caltex,
as the assured shipper of the lost cargo of industrial
fuel oil, but also the amount paid to settle the
insurance claim. The right of subrogation accrues
simply upon payment by the insurance company of
the insurance claim.
Disposition Petition is denied, and the decision of
the CA is affirmed.




FINMAN GENERAL ASSURANCE CORP v.
INOCENCIO
179 SCRA 480
FELICIANO; November 15, 1989

FACTS
- Pan Pacific is a recruitment and employment
agency. It posted surety bond issued by Finman
General Assurance and was granted license to
operate by POEA.
- Inocencio, Palero, Cardones, Hernandez filed with
POEA complaints against Pan Pacific for violation of
Labor Code and for refund of placement fees. POEA
Administrator motu propio impleaded Finman as
surety for Pan Pacific.
- Pan Pacific moved out and no notice of transfer was
furnished to POEA as required. POEA considered
that constructive service of complaints had been
effected.
- Finman denied liability and said that
- POEA had no jurisdiction over surety bonds;
jurisdiction is vested in Insurance Commission
- Finman had not violated Labor Code
- Complainants have no cause of action against
Finman
- Amounts claimed were paid as deposits and not as
placement fees.
- POEA Administrator issued Order that respondents
should pay. Finman appealed to Secretary of Labor.
Secretary upheld the POEA order.

ISSUE
WON Finman can be held liable for complainants
claims against Pan Pacific

HELD
YES
- Under Insurance Code, liability of surety in a
surety bond is joint and several with the principal
obligor.
!"#$%&"'( *+,-
.71

- Conditions of a bond specified and required in the
provisions of a statute providing for submission of
the bond, are incorporated into all bonds tendered
under that statute even though not set out in
printers ink.
- POEA held and Secretary of Labor affirmed that
Pan Pacific had violated Labor Code, and at least one
of the conditions for the grant and continued use of
the recruitment license. POEA and Secretary of
Labor can require Pan Pacific to refund the
placement fees and to impose the fine.
- If Pan Pacific is liable, and if Finman is solidarily
liable with Pan Pacific, then Finman is liable both to
private respondents and to POEA.
- Cash and surety bonds are required from
recruitment companies as means of ensuring prompt
and effective recourse against such companies when
held liable. Public policy will be effectively negated if
POEA and the DoLE were held powerless to compel a
surety company to make good on its solidary
undertaking.

EAGLE STAR INSURANCE CO LTD v. CHIA YU
96 PHIL 696
REYES; March 31, 1955

NATURE
Certiorari

FACTS
- Atkin, Kroll & Co., loaded on the S. S. Roeph
Silverlight owned and operated by Leigh Hoegh &
Co., A/S, of San Francisco California, 14 bales of
assorted underwear valued at P8,085.23 consigned
to Chia Yu in the City of Manila.
- The shipment was insured against all risks by Eagle
Star Ins. Co. of San Francisco, California, under a
policy issued to the shipper and by the latter
assigned to the consignee.
- The vessel arrived in Manila but of the 14 bales
(a.k.a. freights =p) consigned to Chia Yu only 10
were delivered to him as the remaining 3 could not
be found.3 of those delivered were also found
damaged to the extent of 50 per cent.
-Chia Yu claimed indemnity for the missing and
damaged bales. But the claim was declined, first, by
the carrier and afterward by the insurer, whereupon
Chia Yu brought the present action against both,
including their respective agents in the Philippines.
- An action was filed at the CFI after more than 2
years after delivery of the damaged bales and the
date when the missing bales should have been
delivered, the action was resisted by the Atkins and
Eagle Star principally on the ground of prescription.
-TC favored Chia Yu and CA affirmed.
*** CARRIERs defense of prescription is made to
rest on the following stipulation of the bill of lading:
In any event the carrier and the ship shall be
discharged from all liability in respect of loss or
damage unless suit is brought within one year
after the delivery of the goods or the date when
the goods should have been delivered. (This
stipulation is but a repetition of a provision in the
CA 65 which says that bills of lading covering
shipments from the US to the Phils should be
brought w/in one year after the delivery of the
goods or the date when the goods should have
been delivered to hold the carrier liable.)
*** INSURERs claim of prescription is founded upon
the terms of the policy and not upon the bill of
lading. (But in our jurisdiction, as per A1144,
prescription is 10 years after action accrues.)
No suit action on this Policy, for the recovery of
any claim, shall be sustainable in any Court of law
or equity unless the insured shall have fully
complied with all the terms and conditions of this
Policy nor unless commenced with twelve (12)
months next after the happening of the loss . . .

ISSUE
WON ATKIN s action has prescribed

HELD
NO
- Being contrary to the law of the forum, the
stipulation in the policy cannot be given effect as it
would reduce the period allowed the insured for
bringing his action to less than one year (because
the prescription period begins from the happening
of the loss and that before any suit could be
sustained the insured shall have to comply with the
terms and conditions of the policy first TF lessening
the period to less than a year. )
- Insular Government vs. Frank(13 Phil. 236)~
"matters respecting a remedy, such as the bringing
of suit, admissibility of evidence, and statute of
limitations, depend upon the law of the place where
the suit is brought" TF any policy clause repugnant
to this amendment to the Insurance Act cannot be
given effect in an action in our courts.
SEC. 61-A. (Insurance Code) ~ Any condition,
stipulation or agreement in any policy of
insurance, limiting the time for commencing an
action thereunder to a period of less than one year
from the time when the cause of action accrues, is
void.
- The prescription clause could be harmonized with
section 61-A of the Insurance Act by taking it to
mean that the time given the insured for bringing his
suit is twelve months after the cause of action
accrues.
- If so, when did the cause of action accrue? Chia
Yus action did not accrue until his claim was finally
rejected by the insurance company. This is because,
before such final rejection, there was no real
necessity for bringing suit.
- As the policy provides that the insured should file
his claim, first, with the carrier and then with the
insurer, he had a right to wait for his claim to be
finally decided before going to court.
- Furthermore, there is nothing in the record to show
that the claim was rejected in the year 1947, either
by the insurance company in London or its settling
agents in the Philippines.
- For the purpose of this action, Chia Yu's claim was
considered to have been finally rejected by the
insurer on April 22, 1948. Having been filed within
twelve months form that date, the action cannot be
deemed to have prescribed even on the supposition
that the period given the insured for bringing suit
under the prescriptive clause of the policy is twelve
months after the accrual of the cause of action.
- Contractual limitations contained in insurance
policies are regarded with extreme jealousy by
courts and will be strictly construed against the
insurer and should not be permitted to prevent a
recovery when their just and honest application
would not produce that result. (46 C. J. S. 273.)
Disposition Judgment appealed from is REVERSED
with respect to the carrier and its agents but
AFFIRMED with respect to the insurance company
and its agents.





ACCFA v. ALPHA INSURANCE
24 SCRA 151
REYES; July 29, 1968

FACTS
!"#$%&"'( *+,-
.72

- In order to guarantee the Asingan Farmers'
Cooperative Marketing Association, Inc. (FACOMA)
against loss on account of "personal dishonesty,
amounting to larceny or estafa of its Secretary-
Treasurer, Ladines, the appellee, Alpha Insurance &
Surety Company had issued, on 14 February 1958,
its bond, No. P-FID-15-58, for the sum of P5,000
with said Ladines as principal and the appellee as
solidary surety. On the same date, the Asingan
FACOMA assigned its rights to the appellant,
Agricultural Credit Cooperative and Financing
Administration (ACCFA for short), with approval of
the principal and the surety.
- During the effectivity of the bond, Ladines
converted and misappropriated, to his personal
benefit, some P11,513.22 of the FACOMA funds, of
which P6,307.33 belonged to the ACCFA. Upon
discovery of the loss, ACCFA immediately notified in
writing the survey company on 10 October 1958,
and presented the proof of loss within the period
fixed in the bond; but despite repeated demands the
surety company refused and failed to pay.
Whereupon, ACCFA filed suit against appellee on 30
May 1960.
- Defendant Alpha Insurance & Surety Co., Inc.,
(now appellee) moved to dismiss the complaint for
failure to state a cause of action, giving as reason
that (1) the same was filed more than one year after
plaintiff made claim for loss, contrary to the eighth
condition of the bond, providing as follows:
EIGHT LIMITATION OF ACTION: No action, suit or
proceeding shall be had or maintained upon this
Bond unless the same be commenced within one
year from the time of making claim for the loss upon
which such action, suit or proceeding, is based, in
accordance with the fourth section hereof.
(2) the complaint failed to show that plaintiff had
filed civil or criminal action against Ladines, as
required by conditions 4 and 11 of the bond; and (3)
that Ladines was a necessary and indispensable
party but had not been joined as such.
- At first, the Court of First Instance denied
dismissal; but, upon reconsideration, the court
reversed its original stand, and dismissed the
complaint on the ground that the action was filed
beyond the contractual limitation period. Hence, this
appeal.

ISSUE
WON the provision of a fidelity bond that no action
shall be had or maintained thereon unless
commenced within one year from the making of a
claim for the loss upon which the action is based, is
valid, in view of Section 61-A of the Insurance Act
invalidating stipulations limiting the time for
commencing an action thereon to less than one year
from the time the cause of action accrues

HELD
NO
- A fidelity bond is, in effect, in the nature of a
contract of insurance against loss from misconduct,
and is governed by the same principles of
interpretation. Consequently, the condition of the
bond in question, limiting the period for bringing
action thereon, is subject to the provisions of Section
61-A of the Insurance Act (No. 2427), as amended
by Act 4101 of the pre-Commonwealth Philippine
Legislature, prescribing that:
SEC. 61-A: A condition, stipulation or agreement
in any policy of insurance, limiting the time for
commencing an action thereunder to a period of
less than one year from the time when the cause
of action accrues is void.
- Since a "cause of action" requires, as essential
elements, not only a legal right of the plaintiff and a
correlative obligation of the defendant but also "an
act or omission of the defendant in violation of said
legal right," the cause of action does not accrue until
the party obligated refuses, expressly or impliedly, to
comply with its duty (in this case, to pay the amount
of the bond). The year for instituting action in court
must be reckoned, therefore, from the time of
appellee's refusal to comply with its bond; it can not
be counted from the creditor's filing of the claim of
loss, for that does not import that the surety
company will refuse to pay. In so far, therefore, as
condition eight of the bond requires action to be filed
within one year from the filing of the claim for loss,
such stipulation contradicts the public policy
expressed in Section 61-A of the Philippine Insurance
Act.
- Condition eight of the bond, therefore, is null and
void, and the appellant is not bound to comply with
its provisions. The discouraging of unnecessary
litigation must be deemed a rule of public policy,
considering the unrelieved congestion in the courts.
As a consequence of the foregoing, action may be
brought within the statutory period of limitation for
written contracts (New Civil Code, Article 1144).

ANG v. FULTON FIRE INSURANCE CO.
2 SCRA 945
LABRADOR; July 31, 1961

NATURE
Appeal from judgment of the CFI ordering the
defendant Fulton Fire Insurance Co. to pay the
plaintiffs the sum of P10,000.00, with interest, and
an additional sum of P2,000.00 as attorney's fees,
and costs.

FACTS
- The stocks of general merchandise in the store of
the Ang spouses are insured with Fulton. While the
insurance was in force, fire destroyed the goods. The
Angs filed their first claim immediately after the fire.
- Their claim was denied on April 6, 1956. They
received notice on April 19, 1956.
- The Angs brought an action against the agent on
May 11, 1956. The court denied the suit and the mfr
on Sept. 3 and 12, 1957.
- The Angs filed against Fulton on May 26, 1958.
- There was a clause in the policy:
13.If the claim be in any respect fraudulent, or if
any false declaration is made or used in support
thereof, or if any fraudulent means or devices are
used by the Insured or any one acting on his
behalf to obtain any benefit under this Policy, or, if
the loss or damage be occasioned by the wilful act
or with the connivance of the Insured, or, if the
claim be made and rejected and an action or suit
be not commenced within twelve months after
such rejection or (in case of arbitration taking
place in pursuance of the 18th condition of this
Policy) within twelve months after the arbitrator or
arbitrators or umpire shall have made their award
all benefit under this Policy shall be forfeited."

ISSUE
WON the suit against the agent tolled the
prescription period, such that the filing against
Fulton was only 9 months after the claim was
rejected

HELD
NO
- The bringing of the action against the Paramount
Surety & Insurance Company, the agent of the
defendant company, cannot have any legal effect
except that of notifying the agent of the claim.
Beyond such notification, the filing of the action can
serve no other purpose. There is no law giving any
!"#$%&"'( *+,-
.73

effect to such action upon the principal. Besides,
there is no condition in the policy that the action
must be filed against the agent, and the Court can
not by interpretation extend the clear scope of the
agreement beyond what is agreed upon by the
parties.
- Their contract is the law between the parties, and
their agreement that an action on a claim denied by
the insurer must be brought within one year from
the denial, governs, not the rules on the prescription
of actions.
Disposition The judgment appealed from is hereby
set aside and the case dismissed, with costs against
plaintiffs-appellees.

TRAVELLERS INSURANCE & SURETY CORP. v.
CA (MENDOZA)
272 SCRA 536
HERMOSISIMA, JR; May 22, 1997

NATURE
The petition herein seeks the review and reversal of
the decision of respondent Court of Appeals affirming
in toto the judgment of the Regional Trial Court in an
action for damages filed by private respondent
Vicente Mendoza, Jr. as heir of his mother who was
killed in a vehicular accident.

FACTS
-an old lady was hit by a taxicab. The taxicab was
later identified and a case was filed against the
driver and owner. Later, an amendment was filed to
include the insurance company. RTC and CA ordered
that the owner, driver as well as the insurance
company be held solidarily liable.

ISSUE
WON RTC and CA erred
HELD
YES
- Where the contract provides for indemnity against
liability to third persons, then third persons to whom
the insured is liable can sue the insurer. Where the
contract is for indemnity against actual loss or
payment, then third persons cannot proceed against
the insurer, the contract being solely to reimburse
the insured for liability actually discharged by him
thru payment to third persons, said third persons'
recourse being thus limited to the insured alone. But
in the case at bar, there was no contract shown.
What then was the basis of the RTC and the CA
to say that the insurance contract was a third-
party liability insurance policy? Consequently,
the trial court was confused as it did not distinguish
between the private respondent's cause of action
against the owner and the driver of the Lady Love
taxicab and his cause of action against petitioner.
The former is based on torts and quasi-delicts while
the latter is based on contract.
- Even assuming arguendo that there was such a
contract, private respondent's cause of action can
not prevail because he failed to file the written
claim mandated by the Insurance Code (before
it was amended-action must be brought within
six months from date of the accident (this is
whats applicable here) ; after amendment-
"action or suit for recovery of damage due to loss or
injury must be brought in proper cases, with the
Commissioner or the Courts within one year from
denial of the claim, otherwise the claimant's right of
action shall prescribe" ). He is deemed, under this
legal provision, to have waived his rights as against
petitioner-insurer.
Disposition petition granted

SUN INSURANCE v. CA (supra p.57)

COASTWISE v. CA (supra p.70)

CEBU SHIPYARD v. WILLIAM LINES (supra p.3)

MANILA MAHOGANY MANUFACTURING CORP v.
CA (ZENITH INSURANCE CORP)
154 SCRA 652
PADILLA; October 12, 1987

NATURE
Petition to review CA decision ordering Manila
Mahogany Manufacturing Corporation to pay Zenith
Insurance Corporation P5,000 with 6% annual
interest, attorney's fees, and costs of suit

FACTS
- From 6 March 1970 to 6 March 1971, MLA
MAHOGANY insured its Mercedes Benz 4-door sedan
with ZENITH.
- On 4 May 1970, the insured vehicle was bumped
and damaged by a truck owned by San Miguel
Corporation. For the damage caused, ZENITH paid
MLA MAHOGANY P5,000 in amicable settlement. MLA
MAHOGANY's general manager executed a Release of
Claim, subrogating respondent company to all its
right to action against San Miguel Corporation.
- On 11 Dec 1972, ZENITH wrote Insurance
Adjusters, Inc. to demand reimbursement from San
Miguel. Insurance Adjusters, Inc. refused
reimbursement, alleging that San Miguel had already
paid petitioner P4,500, as evidenced by a cash
voucher and a Release of Claim executed by the
General Manager of petitioner.
- ZENITH thus demanded from petitioner
reimbursement of the sum of P4,500 paid by San
Miguel.
- City Court ordered petitioner to pay respondent
P4,500.
- CFI affirmed the City Court's decision in toto.
- CA affirned CFI, with the modification that
petitioner was to pay the total amount of P5,000 it
had earlier received from ZENITH.
Petitioners Claims
> It is not bound to pay P4,500, and much more,
P5,000 to ZENITH as the subrogation in the Release
of Claim it executed in favor of respondent was
conditioned on recovery of the total amount of
damages petitioner had sustained. Since total
damages were valued by petitioner at P9,486.43 and
only P5,000 was received by petitioner, MLA
MAHOGANY argues that it was entitled to go after
San Miguel to claim the additional P4,500.
> It cites Art. 2207
11
and Art. 1304
12
of the Civil
Code, and claims a preferred right to retain the
amount coming from San Miguel, despite the
subrogation in favor of ZENITH.
Respondents Arguments
> There was no qualification to its right of
subrogation under the Release of Claim executed by
petitioner, the contents having expressed all intents
and purposes of the parties.

ISSUE
WON the insurer may recover the sum of P5,000

HELD
YES

!!
?*62#5" ::&@9 If %71 35/0-%066N2 3$+31$%< 7/2 411- 0-2:$198 /-9 71 7/2 $1.10F19 0-91,-0%< 6$+,
%71 0-2:$/-.1 .+,3/-< 6+$ %71 0-_:$< +$ 5+22 /$020-; +:% +6 %71 =$+-; +$ 4$1/.7 +6 .+-%$/.%
.+,35/0-19 +6 %71 0-2:$/-.1 .+,3/-< 27/55 41 2:4$+;/%19 %+ %71 $0;7%2 +6 %71 0-2:$19 /;/0-2% %71
=$+-;9+1$ +$ %71 31$2+- =7+ 7/2 F0+5/%19 %71 .+-%$/.%& U6 %71 /,+:-% 3/09 4< %71 0-2:$/-.1
.+,3/-< 9+12 -+% 6:55< .+F1$ %71 0-_:$< +$ 5+22 %71 /;;$01F19 3/$%< 27/55 41 1-%0%519 %+ $1.+F1$ %71
9160.01-.< 6$+, %71 31$2+- ./:20-; %71 5+22 +$ 0-_:$<.
!E
?*62#5" %A&B9 A creditor, to whom partial payment has been made, may exercise his right for the
$1,/0-91$8 /-9 71 27/55 41 3$161$$19 %+ %71 31$2+- =7+ 7/2 411- 2:4$+;/%19 0- 702 35/.1 0- F0$%:1
+6 %71 3/$%0/5 3/<,1-% +6 %71 2/,1 .$190%&
!"#$%&"'( *+,-
.74

Ratio Since the insurer can be subrogated to only
such rights as the insured may have, should the
insured, after receiving payment from the insurer,
release the wrongdoer who caused the loss, the
insurer loses his rights against the latter. But in such
a case, the insurer will be entitled to recover from
the insured whatever it has paid to the latter, unless
the release was made with the consent of the
insurer.
Reasoning
- Although petitioners right to file a deficiency claim
against San Miguel is with legal basis, without
prejudice to the insurer's right of subrogation,
nevertheless, when Manila Mahogany executed
another release claim discharging San Miguel from
"all actions, claims, demands and rights of action
that now exist or hereafter arising out of or as a
consequence of the accident" after the insurer had
paid the proceeds of the policy - the compromise
agreement of P5,000 being based on the insurance
policy - the insurer is entitled to recover from the
insured the amount of insurance money paid. Since
petitioner by its own acts released San Miguel,
thereby defeating private respondents right of
subrogation, the right of action of petitioner against
the insurer was also nullified.
- As held in Phil. Air Lines v. Heald Lumber Co.,
under Art. 2207, the real party in interest with
regard to the portion of the indemnity paid is the
insurer and not the insured.
SUBROGATION: The right of subrogation can only
exist after the insurer has paid the insured,
otherwise the insured will be deprived of his right to
full indemnity. If the insurance proceeds are not
sufficient to cover the damages suffered by the
insured, then he may sue the party responsible for
the damage for the remainder. To the extent of the
amount he has already received from the insurer
enjoys the right of subrogation.
Disposition Petition DENIED. Judgment appealed
from is AFFIRMED with costs against petitioner.

PIONEER INSURANCE v. CA (BORDER
MACHINERY & HEAVY EQUIPMENT INC)
175 SCRA 668
GUTIERREZ, JR.; July 28, 1989

NATURE
Petitions for review on certiorari of a decision of the
CA

FACTS
- In 1965, Jacob S. Lim was engaged in the airline
business as owner-operator of Southern Air Lines
(SAL), a single proprietorship.
-On May 17, 1965, at Tokyo, Japan, Japan Domestic
Airlines (JDA) and Lim entered into and executed a
sales contract for the sale and purchase of two (2)
DC-3A Type aircrafts and one (1) set of necessary
spare parts for the total agreed price of US
$109,000.00 to be paid in installments.
- On May 22, 1965, Pioneer Insurance and Surety
Corporation, as surety, executed and issued its
Surety Bond No. 6639 in favor of JDA, in behalf of its
principal, Lim, for the balance price of the aircrafts
and spare parts.
-Border Machinery and Heavy Equipment Company,
Inc. (Bormaheco), Francisco and Modesto Cervantes
(Cervanteses) and Constancio Maglana contributed
some funds used in the purchase of the above
aircrafts and spare parts. They executed two (2)
separate indemnity agreements in favor of Pioneer,
one signed by Maglana and the other jointly signed
by Lim for SAL, Bormaheco and the Cervanteses.
- On June 10, 1965, Lim doing business under the
name and style of SAL executed in favor of Pioneer
as deed of chattel mortgage as security for the
latter's suretyship in favor of the former. It was
stipulated therein that Lim transfer and convey to
the surety the two aircrafts.
- Lim defaulted on his subsequent installment
payments prompting JDA to request payments from
the surety.
- Pioneer paid a total sum of P298,626.12.
- Pioneer then filed a petition for the extrajudicial
foreclosure of the said chattel mortgage before the
Sheriff of Davao City.
- The Cervanteses and Maglana, however, filed a
third party claim alleging that they are co-owners of
the aircrafts,
- On July 19, 1966, Pioneer filed an action for judicial
foreclosure with an application for a writ of
preliminary attachment against Lim and
respondents, the Cervanteses, Bormaheco and
Maglana.
**Maglana, Bormaheco and the Cervanteses filed
cross-claims against Lim alleging that they were not
privies to the contracts signed by Lim and, by way of
counterclaim, sought for damages for being exposed
to litigation and for recovery of the sums of money
they advanced to Lim for the purchase of the
aircrafts in question. (this constitutes the second
petition but will no longer be discussed because it is
not relevant to the topic)
- After trial on the merits, a decision was rendered
holding Lim liable to pay Pioneer but dismissed
Pioneer's complaint against all other
defendants.
- CA modified the trial court's decision in that the
plaintiffs complaint against all the defendants
(including Lim) was dismissed.

ISSUE
WON the petition of Pioneer Insurance and Surety
Corporation against all defendants was rightly
dismissed

HELD
YES
- Both the TC and CA made the finding that Pioneer
reinsured its risk of liability under the surety bond it
had executed in favor of JDA, collected the proceeds
of such reinsurance in the sum of P295,000, and
paid with the said amount the bulk of its alleged
liability to JDA under the said surety bond. The total
amount paid by Pioneer to JDA is P299,666.29. Since
Pioneer has collected P295,000.00 from the
reinsurers, the uninsured portion of what it paid to
JDA is the difference between the two amounts, or
P3,666.28. This is the amount for which Pioneer may
sue defendants, assuming that the indemnity
agreement is still valid and effective. But since the
amount realized from the sale of the mortgaged
chattels are P35,000.00 for one of the airplanes and
P2,050.00 for a spare engine, or a total of
P37,050.00, Pioneer is still overpaid by P33,383.72.
Therefore, Pioneer has no more claim against
defendants.
- The payment to the petitioner made by the
reinsurers was not disputed. Considering this
admitted payment, the only question was the effect
of payment made by the reinsurers to the petitioner
- In general a reinsurer, on payment of a loss
acquires the same rights by subrogation as are
acquired in similar cases where the original insurer
pays a loss (Universal Ins. Co. v. Old Time Molasses
Co.).
- The rules of practice in actions on original
insurance policies are in general applicable to actions
or contracts of reinsurance (Delaware, Ins. Co. v.
Pennsylvania Fire Ins. Co.).
- Hence the applicable law is Article 2207 of the new
Civil Code, to wit: Art. 2207. If the plaintiffs
!"#$%&"'( *+,-
.75

property has been insured, and he has received
indemnity from the insurance company for the injury
or loss arising out of the wrong or breach of contract
complained of, the insurance company shall be
subrogated to the rights of the insured against the
wrongdoer or the person who has violated the
contract. If the amount paid by the insurance
company does not fully cover the injury or loss, the
aggrieved party shall be entitled to recover the
deficiency from the person causing the loss or injury
- If a property is insured and the owner receives the
indemnity from the insurer, it is provided in said
article that the insurer is deemed subrogated to the
rights of the insured against the wrongdoer and if
the amount paid by the insurer does not fully cover
the loss, then the aggrieved party is the one entitled
to recover the deficiency. Evidently, under this legal
provision, the real party in interest with regard to
the portion of the indemnity paid is the insurer and
not the insured (. Air Lines, Inc. v. Heald Lumber
Co., and Manila Mahogany Manufacturing
Corporation v. Court of Appeals)
- It is clear from the records that Pioneer sued in its
own name and not as an attorney-in-fact of the
reinsurer. Accordingly, the appellate court did not
commit a reversible error in dismissing the
petitioner's complaint as against the respondents for
the reason that the petitioner was not the real party
in interest in the complaint and, therefore, has no
cause of action against the respondents.
Disposition Petitions dismissed. Questioned decision
of CA affirmed.


PAN MALAYAN INSURANCE CORPORATION v.
CA (FABIE, HER UNKNOWN DRIVER)
184 SCRA 54
CORTES, April 3, 1990

NATURE
PETITION to review the decision of the Court of
Appeals

FACTS
- Pan Malayan Insurance Company (Panmalay)
insured the Mitsubishi Colt Lancer car registered in
the name of Canlubang Automotive Resources
Corporation (Canlubang) under its motor vehicle
insurance policy. Among the provisions of the policy
was a own-damage clause whereby Panmalay
agrees to indemnify Canlubang in cases of damage
caused by accidental collision or overturning, or
collision or overturning consequent upon mechanical
breakdown or consequent upon wear and tear.
- On 1985, the insured car was sideswept and
damaged by a car owned by Erlinda Fabie, driven by
an unknown driver who fled the scene. Panmalay, in
accordance with the policy, defrayed the cost of
repair of the insured car and was subrogated to the
rights of Canlubang against the driver and owner of
the pick-up. Panmalay then filed a complaint for
damages with RTC Makati against Erlinda Fabie and
her driver on the grounds of subrogation, with the
latter failing and refusing to pay their claim. Fabie
filed a Motion for Bill of Particulars.
- RTC: dismissed complaint for lack of cause of
action (payment by PANMALAY of CANLUBANG's
claim under the "own damage" clause of the
insurance policy was an admission by the insurer
that the damage was caused by the assured and/or
its representatives) Panmalay appealed
- CA: dismissed appeal, affirmed RTC (applying the
ejusdem generis rule held that Section III-1 of the
policy, which was the basis for settlement of
CANLUBANG's claim, did not cover damage arising
from collision or overturning due to the negligence of
third parties as one of the insurable risk)

ISSUE
WON the insurer PANMALAY may institute an action
to recover the amount it had paid its assured in
settlement of an insurance claim against private
respondents as the parties allegedly responsible for
the damage caused to the insured vehicle, in
accordance with A2207, NCC

HELD
YES
Ratio Article 2207 of the Civil Code is founded on
the well-settled principle of subrogation. If the
insured property is destroyed or damaged through
the fault or negligence of a party other than the
assured, then the insurer, upon payment to the
assured, will be subrogated to the rights of the
assured to recover from the wrongdoer to the extent
that the insurer has been obligated to pay. Payment
by the insurer to the assured operates as an
equitable assignment to the former of all remedies
which the latter may have against the third party
whose negligence or wrongful act caused the loss.
The right of subrogation is not dependent upon, nor
does it grow out of, any privity of contract or upon
written assignment of claim. It accrues simply upon
payment of the insurance claim by the insurer
Exceptions
(1) if the assured by his own act releases the
wrongdoer or third party liable for the loss or
damage, from liability, the insurer's right of
subrogation is defeated;
(2) where the insurer pays the assured the value of
the lost goods without notifying the carrier who has
in good faith settled the assured's claim for loss, the
settlement is binding on both the assured and the
insurer, and the latter cannot bring an action against
the carrier on his right of subrogation;
(3) where the insurer pays the assured for a loss
which is not a risk covered by the policy, thereby
effecting "voluntary payment", the former has no
right of subrogation against the third party liable for
the loss
Reasoning
- Both TC and CA are incorrect.
ON TC: Own damage (not found in the insurance
policy) simply meant that Panmalay had assumed to
reimburse the cost for repairing the damage to the
insured vehicle. Its different from Third Party
Liability coverage (liabilities arising from the death
of or bodily injuries suffered by 3
rd
parties) and from
Property Damage coverage (liabilities from damage
caused by insured vehicle to properties of 3
rd
parties)
ON CA: the terms of a contract are to be construed
according to the sense and meaning of the terms
which the parties thereto have used. In the case of
property insurance policies, the evident intention of
the contracting parties, i.e., the insurer and the
assured, determine the import of the various terms
and provisions embodied in the policy. It is only
when the terms of the policy are ambiguous,
equivocal or uncertain, such that the parties
themselves disagree about the meaning of particular
provisions, that the courts will intervene. In such an
event, the policy will be construed by the courts
liberally in favor of the assured and strictly against
the insurer
- Both Panmalay and Canlubang had the same
interpretation regarding the coverage of insured risk
regarding accidental collision or overturning to
include damages caused by 3
rd
party to Canlubang so
it was improper for CA to ascribe meaning contrary
to the clear intention and understanding of the
parties.
- Court on several occasions defined accident or
accidental as taking place without ones foresight
!"#$%&"'( *+,-
.76

or expectation, an event that proceeds from an
unknown cause, or is an unusual effect of a known
cause and, therefore, not expected [Dela Cruz v.
Capital Insurance & Surety Co.] The concept
"accident" is not necessarily synonymous with the
concept of "no fault". It may be utilized simply to
distinguish intentional or malicious acts from
negligent or careless acts of man.
- damage/loss to insured vehicle due to negligence
of 3
rd
parties not listed as exceptions to coverage in
the insurance policy
- Interpretation given by Panmalay is more in
keeping with rationale behind rules on interpretation
of insurance contracts in favor of assured or
beneficiary: indemnity or payment
- EVEN if voluntarily indemnified Canlubang, as
interpreted by TC: the insurer who may have no
rights of subrogation due to "voluntary" payment
may never. theless recover from the third party
responsible for the damage to the insured property
under Article 1236 of the Civil Code. [Sveriges
Angfartygs Assurans Forening v. Qua Chee Gan]
Disposition the present petition is GRANTED.
Petitioner's complaint for damages against private
respondents is hereby REINSTATED. Let the case be
remanded to the lower court for trial on the merits.

FIREMAN'S FUND INSURANCE COMPANY v.
JAMILA & COMPANY, INC.
70 SCRA 323
AQUINO; April 1976

FACTS
- Jamila & Co., Inc. or the Veterans Philippine
Scouts Security Agency contracted to supply security
guards to Firestone. Jamila assumed responsibility
for the acts of its security guards. The First Quezon
City Insurance Co., Inc. executed a bond in the sum
of P20,000 to guarantee Jamila's obligations under
that contract.
- On May 18, 1963 properties of Firestone valued at
P11,925 were lost allegedly due to the acts of its
employees who connived with Jamila's security
guard. Fireman's Fund, as insurer, paid to Firestone
the amount of the loss. Fireman's Fund was
subrogated to Firestone's right to get reimbursement
from Jamila. Jamila and its surety, First Quezon City
failed to pay the amount of the loss in spite of
repeated demands.
- Upon defendant's motions, the lower court
dismissed the complaint as to Jamila on the ground
that there was no allegation that it had consented to
the subrogation and, therefore, Fireman's Fund had
no cause of action against it. It also dismissed the
complaint as to First Quezon City on the ground of
res judicata. It appears that the same action was
previously filed in a civil case which was dismissed
because of the failure of the same plaintiffs and their
counsel to appear at the pre-trial.
- Upon an MR, the lower court set aside its order of
dismissal and sustained plaintiff's contention that
there was no res judicata as to First Quezon City
because the civil case was dismissed without
prejudice. However, due to inadvertence, the lower
court did not state in its order why it set aside its
prior order dismissing the complaint with respect to
Jamila. Jamilla had originally moved for the dismissal
of the complaint on the ground of lack of cause of
action. Its basis for its contention were: (1) that the
complaint did not allege that Firestone, pursuant to
the contractual stipulation quoted in the complaint,
had investigated the loss and that Jamila was
represented in the investigation and (2) that Jamila
did not consent to the subrogation of Fireman's Fund
to Firestone's right to get reimbursement from
Jamila and its surety. The lower court in its order of
dismissal had sustained the second ground.
- Jamila in its MR invoked the first ground which
had never been passed upon by the lower court. But
the lower court in its order granting Jamila's motion
for reconsideration, completely ignored that first
ground. It reverted to the second ground which was
relied upon in its order previous order. The lower
court reiterated its order, stating that Fireman's Fund
had no cause of action against Jamila because Jamila
did not consent to the subrogation. The court did not
mention Firestone, the co-plaintiff of Fireman's Fund.
- Firestone and Fireman's Fund filed an MR on the
ground that Fireman's Fund was suing on the basis
of legal subrogation whereas the lower court
erroneously predicated its dismissal order on the
theory that there was no conventional subrogation
because the debtor's consent was lacking.
- The plaintiffs cited article 2207 of the Civil Code
which provides that "if the plaintiff's property has
been insured, and he has received indemnity from
the insurance company for the injury or loss arising
out of the wrong or breach of contract complained
of, the insurance company shall be subrogated to the
rights of the insured against the wrongdoer or the
person who has violated the contract".
- The lower court denied plaintiff's motion. They
filed a second MR, calling the lower court's attention
to the fact that the issue of subrogation was of no
moment because Firestone, the subrogor, is a party-
plaintiff and could sue directly Jamila in its own right.
Without resolving that contention, the lower court
denied plaintiffs' second MR.

ISSUE
WON the complaint of Firestone and Fireman's Fund
states a cause of action against Jamila

HELD
YES
- Fireman's Fund's action against Jamila is squarely
sanctioned by article 2207. As the insurer, Fireman's
Fund is entitled to go after the person or entity that
violated its contractual commitment to answer for
the loss insured against.
- The trial court erred in applying to this case the
rules on novation. The plaintiffs in alleging in their
complaint that Fireman's Fund "became a party in
interest in this case by virtue of a subrogation right
given in its favor by" Firestone, were not relying on
the novation by change of creditors as contemplated
in articles 1291 and 1300 to 1303 of the Civil Code
but rather on article 2207.
- Article 2207 is a restatement of a settled principle
of American jurisprudence. Subrogation has been
referred to as the doctrine of substitution. It is an
arm of equity that may guide or even force one to
pay a debt for which an obligation was incurred but
which was in whole or in part paid by another.
- Subrogation is founded on principles of justice
and equity, and its operation is governed by
principles of equity. It rests on the principle that
substantial justice should be attained regardless of
form, that is, its basis is the doing of complete,
essential, and perfect justice between all the parties
without regard to form.
- Subrogation is a normal incident of indemnity
insurance. Upon payment of the loss, the insurer is
entitled to be subrogated pro tanto to any right of
action which the insured may have against the third
person whose negligence or wrongful act caused the
loss. The right of subrogation is of the highest
equity. The loss in the first instance is that of the
insured but after reimbursement or compensation, it
becomes the loss of the insurer.
- Although many policies including policies in the
standard form, now provide for subrogation, and
!"#$%&"'( *+,-
..7

thus determine the rights of the insurer in this
respect, the equitable right of subrogation as the
legal effect of payment inures to the insurer without
any formal assignment or any express stipulation to
that effect in the policy. Stated otherwise, when the
insurance company pays for the loss, such payment
operates as an equitable assignment to the insurer
of the property and all remedies which the insured
may have for the recovery thereof. That right is not
dependent upon, nor does it grow out of, any privity
of contract, or upon written assignment of claim, and
payment to the insured makes the insurer an
assignee in equity.
- On the other hand, Firestone is really a nominal
party in this case. It had already been indemnified
for the loss which it had sustained. Obviously, it
joined as a party-plaintiff in order to help Fireman's
Fund to recover the amount of the loss from Jamila
and First Quezon City. Firestone had tacitly assigned
to Fireman's Fund its cause of action against Jamila
for breach of contract. Sufficient ultimate facts are
alleged in the complaint to sustain that cause of
action.

TABACALERA v. NORTH FRONT SHIPPING
272 SCRA 527
BELLOSILLO; May 16, 1997

FACTS
- 20,234 sacks of corn grains valued at P3.5M were
shipped on board North Front 777, defendants
vessel. The cargo was consigned to Republic Flour
Mills Corp. under Bill of Lading No. 001 and insured
with Tabacalera, Prudential Guarantee & Assurance,
and New Zealand Insurance.
- Republic Flour was advised of the vessels arrival in
Manila, but did not immediately commence the
unloading operations. Unloading was sometimes
stopped due to varying weather and sometimes for
no apparent reason. Unloading was only completed
20 days after the arrival of the barge; by then, the
cargo was short 26.333 metric tons and the rest was
already moldy and deteriorating.
- Analyses showed that the deterioration was caused
by moisture content from salt water, which could be
arrested by drying. However, Republic Flour rejected
the entire cargo and demanded that defendant North
Front Shipping pay the damages suffered by it. The
demands were unheeded and the insurance
companies were obliged to pay Republic Flour
P2,189,433
- By virtue of the insurance companies payment,
they were subrogated to the rights of Republic Flour.
Petitioners filed a complaint against North Front
Shipping, claiming the loss was exclusively
attributable to the latters fault and negligence.
Having surveyed the vessel, it was found that the
barge had cracks in its bodega. The hatches on the
crates of grain were not sealed and the tarpaulins
used in covering them were not new, contrary to
North Front Shippings claims. North Front Shipping
reiterated that the barge was inspected prior to
loading and found seaworthy and were issued a
permit to sail by the Coast Guard. They further
averred that the grains were farm wet and not
properly dried before loading.
- The court dismissed the complaint, ruling that the
contract entered into was a charter-party
agreement; as such, only ordinary diligence in the
care of the goods was required of North Front
Shipping.

ISSUE
WON defendant is required to observe extraordinary
diligence in its vigilance over the goods it transports

HELD
YES
- As a corporation engaged in the business of
transporting cargo offering its services
indiscriminately to the public, it is without a doubt a
common carrier. As such, it has the burden of
proving that it observed extraordinary diligence to
avoid responsibility for the lost cargo. The clean bill
of lading it issued disprove the master of the vessels
claim that the grains were farm wet when loaded. If
they were wet, the master of the vessel should have
known that the grains would eventually deteriorate
when sealed in hot compartments in hatches of a
ship and should have undertaken precautionary
measures to avoid this. The arrival of the goods at
the place of destination in bad order makes a prima
facie case against the common carrier, which must
prove its non-liability.
- While petitioners presented evidence of the vessels
bad shape and a laboratory analysis revealing that
the grains were contaminated with salt water,
defendants failed to rebut said arguments or even
endeavor to establish that the loss, destruction or
deterioration was due to a fortuitous event; an
act/omission of the owner of the goods; the
character of the goods or defects in their packing; or
an order or act of a competent public authority.
- However, Republic Flour is also found to be guilty
of contributory negligence for not immediately
staring the unloading operations and for providing no
explanation for the delay. As such, it should share at
least 40% of the loss.
Disposition The decision of the CA is REVERSED
and SET ASIDE

PHILIPPINE AMERICAN LIFE INSURANCE
COMPANY v. CA (ELIZA PULIDO)
344 SCRA 360
GONZAGA-REYES; November 15, 2000

NATURE
This petition for review on certiorari seeks to reverse
the Decision of the Special Second Division of the
Court of Appeals

FACTS
- On January 9, 1989, petitioner received from one
Florence Pulido an application for life insurance,
dated December 16, 1988, in the amount of
P100,000.00 which designated her sister, herein
private respondent, as its principal beneficiary.
Because the insurance applied for was non-medical,
petitioner did not require a medical examination and
issued a policy on the sole basis of the application on
February 11, 1989. On April 1992, petitioner
received private respondents claim, which declared
that the insured, Florence Pulido, died of acute
pneumonia on September 10, 1991.
- Petitioner withheld payment on the ground that the
policy claimed under was void from the start for
having been procured in fraud. It is petitioners
contention that even before they received private
respondents claim for death benefits, their
investigation concerning the subject policy yielded
the information that the insured, Florence Pulido,
died in 1988, before the application for insurance on
her life was made. While this was communicated to
private respondent in a letter, private respondent
had already filed her claim earlier that month. In
another letter, however, petitioner confirmed to
private respondent receipt of the claim papers and
assured her that her case was being given
preferential attention and prompt action.
- Petitioner caused another investigation respecting
the subject policy. Pursuant to the findings of this
second investigation, petitioner stood by its initial
!"#$%&"'( *+,-
...

decision to treat the policy as void and not to honor
the claim. On November 9, 1992, private
respondent enlisted the services of counsel in
reiterating her claim for death benefitsPetitioner still
refused to make payment and thus, this action.

ISSUE
WON lower court erred in holding that there was no
fraud

HELD
- The records bear out that since the onset of this
case, the main issue has always been whether there
was fraud in the obtainment of the disputed policy,
or put differently, whether the insured, Florence
Pulido, was in fact dead before the application for
insurance on her life was made. This the lower
courts had effected ruled on, upon a preponderance
of the evidence duly received from both parties. We
see no reversible error in the finding of both
respondent court and the trial court in favor of the
correctness of the entries in Certificate of Death,
duly registered with the Local Civil Registrar of
Bagulin, La Union, which declared that Florence
Pulido died of acute pneumonia on September 10,
1991. Dr. Irineo Gutierrez, the Municipal Health
Officer of Bagulin, La Union whose signature
appeared in the death certificate, testified in addition
that he ministered to the ailing Florence Pulido for
two days immediately prior to her death. This fact is
likewise noted in the death certificate.
- Death certificates, and notes by a municipal health
officer prepared in the regular performance of his
duties, are prima facie evidence of facts therein
stated. A duly-registered death certificate is
considered a public document and the entries found
therein are presumed correct, unless the party who
contests its accuracy can produce positive evidence
establishing otherwise. Petitioners contention that
the death certificate is suspect because Dr. Gutierrez
was not present when Florence Pulido died, and
knew of Florences death only through Ramon
Piganto, does not merit a conclusion of fraud. No
motive was imputed to Dr. Gutierrez for seeking to
perpetuate a falsity in public records. Petitioner was
likewise unable to make out any clear motive as to
why Ramon Piganto would purposely lie. Mere
allegations of fraud could not substitute for the full
and convincing evidence that is required to prove it.
A failure to do so would leave intact the presumption
of good faith and regularity in the performance of
public duties, which was the basis of both
respondent court and the trial court in finding the
date of Florence Pulidos death to be as plaintiff-
private respondent maintained.
- We cannot likewise give credence to petitioners
submission that the inconsistencies in the
testimonies of the witnesses for plaintiff-private
respondent are in themselves evidence of fraud.
Such alleged inconsistencies are matters of
credibility which had been ably passed upon by the
lower court.
Disposition the instant petition is DENIED

ST.PAUL FIRE & MARINE INSURANCE CO v.
MACONDRAY & CO INC
70 SCRA 122
ANTONIO; March 25, 1976

FACTS
- Winthrop Products, Inc. shipped aboard the SS
"Tai Ping", owned and operated by Wilhelm
Wilhelmsen, 218 cartons and drums of drugs and
medicine, with the freight prepaid, which were
consigned to Winthrop-Stearns, Inc. Barber
Steamship Lines, Inc., agent of Wilhelm Wilhelmsen
issued Bill of Lading No. 34, in the name of Winthrop
Products, Inc. as shipper, with arrival notice in-
Manila to consignee Winthrop-Stearns, Inc. The
shipment was insured by the shipper against loss
and/or damage with the St. Paul Fire & Marine
Insurance Company.
- The SS "Tai Ping" arrived at the Port of Manila and
discharged its aforesaid shipment into the custody of
Manila Port Service, the arrastre contractor for the
Port of Manila. The said shipment was discharged
complete and in good order with the exception of
one (1) drum and several cartons which were in bad
order condition. Because consignee failed to receive
the whole shipment and as several cartons of
medicine were received in bad order condition, the
consignee filed the corresponding claim in the
amount of P1,109.67 representing the C.I.F. value of
the damaged drum and cartons of medicine with the
carrier and the Manila Port Service. However, both
refused to pay such claim. Consequently, the
consignee filed its claim with the insurer, St. Paul
Fire & Marine Insurance Co., the insurance company,
on the basis of such claim, paid to the consignee the
insured value of the lost and damagcd goods,
including other expenses in connection therewith, in
the total amount of $1,134.46 U.S. currency.
- As subrogee of the rights of' the shipper and/or
consignee, the insurer, St. Paul Fire & Marine
Insurance Co., instituted an action against the
defendants for the recovery of said amount of
$1,134.46, plus costs.
- The defendants resisted the action. However, for
the purpose only of avoiding litigation without
admitting liability to the consignee, the defendants
offered to settle the latters claim in full by paying
the C.I.F. value of the damaged cargo, but this offer
was declined by the plaintiff.
- The LC rendered judgment ordering the
defendants to pay the plaintiff the sum of P300.00.
The plaintiff filed a MFR contending that it should
recover the amount of $1,134.46 or its equivalent in
pesos at the rate of P3.90, instead of P2.00, but this
was denied. Hence, this appeal.

ISSUES
1. WON in case of loss or damage, the liability of the
carrier to the consignee is limited to the C.I.F. value
of the goods which were lost or damaged
2. WON the insurer who has paid the claim in dollars
to the consignee should be reimbursed in its peso
equivalent on the date of discharge of the cargo or
on the date of the decision

HELD
1. YES
Ratio The purpose of the bill of lading is to provide
for the rights and liabilities of the parties in reference
to the contract to carry. The stipulation in the bill of
lading limiting the common carrier's liability to the
value of the goods appearing in the bill, unless the
shipper or owner declares a greater value, is valid
and binding. This limitation of the carrier's liability is
sanctioned by the freedom of the contracting parties
to establish such stipulations, clauses, terms, or
conditions as they may deem convenient, provided
they are not contrary to law, morals, good customs
and public policy. A stipulation fixing or limiting the
sum that may be recovered from the carrier on the
loss or deterioration of the goods is valid, provided it
is (a) reasonable and just under the circumstances,
and (b) has been fairly and freely agreed upon. In
the case at bar, the liabilities of the defendants-
appellees with respect to the lost or damaged
shipments are expressly limited to the C.I.F. value of
the goods as per contract of sea carriage embodied
in the bill of lading.
!"#$%&"'( *+,-
../

- The plaintiff-appellant, as insurer, after paying the
claim of the insured for damages under the
insurance, is subrogated merely to the rights of the
assured. As subrogee, it can recover only the
amount that is recoverable by the latter. Since the
right of the assured, in case of loss or damage to the
goods, is limited or restricted by the provisions in the
bill of lading, a suit by the insurer as subrogee
necessarily is subject to like limitations and
restrictions.
2. On the date of the discharge of the cargo. The
peso equivalent was based by the consignee on the
exchange rate of P2.015 to $1.00 which was the rate
existing at that time.

PHILAM v. AUDITOR (supra p.59)

FIELDMENS v. ASIAN SURETY (supra p.60)

EQUITABLE v. RURAL INSURANCE (supra p.60)

COQUIA v. FIELDMEN'S INSURANCE CO. INC.
26 SCRA 178
CONCEPCION; November 29, 1968

NATURE
Appeal from the decision of the CFI certified by CA

FACTS
- December 1, 1961, appellant Fieldmen's Insurance
Company, Inc. issued, in favor of the Manila Yellow
Taxicab Co., Inc. a common carrier accident
insurance policy, covering the period from December
1, 1961 to December ,1962. It was stipulated in said
policy that:
"The Company will, subject to the Limits of Liability
and under the Terms of this Policy, indemnify the
Insured in the event of accident caused by or arising
out of the use of Motor Vehicle against all sums
which the Insured will become legally liable to pay in
respect of: Death or bodily injury to any fare-paying
passenger including the Driver, Conductor and/or
Inspector who is riding in the Motor Vehicle insured
at the time of accident or injury."
- While the policy was in force, or on February 10,
1962, a taxicab of the Insured, driven by Carlito
Coquia, met a vehicular accident to which he died.
The Insured filed therefor a claim for P5,000.00 to
which the Company replied with an offer to pay
P2,000.00, by way of compromise. The Insured
rejected it and made a counter-offer for P4,000.00,
but the Company did not accept it.
- On September 18, 1962, the Insured and Carlito's
parents filed a complaint against the Company to
collect the proceeds of the policy. In its answer, the
Company admitted the existence thereof, but
pleaded lack of cause of action on the part of the
plaintiffs.
- TC rendered a decision sentencing the Company to
pay to the plaintiffs the sum of P4,000.00 and the
costs. Hence, this appeal by the Company, which
contends that plaintiffs have no cause of action
because: 1) the Coquias have no contractual relation
with the Company; and 2) the Insured has not
complied with the provisions of the policy concerning
arbitration.

ISSUES
1. WON there was contractual relations between the
Coquias and the Company
2. WON the insured has not complied with the
provisions of the policy concerning arbitration

HELD
1. Although, in general, only parties to a contract
may bring an action based thereon, this rule is
subject to exceptions, one of which is found in the
Art 1311 CC, reading:
"If a contract should contain some stipulation in
favor of a third person, he may demand its
fulfillment provided he communicated his acceptance
of the obligor before its revocation. A mere incidental
benefit or interest of a person is not sufficient. The
contracting parties must have clearly and
deliberately conferred a favor upon a third person."
- Does the policy in question belong to such class of
contracts pour autrui?
In this connection, said policy provides, inter alia:
"Section I Liability to Passengers. 1. The
Company will, subject to the Limits of Liability and
under the Terms of this Policy, indemnify the Insured
in the event of accident caused by or arising out of
the use of Motor Vehicle against all sums which the
Insured will become legally liable to pay in respect
of: Death or bodily injury to any fare-paying
passenger including the Driver. . . who is riding in
the Motor Vehicle insured at the time of accident or
injury.
"Section II. Liability to the Public
"3. In terms of and subject to the limitations of
and for the purposes of this Section, the Company
will indemnify any authorized Driver who is driving
the Motor Vehicle . . . "
"Conditions
"7. In the event of death of any person entitled
to indemnify under this Policy, the Company will, in
respect of the liability incurred by such person,
indemnify his personal representatives in terms of
and subject to the limitations of this Policy, provided,
that such representatives shall, as though they were
the Insured, observe, fulfill and be subject to the
Terms of this Policy insofar as they can apply.
"8. The Company may, at its option, make
indemnity payable directly to the claimants or heirs
of claimants, with or without securing the consent of
or prior notification to the Insured, it being the true
intention of this Policy to protect, to the extent
herein specified and subject always to the Terms of
this Policy, the liabilities of the Insured towards the
passengers of the Motor Vehicle and the Public."
- Thus, the policy under consideration is typical of
contracts pour autrui, this character being made
more manifest by the fact that the deceased driver
paid fifty percent (50%) of the corresponding
premiums, which were deducted from his weekly
commissions. Under these conditions, it is clear that
the Coquias who, admittedly, are the sole heirs of
the deceased have a direct cause of action against
the Company, and, since they could have maintained
this action by themselves, without the assistance of
the Insured, it goes without saying that they could
and did properly join the latter in filing the complaint
herein.
2. Based upon Section 17 of the policy:
"If any difference or dispute shall arise with
respect to the amount of the Company's liability
under this Policy, the same shall be referred to the
decision of a single arbitrator to be agreed upon by
both parties or failing such agreement of a single
arbitrator, to the decision of two arbitrators, one
to be appointed in writing by each of the parties
within one calendar month after having been
required in writing so to do by either of the parties
and in case of disagreement between the
arbitrators, to the decision of an umpire who shall
have been appointed in writing by the arbitrators
before entering on the reference and the costs of
and incidental to the reference shall be dealt with
in the Award. And it is hereby expressly stipulated
and declared that it shall be a condition precedent
to any right of action or suit upon this Policy that
the award by such arbitrator, arbitrators or umpire
!"#$%&"'( *+,-
..0

of the amount of the Company's liability hereunder
if disputed shall be first obtained."
- The record shows that none of the parties to the
contract invoked this section, or made any reference
to arbitration, during the negotiations preceding the
institution of the present case. In fact, counsel for
both parties stipulated, in the trial court, that none
of them had, at any time during said negotiations,
even suggested the settlement of the issue between
them by arbitration, as provided in said section.
Their aforementioned acts or omissions had the
effect of a waiver of their respective right to demand
an arbitration.
Disposition The decision appealed from should be
as it is hereby affirmed in toto, with costs against the
herein defendant-appellant, Fieldmen's Insurance
Co., Inc.

COUNTRY BANKERS INSURANCE CORP v.
LIANGA BAY
DE LEON; January 25, 2002

NATURE
Petition for review on certiorari

FACTS
- Lianga Bay is a duly registered cooperative
judicially declared insolvent and is here represented
by, Cornelio Jamero. Country Bankers Insurance
and Lianga Bay entered into a contract of fire
insurance. Country Bankers insured the
respondents stocks-in-trade against fire loss,
damage or liability during the period starting from
June 20, 1989 at 4:00 p.m. to June 20, 1990 at 4:00
p.m., for the sum of P200,000.00.
- On July 1, 1989, at or about 12:40 a.m., the
respondents building at Barangay Diatagon, Lianga,
Surigao del Sur was gutted by fire, resulting in the
total loss of the respondents stocks-in-trade, pieces
of furnitures and fixtures, equipments and records.
- Due to the loss, the respondent filed an insurance
claim with the petitioner under its Fire Insurance
Policy, submitting: (a) the Spot Report of Pfc. Arturo
V. Juarbal, INP Investigator, dated July 1, 1989; (b)
the Sworn Statement of Jose Lomocso; and (c) the
Sworn Statement of Ernesto Urbiztondo.
- The petitioner, however, denied the insurance
claim on the ground that, based on the submitted
documents, the building was set on fire by 2 NPA
rebels who wanted to obtain canned goods, rice and
medicines as provisions for their comrades in the
forest, and that such loss was an excepted risk under
paragraph No. 6 of the policy conditions of Fire
Insurance Policy No. F-1397, which provides:
This insurance does not cover any loss or damage
occasioned by or through or in consequence,
directly or indirectly, of any of the following
occurrences, namely:
(d) Mutiny, riot, military or popular uprising,
insurrection, rebellion, revolution, military or
usurped power.
Any loss or damage happening during the
existence of abnormal conditions (whether physical
or otherwise) which are occasioned by or through
or in consequence, directly or indirectly, of any of
said occurrences shall be deemed to be loss or
damage which is not covered by this insurance,
except to the extent that the Insured shall prove
that such loss or damage happened independently
of the existence of such abnormal conditions.
- Finding the denial of its claim unacceptable, Lianga
Bay then instituted in the trial court the complaint
for recovery of "loss, damage or liability" against
Country Bankers.
- RTC ruled in favor of the cooperative. CA affirmed.

ISSUE
WON the cause of the loss was an excepted risk
under the terms of the fire insurance policy

HELD
- Where a risk is excepted by the terms of a policy
which insures against other perils or hazards, loss
from such a risk constitutes a defense which the
insurer may urge, since it has not assumed that risk,
and from this it follows that an insurer seeking to
defeat a claim because of an exception or limitation
in the policy has the burden of proving that the loss
comes within the purview of the exception or
limitation set up. If a proof is made of a loss
apparently within a contract of insurance, the burden
is upon the insurer to prove that the loss arose from
a cause of loss which is excepted or for which it is
not liable, or from a cause which limits its liability.

Stated else wise, since Country bank here is
defending on the ground of non-coverage and relying
upon an exemption or exception clause in the fire
insurance policy it has the burden of proving the
facts upon which such excepted risk is based, by a
preponderance of evidence. But petitioner failed to
do so.
- The petitioner relies on the Sworn Statements of
Jose Lomocso and Ernesto Urbiztondo as well as on
the Spot Report of Pfc. Arturo V. Juarbal. A witness
can testify only to those facts which he knows of his
personal knowledge, which means those facts which
are derived from his perception.

Consequently, a
witness may not testify as to what he merely learned
from others either because he was told or read or
heard the same. Such testimony is considered
hearsay and may not be received as proof of the
truth of what he has learned.
Disposition the appealed Decision is MODIFIED.
The rate of interest on the adjudged principal
amount of Two Hundred Thousand Pesos
(P200,000.00) shall be six percent (6%) per annum
computed from the date of filing of the Complaint in
the trial court. The awards in the amounts of Fifty
Thousand Pesos (P50,000.00) as actual damages,
Fifty Thousand Pesos (P50,000.00) as exemplary
damages, Five Thousand Pesos (P5,000.00) as
litigation expenses, and Ten Thousand Pesos
(P10,000.00) as attorney?s fees are hereby
DELETED.

DBP POOL OF ACCREDITED INSURANCE v.
RADIO MINDANAO NETWORK
480 SCRA 314
MARTINEZ; January 27, 2006

NATURE
Petition for certiorari

FACTS
- In the evening of July 27, 1988, the radio station of
Radio Mindanao Network located at the SSS Building
in Bacolod City was burned down causing damage in
the amount of over one million pesos. Respondent
sought to recover under two insurance policies but
the claims were denied on the basis that the case of
the loss was an excepted risk under condition no. 6
(c) and (d), to wit:
6. This insurance does not cover any loss or damage
occasioned by or through or in consequence, directly
or indirectly, of any of the following consequences,
namely:
(c) War, invasion, act of foreign enemies, hostilities,
or warlike operations (whether war be declared or
not), civic war.
(d) Mutiny, riot, military or popular uprising,
insurrection, rebellion, revolution, military or
usurped power.
!"#$%&"'( *+,-
..1

- The insurers maintained that based on witnesses
and evidence gathered at the site, the fire was
caused by the members of the Communist Party of
the Philippines/New Peoples Army. Hence the refusal
to honor their obligations.
- The trial court and the CA found in favor of the
respondent. In its findings, both courts mentioned
the fact that there was no credible evidence
presented that the CCP/NPA did in fact cause the fire
that gutted the radio station in Bacolod.

ISSUE
WON the insurance companies are liable to pay
Radio Mindanao Network under the insurance policies

HELD
YES
- The Court will not disturb the factual findings of the
appellant and trial courts absent compelling reason.
Under this mode of review, the jurisdiction of the
court is limited to reviewing only errors of law.
- Particularly in cases of insurance disputes with
regard to excepted risks, it is the insurance
companies which have the burden to prove that the
loss comes within the purview of the exception or
limitation set up. It is sufficient for the insured to
prove the fact of damage or loss. Once the insured
makes out a prima facie case in its favor, the duty or
burden of evidence shifts to the insurer to controvert
said prima facie case.
Disposition Petition dismissed. Decision of the CA is
affirmed.

LEA MER INDUSTRIES v. MALAYAN INSURANCE
471 SCRA 698
PANGANIBAN; September 30, 2005

NATURE
Petition for Review

FACTS
- Ilian Silica Mining entered into a contract of
carriage with Lea Mer Industries for the shipment of
900 metric tons of silica sand valued at P565,000.
Consigned to Vulcan Industrial and Mining
Corporation, the cargo was to be transported from
Palawan to Manila. The silica sand was placed on
board Judy VII, a barge leased by Lea Mer, the
vessel sank, resulting in the loss of the cargo.

ISSUE
WON Lea Mer is liable for the loss of the cargo

HELD
YES
- Common carriers are bound to observe
extraordinary diligence in their vigilance over the
goods and the safety of the passengers they
transport, as required by the nature of their business
and for reasons of public policy. Extraordinary
diligence requires rendering service with the greatest
skill and foresight to avoid damage and destruction
to the goods entrusted for carriage and delivery.
- Common carriers are presumed to have been at
fault or to have acted negligently for loss or damage
to the goods that they have transported. This
presumption can be rebutted only by proof that they
observed extraordinary diligence, or that the loss or
damage was occasioned by any of the following
causes:
(1) Flood, storm, earthquake, lightning, or other
natural disaster or calamity;
(2) Act of the public enemy in war, whether
international or civil;
(3) Act or omission of the shipper or owner of the
goods;
(4) The character of the goods or defects in the
packing or in the containers;
(5) Order or act of competent public authority.
- To excuse the common carrier fully of any liability,
the fortuitous event must have been the proximate
and only cause of the loss. It should have exercised
due diligence to prevent or minimize the loss before,
during and after the occurrence of the event.
- Petitioner bore the burden of proving that it had
exercised extraordinary diligence to avoid the loss,
or that the loss had been occasioned by a fortuitous
event -- an exempting circumstance.
- The evidence presented by petitioner in support of
its defense of fortuitous event was sorely
insufficient. It was not enough for the common
carrier to show that there was an unforeseen or
unexpected occurrence.
Disposition Petition is DENIED and the assailed
Decision and Resolution are AFFIRMED. Costs
against petitioner.
LOADSTAR SHIPPING CO INC v. PIONEER ASIA
INSURANCE CORP
GR No. 157481
QUISUMBING; January 24, 2006

NATURE
Review on certiorari (1) the Decision dated October
15, 2002 and (2) the Resolution dated February 27,
2003 of CA

FACTS
- June 6, 1984 - Petitioner Loadstar Shipping Co.,
Inc. (LOADSTAR), registered owner and operator of
the vessel M/V Weasel, entered into a voyage-
charter with Northern Mindanao Transport Company,
Inc. for the carriage of 65,000 bags of cement from
Iligan City to Manila. The shipper was Iligan Cement
Corporation, while the consignee in Manila was
Market Developers, Inc. (MARKET)
- June 24, 1984 - 67,500 bags of cement were
loaded on board M/V Weasel and stowed in the cargo
holds for delivery to the consignee. The shipment
was covered by petitioners Bill of Lading dated June
23, 1984.
- Prior to the voyage, the consignee insured the
shipment of cement with respondent Pioneer Asia
Insurance Corporation (PIONEER) for P1,400,000, for
which it issued Marine Open Policy No. MOP-006
dated September 17, 1980, covering all shipments
made on or after September 30, 1980
- June 25, 1984 - Captain Montera of M/V Weasel
ordered the vessel to be forced aground which
rendered the entire shipment of cement as good as
gone due to exposure to sea water. LOASTAR thus
failed to deliver the goods to MARKET in Manila.
- MARKET demanded from LOADSTAR full
reimbursement of the cost of the lost shipment.
LOADSTAR refused to reimburse the MARKET despite
repeated demands.
- March 11, 1985 PIONEER paid the MARKET
P1,400,000 plus an additional amount of P500,000,
the value of the lost shipment of cement. In return,
the MARKET executed a Loss and Subrogation
Receipt in favor of PIONEER concerning the latters
subrogation rights against LOADSTAR.
- October 15, 1986 PIONEER filed a complaint
against LOADSTAR with the RTC Manila alleging
that: (1) the M/V Weasel was not seaworthy at the
commencement of the voyage; (2) the weather and
sea conditions then prevailing were usual and
expected for that time of the year and as such, was
an ordinary peril of the voyage for which the M/V
Weasel should have been normally able to cope with;
and (3) LOADSTAR was negligent in the selection
!"#$%&"'( *+,-
..2

and supervision of its agents and employees then
manning the M/V Weasel.
- LOADSTAR alleged that no fault nor negligence
could be attributed to it because it exercised due
diligence to make the ship seaworthy, as well as
properly manned and equipped and failure to deliver
was due to force majeure.
- February 15, 1993 - RTC decided in favor of
PIONEER and that LOADSTAR , as a common carrier,
bears the burden of proving that it exercised
extraordinary diligence in its vigilance over the goods
it transported. The trial court explained that in case
of loss or destruction of the goods, a statutory
presumption arises that the common carrier was
negligent unless it could prove that it had observed
extraordinary diligence. LOADSTARS defense of
force majeure was found bereft of factual basis as a
PAG-ASA report that at the time of the incident,
tropical storm Asiang had moved away from the
Philippines was presented.
- October 15, 2002 CA affirmed RTC Decision with
modification

ISSUES
1. WON LOADSTAR is a common carrier under
Article 1732 CC
2. Assuming it is a common carrier, WON proximate
cause of the loss of cargo was not a fortuitous event
but was allegedly due to the failure of petitioner to
exercise extraordinary diligence

HELD
1. YES
- A1732 CC defines a common carrier as
follows:
Common carriers are persons, corporations, firms
or associations engaged in the business of carrying
or transporting passengers or goods or both, by
land, water, or air, for compensation, offering their
services to the public.
- LOADSTAR is a corporation engaged in the
business of transporting cargo by water and for
compensation, offering its services indiscriminately
to the public. Thus, without doubt, it is a common
carrier. Even if it entered into a voyage-charter
agreement with Northern Mindanao Transport
Company, Inc, it did not in any way convert the
common carrier into a private carrier.
> Planters Products, Inc. v. CA - public carrier
shall remain as such, notwithstanding the charter of
the whole or portion of a vessel by one or more
persons, provided the charter is limited to the ship
only, as in the case of a time-charter or voyage-
charter. It is only when the charter includes both
the vessel and its crew, as in a bareboat or demise
that a common carrier becomes private, at least
insofar as the particular voyage covering the charter-
party is concerned.
2. YES
- As a common carrier, LOADSTAR is required to
observe extraordinary diligence in the vigilance over
the goods it transports. When the goods placed in its
care are lost, LOADSTAR is presumed to have been
at fault or to have acted negligently. LOADSTAR has
the burden of proving that it observed extraordinary
diligence in order to avoid responsibility for the lost
cargo.
- Compania Maritima V CA - It requires common
carriers to render service with the greatest skill and
foresight and to use all reasonable means to
ascertain the nature and characteristics of goods
tendered for shipment, and to exercise due care in
the handling and stowage, including such methods
as their nature requires.
- A1734 CC enumerates the instances when a carrier
might be exempt from liability for the loss of the
goods.
(1) Flood, storm, earthquake, lightning,
or other natural disaster or calamity;
(2) Act of the public enemy in war,
whether international or civil;
(3) Act or omission of the shipper or
owner of the goods;
(4) The character of the goods or defects
in the packing or in the containers; and
(5) Order or act of competent public
authority
- LOADSTAR claims that the loss of the goods was
due to a fortuitous event under paragraph 1. Yet, its
claim is not substantiated. It is supported by
evidence that the loss of the entire shipment of
cement was due to the gross negligence of
LOADSTAR
- Records show that in the evening of June 24, 1984,
the sea and weather conditions in the vicinity of
Negros Occidental were calm. The records reveal
that LOADSTAR took a shortcut route, instead of the
usual route, which exposed the voyage to
unexpected hazard. LOADSTAR has only itself to
blame for its misjudgment.
Disposition petition is DENIED

You might also like